Sunteți pe pagina 1din 228

!

1. SEÑALE LA RESPUESTA FALSA CON RESPECTO AL LÉNTIGO MALIGNO


MELANOMA:

1. 1. APARECE SOBRE TODO EN EL TRONCO.


2. 2. LAS PERSONAS MAYORES DE 60-65 AÑOS SON LAS MÁS AFECTADAS.
3. 3. ES EL MELANOMA CON MAYOR TIEMPO DE CRECIMIENTO RADIAL.
4. 4. SUELE SER EL DE MEJOR PRONÓSTICO.
Gráfico de respuestas
Comentario

El lentigo maligno aparece casi siempre en personas de más de 60 años, en la cara y sobre una
piel con importante daño actínico. Tiene una fase inicial con crecimiento radial superficial muy
prolongada (hasta más de 10 años). Suele tratarse de una lesión maculosa (por tanto no infiltrada)
que crece superficialmente, de superficie marrón-negra. Aproveche para repasar los 4 tipos
clásicos de melanoma, y recuerde que en el léntigo maligno las metástasis, si ocurren, suelen ser
tardías y limitadas a los ganglios linfáticos cervicales. Que no le engañen con las sinonimias; al
léntigo maligno también se le llama melanosis de Dubreuilh y peca melánica de Hutchinson.(R1)

2. Un niño de 18 meses, quien gozaba de buena salud, fue cargado por los brazos para
subirlo dos escalones; lanzo un grito y después se limito el empleo de su brazo izquierdo,
el diagnostico más probable es:

1. 1. Lesión del plexo braquial.


2. 2. Subluxacion de la cabeza del radio.
3. 3. Luxación del humero.
4. 4. Fractura de la clavícula.
Gráfico de respuestas
Comentario

Se trata de un codo de niñera, por lo que debe recordar la subluxación de la cabeza del radio
Respuesta 2 correcta.(R2)

3. Un niño de 2.2 meses presenta episodio de fiebre, vómitos hipoactividad y mala


succión desde hace 3 días, ¿Cuales son las causas mas probables del presente cuadro?

1. 1. Sepsis.
2. 2. Meningitis.
3. 3. Intoxicación alimentaria.
4. 4. 1 y 2.
Gráfico de respuestas
Comentario

En todo recién nacido o lactante menor con fiebre, vómitos e hipoactividad se debe de descartar un
cuadro infeccioso severo, ya sea una sepsis o meningitis. Hay que tener en cuenta que a esta
edad no podrán dar toda la clínica tan florida de la meningitis, por lo que la respuesta correcta es la
4.(R4)

4. En relación a la epidemiología del cáncer de mama, señalar la FALSA:

!
!
!
!
1. 1. Es 100 veces más frecuente en la mujer que en el hombre.
2. 2. Es más frecuente en mujeres blancas.
3. 3. La menopausia precoz es un factor de riesgo.
4. 4. La lactancia puede tener un efecto protector.
Gráfico de respuestas
Comentario

Pregunta sobre la epidemiología del cáncer de mama nada difícil. Esta pregunta la puede sacar si
recuerda una de las características principales del cáncer de mama como es su
hormonodependencia (recuerde que una de las terapias es el uso de "antiestrógenos"). Así, una
menopausia precoz (opción 3) produciría un déficit estrogénico mucho antes de lo esperado, y
ayudaría a prevenir el cáncer de mama. La lactancia materna y la gestación son efectos
protectores por la diferenciación (opción 4) que suponen sobre el tejido mamario. La distribución
por sexos es de un 99% en mujeres y 1% en el varón, siendo también más frecuente en la raza
blanca.(R3)

5. NO es un objetivo del ultrasonido de tercer trimestre:

1. 1. Identificar la vitalidad y la estática fetal.


2. 2. Diagnostico de anomalías estructurales y marcadores de cromosomopatía.
Diagnosticar anomalías de la localización placentaria y del volumen del líquido
3. 3.
amniótico.
4. 4. En casos indicados, estudios de flujo feto-placentarios.
Gráfico de respuestas
Comentario

Pregunta muy importante y sencilla sobre el papel del ultrasonido. En el primer trimestre nos sirve
para identificar el número de embriones, en el caso de gestación múltiple, diagnostico de
cigosidad, identificación del latido cardiaco embrionario, estimación de la edad de gestación,
detección y medida de la translucencia nucal (marcador de cromosomopatía fetal), observación de
la morfología embrionaria y por último identificar la existencia de patología uterina y de los anejos.
En el segundo trimestre el objetivo es el diagnostico de anomalías estructurales y marcadores de
cromosomopatías y si no se ha realizado la ecografía de nivel básico del primer trimestre, incluye
sus objetivos. En el tercer trimestre los objetivos son identificar la vitalidad y la estática fetal,
estimar el crecimiento fetal, diagnostico de anomalías de la localización placentaria (placenta
previa), diagnosticar anomalías del volumen del liquido amniótico y en casos indicados, estudios de
flujo feto-placentarios.(R2)

6. Lactante de 3 meses de edad que presenta vómitos atónicos en todas las tomas desde
hace un mes. Presenta buen estado general con buena ganancia ponderoestatural
encontrándose en un p 50 de peso sin referir otra sintomatología. ¿Cuál sería la primera
prueba a realizar según su sospecha diagnóstica?

1. 1. pHmetría de 24 horas.
2. 2. Tránsito digestivo superior.
3. 3. No realizar ninguna prueba complementaria.
4. 4. Endoscopia y toma de biopsias.
Gráfico de respuestas
Comentario
Ante un paciente menor de 2 años con vómitos atónicos de contenido alimentario sin presentar
retraso en su crecimiento con buen estado general y sin presentar síntomas atípicos se debe
sospechar un reflujo gastroesofágico fisiológico que no requiere pruebas complementarias. Si este

!
!
!
!
lactante hubiese presentado retraso ponderoestatural o síntomas atípicos como clínica respiratoria,
sangrado digestivo, etc. deberíamos sospechar una enfermedad por reflujo gastroesofágico
(ERGE) en cuyo caso la primera prueba a realizar sería un tránsito digestivo superior aunque la
prueba más sensible y específica para el diagnóstico de ERGE es la pHmetría.(R3)

7. En relación con las enfermedades del ano, indique la afirmación INCORRECTA:

1. 1. Las hemorroides internas suelen ser dolorosas sin necesidad de complicación.


2. 2. La ligadura de hemorroides son útiles en el tratamiento de las hemorroides internas.
3. 3. El dolor con la defecación debe hacer pensar en fisura anal o úlcera.
4. 4. Las hemorroides externas suelen causar dolor muy a menudo.
Gráfico de respuestas
Comentario
Las hemorroides internas están producidas por la dilatación del plexo hemorroidal interno
consitituido por la vena rectal superior y media, que se encuentran en el espacio submucoso
superiores a la válvula de Morgagni. Su manifestación más frecuente es la rectorragia y el dolor,
pero para que este exista debe haber alguna complicación como la trombosis de las venas. La
ligadura es útil en los grados más leves de hemorroides internas. La externas si causan dolor a
menudo porque la trombosis es más frecuente.(R1)

8. ¿Cuál es el tratamiento de elección de un esguince aislado del ligamento colateral


medial de la rodilla?:

1. 1. Reconstrucción mediante injerto tendinoso.


2. 2. Sutura de las fibras completamente interrumpidas.
3. 3. Vendaje compresivo y descarga con movilización precoz.
4. 4. Ejercicios isométricos para potenciar la musculatura cuadricipital.
Gráfico de respuestas
Comentario

El esguince aisaldo del ligamento colateral interno o medial de la rodilla se trata mediante un
vendaje compresivo o rodillera y descarga con movilización precoz. Se permite la carga hacia las
2-3 semanas con ayuda de un bastón. No está indicada la cirugía y tampoco es preciso un
programa de rehabilitación prolongado para la recuperación funcional.(R3)

9. En una paciente gestante de 8 semanas, portadora de distrofia muscular de Duchenne,


¿qué método de diagnóstico prenatal es el indicado?:

1. 1. Determinación de alfafetoproteína sérica materna.


2. 2. Biopsia de corion.
3. 3. Funiculocentesis.
Triple cribado de alfafetoproteína sérica materna, fracción beta de la gonadotropina
4. 4.
coriónica humana y estriol no conjugado plasmático.
Gráfico de respuestas
Comentario

Pregunta de dificultad media sobre los métodos de diagnóstico prenatal. Se nos presenta un caso
clínico de una paciente en el primer trimestre (8 semanas) en la que hay que descartar que el feto
sea portador de distrofia muscular de Duchenne. Se puede sacar esta pregunta por descarte, ya

!
!
!
!
que todas las opciones excepto la 2 (biopsia corial) se realizan en el segundo trimestre. Así, la
opción correcta es la 2, porque además de ser la única que se realiza a las 8 semanas de
gestación, está indicada para el diagnóstico de trastornos bioquímicos como la enfermedad de
Gaucher, Niemann- Pick, Tay-Sachs, distrofia muscular de Duchenne o de Becker,
cromosomopatías, infecciones fetales (TORCH). En resumen, siempre que necesite un diagnóstico
citogenético prenatal antes de las 12 semanas de gestación, recurra a la biopsia corial.

A partir de la semana 12 puede realizar una amniocentésis y a partir de la semana 18 una


funiculocentesis.(R2)

10. Masculino de 32 años es remitido para estudio diagnóstico al presentar en extudios


de laboratorio de rutina, en su empresa, unas pruebas de función hepática alteradas. Está
asintomático y no toma medicamentos. Presenta durante más de seis meses discreta
hipertransaminasemia GOT 75, GPT 89, colesterol 221, el resto fue normal. Una correcta
evaluación del paciente incluiría lo siguiente, EXCEPTO:

1. 1. CPRE.
2. 2. Ecografía abdominal.
3. 3. Serología virus hepatitis.
4. 4. Biopsia hepática.
Gráfico de respuestas
Comentario

En este caso, la prueba diagnóstica que no tiene sentido es la CPRE. Dese cuenta de que los
laboratorios revelan cierto grado de citolisis, pero no de colestasis, por lo que resultaría muy
improbable encontrarnos una alteración morfológica de las vías biliares extrahepáticas.(R1)

11. Todas las afirmaciones siguientes sobre la difteria son ciertas, EXCEPTO:

1. 1. La enfermedad se debe a la invasión de los tejidos por la bacteria.


2. 2. La difteria nasal se observa casi siempre en los lactantes.
3. 3. No debe esperarse la confirmación del laboratorio para iniciar el tratamiento.
Los pacientes que curan deben ser inmunizados con toxoide, dada la falta de formación de
4. 4.
anticuerpos contra la toxina.
Gráfico de respuestas
Comentario

Sabiendo lo fundamental de las patologías podemos llegar a seleccionar la opción correcta sin que
tengamos que saber de memoria los datos que aparecen en las otras opciones.

En esta pregunta lo fundamental es saber que la patogenia de la Difteria se debe a una toxina
producida por la bacteria y no a la invasión de los tejidos por la propia bacteria.(R1)

12. With regard to orbital cellulitis, point out the sign that distinguishes this process from
preseptal cellullitis:

1. 1. Diplopia.
2. 2. Chorea.
3. 3. Ptosis.
4. 4. Eyelid erythema.
Gráfico de respuestas

!
!
!
!
Comentario
Diplopia. Ptosis, eyelid erythema and fever occur both in preseptal and orbital cellulitis. Orbital
cellulitis is a deeper infection, and therefore extraocular muscles are involved causing diplopia.(R1)

13. ¿Cuál de las siguientes características NO aparece en la galactosemia por déficit de


galactoquinasa?:

1. 1. Cataratas.
2. 2. Retraso mental.
3. 3. Galactosemia.
4. 4. Galactosuria.
Gráfico de respuestas
Comentario

Concepto poco importante para fines del ENARM. La galactosemia por déficit de galactoquinasa es
un error congénito del metabolismo de la galactosa, de herencia autosómica recesiva, que afecta al
primer paso de la ruta, la conversión de galactosa en galactosa-1-fosfato. Está causado por
mutaciones en el gen GALK1.

La deficiencia de GALK bloquea la ruta metabólica derivando la galactosa acumulada hacia


compuestos alternativos, el galactitol y el galactonato. El galactitol produce un edema de las fibras
del cristalino, dando lugar a cataratas durante las primeras semanas o meses de la vida. Se ha
descrito algún paciente con cierto déficit intelectual.

El diagnóstico se realiza por sospecha clínica ante un niño con cataratas y sin afectación hepática
ni renal, que excreta elevadas concentraciones de galactosa en orina. Se comprueba demostrando
el acúmulo de galactosa y galactitol en plasma y la ausencia de Gal-1-P en eritrocitos y mediante la
demostración del defecto enzimático de GALK. El estudio mutacional del gen GALK completa el
estudio.

El tratamiento consiste en la eliminación de la leche y sus derivados de la dieta.(R2)

14. Un lactante de 4 meses de vida presenta un ECG con eje del QRS de +150º, ondas R
altas en V1, S profundas en V6 y la onda T positiva en V1. ¿Qué diagnóstico
electrocardiográfico establecería?:

1. 1. Signos de hipertrofia ventricular izquierda.


2. 2. Signos de hipertrofia ventricular derecha.
3. 3. Signos de hipertrofia biventricular.
4. 4. Signos de isquemia.
Gráfico de respuestas
Comentario

No se preocupe si ha fallado esta pregunta porque nunca se han preguntado las características
electrocardiográficas de la infancia y es un tema poco útil. El eje medio de QRS en el RN está a +
125º. El eje medio de QRS de + 90º se alcanza cuando el niño tiene 1 mes de edad y hay un
cambio hacia la izquierda durante la infancia , acercándose a los 3 años al valor medio del adulto
de + 50º. Como sabe, la onda T está generada por la repolarización del miocardio ventricular. La
onda T puede ser (-) en DI y (+) en aVR durante los primeros días de vida. Por encima de 1 mes de
edad siempre es (+) en DI - DII - aVF y (- ) en aVR, pudiendo ser (+) (- ) ó difásica en DIII - aVL.

!
!
!
!
En las precordiales: en V1 las primeras horas de vida puede ser la onda T (+), después de 24
horas a 4 dias será (- ). Por todo lo anterior, se deduce que el lactante d(e 4 meses de este caso
presenta una hipertrofia del VD con eje eléctrico desviado a la derecha.(R2)

15. El diafragma pélvico está formado por los siguientes músculos:

1. 1. El elevador del ano y coccígeo.


2. 2. Transverso superficial del periné y elevador del ano.
3. 3. Elevador del ano y puborectal.
4. 4. Bulbocavernosos y coccígeo.
Gráfico de respuestas
Comentario

Pregunta muy dificil sobre anatomía del suelo pélvico. La respuesta es la 1 elevador del ano y
coccígeo.(R1)

16. La forma más tardía de presentación de la tuberculosis es:

1. 1. Osteomielitis tuberculosa.
2. 2. Tuberculosis genitourinaria.
3. 3. Meningitis tuberculosa.
4. 4. Tuberculosis pulmonar.
Gráfico de respuestas
Comentario

La TB pulmonar puede presentarse de forma precoz - primaria- , o más tardía, pero no destaca por
ser la más tardía. La forma de meningitis es más frecuente en niños pequeños. Por lo que la duda
más lógica esta entre la opción 1 y 2. La TB genitourinaria suele ser una forma de presentación
bastante insidiosa por lo que puede presentarse cuando se ha producido importante daño renal. Es
una forma que se suele manifestar muchos años después de la primoinfección.(R2)

17. A full-term newborn who is still in the delivery room presents with progressive
respiratory distress and cyanosis after ligation of the umbilical cord. Oxymetric results
show marked hypoxemia that does not improve despite 100% oxygen administration. He
also presents with progressive unresponsiveness. Cardiac auscultation shows no
murmurs, but a single and loud S2 with normal S1 are present. What treatment is indicated
in this case?

1. 1. Ibuprofen.
2. 2. Nitric oxide and prostaglandin E.
3. 3. Antibiotics.
4. 4. Saline infusion.
Gráfico de respuestas
Comentario
Nitric oxide and prostaglandin E. In this situation of vital risk to the newborn it is necessary to act
quickly. The first step is the administration of vasodilators such as nitric oxide or prostaglandins to
maintain the ductus open and improve oxygenation. Ibuprofen would produce the opposite
effect.(R2)

!
!
!
!
18. Entre los siguientes cuadros amnésicos, señale el que suele ser permanente
(duración mayor de un mes):

1. 1. Amnesia post-TEC.
2. 2. Amnesia asociada a migraña.
3. 3. Amnesia asociada a ingesta de benzodiacepinas.
4. 4. Amnesia asociada al síndrome de Korsakov.
Gráfico de respuestas
Comentario

Si no es capaz de contestar a la primera, deberá acertarla por descarte, no es complicado. R1: es


cierto que produce amnesia, retrograda, pero suele subsanarse (hasta que las neuronas se
"recolocan", parecido a una crisis epiléptica). R2: efímera, transitoria al igual que una migraña (sino
sospechar EVC); R3: amnesia anterógrada, lo que dura el efecto de la BZD. R4: continúa a la
encefalopatía de Wernicke, amnesia anterógrada, que se produce por una lesión (diencefálica), por
lo que permanente. Si no responde a tiamina, es permanente.(R4)

19. Señale el hallazgo que iría en contra de una situación de insuficiencia renal prerrenal
en un niño con deshidratación severa:

1. 1. Densidad urinaria > 1.020.


2. 2. Excreción fraccionada de sodio < 1%.
3. 3. Hipotensión arterial.
4. 4. Disminución de la creatinina plasmática con la administración de furosemida.
Gráfico de respuestas
Comentario

Es muy importante de cara al examen que sepa diferenciar una IRA prerrenal de una
parenquimatosa. La clínica de deshidratación e hipotensión y la mejora de la diuresis tras la
expansión con volumen sugieren IRA prerrenal. Los datos de laboratorio que van a favor de ella
son: Densidad urinaria > 1.018, osmolaridad urinaria >500, creatinina orina/plasma >40 urea, orina
/plasma>1. El mejor: IFR <1 y el más útil para responder preguntas: FENa < 1%. La administración
de furosemida en la IRA prerenal se realiza cuando, una vez expandido el volumen, el enfermo
continua sin diuresis adecuada para transformar el fallo en poliúrico, de más fácil manejo. En un
caso de deshidratación severa el administrar furosemida supone aumentar la pérdida de líquidos,
lo que descendería aun más la perfusión renal y empeoraría la insuficiencia, aumentando así las
cifras de creatinina. Recuerda que la IR prerrenal es la causa más frecuente de IRA y que, si la
causa no se corrige, puede producir necrosis tubular.(R4)

!
!
!
!

20. A la vista de estos hallazgos, ¿cuál es


la actitud a realizar?:

1. 1. Biopsia con control estereotáxico.


2. 2. Cirugía radical.
3. 3. Tratamiento con antiestrógenos.
4. 4. Control rutinario en el programa de cribado mamográfico.
Gráfico de respuestas
Comentario

Pregunta sobre un tema fundamental: actitud diagnóstico-terapéutica en el cáncer de mama. Con


los resultados de la mastografía de la imagen, necesitamos confirmación histológica, por lo que la
respuesta correcta es la 1. El tratamiento con antiestrógenos seria una opción para el tratamiento
post-quirúrgico, la cirugía radical precisa confirmación anatomopatológica previa y el control
mamográfico no tiene sentido ya.(R1)

!
!
!
!

21. Mujer de 52 años, asintomática, que


acude a un programa de detección precoz de cáncer de mama. Para completar el estudio
se realiza una radiografía ampliada que se muestra en la imagen n° 15. ¿Cuál es el signo
radiográfico que define mejor los hallazgos?:

1. 1. Mastografía normal.
2. 2. Nódulos con microcalcificaciones.
3. 3. Microcalcificaciones segmentarias sospechosas de malignidad.
4. 4. Microcalcificaciones groseras sugestivas de mastopatía fibroquística.
Gráfico de respuestas
Comentario

Pregunta directa en la que hay que descrbir la imagen, pero no por ello difícil. En la parte superior
se observan múltiples imágenes hiperdensas, compatibles con la opción 3 (correcta). Las demás
son incorrectas: No es una mastografía normal, no se aprecian nódulos, ni tampoco microquistes, y
las calcificaciones no son groseras, sino finas.(R3)

22. Indique cuál es la conducta terapéutica establecida para la endocarditis infecciosa


sobre válvula protésica con cultivo positivo para Cándida:

1. 1. Cirugía.
2. 2. Fluconazol.
3. 3. Anfotericina B con cirugía.
4. 4. Anfotericina B.
Gráfico de respuestas
Comentario
La endocarditis por hongos suele precisar siempre cirugía además del antifúngico por su escasa
respuesta. Como antifúngicos se usa anfotericina B o algún imidazol como el fluconazol, ambos i.v.

!
!
!
!
Ambos tratamientos, farmacológico y quirúrgico se utilizan en combinación, por eso la opción válida
es la 3. Aún así, la respuesta es regular, por lo que es una endocarditis de mal pronóstico.(R3)

23. A 33-year-old
woman presents to the emergency department with a 3-day history of dyspnea, cough
and fever. Past medical history is remarkable for recurrent tonsillitis. The patient says
that she started feeling ill and febrile 4 days ago after cleaning the kindergarten where
she works. She visited her primary care physician and was prescribed
amoxicillin/clavulanic acid. 24 hours later she developed an itchy generalized rash. Her
physician then changed the antibiotic treatment for a macrolide agent. However, the
patient comes today to the emergency department because the symptoms have
worsened, with high fever and dyspnea. Physical examination reveals a generalized and
confluent rash that affects palms and soles accompanied by intense pruritus. Oral
mucosa examination reveals no signs of tonsillitis. However, the resident physician notes
the presence of a few clustered erythematous lesions with a porcelain-white center in the
buccal mucosa in front of the superior molar teeth (Shown in the image). Which of the
following best describes these lesions?

1. 1. Koplik's spots.
2. 2. Oral aphthae.
3. 3. Oral lichen planus.
4. 4. Leukoplakia.
Gráfico de respuestas
Comentario

Aunque se trate de una mujer de 33 años, a la hora de responder esta pregunta no se puede
obviar que trabaja en una escuela infantil (en el ENARM todos los datos suelen tener su
importancia). Ante una enfermedad exantemática el diagnóstico diferencial es amplio, pero en este
caso nos dan un dato patognomónico como es la presencia de lesiones eritematosas con centro
blanquecino frente a arcadas dentarias superiores vinculadas a una imagen característica que se
corresponde con Manchas de Koplik. El resto de datos (fiebre, afectación del estado general,
cuadro respiratorio, exantema que afecta a palmas y plantas) no hacen más que corroborar el
diagnóstico de sarampión, enfermedad de aparición típicamente infantil y que ha aumentado de

!
!
!
!
forma importante su incidencia en los últimos años gracias al auge del “movimiento antivacunas”.
El contexto clínico descarta las tres opciones restantes.

24. The previously described patient was admitted in the hospital for further treatment.
Auscultation revealed crackles in both lungs, especially in the lower lobes. The patient
had an oxygen saturation of 90%. Chest X-ray showed interstitial lung pattern. Serology
for HIV was negative. Levofloxacin therapy was administered. Five days after admission,
the patient's condition starts to improve with decreasing dyspnea and progressive fading
of the cutaneous rash. Which of the following tests would confirm the diagnosis?

1. 1. Serology for influenza virus H1N1.


2. 2. Measles serologic testing (IgM).
3. 3. Parvovirus B-19 serologic testing (IgM)
4. 4. Treponemic tests and reaginic luetic serology.
Gráfico de respuestas
Comentario

Nos preguntan qué prueba nos permitirá llegar al diagnóstico definitivo. De entre todas las
opciones solo la respuesta 2 nos permitiría aclarar la etiología del cuadro de la paciente a través de
pruebas serológicas que determinen anticuerpos IgM frente a sarampión. Hay que recordar que la
neumonía es una de las complicaciones más frecuentes del sarampión, resultando más habitual la
sobreinfección bacteriana secundaria que la neumonía por el propio virus del sarampión (neumonía
de células gigantes de Hecht) de curso grave y más típica de inmunodeprimidos.(R2)

25. Paciente de 75 años de edad que acude por hemorragia digestiva baja, detectándose
en la colonoscopia múltiples lesiones similares a arañas vasculares en ciego y colon
ascendente de menos de 5 mm de tamaño. Respecto a la enfermedad que padece este
paciente señale la FALSA:

1. 1. Puede asociarse a insuficiencia aórtica.


2. 2. Son lesiones degenerativas con dilatación y distorsión de vénulas y capilares.
3. 3. Generalmente el sangrado suele ceder espontáneamente.
4. 4. El método diagnóstico de elección es la colonoscopia.
Gráfico de respuestas
Comentario

La angiodisplasia de colon es una lesión degenerativa que consiste en dilatación y distorsión de


vasos de pared fina en colon derecho, contituyendo la causa más frecuente de este tipo de
hemorragia a partir de los 60 años. Debe sospecharse con antecedentes de estenosis aórtica y no
de insuficiencia como se indica en la primera opción. El diagnóstico se realiza mediante una
colonoscopia completa, pudiendo realizarse tratamiento con laser- rgón para "quemar" las lesiones
y evitar los episodios de sangrado. Otra opción de tratamiento es la administración de estrógenos-
progestágenos que interviene en la reparación vascular. En caso de mala respuesta a tratamiento,
se indica hemicolectomía derecha.(R1)

26. Señale la opción que considere INCORRECTA sobre los tumores del estroma
gastrointestinal:

1. 1. Es un tumor desarrollado a partir de las células intersticiales de Cajal.


2. 2. Un índice mitótico alto aumenta la probabilidad de recurrencia y metástasis.

!
!
!
!
Su forma más frecuente de aparición es una masa dependiente de mucosa con crecimiento
3. 3.
endoluminal.
4. 4. El GIST irresecable puede controlarse con imatinib mesilato.
Gráfico de respuestas
Comentario

Los GIST son tumores raros de origen mesenquimal cuya forma más frecuente de aparición es en
forma de una lesión submucosa que puede ocasionalmente ulcerarse. El potencial maligno de este
tumor viene fundamentalmente determinado por el índice mitogénico (> 25 mitosis por campo de
gran aumento) y un tamaño mayor de 3 cm. En ellos el tratamiento con imatinib tiene cierta utilidad,
debido a la presencia de la mutación de c-kit.(R3)

27. La realización de una colecistectomía de forma programada sería, desde el punto de


vista del grado de contaminación, una cirugía:

1. 1. Limpia.
2. 2. Sucia.
3. 3. Limpia-contaminada.
4. 4. Contaminada-sucia.
Gráfico de respuestas
Comentario

Una cirugía limpia-contaminada es aquella en la que se abre el tubo digestivo, respiratorio o


genitourinario de forma controlada, sin salida de material. Una colecistectomía programada, por
ejemplo. Si se tratase de una colecistectomía en el contexto de una colecistitis aguda, estaríamos
ante una cirugía contaminada.(R3)

28. La estenosis mitral se considera crítica o grave cuando el área valvular es:

1. 1. Inferior a 2 cm2.
2. 2. Inferior a 1.5 cm2.
3. 3. Inferior a 1 cm2.
4. 4. Inferior a 0.4 cm2.
Gráfico de respuestas
Comentario

Se considera que, en función del área la estenosis mitral es ligera si tiene un área entre 1.5 y 2
cm2, estenosis mitral moderada si tiene un área entre 1 y 1.5 cm2 y estenosis mitral severa si el
área es menor de 1 cm2. La cuantificación del area de apertura mitral puede hacerse por
ecocardiograma, que además puede medir el gradiente diastólico medio. Y evaluar la morfología
para ver el grado de calcificación y en definitiva decidir si es una válvula conservable o no. Además
del ecocardiograma también se puede evaluar el área por cataterismo.(R3)

29. Todas las siguientes son características del síndrome de West, EXCEPTO:

1. 1. Patrón de hipsarritmia en el EEG.


2. 2. En el 90% de los casos son criptogenéticos o idiopáticos.
3. 3. Los espasmos pueden ser flexores, extensores o mixtos.
4. 4. En los casos sintomáticos el riesgo de retraso mental es del 80-90%.
Gráfico de respuestas
Comentario

!
!
!
!
El síndrome de West aparece en el primer año de edad y en la mayoría de los casos se da en
niños con patología cerebral subyacente (respuesta 2 FALSA). La triada típica del síndrome de
West es: 1. Retraso en el desarrollo psicomotor del niño (importante riesgo de retraso mental). 2.
Espasmos flexores, extensores o mixtos. 3. Hipsarritmia: patrón en el EEG entre las crisis que
consiste en ondas lentas de alto voltaje entre las que se intercalas salvas de puntas.(R2)

30. ¿Cuál de las siguientes alteraciones no forman parte de la clínica habitual del
síndrome nefrótico?

1. 1. Edemas.
2. 2. Hipoproteinemia.
3. 3. Aumento del riesgo de hemorragia.
4. 4. Mayor riesgo de infecciones.
Gráfico de respuestas
Comentario

La pérdida de factores de coagulación, especialmente la antitrombina III, conducen a un mayor


riesgo de fenómenos trombóticos, cosa que no sucede en el síndrome nefrótico recuerde que lo
más característico es la proteinuria +++, que genera edema por pérdida de la presión oncótica.(R3)

31. De entre las distintas funciones de la placenta, destaca la de la trasferencia de


sustancias. Respecto a esta función, señale lo FALSO:

En la difusión simple, la velocidad de transferencia depende del tamaño molecular y de la


1. 1.
hidrosolubilidad de la sustancia.
En el transporte activo hay gasto energético, aunque no se pueda realizar a contracorriente
2. 2.
de las concentraciones relativas.
3. 3. Las lipoproteínas y fosfolípidos atraviesan por pinocitosis.
4. 4. El paso directo por solución de continuidad es usado por los eritrocitos.
Gráfico de respuestas
Comentario
El mecanismo de transporte de la placenta se puede dividir en 5 tipos. El primero sería la difusión
simple, en la cual la velocidad de transferencia depende del tamaño molecular y de la
hidrosolubilidad de la sustancia, este mecanismo lo utilizan los gases. La difusión facilitada utiliza
una tasa de trasferencia mayor que en la difusión simple, este mecanismo es usado por la glucosa
y el lactato. En el transporte activo hay gasto energético, pudiéndose realizar a contracorriente de
las concentraciones relativas (cationes, vitaminas, aminoácidos). Las lipoproteínas y fosfolípidos
atraviesan por pinocitosis, que es la absorción de gotitas microscópicas del plasma materno del
espacio intervelloso por las células de la membrana placentaria. El paso directo por solución de
continuidad es usado por elementos corpusculares como los hematíes.(R2)

32. Un paciente de 46 años de edad acude a Urgencias por vómitos en posos de café y
melena. El hematocrito de entrada es del 33%, su presión arterial es de 110/70 mmHg y
su frecuencia cardíaca de 87 latidos por minuto. Un estudio endoscópico practicado 5
horas después de su ingreso revela alguna erosión superficial limpia en el tercio distal
del esófago y varias erosiones superficiales en la zona antral prepilórica, una de ellas con
un punto de hematina en su base. No quedan restos hemáticos en la cavidad gástrica. El
paciente se había medicado con diclofenaco 150 mg/día los últimos 8 días por una
ciatalgia. ¿Cuál sería la actitud más recomendable?

!
!
!
!
1. 1. Tratamiento con inhibidores de la bomba de protones por vía oral y alta hospitalaria.
Tratamiento hemostático con sonda de calor de la erosión con hematina en su base y
2. 2.
perfusión endovenosa de inhibidores de la bomba de protones durante tres días.
Ayuno total y tratamiento con perfusión endovenosa de inhibidores de la bomba de
3. 3.
protones durante 48 horas.
Colocación endoscópica de un hemoclip en la lesión erosiva con signos de hemostasia
4. 4.
reciente.
Gráfico de respuestas
Comentario

El caso que nos describen corresponde a una hemorragia digestiva alta, en el contexto de un
paciente tratado con diclofenaco. Dado este antecedente, habría que asumir que el AINE es la
causa de la hemorragia. Según los datos que nos ofrecen, no existen signos de sangrado activo en
el momento de la exploración endoscópica y el paciente no está hemodinámicamente inestable.
Teniendo esto en cuenta, y que conocemos la causa del problema, bastaría recomendar el uso de
omeprazol u otro inhibidor de la bomba de protones, que incluso podría haber sido útil
profilácticamente, si se hubiera administrado junto con el diclofenaco desde un principio. Por otra
parte, dado que el paciente no reúne criterios de ingreso, la actitud más correcta sería la respuesta
1.(R1)

33. ¿Cuál de las siguientes situaciones se asocia con mayor frecuencia a bacteremia por
Yersinia enterocolítica?:

1. 1. Tratamiento con corticoides.


2. 2. Hemocromatosis.
3. 3. Intoxicación por plomo.
4. 4. Síndrome de DiGeorge.
Gráfico de respuestas
Comentario

Pregunta díficil sobre Yersinia enterocolítica, agente productor de diarrea inflamatoria, que es
aquella que cursa con sangre, pus y moco en heces y con visualización de leucocitos al
microscopio. Los enfermos con diabetes, anemia grave, hemocromatosis, cirrosis hepática o
neoplasia, o bien los de edad avanzada, pueden padecer una sepsis por Y. enterocolítica. La
sepsis se ha relacionado también con estados de sobrecarga férrica, como la talasemia y el
tratamiento con desferroxiamina. Los pacientes con sepsis presentan abscesos viscerales múltiples
o meningitis y endocarditis asociadas. Opción 2 correcta.(R2)

34. Which of the following statements is false in relation to neonatal clavicular fracture?

1. 1. It is more frequent in sons of mothers with type 2 diabetes.


2. 2. It is more frequent in newborns with a history of prolonged labor.
3. 3. It is a typical condition of newborns affected by Down's syndrome.
4. 4. No treatment is required in the majority of cases.
Gráfico de respuestas
Comentario
It is a typical condition of newborns affected by Down's syndrome. Clavicle fracture is a common
complication of childbirth and resolves without sequelae in the vast majority of newborns without
requiring specific treatment. Children with Down's syndrome have no increased risk of this
complication. In fact, these infants have usually low weight at birth and would have a lower risk of
prolonged labor and dystocia, so this complication would be expected to be less frequent in
theory.(R3)

!
!
!
!

35. Durante el primer trimestre de la gestación pueden aparecer los siguientes síntomas,
EXCEPTO uno. Señálelo:

1. 1. Reflujo gastroesofágico.
2. 2. Estreñimiento.
3. 3. Disuria.
4. 4. Tensión mamaria.
Gráfico de respuestas
Comentario

La disuria no es un síntoma fisiológico del embarazo. Su presencia debe hacernos descartar una
infección urinaria. Recordemos que las infecciones de vías urinarias en las mujeres embarazadas
se consideran complicadas.(R3)

36. Con respecto al síndrome de abstinencia del RN secundario a la adicción materna a


drogas es cierto todo, EXCEPTO:

La incidencia de enfermedad de membrana hialina y de hiperbilirrubinemia es mayor en


1. 1.
hijos de madres adictas a heroína.
2. 2. Los síntomas más llamativos son los temblores y la hiperexcitabilidad.
Los hijos de madres adictas a metadona tienen más incidencia de convulsiones y un inicio
3. 3.
de la sintomatología más tardío que los hijos de madres adictas a heroína.
4. 4. En la adicción a cocaína, no es frecuente el síndrome de abstinencia.
Gráfico de respuestas
Comentario

El estrés sufrido de forma continuada por los hijos de madre consumidora habitual de drogas sirve
de estímulo para que sinteticen precozmente surfactante: de ahí que estos niños padezcan menos
enfermedad de membrana hialina. El resto de las opciones es correcta: recuerde que el síndrome
de abstinencia neonatal cursa como un síndrome de hiperexcitabilidad.(R1)

37. Señale cuál de las siguientes anemias NO suele ser microcítica:

1. 1. Mielodisplasia.
2. 2. Talasemias.
3. 3. Sideroblástica hereditaria.
4. 4. Saturnismo.
Gráfico de respuestas
Comentario

Pregunta de baja dificultad. Las anemia se clasifican de acuerdo a su VCM en microciticas (VCM
<80 fl), normaciticas (VCM 80-100 fl) y macrociticas (VCM>100). Es importante conocer ejemplos
de cada una de estas anemias, para poder responder preguntas como esta. Les recuerdo que las
anemia sideroblasticas hereditarias son anemia microciticas (opción 3 correcta), mientras que las
anemia sideblasticas adquiridas (como los síndromes mielodisplásicos) son anemia normo o
macrociticas (opción 1 falsa).(R1)

38. El ácido hialurónico, que se encuentra en alta proporción en el líquido sinovial,


procede de:

!
!
!
!
1. 1. Sinoviocitos A.
2. 2. Sinoviocitos B.
3. 3. Ultrafiltrado del plasma.
4. 4. Hueso subcondral.
Gráfico de respuestas
Comentario

La membrana sinovial se compone de una capa de tejido conectivo compuesta por colágeno tipo I
y III, reticulina y fibronectina...etc y de una capa de células de revestimiento o sinoviocitos. Los
sinoviocitos ultraestructuralmente pueden ser tipo A, que son células con un aparato de Golgi muy
desarrollado, con escaso retículo endoplásmico y que proceden de la médula ósea, de la línea
monocito-macrófago, con función fagocítica, y los sinoviocitos B, que poseen un abundante retículo
endoplásmico, proceden de células del mesénquima local, producen ácido hialurónico, colagenaza,
fibronectina, lubricina...(R2)

39. Paciente primigesta de 31 semanas con un embarazo gemelar monocorial biamniótico


con el primer gemelo en cefálica y un peso fetal estimado de 1300 g y el segundo gemelo
en cefálica con un peso fetal estimado de 1150 g, que acude a Urgencias con dinámica
uterina regular y una dilatación cervical de 3 cm. La vía del parto más adecuada será:

1. 1. Cesárea por edad gestacional inferior a 32 semanas.


2. 2. Cesárea por gestación gemelar monocorial.
3. 3. Parto vaginal de ambos gemelos.
4. 4. Parto vaginal con versión interna y gran extracción del segundo gemelo.
Gráfico de respuestas
Comentario
Aunque la edad gestacional sea inferior a 32 semanas o el peso estimado fetal sea <1500 g,
siempre que ambos gemelos estén en cefálica se intentará la vía vaginal. La versión interna y gran
extracción del segundo feto se realiza si éste está en situación transversa.(R3)

40. ¿Cuál de los siguientes es el factor etiológico más constante en el cáncer de


páncreas?:

1. 1. Alcohol.
2. 2. Dieta rica en grasas.
3. 3. Dieta rica en proteínas.
4. 4. Tabaco.
Gráfico de respuestas
Comentario
El factor de riesgo más consistentemente observado para el desarrollo de un cáncer de páncreas
es el consumo de tabaco. La mayoría de los estudios demuestran que el hábito de fumar dobla o
triplica el riesgo de una persona para padecer cáncer de páncreas. El mecanismo es desconocido
pero se ha demostrado que los carcinógenos que contiene el humo de los cigarrillos provoca
cáncer de páncreas en los animales de laboratorio. El alcohol ha sido implicado en algunos
estudios de casos control de cáncer de páncreas pero las pruebas globales son inconsistentes y el
alcohol no es probable que sea el principal factor de riesgo para el desarrollo de esta
enfermedad.(R4)

41. Femenino de 31 años, primigesta de 38 semanas de gestación, que acude a consulta


por presentar desde
!
!
!
!
el día anterior cefalea frontal que no cede con paracetamol. En la exploración destaca la
existencia de dolor epigástrico y TA de 150/100 mmHg. Laboratorio: Hb 11.8 g/dl,
leucocitos: 12,500/mm3, plaquetas: 100,000/mm3, ácido úrico: 8.2 mg/dl y creatinina: 1.4
mg/dl. Mientras está recogiendo en la historia todos estos datos, la paciente presenta una
convulsión generalizada. ¿Cuál de los pasos que se refieren a continuación NO está
indicado ante esta situación?:

1. 1. Administrar hidralacina intravenosa si la PA diastólica supera 110 mmHg.


2. 2. Inyectar inmediatamente diacepam intravenoso a ritmo lento para yugular la convulsión.
Administración de furosemida de forma sistemática para evitar posibles complicaciones
3. 3.
como el edema agudo de pulmón.
4. 4. Terminar la gestación por la vía menos perjudicial en el plazo más corto posible.
Gráfico de respuestas
Comentario

La paciente de esta pregunta padece un cuadro de eclampsia. Para el control tensional, es


adecuada la hidralacina, un vasodilatador arterial directo, pero no la furosemida . Recuerde que los
diuréticos, al disminuir la volemia, reducirían el flujo placentario. En un caso puntual, con un grave
edema pulmonar ya establecido, podría plantearse su uso si fracasan otras medidas… Pero no
pueden emplearse “de forma sistemática”, como dice la respuesta 3

Una observación sobre la respuesta 2. Aunque el diacepam no es una mala opción para el
tratamiento de las convulsiones de la preeclampsia, es preferible el sulfato de magnesio. En caso
de intoxicación por este fármaco, el antídoto sería el gluconato cálcico.(R3)

42. RNPT 24 SDG de 26 horas de vida, respirando espontáneamente, sin O2, nacido por
cesárea y con Apgar 8/9. ¿Qué método utilizaría para valorar la edad gestacional?

1. 1. Método de Capurro y Usher.


2. 2. Método de Ballard y Capurro.
3. 3. Ballard Modificado.
4. 4. Método de Usher solamente.
Gráfico de respuestas
Comentario

Tema importante en el bloque de neoatología. La respuesta correcta es la 3, ya que el Ballard


modificado sirve para RNPT < 29 SDG, mientras que Capurro para > 29 SDG. Respuesta 3
correcta.(R3)

43. En una mujer gestante de 40 semanas, primigesta, con diabetes gestacional con
aceptable control metabólico hasta el momento actual, que ha precisado tratamiento con
insulina en el último periodo de gestación y que presenta un peso estimado fetal de 4120
g, en la cual se decide finalizar la gestación, dado que no se ha puesto de parto de manera
espontánea hasta la fecha, señale cuál de las siguientes afirmaciones es INCORRECTA:

Suele ser necesario un control intraparto de la glucemia, con administración de


1. 1.
sueroterapia y valorar la necesidad de insulina en función de las glucemias horarias.
En mujeres primíparas, con diabetes gestacional y fetos macrosómicos, pese a un
2. 2. adecuado control metabólico, es de elección la cesárea como vía de parto, para evitar las
posibles complicaciones asociadas.

!
!
!
!
En pacientes con retinopatía proliferativa severa está indicado abreviar el período
3. 3. expulsivo, mediante cesárea o mediante parto instrumental, por riesgo de desprendimiento
de retina.
Se recomienda realizar una sobrecarga oral con 75 g de glucosa tras finalizar el parto y la
4. 4.
lactancia.
Gráfico de respuestas
Comentario
En una mujer diabética con buen control y sin otros factores asociados, se intenta que el parto sea
vía vaginal y la indicación de cesárea es la misma que en mujeres no diabéticas, aunque es
conocido el mayor riesgo de complicaciones relacionadas con la macrosomía fetal como la distocia
de hombros, la tasa de cesáreas por desproporción pelvicocefálica y la tasa de partos
instrumentales.(R2)

44. Niña de 5 años diagnosticada de


asma y síndrome de lóbulo medio. Presenta como antecedentes dificultad respiratoria
neonatal y rinorrea. Tos crónica húmeda, infiltrados radiológicos cambiantes, pobre
respuesta al tratamiento antiasmático. Rinitis crónica. Otitis de repetición con otorrea
persistente a pesar de los drenajes. Se le realiza la siguiente radiografía ¿qué le sugiere?

1. 1. Neumonía en lóbulo superior izquierdo.


2. 2. Discinesia ciliar primaria.
3. 3. Neumonía intersticial.
4. 4. Cardiomegalia sugestivo de cardiopatía congénita.
Gráfico de respuestas
Comentario

La historia de rinitis, otitis y neumonías podría ser sugestiva de una inmunodeficiencia humoral,
pero esta posibilidad ni siquiera se observa entre las opciones. Sin embargo, nos ofrecen una
respuesta que podría justificar todas estas infecciones respiratorias persistentes: la discinesia ciliar
primaria, que alteraría la capacidad del epitelio respiratorio para evacuar mecánicamente las
secreciones mucosas impregnadas en ciertos microorganismos. De ahí que la respuesta correcta
sea la 2.

!
!
!
!
Ojo con la radiografía, porque no está al revés... Se trata de un caso de situs inversus (observemos
la dextrocardia y la burbuja gástrica del lado contrario). ¿Cuál es el diagnóstico exacto del
paciente? Se trata de un síndrome de Kartagener.(R2)

45. ¿Cuál es el pronóstico de la enfermedad que padece?

1. 1. Tras tratamiento antibiótico, rápida resolución, sin secuelas.


2. 2. Infecciones respiratorias recurrentes y probable evolución a bronquiectasias.
3. 3. Con tratamiento escalonado de asma crónico evolución favorable.
4. 4. Según el número de otitis definirá el pronóstico.
Gráfico de respuestas
Comentario

Como se puede imaginar, si no funcionan los cilios del epitelio respiratorio, la niña continuará con
infecciones de repetición. Por otra parte, la retención mucosa justificaría, a largo plazo, la aparición
de bronquiectasias, complicación frecuente en la enfermedad que padece.(R2)

46. Ante una tumoración redondeada situada en línea media, unos pocos centímetros por
debajo del ombligo, a través de la cual emerge un líquido ácido, de color amarillo-
transparente, ¿en qué patología pensaría?:

1. 1. Megavejiga idiopática.
2. 2. Agenesia de uretra.
3. 3. Extrofia vesical.
4. 4. Quiste del uraco.
Gráfico de respuestas
Comentario

La persistencia anómala del conducto alantoideo o uraco (recuerden que este conducto conecta el
sistema urinario primitivo con el saco vitelino) da lugar a un tracto fistuloso, que puede aparecer
aislado, o en asociación con un pólipo o un quiste. La clínica de estos procesos es común: la
madre comenta que el ombligo de su hijo está perennemente mojado por un líquido con
características de orina: ácido, amarillo-transparente. La solución a estos trastornos es
quirúrgica.(R4)

47. En mujeres jóvenes con cuadro clínico de dolor abdominal en fosa ilíaca derecha y
fiebre, ¿cuál debe ser la técnica de imagen inicial?

1. 1. La tomografía computarizada con contraste i.v.


2. 2. Ultrasonido abdominal.
3. 3. Ultrasonido transvaginal.
4. 4. La tomografía computarizada sin contraste i.v.
Gráfico de respuestas
Comentario

Una mujer joven con dolor abdominal en fosa ilíaca derecha y fiebre podría padecer una apendicitis
aguda, pero habría muchas más posibilidades en el diagnóstico diferencial. Podría tratarse de una
enfermedad inflamatoria pélvica, la rotura de un folículo de De Graaf en la mitad del ciclo
menstrual, u otros procesos ginecológicos.

!
!
!
!
Aunque el diagnóstico de apendicitis aguda es clínico en el 80% de los casos, en los casos en los
que existe duda, puede recurrirse a pruebas de imagen. Aunque la TC es más sensible, en niños y
mujeres jóvenes se prefiere comenzar por el ultrasonido, ya que es una técnica que no emplea
radiaciones ionizantes. Recuerde, por otra parte, que la radiografía de abdomen más frecuente en
la apendicitis aguda es una Rx normal.(R2)

48. A los 5 minutos de vida un neonato presenta estos signos: 130 pulsaciones por
minuto, manos y pies cianóticos, buen tono muscular, llanto poderoso, reactividad
normal, frecuencia respiratoria 60 por minuto. El Índice de apgar en este niño es :

1. 1. 6.
2. 2. 7.
3. 3. 8.
4. 4. 9.
Gráfico de respuestas
Comentario

El índice Apgar valora 5 puntos, que siguen la regla mnemotécnica "apgar":

- Actividad (tono muscular): ausente (0 puntos), flexión de extremidades (1 punto) y movimiento


activo (2 puntos).

- Pulso (frecuencia cardíaca): ausente (0 puntos), <100 lpm (1 punto), >100 lpm (2 puntos).

- Gesto (respuessta a estímulos con sonda nasogástrica y estímulo tactil): para cada uno, si no hay
respuesta (0 puntos), leve mueca o leve flexión respectivamente (1 punto), llanto y estornudo o
retirada activa respectivamente (2 puntos).

- Apariencia (color): palidez (0 puntos), acrocianosis (1 punto), sonrosado (2 puntos).

- respiracion: ausente (0 puntos), irregular (1 punto), llanto (2 puntos).(R4)

49. Niño de 8 meses con historia de 3 días de evacuaciones liquidas, sin moco, sin sangre,
en número de 6 a 7 al día. La madre ha repuesto con líquidos caseros y lo ha alimentado
con leche materna y arroz, leguminosas, vegetales, y algo de pollo o pescado, añadiendo
una cucharadita de aceite a las comidas. Señale lo CORRECTO:

No es recomendable añadir aceite a los alimentos durante la diarrea aguda porque este no
1. 1.
es bien absorbido.
Debe continuarse con alimentación durante la diarrea. A mayor ingesta se aprecia mayor
2. 2.
beneficio nutricional durante el episodio agudo.
Debe recomendarse una formula a base de soya en lugar de la leche que esta recibiendo el
3. 3.
niño.
Debe recomendarse reducir la cantidad de ingestia de alimentos por uno o dos días ya que
4. 4.
ayuda a reducir el flujo de la diarrea.
Gráfico de respuestas
Comentario

Nos presentan un cuadro de diarrea crónica inespecifíca. Es una diarrea motora debida a un
peristaltismo intestinal aumentado de etiologia desconocida. Es la causa más frecuente de diarrea
crónica en la infancia, de presentación entre los seis meses y los tres años. El paciente presenta

!
!
!
!
entre tres y diez evacuaciones al día, líquidas, no malolientes, con moco, pero sin sangre,
leucocitos ni eosinófilos.

A pesar de la diarrea, no se produce decaimiento, ni deshidratación y el paciente sigue ganando


peso. No hay diarrea nocturna.

Hay que explicar a los padres la benignidad y el caracter autolimitado del proceso. Se aconseja
dieta normal, con restricción del consumo de jugos de frutas industriales y aumento de la ingesta
de grasas.(R2)

50. La marcada calcificación y osificación de los ligamentos paraespinales anteriores


dando la imagen característica en "cera derretida" es típica de:

1. 1. Espondilitis anquilosante.
2. 2. Hiperostosis vertebral idiopática difusa.
3. 3. Artrosis.
4. 4. Osteomielitis vertebral.
Gráfico de respuestas
Comentario

La imagen característica que nos describen es propia de la hiperostosis anquilosante vertebral


difusa (enfermedad de Forestier), un proceso frecuentemente asintomático propio de varones de
edad avanzada. La lesión radiológica típica de la espondilitis anquilosante son los sindesmofitos,
formaciones más “finas”, uniformes y simétricas. En la espondiloartrosis la lesión radiológica son
los osteofitos. Y en la osteomielitis vertebral, lo típico es la destrucción del disco intervertebral. En
cuanto a la artritis reumatoide, no es propia de ella la afectación del esqueleto axial, excepto a nivel
de la columna cervical.(R2)

51. Mujer de 55 años, hipertensa y diabética con IMC 30 kg/m2. Acude a consulta de
ginecología por sangrado vaginal de varios días de evolución tras 5 años de amenorrea.
La línea endometrial medida por USG es de 10 mm. La biopsia tomada por histeroscopia
informa de hiperplasia de endometrio compleja atípica. ¿Cuál es la mejor opción
terapéutica?

1. 1. Histerectomía total con salpingooforectomía bilateral.


2. 2. Análogos GnRH.
3. 3. Dispositivo liberador de levonorgestrel.
4. 4. Progestágenos.
Gráfico de respuestas
Comentario

Nos presentan una hiperplasia endometrial que, según el estudio histológico, merece ser
clasificada como atípica. Si no existieran atipias, la probabilidad de desarrollar un cáncer de
endometrio sería únicamente de un 2%. Sin embargo, cuando éstas aparecen (sean simples o
complejas), el porcentaje de malignización alcanza casi un 25%. Por tanto, ante una hiperplasia
atípica, la indicación de histerectomía es indiscutible.

Por otra parte, nos hablan de una paciente de 55 años postmenopáusica. En consecuencia, la
histerectomía debe asociar anexectomía bilateral (esto también le evitará el riesgo de padecer más
tarde un cáncer de ovario).(R1)

!
!
!
!
52. La acción fundamental del vacuoextractor es:

1. 1. Extractora.
2. 2. Rotación de la cabeza.
3. 3. Deflexión de la cabeza.
4. 4. Dilatación del cuello uterino.
Gráfico de respuestas
Comentario

Preguntas de este tipo en el examen que por más que estudie lo más probable que no la sepa pero
ni usted ni nadie, asi que no se estrese.

El vacuoextractor o ventosa es uno de los instrumentos que podemos utilizar para abreviar el
expulsivo. El mecanismo de acción consiste en aplicar una campana que suele ser de silicona
sobre la cabeza fetal y hacer un efecto de vacío. De esta forma, se podrá ejercer tracción
siguiendo el eje del canal del parto.

Los requisitos para poder aplicar la ventosa son:

- Dilatación completa.

- Bolsa rota.

- Gestación de >37 semanas.

- Presentación fetal rotada o casi rotada a partir del II plano de Hodge.

La diferencia fundamental de la ventosa con respecto al fórceps es que no es un instrumento


rotador (aparte de que el fórceps no se emplearía en un II plano de Hodge).(R1)

53. Un paciente de 71 años acude a su consulta porque “nunca se ha revisado la próstata”


y desea saber si debe realizarse algún estudio al respecto. Respecto al cribado
poblacional del cáncer de próstata, señale la FALSA:

En la actualidad no existen datos concluyentes para aconsejar o desaconsejar realizar


1. 1.
cribado sistemático en población sana a partir de una cierta edad.
De forma individualizada se debe informar al paciente de las ventajas e inconvenientes de
2. 2.
realizar cribado mediante el tacto rectal y la determinación del APE.
3. 3. La OMS lo recomienda en todos los pacientes menores de 75 años.
Se recomienda en pacientes de raza negra, con antecedentes familiares o en aquellos bien
4. 4.
informados que deseen realizarlo.
Gráfico de respuestas
Comentario
Según la OMS en la actualidad no hay evidencia suficiente ni a favor ni en contra del cribado en
hombres menores de 75 años de edad y no recomienda que se realice en los mayores de 75 años.
El resto de las opciones son ciertas y otorgan datos fundamentales para el estudio.(R3)

54. De entre los siguientes tipos de cáncer de tiroides, uno de ellos se relaciona con la
tiroiditis de Hashimoto o bocio linfoide. Señálelo:

!
!
!
!
1. 1. Carcinoma papilar.
2. 2. Carcinoma folicular.
3. 3. Carcinoma de células de Hurthle.
4. 4. Linfoma tiroideo.
Gráfico de respuestas
Comentario

Detalle importante sobre la tiroiditis de Hashimoto. El linfoma tiroideo es muy poco frecuente (1%
de todos los tumores malignos de tiroides) y asienta en más del 80% de los casos sobre tiroiditis
autoinmune. La supervivencia a los 5 años ronda el 70%. Sin embargo, es el tumor que
típicamente se asocia a esta forma de tiroiditis.(R4)

55. Sobre la profilaxis de contacto de la meningitis por Haemophilus influenzae tipo B,


señale la opción CORRECTA:

1. 1. Deben recibir profilaxis todos los adultos que hayan tenido contacto.
2. 2. Deben recibir profilaxis todos los niños que hayan tenido un contacto.
3. 3. El fármaco de elección es la rifampicina.
4. 4. No existe profilaxis postcontacto eficaz.
Gráfico de respuestas
Comentario

Pregunta importante para fines del ENARM.

Se aconseja profilaxis de la meningitis por Hib en contactos íntimos (familia, guardería), menores
de seis años y que no estén vacunados; si el contacto fuese mayor de seis años, pero convive con
menores de esa edad, también debería recibir profilaxis.

La quimioprofilaxis se realiza con rifampicina oral, en dosis única diaria y durante cuatro días.(R3)

56. Which of the following is the best method to diagnose factor XIII deficiency?

1. 1. The 5 M Urea clot solubility assay.


2. 2. Measurement of serum levels of fibrinogen.
3. 3. Prothrombin time.
4. 4. Measurement of the fibrinogen/fibrin degradation products.
Gráfico de respuestas
Comentario

El factor XIII es el encargado de la estabilización del coágulo. Por tanto, la prueba más indicada
para detectar su déficit es la solubilidad del coágulo en urea ó en ácido monoacético. No obstante,
no debe preocuparse si falló esta pregunta. Lo importante es que sepa para qué sirve el tiempo de
cefalina (heparina, vía intrínseca) y el de protrombina (anticoagulantes orales, vía extrínseca).(R1)

57. ¿De qué patología renal estaríamos hablando, si se indica que es la causa más
frecuente de síndrome nefrótico en afroestadounidenses, que la lesión de daño
histológico es parcheado, que la proteinuria marca su pronóstico, que recidiva en el
transplante renal en un 20-40% de los casos y qué suele tener resistencia al tratamiento
esteroideo?:

!
!
!
!
1. 1. GN mesangial Ig A.
2. 2. Glomeruloesclerosis segmentaria y focal.
3. 3. GN membranosa.
4. 4. GN membranoproliferativa.
Gráfico de respuestas
Comentario

Pregunta de dificultad considerable acerca de un síndrome nefrótico. Puede existir confusión, ya


que no debe olvidar que, aisladamente, la GN membranosa es la causa más frecuente de
síndrome nefrótico en el adulto. Pero en este caso nos matizan que se trata de
afroestadounidenses. Cuando nos dicen que la lesión es parcheada, nos están diciendo que es
"focal y segmentaria". Además añaden el dato de la recidiva en el trasplante en un 20-40% y la
resistencia al tratamiento esteroideo (por lo que se le añade ciclosporina con frecuencia). Todas
estos datos nos están sugiriendo que LA RESPUESTA CORRECTA ES LA 3, la hialinosis
segmentaria y focal.(R2)

58. Masculino de 56 años que acude a su consulta por hepatomegalia y los siguientes
laboratorios: Hb 13.2 g/dl, leucocitos 6500/mm3, plaquetas 175,000/mm3, glucosa 111
mg/dl, urea 50 mg/dl, creatinina 0.7 mg/dl, GOT 178 U/L, GPT 97 U/L, GGT 100 U/L,
bilirrubina total 1.0 mg/dl, fosfatasa alcalina 111 U/L, alfafetoproteína 1080 ng/ml. AntiHBs
-, AgHBs +, AntiHBc +, AgHBe -, antiHBe+, DNA-VHB +, anti-VHC -, AgVHD -. Se realiza
una ecografía abdominal en la que se observa un hígado heterogéneo e irregular con una
lesión nodular de 4.5 cm. En un TC se observan los mismos hallazgos, siendo una lesión
con realce en fase arterial y lavado precoz en fase venosa. ¿Cuál sería el diagnóstico más
probable?:

1. 1. Metástasis de cáncer de colon.


2. 2. Hepatocarcinoma.
3. 3. Angioma hepático.
4. 4. Coinfección VHB-VHD.
Gráfico de respuestas
Comentario
Se trata de un paciente con hepatitis crónica por VHB que ha desarrollado un hepatocarcinoma. El
hepatocarcinoma es un tumor que típicamente produce alfafetoproteína aunque no es una
constante. El diagnóstico puede hacerse por el comportamiento en 1 ó 2 técnicas de imagen en las
que se observe una lesión con realce en fase arterial y lavado precoz en fase venosa.(R2)

59. En un paciente con un traumatismo pancreático, la principal causa de mortalidad es:

1. 1. Lesiones vasculares asociadas.


2. 2. Fístula pancreática.
3. 3. Sepsis intraabdominal.
4. 4. Insuficiencia respiratoria progresiva.
Gráfico de respuestas
Comentario

Esta es una pregunta que cuando estudie el manual CTO no le supondrá ningún problema. Así que
no se preocupe. El trauma de páncreas es raro. Es muy frecuente que se acompañe de lesiones
de órganos y vasos de la vecindad. La causa principal de la mortalidad se debe a lesiones
vasculares asociadas (opción 1 cierta). El tratamiento vendrá dado por la magnitud de la lesión. En

!
!
!
!
las lesiones pequeñas bastará con un drenaje adecuado. Cuando hay laceración considerable será
necesaria la cirugía resectiva.(R1)

60. A paraspinal mass


puncture was obtained and pathology and microbiology studies were performed. Image
shows the result of the Ziehl-Neelsen staining. Mark the CORRECT answer:

Gram-negative coccobacilli are observed, which suggests the diagnosis of infection by


1. 1.
Brucella melitensis.
Gram-positive cocci arranged in grape-like clusters are observed, suggesting the diagnosis
2. 2.
of infection by Staphylococcus aureus, probably secondary to endocarditis.
Gram-negative bacilli are observed, which is consistent with an infection by Escherichia
3. 3.
coli, probably due to a complicated urinary tract infection.
Acid-fast bacilli are observed. This finding is consistent with infection by Mycobacterium
4. 4.
tuberculosis. Antituberculosis drugs should be prescribed.
Gráfico de respuestas
Comentario
Cuando la tinción de Ziehl-Neelsen es positiva, se aprecian los bacilos tal como se muestra en la
sección derecha de la imagen: rojizos sobre fondo azul. De las opciones que nos ofrecen, Nocardia
asteroides es también un microorganismo ácido alcohol resistente, pero el contexto clínico sería el
de un paciente con abscesos pulmonares y cerebrales al mismo tiempo. Sin embargo, el caso
clínico y la imagen microbiológica sí encajan bien en una posible tuberculosis, que actualmente se
debe tratar con cuatro fármacos.(R4)

61. Femenino de 60 años que presenta un cuadro clínico de obstrucción intestinal junto
con dolor en la cara interna del muslo, que se alivia con la flexión de la cadera. A la
exploración, no se encuentra masa palpable en la ingle pero presenta dolor que se
extiende medialmente al muslo. Indique el diagnóstico más probable:

1. 1. Hernia directa.
2. 2. Hernia indirecta.
3. 3. Hernia obturatriz.
4. 4. Hernia femoral.
Gráfico de respuestas
Comentario

Es habitual encontrar preguntas sobre hernias en el ENARM. Es importante que conozca todos los
tipos posibles y además añada algunos datos epidemiológicos que seguro serán de mucha utilidad.
En este caso estamos hablando de una hernia un tanto especial ya que rara vez es palpable.

!
!
!
!
Produce dolor que se extiende medialmente al muslo. Se conoce como hernia obturatriz y el
diagnóstico suele confirmarse por una TAC.(R3)

62. Masculino de 24 años comienza con fiebre elevada (40ºC) que se acompaña
inicialmente de dolorimiento abdominal. Posteriormente aparecen “manchas rosadas”
maculares en tórax y abdomen que palidecen con la presión. En los laboratorios
leucopenia, y en la exploración física, Tª: 40 ºC, FC: 70 lpm. Responde a tratamiento con
quinolonas. ¿Cuál es el diagnóstico más probable?:

1. 1. Viriasis.
2. 2. Rickettsiosis.
3. 3. Fiebre tifoidea.
4. 4. Leptospirosis.
Gráfico de respuestas
Comentario

Conviene que recuerde este caso clínico, pues es la típica “foto” con la que cursa una fiebre
tifoidea, enfermedad causada porSalmonella typhi. Cursa con escasa clínica a nivel digestivo, pero
con abundante clínica sistémica: fiebre, cefalea, dolor abdominal, esplenomegalia, leucopenia y
bradicardia relativa. A partir de la segunda semana, el cuadro se puede acompañar de roseóla
tifoidea (manchas rosadas) o alteración del nivel de conciencia, pues el bacilo invade el torrente
circulatorio desde el intestino. Salmonella typhipenetra en el organismo por la ingesta de agua y
verduras contaminadas. El diagnóstico precoz se basa en el hemocultivo (en las dos primeras
semanas del cuadro es el procedimiento más útil) o en el coprocultivo (a partir de la tercera
semana). Respecto al tratamiento, debido a la aparición de cepas de Salmonella typhiresistentes a
diversos antibióticos, el tratamiento actualmente recomendado son las quinolonas o cefalosporinas
de 3ª generación.(R3)

63. Traen a urgencias a un paciente que ha sufrido un accidente de tráfico. El paciente


está inconsciente y hemodinámicamente inestable, intubado, ventilándose con ambú y
se le han colocado dos vías periféricas por las que se están administrando expansores
plasmáticos. El paciente presenta: traumatismo craneoencefálico, neumotórax a tensión
izquierdo, múltiples fracturas costales, fractura de húmero izquierdo, fractura de pelvis
inestable. Lo primera medida a tomar es:

1. 1. Realizar TC craneal para evaluar el traumatismo craneoencefálico.


2. 2. Evaluar lesiones viscerales, sobre todo para descartar rotura esplénica o hepática.
3. 3. Colocar un tubo de tórax para drenar el neumotórax.
4. 4. Colocar un fijador externo para estabilizar la fractura y reducir la hemorragia.
Gráfico de respuestas
Comentario

Esta pregunta es muy importante ya que el manejo del politraumatizado es un tema muy
preguntado. Para valorar el orden de priorización ante un accidentado utilizamos el acrónimo
ABCDE. Lo prioritario es mantener la vía aérea permeable (airway) y en segundo lugar controlar la
ventilación (breathing), por lo que se deben tratar aquellas lesiones que la alteren, como es el caso
del neumotórax a tensión, colocando un tubo de drenaje de manera urgente.(R3)

64. El síndrome de ovario poliquístico se caracteriza por todo lo siguiente, EXCEPTO:

1. 1. Hirsutismo.

!
!
!
!
2. 2. Esterilidad.
3. 3. Hipertensión.
4. 4. Obesidad.
Gráfico de respuestas
Comentario

Esta pregunta es muy importante porque debemos conocer perfectamente el síndrome de ovario
poliquístico y sus alteraciones hormonales. Es una afección muy frecuente en que está aumentada
la LH con niveles de FSH bajos o inferiores a los normal, por lo que aumenta la relación LH/FSH.
Hay un aumento leve de andrógenos, aumento de la estrona (los andrógenos circulantes son
convertidos a estrona en la grasa periférica) y descenso del estradiol. La LH aumentada estimula la
teca, produciendo más andrógenos (también hay una sobreproducción suprarrenal) que va a
provocar obesidad, hirsutismo y anovulación. La esterilidad es el síntoma más frecuente (73%) y se
debe precisamente a la falta de ovulación. Existe insulinresistencia. La tendencia actual para el
tratamiento de la esterilidad en el SOP es usar como primera opción clomifeno y metformina. Si
falla usar gonadotropinas como 2º opción y, si nuevamente fracasamos, destrucción parcial del
ovario por vía laparoscópica.(R3)

65. Recién nacida de 36 horas de vida trasladada a la unidad de neonatología de su


hospital para estudio y tratamiento de ictericia progresiva y masa abdominal izquierda.
El embarazo ha sido controlado y normal. El parto ha tenido lugar a las 42 semanas de
gestación, tras un tiempo de rotura de membranas de 9 horas y sin fiebre materna
intraparto. Fue eutócico, en presentación cefálica, prolongado y traumático, precisando
instrumentación. Apgar 9/10. PRN: 4,180 g. Talla: 51 cm. PC: 37 cm. A su llegada se realiza
una exploración neonatal completa en la que se detecta coloración ictérica de piel y
mucosas, masa abdominal a nivel de hipocondrio izquierdo y crepitación de clavícula
izquierda. Se realizan exámenes de laboratorio donde se observa una hemoglobina de 12
g/dl, bilirrubina total 14.3 mg/dl, bilirrubina indirecta 13.9 mg/dl. En la bioquímica
destacan un sodio de 131 mEq/l, un potasio de 5.9 mEq/l y una glucemia 55 mg/dl.
Respecto a la patología que sospecha en este neonato, señale la opción INCORRECTA

1. 1. El diagnóstico se confirma mediante un ultrasonido abdominal.


2. 2. Es más frecuente en fetos macrosómicos nacidos de nalgas.
3. 3. El tratamiento es sintomático y de sostén.
4. 4. Se debe a una trombosis de la vena renal.
Gráfico de respuestas
Comentario

Tema muy rentable. El caso que se describe es el típico de una hemorragia suprarrenal. El
sangrado de esta víscera ocurre en fetos macrosómicos que nacen de nalgas (aunque también
puede ocurrir en partos en cefálica como es el caso) y han sufrido un expulsivo traumático (nuestro
paciente requirió instrumentación y tiene una fractura de clavícula). La clínica se debe por un lado
al sangrado, que da lugar a la anemización y la ictericia (el neonato no logra aclarar toda la
hemoglobina y por ello se elevan la bilirrubina total y la indirecta); y por otro lado a la lesión de la
glándula que da lugar a un déficit de cortisol (hormona contrarreguladora que eleva la glucemia,
por lo que el paciente se encuentra hipoglucémico) y aldosterona (que en condiciones normales
retiene sodio en el túbulo renal a expensas de perder potasio y protones, por lo que si esta
hormona es deficitaria se producirá lo contrario: aumentarán potasio y protones (hiperpotasemia y
acidosis) y disminuirá el sodio (hiponatremia)). El diagnóstico se confirma con ultrasonido y no
precisa cirugía de entrada, siendo suficiente un tratamiento de sostén. La afirmación claramente
falsa es la 4, ya que el cuadro se debe a la lesión directa y sangrado posterior de la suprarrenal, no
a la trombosis de la vena renal.(R4)

!
!
!
!
66. ¿Qué dato NO es característico de la lepra lepromatosa?:

1. 1. Amiloidosis visceral.
2. 2. Máculas anestésicas unilaterales.
3. 3. Unión dermoepidérmica respetada.
4. 4. Eritema nodoso leproso.
Gráfico de respuestas
Comentario

La lepra es un tema complicado para el ENARM. Dependiendo del estado inmune del huésped
infectado, se producen unas u otras formas de lepra. La lepra lepromatosa es la típica de las
películas. Es la forma que afecta a sujetos con una mala inmunidad celular y que se ven
abiertamente superados por el bacilo. Estos sujetos tendrán altas cargas de bacilos, múltiples
lesiones, simétricas, de distribución acral, pequeñas, con alteración visceral. El compromiso
neurológico sensitivo es algo típico que debemos recordar de la lepra. Típicamente este
compromiso es precoz en la lepra tuberculoide (la de los sujetos con buena inmunidad) y tardío en
la lepra lepromatosa (y por tanto menos característico). Un dato típico de la histología en la lepra
lepromatosa es que existe un infiltrado linfocitario dérmico que respeta la unión dermoepidérmica,
denominada "banda de Unna". El eritema nodoso es típico de la leproreacción tipo II, que a su vez
es propio de la lepra lepromatosa.(R2)

67. ¿Con qué cuadro clínico de los siguientes NO se relaciona la infección por virus
respiratorio sincitial en la infancia?:

1. 1. Neumonía.
2. 2. Síndrome de muerte súbita del lactante.
3. 3. Pausas de apnea en prematuros.
4. 4. Meningoencefalitis.
Gráfico de respuestas
Comentario

De el VRS es importante que recuerde que es el principal patógeno respiratorio en los niños
pequeños, principalmente su asociación con la bronquilitis del lactante. No causa enfermedad fuera
del aparato respiratorio.

La infección por VRS suele ocasionar un cuadro catarral de vías altas. En la mitad de los casos
progresa hacia el tracto respiratorio inferior causando fundamentalmente bronquiolitis, con menor
frecuencia neumonía y, en un pequeño porcentaje de casos, crup.

La asociación entre bronquiolitis por VRS y episodios recurrentes de sibilancias o asma en la


infancia ha sido puesta de manifiesto en numerosos estudios.

Los lactantes < de 3 meses son de alto riesgo de enfermedad grave por VRS y presentan un alto
riego de apneas.

Entre los diversos mecanismos que se han propuesto para explicar el SMSL se han considerado la
alteración de los reflejos respiratorios y la obstrucción de la vía respiratoria. No es raro encontrar
una IRS previa.(R4)

68. Una de las siguientes enfermedades NO se produce por mutación de algún elemento
del receptor hormonal:

!
!
!
!
1. 1. Diabetes insípida central.
2. 2. Síndrome de Morris o feminización testicular.
3. 3. Pseudohipoparatiroidismo.
4. 4. Síndrome de resistencia a hormonas tiroideas.
Gráfico de respuestas
Comentario

Analicemos cada respuesta una a una:

- El síndrome de Morris se debe a una insensibilidad de los receptores androgénicos (respuesta 2


incorrecta).

- El pseudohipoparatiroidismo comparte características clínico-laboratorio con el


hipoparatiroidismo. Sin embargo, lo que se produce no es un déficit de PTH, sino una
insensibilidad de su receptor (opción 3 incorrecta), que no responde a ella.

- La opción 4 es claramente falsa. Una resistencia a una hormona se debe a fallos en el receptor,
no a la falta de la propia hormona.

- Por último, en cuanto a la diabetes insípida CENTRAL, el fallo radica en un déficit de ADH, sin
que exista alteración alguna en su receptor. La respuesta correcta por tanto sería la 1. No olvide
que la diabetes insípida nefrogénica sería un problema distinto, debido a una insensibilidad del
receptor.(R1)

69. La aparición en una placa de tórax de un patrón micronodular difuso en ambos


campos pulmones es MENOS probable:

1. 1. Metástasis de carcinoma de tiroides.


2. 2. Sarcoidosis.
3. 3. Silicosis.
4. 4. Metástasis de carcinoma colo-rectal.
Gráfico de respuestas
Comentario

Pregunta no muy dificil. El síntoma más frecuente del cancer de pulmón es la tos y hemoptisis. En
un paciente con NPS con criterios de malignidad (fumador, edad avanzada...) se debe sospechar
un cancer. En el cancer de pulmón aparecen frecuentemente síndromes paraneoplásicos, casi
siempre por el oat-cell, entre los que se encuentran SIADH, astenia, síndrome carcinoide,
polineuropatias... Dentro de las neumoconiosis es importante recordar que la asbestosis aumenta
el cancer de pulmón (recuerde la regla: asbesto-adenocarcinoma, a lo mejor parece estúpido pero
cada uno hace lo que puede...) además de aumento de incidencia del mesotelioma; sin embargo la
silicosis no aumenta el riesgo de cancer de pulmón, aunque es típica la asociación con la
tuberculosis. Lo más importante de la pregunta es que recuerde que ante todo paciente en el que
haya sospecha de cancer de pulmón se debe hacer broncoscopia más biopsia de la lesión
sospechosa, no la citología.(R4)

70. En los retrasos de crecimiento asimétrico, el último parámetro en afectarse es:

1. 1. Peso.
2. 2. Grasa subcutánea.
3. 3. Perímetro abdominal.

!
!
!
!
4. 4. Perímetro cefálico.
Gráfico de respuestas
Comentario

Tema muy importante para el ENARM.

Existen dos tipos de crecimiento intrauterino retardado (CIR):

- CIR tipo I o CIR simétrico: Se produce desde el comienzo de la gestación. El crecimiento


longitudinal, los diámetros abdominales y otras medidas guardan entre sí las proporciones
adecuadas. Entre sus causas, destacan las cromosomopatías y las infecciones connatales.

- CIR tipo II o CIR asimétrico: Es mucho más frecuente que el tipo I. Consiste en una reducción de
los diámetros abdominales, quedando preservados los parámetros óseos de medida (diámetro
biparietal y longitud femoral). La causa más frecuente de CIR II son las enfermedades placentarias,
como la preeclampsia u otras condiciones que puedan producir una disminución del intercambio
útero- placentario. En definitiva, el peso de la placenta no guarda la debida proporción con el del
feto.(R4)

71. ¿Cuál es la incidencia de las convulsiones febriles que se presentan en los niños
entres los 3 meses y los 5 años de edad?

1. 1. 15-20%.
2. 2. 8-10%.
3. 3. 12-15%.
4. 4. 3-5%.
Gráfico de respuestas
Comentario

Pregunta dificil y memorística, por lo que no debe proecuparle haberla fallada. La respuesta es 3-
5%, respuesta 4 correcta.(R4)

72. Primigesta de 29 años, en su 32ª semana de gestación, que acude al obstetra a


revisión. En las últimas dos semanas ha ganado 2 kg, refiere ligera astenia, y en la toma
de TA se hallan valores de 145/95 mmHg. En los exámenes de orina solo destaca una
proteinuria de ++. Señale cuál de las siguientes premisas, acerca del manejo de la
enfermedad que tiene la paciente, NO es correcta:

Dado que el problema fisiopatológico reside en la placenta, el único tratamiento definitivo


1. 1.
es terminar la gestación.
Es aconsejable administrar una dieta normosódica y rica en proteínas aunque exista
2. 2.
retención hídrica.
3. 3. Los hipotensores permiten controlar la enfermedad si se administran desde fases precoces.
Los diuréticos están contraindicados ya que al disminuir el flujo plasmático también
4. 4.
disminuyen el flujo útero-placentario.
Gráfico de respuestas
Comentario

Como regla general, la terminación de la gestación sigue siendo el único tratamiento definitivo para
la preeclampsia (opción 1) y la opción terapéutica de elección ante cualquier paciente con

!
!
!
!
preeclampsia a término (cualquier grado de severidad) o en gestación pretérmino con enfermedad
grave no controlable.

No se aconseja la restricción de sodio en la preeclampsia (opción 2).

El tratamiento hipotensor no influye en el curso de la enfermedad, y por tanto, ningún fármaco


hipotensor previene la evolución a preeclampsia grave (opción 3 falsa).

Los diuréticos disminuyen el flujo útero-placentario y no están indicados en la gestación (opción


4).(R3)

73. ¿Cuál es el fármaco de elección en una esclerodermia sistémica con crisis renal y TA
de 200/100 mmHg?:

1. 1. Nifedipina.
2. 2. IECAs.
3. 3. Clonidina.
4. 4. Doxazosina.
Gráfico de respuestas
Comentario

Esta pregunta es muy importante y muy preguntable de cara al ENARM. Debe saber que los IECAs
han modificado la historia natural de las crisis renales de la esclerodermia difusa, que hasta hace
poco, se proclamaban como primera causa de muerte. Se frena así la progresión de la HTA
maligna por intervención en el eje renina-angiotensina, alterado ante la necrosis fibrinoide renal. Es
gracias a los IECAs que la fibrosis pulmonar es hoy la primera preocupación para los médicos que
tienen a su cargo pacientes con esclerodermia difusa, porque en casos moderados-avanzados
carece de tratamiento y constituye la primera causa de muerte.(R2)

74. El fenómeno de Koebner NO suele aparecer en:

1. 1. Psoriasis vulgar.
2. 2. Vasculitis leucocitoclástica.
3. 3. Impétigo contagioso.
4. 4. Lupus eritematoso discoide.
Gráfico de respuestas
Comentario

Se trata de una pregunta de dificultad media. El fenómeno de Koebner es la aparición de lesiones


cutáneas propias de una determinada dermatosis en las zonas de roce, presión o traumatismo. Es
típico de las enfermedades eritematoescamosas (psoriasis, liquen plano, enfermedad de Darier,
pitiriasis rubra pilaris), algunas infecciones (molluscum contagioso, verrugas) y otras dermatosis
más raras (vasculitis, síndrome de Sweet). La única enfermedad de las que se describen que NO
presenta este signo es el lupus eritematoso discoide (respuesta 4 correcta).(R4)

75. Una paciente de 34 años ha sido diagnosticada de enfermedad celíaca con histología
y serología; aunque cumple el tratamiento estrictamente, no ha mejorado después de tres
meses, e incluso ha perdido 7 kilos de peso y tiene febrícula. ¿En qué pensaría usted en
primer lugar?:

!
!
!
!
1. 1. El diagnóstico de celíaca no es cierto.
2. 2. Realmente no hace bien la dieta.
3. 3. Tres meses es poco tiempo.
4. 4. Descartaría un linfoma intestinal.
Gráfico de respuestas
Comentario

Si un paciente no responde al tratamiento, habrá que plantearse, entre otras cosas: que el paciente
no sigue bien la dieta, diagnóstico incorrecto, existe otra causa concurrente o ha desarrollado un
linfoma (como ocurre en este caso).

Los pacientes con enfermedad celíaca tienen un aumento del riesgo de tumores, sobre todo de
linfoma intestinal de tipo T. Nos dicen que el paciente sigue estrictamente la dieta y aún así ha
perdido 7 kilos y además presenta febrícula, por lo que lo primero que debemos hacer es descartar
la presencia de un linfoma.(R4)

76.
Masculino de 53 años, fumador e hipertenso, que acude a su consulta por presentar
desde hace 7 meses episodios de molestias precordiales que define como un peso,
irradiado a hombro izquierdo coincidiendo con la subida de cuestas. Explica que le
calman después de 2-3 minutos de reposo. El último episodio de dolor fue hace 2 días.
Usted realiza un ECG que se muestra en la imagen, cuyos hallazgos ya estaban presentes
en ECG previos. De los siguientes enunciados señale la respuesta CORRECTA:

1. 1. El paciente presenta un síndrome coronario agudo sin elevación del segmento ST.
En estos momentos estaría indicado iniciar tratamiento con antiagregantes plaquetarios y
2. 2.
heparina de bajo peso molecular.
3. 3. La ergometría simple es la prueba de detección de isquemia de elección en este paciente.
El tratamiento con nitratos transdérmicos puede mejorar la calidad de vida de los
4. 4.
pacientes.
Gráfico de respuestas
Comentario

Una pregunta de dificultad media sobre cardiopatía isquémica. Veamos qué es lo que hace falsa
cada respuesta:

!
!
!
!
R1. Se trata de un cuadro de angina estable, no de un síndrome coronario aguda. Ténga en cuenta
que el paciente lleva siete meses con síntomas, que se desencadenan con esfuerzos de intensidad
parecida.

R2. No precisa heparina de bajo peso molecular. Lo que sí tiene sentido, teniendo en cuenta el
diagnóstico correcto, es la antiagregación.

R3. La ergometría simple puede no ayudarnos demasiado en este paciente, debido a la existencia
de alteraciones en la repolarización en su ECG (p.e. en aVL, V4- V6). Por otra parte, parece existir
cierto grado de hipertrofia ventricular.

Tal como dice la respuesta 4, los nitratos transdérmicos pueden mejorar su calidad de vida, dado
que también aliviarían su dolor torácico. Recuerde que los nitratos, sin embargo, no han
demostrado impacto sobre la supervivencia de estos pacientes.(R4)

77. Femenino de 32 años, embarazada de gemelos y diagnosticada de polihidramnios,


comienza después del parto con una importante hemorragia. A la palpación se detecta la
ausencia de retracción uterina. Debemos pensar en:

1. 1. Desprendimiento de placenta normalmente inserta.


2. 2. Placenta acreta.
3. 3. Desgarro de partes blandas.
4. 4. Atonía uterina.
Gráfico de respuestas
Comentario

La causa más frecuente de las hemorragias del alumbramiento es la atonía uterina. Además, esta
paciente que nos presentan tiene un riesgo aumentado, por la sobredistensión uterina mantenida
por el polihidramnios y el embarazo gemelar. La clínica de un útero que no se contrae nos apoya
en nuestra sospecha, y lo único que tenemos que descartar es la presencia de una rotura uterina
como causa de atonía uterina. Para ello, se revisaría la cavidad uterina y, si no existe solución de
continuidad, comenzaría el tratamiento de la atonía con masaje uterino, fármacos uterotónicos,
taponamiento uterino… Si las medidas conservadoras fracasan, la histerectomía sería el
tratamiento definitivo.(R4)

78. Señale cuáles son los elementos del triángulo de Hesselbach.

1. 1. Los vasos epigástricos, ligamento de Cooper, borde lateral de los rectos.


2. 2. Ligamento inguinal, vasos epigástricos y borde lateral de los rectos.
3. 3. Ligamento inguinal, vasos epigástricos, borde interno de los rectos.
4. 4. Vasos epigástricos, ligamento inguinal, borde del arco de Douglas.
Gráfico de respuestas
Comentario

Pregunta fácil de anatomía. La respuesta correcta es la 2.(R2)

79. ¿A qué nos referimos al hablar de la enfermedad de Paget?:

1. 1. A una enfermedad que cursa con galactorrea.


2. 2. A un tipo de cáncer mamario.
3. 3. A una malformación mamaria.

!
!
!
!
4. 4. A una linfangitis mamaria.
Gráfico de respuestas
Comentario

La enfermedad de Paget se considera como la extensión de un tipo de cáncer de mama ductal a la


epidermis suprayacente, afectando pezón y la areola. Es poco frecuente, representando el 1-3% de
todos los cánceres de mama. Clínicamente, consiste en una lesión eccematosa, asociando
también prurito. Por ello, el eccema es su principal diagnóstico diferencial. Una de las
características por las que puede distinguirse del eccema es por ser unilateral (el eccema suele
afectar a ambas mamas). Otro rasgo diferencial son los límites, que suelen ser difusos en el
eccema y mejor definidos en el Paget mamario.

Recuerde que, histológicamente, son características las células de Paget. Son grandes,
redondeadas, PAS-positivas, con núcleos de gran tamaño y sin puentes intercelulares que las
unan.(R2)

80. A 35-year-old woman makes an appointment with her physician, due to intermittent
bloody discharge from her left breast and staining of her bra. Her medical history is
unremarkable. Physical examination reveals no lumps or masses in either breast, and no
adenopathies are palpable. Ultrasongraphy is performed, and it is reported as normal.
What is the most likely diagnosis?

1. 1. Fibroadenoma.
2. 2. Fibrocystic changes.
3. 3. Paget's disease.
4. 4. Intraductal papilloma.
Gráfico de respuestas
Comentario
Intraductal papilloma. In the breast, intraductal papilloma (IDP) is a benign lesion that consists of
branching fibrovascular cores with overlying layers of epithelial and myoepithelial cells. It frequently
presents as a unilateral serous or bloody nipple discharge. It may also present as a palpable breast
mass and it might cause breast pain. Mammography reveals no abnormality in most cases but may
show duct ectasia, microcalcifications, or a mass. Most studies have shown a small risk (0-3%) of
finding ductal carcinoma in situ (DCIS).(R4)

81. En relación a las masas suprarrenales asintomáticas, ¿cuál de los siguientes estudios
diagnósticos considera que es el menos útil?

1. 1. Medición de catecolaminas y metanefrinas en orina de 24 horas.


Punción-aspiración con aguja fina para diagnóstico diferencial de tumores suprarrenales
2. 2.
primarios benignos y malignos.
3. 3. Test de supresión con 1 mg de dexametasona para cortisol.
4. 4. Medición de 17-OH progesterona.
Gráfico de respuestas
Comentario

Esta pregunta hace referencia a la actitud que hay que adoptar ante el diagnóstico de una masa
suprarrenal asintomática (incidentaloma suprarrenal). Lo primero es descartar la funcionalidad de la
masa; para eso, se realizan varias determinaciones: catecolaminas y metanefrinas en orina de 24
horas, cociente aldosterona/ARP y electrolitos en sangre y orina, supresión con 1 mg de

!
!
!
!
dexametasona o cortisoluria de 24 horas y DHEA-s y 17 cetoesteroides. Pueden existir dudas entre
las opciones 2 y 4. La determinación de 17-hidroxiprogesterona es de utilidad para descartar
hiperplasia suprarrenal congénita, donde en ocasiones, puede aparecer una imagen de adenoma
unilateral en las técnicas de imagen. La PAAF es útil para distinguir la enfermedad primaria de la
metastásica, pero no la enfermedad primaria benigna de la maligna.(R2)

82. A four year old boy falls while running in the street. He used both hands to stop the
fall, and he now complains of pain in the wrist. Hand X-ray reveals a fracture through the
distal radius growth plate and the metaphysis, sparing the epiphysis. Which of the
following is incorrect about this kind of injury?

The relevance of the lesion lies in the potential injury of epiphyseal cartilage, which can
1. 1.
cause growth arrest in limb length.
2. 2. Because it affects the growth plate, the treatment is surgical.
The deviation of the growth in length of the radius is another possible complication
3. 3.
essential to consider the different therapeutic alternatives.
4. 4. The fracture is not unstable because it does not affect the articular surface.
Gráfico de respuestas
Comentario

Pregunta tipo caso clínico sobre las epifisiolisis. Recuerde que este tipo de fracturas que discurren
sobre una de las zonas de crecimiento del hueso (fisis), en los extremos de los huesos largos.
Existen 5 tipos según la clasificación de Salter y Harris:

- Tipo I, que discurren a lo largo de la fisis.

- Tipo II, que tienen un trazo ascendente metafisario (típicas del radio distal).

- Tipo III, que son fragmentos epifisarios (tibia distal).

- Tipo IV, que son de arriba abajo (metáfiso-epifisarias, como en los cóndilos humerales).

- Tipo V, que son similares a las fracturas en rodete (por compresión axial).

Las de tipo I y II se tratan de forma conservadora; las tipo III y IV, de forma quirúrgica. Recuerde
que la principal complicación es el cierre precoz de la fisis, originando una epifisiodesis.(R2)

83. Niña de 18 meses que es traída a Urgencias por presentar crisis de tos,
desencadenadas por la ingesta de agua y alimento, y que suelen terminar con una pausa
de apnea o con vómitos de características mucosas. Como antecedentes de interés sólo
destaca un cuadro catarral que apareció hace una semana. A la exploración física destaca
una hemorragia subconjuntival y una discreta hernia umbilical. Respecto a la enfermedad
que usted sospecha, señale cuál de las siguientes afirmaciones es CIERTA:

1. 1. El agente causal en un Paramyxovirus.


2. 2. La enfermedad suele conferir inmunidad duradera, de por vida.
En los exámenes de laboratorio es típica la existencia de una importante leucocitosis con
3. 3.
linfocitosis absoluta.
4. 4. La profilaxis se realiza mediante la administración de gammaglobulina específica.
Gráfico de respuestas

!
!
!
!
Comentario

El cuadro que nos describen corresponde a una tos ferina.

Esta enfermedad es una infección producida principalmente por Bordetella pertussis. Afecta
preferentemente a menores de un año. Clínicamente, produce un pródromo inespecífico, de tipo
catarral, y seguidamente aparece la fase que caracteriza a la enfermedad, que es la de tos
paroxística. Produce accesos repentinos de tos, con series repetitivas de tos enérgica,
acompañándose después de un ruido inspiratorio que recibe el nombre de gallo, al pasar aire a
través de una glotis cerrada. Las complicaciones de la tos ferina son:

- Neumonía: La más frecuente. Suele ser por sobreinfección bacteriana secundaria.

- Convulsiones, cuya causa no está bien establecida.

- Otras: Prolapso rectal, hemorragia subconjuntiva, hernia umbilical, epistaxis… Estas


complicaciones se deben a la intensa presión positiva intratorácica que se produce en relación con
la tos.

A pesar de tratarse de una infección bacteriana, es característica la presencia de linfocitosis en la


biometría hemática. El tratamiento antibiótico de la tos ferina es la eritromicina durante dos
semanas.(R3)

84. Respecto al adenoma pleomorfo de las glándulas salivales señale lo CORRECTO:

1. 1. Se domina también tumor mixto benigno.


2. 2. Presenta mayor prevalencia en hombres.
3. 3. La glándula más frecuentemente afectada es la submaxilar.
4. 4. Tiene una alta tasa de malignización hacia adenocarcinoma (30-40%).
Gráfico de respuestas
Comentario

El adenoma pleomorfo es el tumor más frecuente de las glándulas salivales (80%) que afecta
sobre todo a la parótida (95%), más habitualmente en mujeres. Es una tumoración indolora, firme y
de crecimiento lento en el lóbulo superficial de la parótida, templado en la gammagrafía. Recidiva
con frecuencia tras la cirugía si no se realiza una parotidectomía superficial, y tiene riesgo de
malignización en un 5-10% de los casos.(R1)

85. ¿Cuál de los siguientes hechos es CORRECTO en el cáncer de ovario?

1. 1. Es más frecuente en las multíparas.


2. 2. Es más frecuente en las mujeres que han tomado varios años anticonceptivos orales.
3. 3. Suelen dar precozmente metrorragias.
4. 4. Se implanta fácilmente por la serosa peritoneal.
Gráfico de respuestas
Comentario

Pregunta directa sobre cáncer de ovario.

!
!
!
!
La vía de diseminación más normal del cáncer de ovario es la implantación directa por siembra
peritoneal de células tumorales sobre peritoneo o epiplón, siendo la extensión peritoneal difusa la
forma más frecuente de diseminación del cáncer de ovario.(R4)

86. Niño de 7 años, acude a Urgencias por presentar un exantema ligeramente


pruriginoso en tronco y extremidades, sin afectación palmoplantar y con aspecto de
"encaje". Afebril. Los padres refieren que el día previo al exantema tenía las mejillas
muy "rojas". Señale lo CORRECTO:

1. 1. El virus que lo produce es ARN.


2. 2. Los síntomas generales son más graves en niños que en adultos.
3. 3. Este niño ya no es contagioso.
4. 4. El agente responsable no se ha relacionado con hydrops fetal.
Gráfico de respuestas
Comentario

Pregunta de dificultad media sobre el eritema infeccioso o 5ª enfermedad, diagnóstico que


podemos hacer con los datos característicos que nos dan en el enunciado, como son exantema
en encaje, afectación de las mejillas (“bofetón”) y el estado afebril del niño, que además suele
estar en edad escolar (5-15 años).

La opción correcta es la 3, ya que el periodo de contagio es máximo antes de la aparición del


exantema, y a partir de este momento desaparece la contagiosidad.

El parvovirus es un virus de DNA, los síntomas generales son más frecuentes en adultos,
aunque es una infección tipica de la edad escolar.

Entre sus complicaciones, destacan los abortos y el hydrops fetal en embarazadas, las crisis
aplásicas en pacientes con anemias hemolíticas crónicas y, en inmunodeprimidos, la
cronificación de la enfermedad.(R3)

87. Señale el agente causal de la escarlatina:

1. 1. Estreptococo beta-hemolítico del grupo A.


2. 2. Herpes simple tipo 6.
3. 3. Virus coxsackie B.
4. 4. Staphylococcus aureus.
Gráfico de respuestas
Comentario

Pregunta directa e importante para el ENARM que debe quedar clara ya en la primera vuelta.

Es fundamental conocer el agente causal de cada una de las enfermedades exantemáticas en


pediatría.

La respuesta correcta es la 1. De la escarlatina es también importante recordar que es la que


tiene el periodo de incubación más corto (3 días) y que son signos característicos las líneas de
Pastia y la fascies de Filatov.(R1)

!
!
!
!

88. Femenino de 49 años que acude a consulta de ginecología para revisión. Entre sus
antecedentes destaca la existencia de mastodinia cíclica, con un aumento de densidad
mamaria, que se ha complicado con la aparición de dos tumoraciones en cuadrante
externo derecho de mama derecha, bien delimitadas y de consistencia elástica. Se
realiza ultrasonido mamario que confirma el diagnóstico de quistes mamarios, en el
seno de una mastopatía fibroquística. ¿Cuál sería la conducta terapéutica más
apropiada?:

1. 1. Aplicación percutánea de un gel de progesterona.


2. 2. Progesterona por vía percutánea más gestágeno por vía oral.
3. 3. Punción de ambos quistes, con citología del líquido.
4. 4. Extirpación quirúrgica de ambas tumoraciones.
Gráfico de respuestas
Comentario

Pregunta muy importante. El tratamiento óptimo para una mastopatía fibroquística en el que se
vislumbran claramente quistes (en este caso son tan grandes que incluso se palpan) es
drenarlos de manera percutánea y posteriormente su estdio anatomopatológico.(R3)

89. Gestante de 32 semanas con sangrado vaginal oscuro y escaso. A la palpación


abdominal se aprecia una marcada hipertonía uterina. No se aprecia latido fetal. El
estado materno es malo, con signos de choque y los exámenes de laboratorio de
Urgencias son compatibles con el inicio de una coagulopatía. ¿Qué diagnóstico le
sugeriría?

1. 1. Placenta previa sangrante.


2. 2. Abdomen agudo de causa no obstétrica.
3. 3. Desprendimiento prematuro de placenta.
4. 4. Amenaza de parto pretérmino.
Gráfico de respuestas
Comentario

Pregunta muy importante acerca de las hemorragias de tercer trimestre. De las que no se deben
fallar.

Nos presentan un caso clínico compatible con desprendimiento prematuro de placenta


normoinserta: comienzo brusco, sangrado escaso y oscuro, mal estar general de la paciente
(choque y coagulopatía), riesgo de sufrimiento fetal, dolo e hipertonía uterina.(R3)

90. Señale qué patología NO asocia fontanela grande en el recién nacido:

1. 1. Acondroplasia.
2. 2. Síndrome de Down.
3. 3. Craneosinostosis.
4. 4. Hipotiroidismo.
Gráfico de respuestas
Comentario

!
!
!
!

La craneosinostosis consiste en el cierre precoz de alguna fontanela. Por ende, en esta


patología las fontanelas estarán disminuidas de tamaño, y no aumentadas.(R3)

91. Un niño de 4 años presenta fiebre, faringoamigdalitis con exudados,


hepatoesplenomegalia y adenopatías cervicales, inguinales y axilares. Los exámenes
de laboratorio muestran leucocitosis, con linfocitosis y 20% de linfocitos atípicos.
¿Cuál de los siguientes es el germen responsable de esta enfermedad?:

1. 1. Citomegalovirus.
2. 2. Estreptococo beta-hemolítico.
3. 3. Virus de la rubéola.
4. 4. Virus de Epstein-Barr.
Gráfico de respuestas
Comentario

Un cuadro clínico típico, que no es muy difícil de reconocer, pero que quiza no nos acaba de
cuadrar la edad.

Vamos a analizar las distintas opciones más detalladamente.

- La 2, un cuadro bacteriano que podríamos confundir por la faringoamigdalitis, queda


descartada por las alteraciones de laboratorio, ya que cursaría más típicamente con leucocitosis
con neutrofilia y no con linfocitosis. Pensamos entonces en un virus.

- ¡La rubéola es una enfermedad exantemática y este niño no tiene exantema!

- El citomegalovirus es la duda razonable, pero el dato que no deja lugar a duda es que entre la
linfocitosis, haya un ¡20% de linfocitos atípicos! Eso nos tiene que sonar a VEB pero esta era la
enfermedad "del beso" típica de adolescente y el paciente que se nos presenta tiene 4 años.
Hay que tener en cuenta que aunque el beso sea la vía más frecuente de transmisión (típica de
adolescentes) no es la única, y puede, como vemos aparecer a otras edades.(R4)

92. RNT que a las 3 semanas de vida comienza con fiebre, irritabilidad y rechazo de las
tomas. Doce horas después se encuentra pálido, letárgico, hipotónico y con reflejo de
succión débil. En la exploración presenta una temperatura de 40°C, FC de l80. lpm, FR
de 60 rpm; auscultación cardiopulmonar normal; abdomen no doloroso, sin masas ni
megalias; fontanela a tensión. BH: leucocitos 2,800, segmentados 12%, bandas 13%.
La prueba diagnóstica que debe realizarse a continuación es:

1. 1. Hemocultivo.
2. 2. Urocultivo.
3. 3. Ultrasonido cerebral.
4. 4. Punción lumbar.
Gráfico de respuestas
Comentario

El caso de este neonato nos obliga a descartar la posibilidad de que este sufriendo una
meningitis neonatal secundaria a sepsis neonatal. Respuesta 4 correcta.(R4)

!
!
!
!

93. En el niño, ¿Cuál es la causa más común de retraso de crecimiento?

1. 1. Hipoxia secundaria a cardiopatía congénita.


2. 2. Infecciones respiratorias de repetición.
3. 3. Desnutrición primaria.
4. 4. Anomalías cromosómicas.
Gráfico de respuestas
Comentario

A nivel mundial, la desnutrición es la causa más común de retraso de crecimiento, pues dos
tercios de la población mundial sufren esta problemática. La falta de nutrientes también puede
ser provocada por restricción calórica voluntaria (adolescentes temerosas de ser obesas,
bailarinas de ballet, gimnastas), por cuadros psiquiátricos (anorexia nervosa) o por anorexia
secundaria a enfermedades crónicas. La desnutrición también puede ser secundaria a pérdidas
exageradas, como ocurre en los síndromes de malabsorción, o bien a un gasto metabólico muy
alto no suficientemente cubierto con una alimentación habitual (cardiopatías, cuadros
infecciosos crónicos).(R3)

94. ¿Cuál de las siguientes afirmaciones sobre el bocio simple es FALSA?:

1. 1. Se trata de un aumento de la glándula tiroides que no se acompaña de hipertiroidismo.


2. 2. El estudio citológico suele mostrar un bocio coloide.
3. 3. Favorece la aparición de neoplasias tiroideas.
4. 4. El déficit de aporte yódico es el factor etiológico más importante.
Gráfico de respuestas
Comentario
El bocio simple es aquel aumento de la glándula tiroidea que no se debe a causa tumoral,
fármacos o enfermedad autoinmune. La causa más frecuente del bocio simple es el déficit de
yodo. Otras son los bociógenos, las alteraciones de las enzimas encargadas de la organificación
o del transporte de yodo en el tiroides, etc. Los niveles de hormonas tiroideas son normales, al
igual que las cifras de TSH. La captación gammagráfica suele ser normal o levemente
aumentada. Suele presentar una palpación difusa, sin nódulos, aunque con el tiempo puede
hacerse nodular y progresar a un bociomultinodular. El tratamiento depende de la clínica
compresiva existente o no en el paciente. Si existe compresión el tratamiento es la tiroidectomía
subtotal y si no, se suele administrar levotiroxina en dosis supresoras si no existen
contraindicaciones. Otras terapias son el radioyodo o la administración de sal yodada en
pacientes con déficit del mismo.(R3)

95. Señale cuál de las siguientes complicaciones de las hernias es la MENOS frecuente:

1. 1. Estrangulación de la hernia femoral.


2. 2. Estrangulación de la hernia inguinal directa.
3. 3. Estrangulación de la hernia crural.
4. 4. Estrangulación de la hernia umbilical.
Gráfico de respuestas
Comentario

!
!
!
!

Es importante que conozca todos los tipos posibles y además añada algunos datos
epidemiológicos que seguro serán de mucha utilidad. La hernia con más riesgo de
estrangulación es la hernia femoral o crural. Le sigue en orden descendente la hernia inguinal
indirecta, la hernia umbilical. La hernia inguinal directa se debe a una debilidad difusa de la
fascia siendo muy raro que se estrangule. Recuerde que la hernia inguinal externa es lo mismo
que la hernia inguinal indirecta; se llama así porque en su origen es externa a los vasos
epigástricos.(R2)

96. Mujer de 26 años


que acude a consulta por dismenorrea progresiva (que requiere tratamiento con AINES
cada 4-6 horas) y dispareunia ocasional. Antecedentes personales: enfermedad
celíaca, apendicetomía y un legrado. Antecedentes gineco-obstétricos: menarquia a
los 10 años, ciclos menstruales=7/26 y una gestación que interrumpió de forma
voluntaria. La especuloscopía y el tacto bimanual no detecto ninguna alteración. Se le
realiza USG pélvico con sonda vaginal y usted observa en la región anexial derecha la
imagen que se muestra en la imagen. Señale opción CORRECTA:

Es probable que se trate de un folículo hemorrágico. Hay que realizar analítica hormonal
1. 1.
en fase folicular precoz e iniciar tratamiento con anticonceptivos a altas dosis.
Para filiar esta masa anexial hay que extraer marcadores tumorales, realizar TAC
2. 2.
abdomino-pélvico y valorar eventual PAAF.
Para filiar esta masa anexial hay que realizar una PAAF guiada por ecografía y según la
3. 3.
estirpe celular realizar el estudio de extensión conveniente.
Se completaría el estudio con ecografía Doppler, marcadores tumorales y la programación
4. 4.
de una laparoscopia diagnóstico terapéutica.
Gráfico de respuestas
Comentario

El cuadro clínico corresponde con una endometriosis, fundamentalmente por la clínica que
presenta (dismenorrea y dispareunia) y las características de la imagen ovárica apoyan dicha
sospecha diagnóstica. Es conveniente completar el estudio USG con un doppler y con los
marcadores tumorales para descartar posible malignidad asociada, aunque el diagnóstico
definitivo se realizará mediante laparoscopía que nos permitirá visualizar la lesión y su
extensión. Además nos permitirá obtener muestra para estudio anatomopatológico.(R4)

!
!
!
!

97. Respecto a la paciente del caso clínico anterior, ¿qué tratamiento le parece
indicado como primera opción terapéutica?:

1. 1. Quistectomía ovárica vía laparoscópica.


2. 2. Anexectomía vía laparoscópica.
Anexectomía vía laparotómica con biopsia intraoperatoria para valorar una eventual
3. 3.
cirugía completa reglada.
Análogos GnRH durante 6 meses y posteriormente, según el tamaño de la lesión, realizar
4. 4.
quistectomía u ooforectomía laparoscópica.
Gráfico de respuestas
Comentario

El tratamiento de la endometriosis sintómática es eminentemente quirúrgico. Es el mejor


tratamiento que podemos ofertar para esta enfermedad. No olvide que es una entidad benigna
que suele afectar a mujeres jóvenes en edad fértil por lo que el tratamiento, aunque sea
quirúrgico, debe ser lo más conservador posible realizando una quistectomía ovárica. Los
tratamientos médicos están indicados tras la cirugía, no antes, y buscan fundamentalmente
controlar los síntomas y evitar las recidivas.(R1)

98. A male patient with a history of long-standing arterial hypertension for the past 20
years presents with symptoms of heart failure. Echocardiogram shows concentric left
ventricular hypertrophy. Which of the following would be the least appropriate
treatment for this patient?

1. 1. Verapamil.
2. 2. Atenolol.
3. 3. Enalapril.
4. 4. Digoxin.
Gráfico de respuestas
Comentario

En este paciente, debemos asumir una insuficiencia cardíaca diastólica (deterioro de la


relajación ventricular), como consecuencia de la hipertrofia debida a su cardiopatía hipertensiva.
En otras palabras, la rigidez del miocardio dificulta su llenado. Por este motivo, serán
beneficiosos los fármacos que disminuyan la frecuencia cardíaca (verapamil, diltiazem, atenolol),
que además son inotropos negativos, con lo que favorecen esta relajación. Por otra parte,
merece la pena utilizar un IECA, que ha demostrado prolongar la supervivencia en la
insuficiencia cardíaca.

En este caso, la digoxina no sería útil. No ha demostrado prolongar la supervivencia en la


insuficiencia cardíaca y, para este paciente en concreto, su carácter inotrópico positivo no nos
aporta nada. Recuerde que está más indicada en la insuficiencia cardíaca sistólica que en la
diastólica.(R4)

99. Un paciente HIV positivo en tratamiento profiláctico por una neumonía intersticial
que padeció hace dos meses, presenta hipotensión ortostática, hiperglucemia,
hiperazoemia y neutropenia. El motivo más posible de esta clínica será:

1. 1. Diseminación del foco pulmonar con clínica sistémica.

!
!
!
!

2. 2. Sobreinfección por CMV.


3. 3. Infección oportunista por Legionella.
4. 4. Profilaxis errónea con pentamidina vía oral.
Gráfico de respuestas
Comentario

Esta pregunta es difícil, aunque permite repasar algunos conceptos importantes en el tema de la
infección por el VIH y sus enfermedades oportunistas. La clínica que se nos presenta coincide
con los efectos secundarios de la pentamidina. Lo que hay que recordar es que ésta puede
utilizarse por vía endovenosa como tratamiento alternativo de la neumonía por Pneumocystis, y
que en la profilaxis se utiliza en aerosoles, no por vía oral. No obstante, debes saber que es
preferible el TMP-SMX.(R4)

100. A 21 years old patient with a history of


myopia undergoes a fundus photography with these findings. What is true about the
changes in myopic eyes?

1. 1. There is higher risk to develop closed angle glaucoma.


If myopdesopsy occurs, it will probably be due to a posterior vitreous with added retinal
2. 2.
tear.
During cataract surgery, after the removal of the cataract, an intraocular lens of less
3. 3.
graduation is implanted.
4. 4. When photopsia occurs it doesn’t mean any pathological state.
Gráfico de respuestas
Comentario

!
!
!
!

El fondo de ojo que nos presentan corresponde a un paciente con miopía magna y algunos
signos de neovascularización. En cualquier caso, no era necesario que fuera capaz de
interpretarlo. Bastaba la palabra “miopía”, que sale en el propio enunciado.

R1: Los miopes tienen mayor riesgo de glaucoma de ángulo abierto. El de ángulo cerrado se
relaciona con los hipermétropes.

R2: Las miodesopsias pueden producirse, simplemente, por desprendimientos del vítreo
posterior, dato frecuente en el paciente miope, y no necesariamente desgarros.

R4: Las fotopsias deberían hacernos pensar en ciertas complicaciones de la miopía. Puede
tratarse de algo relativamente poco importante, como el desprendimiento del vítreo posterior,
pero también podrían aparecer en un desgarro retiniano.(R3)

101. Mujer de 40 años, con un embarazo y parto normal hace 4 años, que tras 3
laparoscopías para realizar quistectomías ováricas por endometriosis presenta
hipermenorrea y dismenorrea intensa, a pesar de tratamiento con anticonceptivos y
AINE, junto a dolor intermenstrual que le ha obligado a pedir varias permisos laborales.
Además, en los últimos 4 meses presenta disquecia los 2 primeros días de
menstruación. En el USG transvaginal se identifican 2 quistes endometriósicos en
ovario izquierdo (de 3 y 4 cm) y otro en ovario derecho (de 4 cm). ¿Qué opción
terapéutica recomendaría?:

1. 1. Tratamiento con análogos de GnRH.


2. 2. Laparotomía: histerectomía y doble anexectomía.
3. 3. Laparoscopía: quistectomía ovárica bilateral.
4. 4. Colocación de DIU-Levonorgestrel.
Gráfico de respuestas
Comentario

Se presenta un caso de endometriosis refractaria a tratamiento médico y a tratamiento


quirúrgico conservador y que provoca una clínica eventualmente importante por lo que la mejor
opción, dada la edad y deseos genésicos cumplidos, es proponer tratamiento radical:
histerectomía y doble anexectomía.(R2)

102. Se considera el patrón de reactividad en la monitorización fetal no estresante


cuando se cumplen todos los siguientes requisitos, EXCEPTO uno. Indique cuál:

1. 1. Línea de base de la FCF entre 120 y 160 latidos por minuto.


2. 2. Dos o más ascensos transitorios en el registro.
3. 3. El feto se ha movido un mínimo de cinco veces.
4. 4. Aparición de un ritmo saltatorio.
Gráfico de respuestas
Comentario

Pregunta sobre la reactividad fetal y el registro cardiotocográfico que le habrá resultado fácil.

!
!
!
!

Los parámetros de normalidad son una frecuncia cardiaca normal (entre 120 y 160 lpm),
variabilidad normal (de 10 a 25 lpm), presencia de dos o más ascensos transitorios en 20
minutos, movimientos fetales registrados al menos cinco veces.

Un ritmo saltatorio (variabilidad de >25 lpm) significa buena respuesta fetal ante una situación
estresante no muy intensa como una hipoxia leve, hipovolemia o la fiebre materna, por lo tanto,
la opción 4 es la que debemos escoger.(R4)

103. Señale la opción FALSA respecto al tumor de Wilms:

Se asocia a deleción del cromosoma 11, ya sea en las células tumorales o en todas las
1. 1.
células del organismo.
2. 2. Se asocia a aniridia y anomalías genitourinarias.
3. 3. La HTA es un hallazgo frecuente.
4. 4. En el estado inicial es preciso realizar una biopsia.
Gráfico de respuestas
Comentario

El tumor de Wilms es el tumor abdominal más frecuente en los niños y, al originarse a partir de
las células epiteliales de los túbulos renales, es también el tumor renal más frecuente.

La mayoría de los casos se manifiestan inicialmente como una masa abdominal que,
característicamente y a diferencia del neuroblastoma, no cruza la línea media (aunque existen
formas familiares que pueden ser bilaterales). Se descubren metástasis a distancia
(fundamentalmente pulmonares) con cierta frecuencia en el momento del diagnóstico, pero
dichas metástasis pueden responder al tratamiento oncológico.

Al ser un tumor renal, deforma los vasos sanguíneos renales y las vías urinarias, por lo que la
arteriografía renal y la urografía pueden ayudar en el diagnóstico diferencial con otros tumores
abdominales

Recuerde que el diagnóstico definitivo lo da el estudio anatomopatológico, pero que no debe


hacerse biopsia sino resección del tumor completo (con o sin nefrectomía total asociada), ya que
la rotura de la cápsula tumoral aumenta el riesgo de diseminación.

Respuesta correcta 4.(R4)

104. Una mujer de 56 años presenta un cuadro de fiebre con escalofríos. El examen de
orina muestra que su pH es 8.5. El sedimento urinario contiene cristales en ataúd,
presumiblemente de estruvita (MgNH4PO4). En relación con este hallazgo, el
microorganismo más probablemente implicado en su infección del tracto urinario
sería:

1. 1. Escherichia coli.
2. 2. Proteus spp.
3. 3. Klebsiella spp.
4. 4. Pseudomonas fluorescens.
Gráfico de respuestas
Comentario

!
!
!
!

En esta pregunta se resumen las características más importantes de la litiasis infectiva: más
frecuente en mujeres, presencia de cristales de estruvita en forma de ataud, orina alcalina, y
producido por gérmenes ureasa +, siendo el más frecuente el género Proteus. En el tratamiento
se usan el ácido acetohidroxámico y el ácido propiónico.(R2)

105. ¿Cuál de las siguientes pruebas se considera como diagnóstico definitivo del
embarazo?

1. 1. Amenorrea.
2. 2. Aumento del tamaño uterino.
3. 3. Auscultación de latido cardiaco.
4. 4. Signo de Hegar.
Gráfico de respuestas
Comentario

Una pregunta que puede resolver por sentido común, por las siguientes razones:

- La causa más frecuente de amenorrea secundaria es la gestación, pero puede deberse a


muchas otras causas (anovulación crónica, síndrome de Asherman, etc…).

- Una prueba de embarazo se basa en la detección de beta-hCG… Por ello, también puede dar
positiva en la enfermedad trofoblástica, en la gestación ectópica, en tumores productores de
beta-hCG…

- El aumento del tamaño uterino y el signo de Hegar (reblandecimiento del segmento inferior)
pueden verse en la enfermedad trofoblástica, en miomas, etc…

Sin embargo, la presencia de latido cardíaco fetal implica la presencia de un feto viable, por lo
que necesariamente debe tratarse de un embarazo.(R3)

106. Todos los siguientes enunciados sobre la tos ferina son ciertos, EXCEPTO:

1. 1. Algunos pacientes, en especial los lactantes, no tienen gallo respiratorio.


2. 2. Suele afectar a lactantes menores de un año.
3. 3. El síndrome se asocia con infección por Bordetella bronchiseptica en ocasiones.
4. 4. De forma característica hay fiebre en la fase de tos paroxística.
Gráfico de respuestas
Comentario

La aparición de la tos ferina es posible a cualquier edad pero los más afectados son los niños
menores de cinco años. Resto de respuestas son correctas.(R2)

107. Ante una paciente perimenopáusica de 42 años, con cáncer de mama, a la que se
le ha realizado tumorectomía y linfadenectomía, con ganglios positivos para
infiltración tumoral y receptores estrogénicos positivos, ¿qué actitud debemos tomar
a continuación?

1. 1. Radioterapia sólo.

!
!
!
!

2. 2. Radioterapia + tratamiento antiestrogénico.


3. 3. Quimioterapia sólo.
4. 4. Radioterapia + quimioterapia + tratamiento antiestrógenico.
Gráfico de respuestas
Comentario

Caso clínico sencillo sobre el tratamiento del cáncer de mama, que puede resolver si tiene claro
el esquema terapéutico. Tengamos en cuenta que a esta paciente se le ha practicado una
cirugía conservadora (tumorectomía) acompañada de linfadenectomía. Todas las cirugías
conservadoras deben acompañarse de radioterapia, por lo que descartamos la opción 3, que no
incluyen la radioterapia. Dado que los receptores son positivos, añadimos hormonoterapia, así
sólo nos quedan las opciones 2 y 4. Como la linfadenectomía nos informa de ganglios afectos,
es obligado el uso de quimioterapia adyuvante. Por tanto, la opción correcta es la 4(R4)

108. Respecto a las disomnias por movimientos durante el sueño es FALSO:

Una de las disomnias más frecuentes es el síndrome de piernas inquietas, que aparece al
1. 1.
acostarse y solo se calma con el movimiento como en una neuropatía periférica.
El síndrome de piernas inquietas se asocia con el embarazo, la anemia, la insuficiencia
2. 2.
renal y el mioclonus nocturno.
3. 3. El mioclonus nocturno es otra disomnia que aparece en las fases 1 y 2 del sueño.
Aunque el origen de las mioclonías nocturnas se desconoce, responden a clonacepam o L-
4. 4.
Dopa.
Gráfico de respuestas
Comentario

El síndrome de las piernas inquietas es un trastorno neurológico que repercute mucho sobre la
calidad del sueño. El paciente padece sensaciones desagradables en las piernas (hormigueo,
frío, calor e incluso dolor), normalmente en la zona de la pantorrilla. Cuando experimentan estas
sensaciones, padecen la necesidad irresistible de mover las piernas. En algunos casos, sólo
cursa con necesidad de movimiento, sin sensación desagradable asociada, aunque esto es
poco habitual.
La respuesta falsa sería la 1. En una neuropatía periférica, cuando es de tipo sensitivo (lo más
frecuente), se producen molestias tipo hormigueo en las zonas más distales de las extremidades
(manos y pies), sensaciones que difícilmente se ven influenciadas por el movimiento.(R1)

109. Una paciente de 60 años de edad consulta por cefalea, pérdida brusca de visión y
claudicación mandibular. A la exploración presenta la arteria temporal derecha
engrosada, dolorosa y sin pulso. Junto a este cuadro clínico, señale cuál de los
siguientes hallazgos de laboratorio cree que tengan la mayor especificidad
diagnóstica:

1. 1. Elevación de la fosfatasa alcalina.


2. 2. Elevación de la velocidad de sedimentación globular.
3. 3. Hipergammaglobulinemia IgG.
4. 4. Hipocomplementemia.
Gráfico de respuestas
Comentario

!
!
!
!

Esta pergunta no se le deberia pasar, pues es un caso típico de arteritis de temporal, también
conocida como arteritis de células gigantes. Llegamos al diagnóstico por los siguientes datos
característicos:

- Cefalea en persona mayor.

- Pérdida brusca de visión.

- Claudicación mandibular.

- Arteria temporal engrosada y sin pulso a la palpación.

Recuerde que es característico de esta enfermedad una gran elevación de la VSG (muchas
veces superior a 100 mm/h), aunque puede ser normal hasta en un 20% de pacientes.(R2)

110. Recién nacido de 36 semanas de gestación, con 7 días de vida, que desde el
tercero presenta una ictericia que ha ido en aumento. La madre es primigesta, tiene un
grupo sanguíneo A (Rh negativo) y el niño es O (Rh positivo). El 7° día tiene una
bilirrubina total de 12 mg/dL, a expensas de la fracción indirecta. El niño tiene buen
estado general y los valores de hematocrito, hemoglobina y reticulocitos son
normales. ¿En qué causas de hiperbilirrubinemia entre las siguientes hay que pensar
en primer lugar?:

1. 1. Ictericia fisiológica.
2. 2. Hepatitis neonatal.
3. 3. Atresia de vías biliares.
4. 4. Enfermedad hemolítica ABO.
Gráfico de respuestas
Comentario

Las características de la ictericia no fisiológica son las siguientes:

• Inicio en las primeras 24 horas de vida.

• Duración superior a 10-15 días.

• Bilirrubina total mayor de 12 mg/dl en RNT o mayor de 15 mg/dl en

RNPT.

Incremento de la bilirrubina superior a 5 mg/dl/24 horas.

• Bilirrubina directa mayor a 1 mg/dl o superior al 20 % de la bilirrubina

total.

!
!
!
!

La ictericia fisiológica nunca comienza el primer día de vida, ni se prolonga más allá de los días
10-15. Cursa con buen estado general. La billrrubina no supera los 12 mg/dl en los RNT ni los
14 mg/dl en los RNPT.

En los RNPT, la ictericia fisiológica suele ser de inicio algo más tardío que en el RNT, y también
habitualmente es algo más prolongada, así como también puede normalmente alcanzar niveles
más altos.(R1)

111. El manejo de un recién nacido con hipoglicemia sintómatica:

1. 1. Dextrosa 10% VO 10 mg/kg/dosis.


2. 2. Fórmula artificial del 1er semestre 80 ml/kg/dosis dividido cada 3 horas.
3. 3. Dextrosa al 10% 2ml/kg vía IV.
4. 4. Proporcionar un glucosa 5% a 5 mg/kg/min.
Gráfico de respuestas
Comentario

En el caso de hipoglucemia < 45mg/dl, sintomática, el primer paso es administrar un bolo IV de


suero glucosado 10% a 2 ml/kg a pasar en 2 minutos. Luego una infusión de glucosa IV a 4-6
mg/kg/min y monitorización horaria.(R3)

112. Paciente de 69 años, sin revisiones ginecológicas recientes, que acude a


urgencias por molestias abdominales. En la exploración se palpa masa pélvica de
aproximadamente 9 cm. En el USG transvaginal se observa imagen quística
multiloculada en anexo derecho, de 91 x 78 mm, con septos en su interior y vasos con
IR doppler de 0.3. Marcadores tumorales: Ca 125: 65 U/ml, Ca 19.9: 3 U/ml. ¿Cuál sería
la actitud más CORRECTA en este caso?:

Ampliar el estudio solicitando citología y SCC (antígeno de crecimiento de células


1. 1.
escamosas).
2. 2. Realizar nuevo USG y lboratorio analítica de control en 3 meses.
Realizar analítica completa, ampliando el estudio con otros marcadores tumorales (alfa-
3. 3.
fetoproteína, BHCG) y estudio de hormonas tiroideas.
4. 4. Cirugía: lavado peritoneal, anexectomía derecha y biopsia intraoperatoria de la lesión.
Gráfico de respuestas
Comentario

Cuando existe una masa ovárica altamente sospechosa de malignidad mediante pruebas de
laboratorio y/o de imagen, es precisa la confirmación diagnóstica mediante estudio
anatomopatológico. Por este motivo, la respuesta correcta es la número 4. La citología cervical y
el SCC sirven fundamentalmente para el diagnóstico de patología de cérvix. En el caso de
tumoraciones ováricas de gran tamaño y sugestivas de malignidad, no se realiza control
periódico, que se reserva para tumoraciones de menor tamaño y de características benignas. En
este caso, al ser una tumoración sospechosa de malignidad, es necesario el manejo quirúrgico,
con biopsia intraoperatoria de la lesión, para valorar la necesidad o no de completar el protocolo
quirúrgico de cáncer de ovario en caso de confirmación diagnóstica. Recuerde que la cirugía en
el cáncer de ovario tiene una doble finalidad: diagnóstica y de estadificación.(R4)

113. Los siguientes datos histológicos son característicos de: Inflamación glomerular
mediada por células inflamatorias infiltrantes en el glomérulo y en el intersticio renal.

!
!
!
!

Proliferación de células mesangiales, rotura del glomérulo hacia el espacio de


Bowman, formación de semilunas en más de un 50%. Cilindros eritrocitarios
intratubulares:

1. 1. Nefropatía tubulointersticial crónica.


2. 2. Nefropatía de cambios mínimos.
3. 3. Glomerulonefritis aguda (posiblemente rápidamente progresiva).
4. 4. Síndrome de Alport.
Gráfico de respuestas
Comentario
Pregunta de dificultad media en la que nos presentan lo que podría ser perfectamente un
informe de un servicio de anatomía patológica de una biopsia renal que nosotros debemos
interpretar, siempre ayudados por una buena tabla de diagnóstico diferencial en GN. En
resumen nos están diciendo que al microscopio óptico se visualiza una proliferación endocapilar,
es decir, una proliferación endotelial y mesangial difusa. Asimismo describen inflamación, que
podría considerarse el elemento exudativo de la GN aguda (RESPUESTA 4 CORRECTA). Nos
añaden la presencia de semilunas, que es el componente anatomopatológico que traduce la
rápida progresión (mal pronóstico).(R3)

114. Son factores favorecedores de la formación de callo óseo todos los siguientes,
EXCEPTO:

1. 1. Glucocorticoides
2. 2. Hormona de crecimiento.
3. 3. Hormona tiroidea.
4. 4. Vitamina D.
Gráfico de respuestas
Comentario

Recuerde que los glucocorticoides, así como otras drogas antiinflamatorias (indometacina),
retrasan la consolidación del callo óseo (recuerde que además, los corticoides producen
osteoporosis).(R1)

115. La ingesta de AINES puede afectar al riñón de diferentes formas. Es excepcional


que de lugar a:

1. 1. Nefritis por hipersensibilidad.


2. 2. Glomerulonefritis por cambios mínimos.
3. 3. GN extracapilar.
4. 4. Uroteliomas
Gráfico de respuestas
Comentario
Pregunta sencilla sobre AINES y riñón. Se han asociado al desarrollo de nefritis intersticial
inmunoalérgica y GNCM pero no al desarrollo de GNRP con semilunas. El consumo crónico se
ha asociado a la nefropatía crónica por analgésicos y aumentan el riesgo de padecer
uroteliomas.(R3)

!
!
!
!

116. El test de adaptación auditiva es un:

1. 1. Audiometría tonal liminar.


2. 2. Audiometría supraliminar.
3. 3. Reflejo estapedial.
4. 4. Timpanometría.
Gráfico de respuestas
Comentario

La audiometría supraliminar se utilizan para estudiar las distorsiones de la sensación sonora:

- La distorsión de sensación de intensidad (por ejemplo el reclutamiento) se estudia con el


test de Metz, la prueba de Fowler y el SISI.

- La distorsión de la sensación de duración (por ejemplo fatiga y adaptación patológica) se


estudia con el Tone Decay Test. Las hipoacusias cocleares presentan reclutamiento; las
retrococleares, fatiga y adaptación patológica, mientras que las de transmisión no presentan
ninguno de los fenómenos anteriores.(R2)

117. ¿Cómo se denomina el tumor androgénico que deriva de la médula ovárica?:

1. 1. Androblastoma.
2. 2. Gonadoblastoma.
3. 3. Meduloblastoma.
4. 4. Tumor de la teca-fibroma.
Gráfico de respuestas
Comentario

Esta pregunta sobre tumores ováricos no tiene mucha relevancia.

Los tumores de cordones - estroma normalmente producen hormonas que ayudan a


diagnosticarlos. El 50% son androblastomas o tumores de celulas de Sertoly-Leydig. Recuerde
que es la primera causa de virilización de origen ovárico. Los tumores de la teca-fibroma
producen estrógenos y andrógenos, los de la granulosa estrógenos. Los gonadoblastomas son
muy poco frecuentes y aparecen casi exclusivamente en individuos con disgenesia gonadal.
Contienen zonas semejantes al disgerminoma y elementos del estroma gonadal y derivan de la
célula germinal. El meduloblastoma es tumor cerebral maligno más frecuente en la infancia.(R1)

118. Masculino de 23 años que trabaja en una tienda de mariscos, se pico con una
espina de pescaso y 5 días después presenta una mácula eritematosa, bien definida,
en dorso de mano derecha. Presenta dolor a la presión y en el cultivo de la lesión se
encuentran bacilos gram positivos. Señale la opción CORRECTA:

1. 1. El tratamiento se fundamenta en la administración de eritromicina.


2. 2. No suelen existir adenopatías regionales.
3. 3. Los pacientes desarrollan inmunidad frente a la infección.
4. 4. El tratamiento de elección es con penicilina oral.
Gráfico de respuestas

!
!
!
!

Comentario
Pregunta sencilla sobre el erisipeloide. El antecedente de pinchazo/herida con espina de
pescado, en este caso con un marisco, es típico de esta patología (se ve en pescaderos, pero
también en matarifes y ganaderos). También lo es la localización en dorso de manos, que no
suele progresar más allá de la muñeca. Esta enfermedad es producida por un bacilo gram
positivo denominado Erisipelthrix rusiopathiae, su tratamiento de elección es la penicilina.(R4)

119. De las siguientes opciones, todas son contraindicaciones absolutas para el uso
de anticonceptivos orales, EXCEPTO una. Señálela:

1. 1. Embarazo confirmado, o sospecha del mismo.


2. 2. Dismenorrea.
3. 3. Antecedentes personales de cáncer de mama.
4. 4. Antecedentes de colestasis durante un embarazo.
Gráfico de respuestas
Comentario

Pregunta sencilla sobre los anticonceptivos orales. Realmente está clara la indicación en casos
de dismenorrea, en que controlan y regulan el ciclo menstrual de la paciente. En el resto de
casos es clara la contraindicación. Recuerde especialmente la relación de los ACO con el
tromboembolismo, ya que este es un aspecto importante.(R2)

120. Paciente de 55 años que refiere hipoacusia progresiva de oído derecho desde hace
1.5 años junto con sensación leve de inestabilidad y acúfeno en dicho oído. La
exploración muestra tímpanos normales y la audiometría tonal indica una hipoacusia
derecha moderada severa. ¿Qué actitud tomaría?

1. 1. Control evolutivo.
2. 2. Realizaría una resonancia magnética para descartar un neurinoma.
3. 3. Le indicaría la adaptación de prótesis auditiva.
4. 4. Le indicaría cirugía otológica de otosclerosis.
Gráfico de respuestas
Comentario

El neurinoma del acústico es el tumor más frecuente del ángulo bulbopontocerebeloso.


Clínicamente, cabría sistematizar los signos y síntomas en función de la fase de crecimiento:

•! Fase intracanalicular: hipoacusia neurosensorial unilateral, con predominio en tonos


agudos y pérdida de la inteligibilidad. Habría también pérdida de la discriminación tonal-
verbal, como corresponde a una hipoacusia neurosensorial retrococlear. Tal como
sucede en nuestro paciente, es típica la aparición de un acúfeno unilateral y síntomas
vestibulares (sobre todo inestabilidad, más que un vértigo franco, ya que el lento
crecimiento del tumor permite la compensación central).

•! Fase cisternal: ocupa el ángulo pontocerebeloso pero sin comprimir el tronco. Afecta a
pares craneales vecinos: trigémino, facial y pares bajos (IX, I, XI, XII), por este orden de
frecuencia.

!
!
!
!

•! Fase compresiva: comprime el tronco e incluso el cerebelo. Clínicamente, se manifiesta


como hipertensión intracraneal con síndrome cerebeloso.

Para el diagnóstico definitivo, el método de elección sería la RMN con gadolinio. No obstante,
antes de recurrir a ella, suele recurrirse a pruebas auditivas, vestibulares y los PTEAC. Si hs
leído con atención, verás que la descripción del caso coincide con la fase intracanalicular.(R2)

121. La presencia de placas hialinas pleurales, calcificadas en forma de semiluna sobre


las cúpulas diafragmáticas:

1. 1. Indica necesariamente la existencia previa de exposición al asbesto.


2. 2. Es sugestiva de exposición al asbesto.
3. 3. Es indicación de pleurectomía.
4. 4. Degenera posteriormente a mesotelioma maligno difuso de la pleura.
Gráfico de respuestas
Comentario

La placas pleurales (calcificaciones de la pleura parietal) no son específicas de exposición al


asbesto; también aparecen en la TB, hemorragias o conectivopatías. Sugieren exposición al
asbesto en pacientes con antecedentes epidemiológicos compatibles. No producen síntomas, no
aumentan el riesgo de mesotelioma y no requieren tratamiento.(R2)

122. Señale la conducta ante un lactante de 5 meses, con vómitos frecuentes (6-8
diarios) desde los primeros días de vida, que han disminuido de frecuencia desde la
introducción de los cereales. Peso y talla en percentiles estables en todo momento,
con buen apetito y estado general:

1. 1. Tránsito digestivo superior en busca de una hernia de hiato.


2. 2. pHmetría intraesofágica de 24 horas para documentar el reflujo.
3. 3. Manometría esofágica para cuantificar el tono del esfínter esofágico inferior.
4. 4. Observación y control evolutivo esperando una remisión espontánea.
Gráfico de respuestas
Comentario

Se trata de un caso de reflujo gastroesofágico no complicado, con regurgitaciones frecuentes sin


repercusión sobre el estado y el desarrollo del niño y sin otros síntomas asociados. En estos
casos, no se requieren pruebas complementarias y la evolución es hacia la curación, mejorando
con medidas posturales y dietéticas. El esofagograma con bario es la prueba más accesible, que
permite descartar anomalías anatómicas, careciendo de buena sensibilidad y especificidad para
el diagnóstico de reflujo. La pHmetría esofágica de 24 horas es la prueba más sensible y
específica para el diagnóstico de reflujo, pero es una prueba invasiva y menos disponible. Su
utilidad queda relegada para casos graves, con importante sintomatología extradigestiva o como
control de la eficacia del tratamiento. La endoscopia es la prueba de elección ante la sospecha
clínica de esofagitis (rechazo del alimento, irritabilidad, vómitos con sangre…). La manometría
es útil cuando se sospechan trastornos motores esofágicos.(R4)

123. El tratamiento de elección en el momento actual para el tratamiento de una


esterilidad derivada de un SOP es:

1. 1. Resección en cuña.

!
!
!
!

2. 2. Citrato de clomifeno.
3. 3. Gonadotropinas.
4. 4. Inductores de la aromatasa.
Gráfico de respuestas
Comentario

Esta pregunta es muy importante porque debe conocer perfectamente el síndrome de ovario
poliquístico y sus alteraciones hormonales y dirigir el tratamiento en una paciente con deseos
genésicos como es esta paciente, debe leer muy bien el en examen lo que le preguntan. Es una
afección muy frecuente en que está aumentada la LH con niveles de FSH bajos o inferiores a los
normal, por lo que aumenta la relación LH/FSH. Hay un aumento leve de andrógenos, aumento
de la estrona (los andrógenos circulantes son convertidos a estrona en la grasa periférica) y
descenso del estradiol. La LH aumentada estimula la teca, produciendo más andrógenos
(también hay una sobreproducción suprarrenal) que va a provocar obesidad, hirsutismo y
anovulación. Existe insulino resistencia. La tendencia actual para el tratamiento de la esterilidad
en el SOP es usar como primera opción clomifeno y metformina. Si falla usar gonadotropinas
como 2º opción y , si nuevamente fracasamos, destrucción parcial del ovario por vía
laparoscópica.(R2)

124. Masculino de 60 años tiene un síndrome nefrótico, sin evidencia de enfermedad


sistémica. ¿Qué proceso glomerular es el más probable de su síndrome nefrótico
idiopático?:

1. 1. Amiloidosis.
2. 2. GN membranosa.
3. 3. GN membranoproliferativa.
4. 4. Hialinosis focal y segmentaria.
Gráfico de respuestas
Comentario

Sencilla pregunta acerca de la epidemiología de las GN. Si la ha fallado, por favor, repase el
tema y los desgloses. LA RESPUESTA CORRECTA ES LA 2, ya que la GN membranosa es la
causa más frecuente de síndrome nefrótico en adultos. No obstante, algunos autores apuntan
que comienza a ser más frecuente en algunas series la nefropatía diabética. Recordemos que
TODO síndrome nefrótico en el adulto se debe biopsiar, a diferencia del niño. No olvide que si
nos hubieran hecho la misma pregunta pero en la infancia, la respuesta hubiera sido la GN de
cambios mínimos.(R2)

125. A 78-year-old woman comes to your office complaining of acute loss of vision in
her left eye. In the last 3 weeks she has presented low-grade fever, pain in shoulders,
neck and hips as well as moderate headache. The fundus examination reveals a pale
and edematous optic disc. Mobilization of shoulders and hips is very painful. No
abnormalities were observed on palpation of the temporal arteries. Lab tests show
hemoglobin 9.7 g / dl, ferritin 450 ng / ml, ESR 115 mm / h. What is the most appropriate
immediate action in this patient?

1. 1. Start treatment with prednisone 60 mg daily and aspirin 100 mg daily.


2. 2. Start treatment with prednisone 10 mg daily.
3. 3. Biopsy of the temporal artery.
4. 4. Request a brain MRI.
Gráfico de respuestas

!
!
!
!

Comentario

Pregunta sencilla sobre arteritis de la temporal. Se trata de un adulto mayor con clínica dolorosa
en hombros, fiebre y cefalea, posteriormente se agrega pérdida de la agudeza visual, asociado a
un disco óptico pálido y edematoso. En los exámenes de laboratorio destaca una anemia de
procesos crónicos, con ferritina elevada y una VSG muy elevada. Recuerde que la primer
maniobra terapéutica es la administración de corticoides a altas dosis, apra evitar el daño óptico,
posteriormente se podrá realizar la biopsia de la arteria temporal.(R1)

126. Paciente de 50 años que presenta en la radiografía de tórax un patrón intersticial


instaurado tras una clínica de disnea de meses de duración. Al no realizarse
diagnóstico se le practica una broncoscopía con biopsia transbronquial en la que,
entre otras lesiones, se muestran granulomas. El lavado broncoalveolar demuestra un
predominio de linfocitos (60%) con un predominio de los linfocitos T supresores
citotóxicos (CD8). El diagnóstico más probable es de:

1. 1. Neumonía intersticial linfocitaria.


2. 2. Neumonitis por hipersensibilidad.
3. 3. Sarcoidosis.
4. 4. Tuberculosis miliar.
Gráfico de respuestas
Comentario

Pregunta muy típica sobre la alveolitis alérgica extrínseca (neumonitis por hipersensibilidad). El
patrón característico del lavado broncoalveolar podría ser preguntado, y es contrario al de la
sarcoidosis, que siempre aparece entre los posibles diagnósticos diferenciales.

•! Neumonitis por hipersensibilidad: predominio de los CD8 sobre los CD4.


•! Sarcoidosis: predominan los CD4 sobre los CD8.

(R2)

127. Regarding the squamous-columnar junction of the cervix, which of the following
statements is CORRECT?

1. 1. It is located in the upper endocervical canal before puberty.


2. 2. Transformation zone is where the transitional epithelium becomes squamous.
3. 3. It is the least common site of cervical carcinoma.
4. 4. It often shows squamous metaplasia.
Gráfico de respuestas
Comentario

La unión escamocolumnar está localizada en el punto donde el epitelio plano y el epitelio


cilíndrico se encuentran. La localización de este punto varía durante la vida de la mujer debido a
los cambios metaplásticos en el epitelio cervical que ocurren después de la pubertad y durante
el embarazo. La zona de transformación es el nombre asignado al área del cuello uterino
compuesta de epitelio que ha sufrido cambio metaplástico, a menudo escamoso.(R4)

!
!
!
!

128. ¿A partir de qué semana del embarazo puede verse por USG los movimientos
cardiacos fetales?

1. 1. 5ª semana.
2. 2. 7ª semana.
3. 3. 10ª semana.
4. 4. 14ª semana.
Gráfico de respuestas
Comentario

Pregunta sobre el USG del primer trimestre. Esta pregunta no tiene ninguna forma de ser
razonada, y sólo podemos sacarla si sabemos un poco de la cronología del ultrasonido del
primer trimestre. Podemos ver el saco gestacional al final de la 5ª semana por vía vaginal y al
final de la 6ª por vía abdominal. Con el latido cardíaco pasa algo similar; así, lo veremos a la 6ª
semana por vía vaginal y a la 7ª por vía abdominal. Por lo tanto, la opción 2 es la correcta.(R2)

129. ¿Qué lesión sospecharía ante un traumatismo abdominal cerrado con signos de
hemorragia e irritación peritoneal?:

1. 1. Rotura vesical.
2. 2. Rotura esplénica.
3. 3. Perforación intestinal.
4. 4. Fractura de pelvis.
Gráfico de respuestas
Comentario

El órgano más frecuentemente lesionado en traumatismos abdominales no penetrantes es el


bazo. Clínicamente se observan signos generales de hemorragia y locales de irritación
peritoneal en el área esplénica. En raros casos (<5%) puede haber una rotura esplénica diferida
por un hematoma eventualmente contenido por la cápsula, manifestándose generalmente dentro
de la primera semana tras el trauma. El diagnóstico se establece por USG o TC. Si el paciente
está hemodinámicamente inestable, la punción-lavado peritoneal es indicación de cirugía sin
dilación. El tratamiento en ausencia de lesiones significativas y de hemorragia persistente puede
ser consevador (esplenorrafia), reservando la esplenectomía para lesiones extensas del
parénquima.(R2)

130. ¿Cuál es el tratamiento de elección de la hipercalciuria idiopática?

1. 1. Furosemida.
2. 2. Restricción de calcio en la dieta.
3. 3. Hidroclorotiazida.
4. 4. Calcitonina.
Gráfico de respuestas
Comentario

Pregunta directa sobre las urolitiasis acerca del tratamiento de elección de la hipercalciuria
idiopática.

!
!
!
!

•! R1: la furosemida es un diurético de asa y, como tal, aumenta la excreción renal de


calcio, así pues no estaría indicada.
•! R2: se trata de una hipercalciuria idiopática, de modo que la reducción de la ingesta de
calcio no será suficiente.
•! R3: respuesta correcta. Las tiacidas disminuyen el calcio urinario y están aconsejadas
en la hipercalciuria idiopática.
•! R4: la calcitonina favorece la entrada de Ca en el hueso y su reabsorción renal. No
estaría, pues, indicada.

(R3)

131. Todas las siguientes enfermedades, EXCEPTO una, afectan preferentemente a los
campos pulmonares inferiores. Señale de cuál se trata:

1. 1. Enfisema panacinar.
2. 2. Fibrosis pulmonar por artritis reumatoide.
3. 3. Esclerosis sistémica progresiva.
4. 4. Histiocitosis X.
Gráfico de respuestas
Comentario

Las enfermedades expuestas afectan en mayor medida a los campos pulmonares inferiores,
menos la Histiocitosis X que lo hace en los superiores. En este sentido, recuerda la regla
nemotécnica FITNESS (F. quística, histiocitosis X, tuberculosis, neumonía por hipersensibilidad,
espondilitis anquilosante, silicosis, sarcoidosis), que recoge las principales patologías que
afectan a los ápices pulmonares.(R4)

132. La parálisis facial da lugar a:

1. 1. Entropion.
2. 2. Lagoftalmos.
3. 3. Blefaroesclerosis.
4. 4. Triquiasis.
Gráfico de respuestas
Comentario

Esta pregunta sobre la parálisis facial es de baja dificultad y debe saberse porque es bastante
preguntado en el ENARM. Recuerde que la parálisis facial produce en reposo, una parálisis
motora de la hemicara con desviación de la comisura bucal al lado sano y babeo en el lado de la
lesión. La piel del lado paralizado aparece notablemente tensa, sin arrugas. El párpado inferior
cae produciendo el cierre incompleto del párpado, fenómeno que se conoce como lagoftalmos.
Al intentar cerrar el ojo, el globo ocular del lado paralizado se desvía hacia arriba y ligeramente
hacia dentro (fenómeno de Bell).(R2)

133. ¿En cuál de las siguientes, el diagnóstico de hemorragia uterina disfuncional es


inapropiado?

1. 1. Pólipo endometrial.
2. 2. Endometritis por tuberculosis.
3. 3. Mioma submucoso.

!
!
!
!

4. 4. Cualquiera de las anteriores.


Gráfico de respuestas
Comentario

El sangrado uterino disfuncional es la aparición de sangrado uterino no relacionado con


anomalías estructurales del útero o del endometrio. Es un diagnóstico de exclusión realizado
luego descartar las causas estructurales de sangrado y las enfermedades médicas crónicas. La
respuesta correcta es la 4.(R2)

134. Cuando observa un pulso arterial que descienda más de 10 mmHg durante la
inspiración, pensará que la causa más probable es:

1. 1. Crisis asmática.
2. 2. Pericarditis constrictiva.
3. 3. Enfermedad pulmonar obstructiva agudizada.
4. 4. Taponamiento cardiaco.
Gráfico de respuestas
Comentario

Pregunta de baja dificultad sobre el pulso paradójico. Consiste en la disminución en más de 10


mmHg de la presión sistólica con la inspiración profunda. Es la exageración de un fenómeno
fisiológico por el abombamiento del septo interventricular hacia el VI al aumentar el llenado del
VD, cuya pared lateral no puede distenderse. El ejemplo más típico es el taponamiento cardíaco,
pero también puede aparecer en otras patologías como obstrucciones bronquiales graves,
tromboembolismo pulmonar, obstrucción de la cava superior, pericarditis constrictiva, etc...(R4)

!
!
!
!

135. Niño de 6 años acude a urgencia por fiebre


de 6 días de evolución, exantema como el que se muestra en la imagen, lengua
aframbuesada y adenopatía cervical. ¿Cuál es el diagnostico más probable?

1. 1. Faringoamigdalitis por estreptococo grupo A.


2. 2. Infección por parvovirus B19.
3. 3. Enfermedad de Kawasaki.
4. 4. Sarampión.
Gráfico de respuestas
Comentario

La enfermedad de Kawasaki está bastante de moda, tal vez por la implicación de mecanismos
inmunológicos en su patogenia, ya que la inmunología también se pregunta cada vez con más
frecuencia.

La enfermedad de Kawasaki se define por una serie de criterios clínicos. Como puedes ver, se
adaptan bastante bien a lo que nos cuentan en esta pregunta (respuesta 3 correcta).(R3)

!
!
!
!

136. Tras realizar exámenes complementarios, ¿qué esperaría encontrar?


1. 1. Leucocitosis con linfocitosis y linfocitos atípicos.
2. 2. Leucopenia, acompañada de neutropenia, trombopenia y anemia.
3. 3. Aumento de PCR, leucocitosis con desviación izquierda y aumento de transaminasas.
4. 4. Cultivo faríngeo para SGA positivo.
Gráfico de respuestas
Comentario

Desde el punto de vista en la enfermedad de Kawasaki los hallazgos de laboratorio, es


frecuente encontrarnos con una leucocitosis con desviación izquierda. También es habitual
encontrarnos con trombocitosis y anemia. Por otra parte, la VSG y la proteína C reactiva suelen
estar muy elevadas. En cuanto a los niveles de transaminasas y bilirrubina, pueden estar
ligeramente elevados, secundario a una afectación hepática leve, en la mayoría de los casos sin
ninguna traducción clínica(R3)

137. Constituyen factores de riesgo para síndrome de aspiración meconial, todo lo


siguiente, EXCEPTO:

1. 1. Prematuridad.
2. 2. Hipertensión inducida por el embarazo.
3. 3. Restricción de crecimiento intrauterino.
4. 4. Embarazo prolongado.
Gráfico de respuestas
Comentario

Factores de riesgo para SAM.

1. Hipoxia aguda intraparto.

2. Hipoxia perinatal crónica.

3. Frecuencia cardiaca fetal anormal.

4. Recién nacidos pequeños para la edad gestacional.

5. Perfil biofísico igual o menor a 6.

6. Historia materna:

a) Embarazo prolongado.

b) Preeclampsia – eclampsia.

c) Hipertensión materna.

d) Diabetes mellitus materna.

!
!
!
!

e) Tabaquismo importante, enfermedad respiratoria crónica o enfermedad cardiovascular de la


madre.

f) Hemorragia materna, hipotensión.

g) Abruptio placentario.

h) Insuficiencia placentaria crónica.

El ser prematuro no es factor de riesgo.

(R1)

138. De los siguientes anticuerpos, señale los más prevalentes en el síndrome de


Sjögren primario:

1. 1. Anti-Ro y anti-La.
2. 2. Anti-Ro y anti-Sm.
3. 3. Anti-RNP y anti-La.
4. 4. Anti-Scl 70.
Gráfico de respuestas
Comentario

Los anticuerpos son bastante preguntados en el ENARM, y aunque el síndrome de Sjögren no


es de los más importantes, sí debe conocerlos: son el anti SS-A y anti SS-B o también
conocidos como anti-Ro y anti-La los que más se han asociado al síndrome de Sjögren, y en
concreto a las formas primarias, que con mayor frecuencia que las secundarias presentan
positividad para estos anticuerpos. Recuerde que los anti-DNA se presentan en muchas
entidades: LES, ESP, EMTC; los anti-Scl-70 o antitopoisomerasa I son propios de la
esclerodermia difusa; los anti-RNP son imprescindibles en el diagnóstico de enfermedad mixta
del tejido conectivo; los anti-Sm y anti- DNAds son bastante específicos del LES. Tenga en
cuenta que los anti-SS-A y SS-B también pueden aparecer en diversas formas de lupus.(R1)

139. Mujer de 27 años con dolor en hipogastrio y sangrado genital escaso. La prueba
de embarazo es negativo y en los exámenes de laboratorio destacan unos leucocitos
de 11,500/mm3. En la exploración, el útero está ligeramente aumentado de tamaño y
existe hipersensibilidad anexial. ¿Cuál es el primer diagnóstico en el que pensaría?:

1. 1. Amenaza de aborto.
2. 2. Apendicitis aguda.
3. 3. Gestación ectópica.
4. 4. Enfermedad inflamatoria pélvica.
Gráfico de respuestas
Comentario

Cuadro clásico de EIP.

Mujer jóven con dolor abdominal, leucocitosis, hipersensibilidad anexial, con prueba de
embarazo negativa.(R4)

!
!
!
!

140. Acude a su consulta un paciente de 33 años por tos seca escasa en el último mes.
En la exploración encuentra unas lesiones rojo vinosas en la cara anterior de ambas
piernas, y la radiografía de tórax muestra un patrón intersticial con adenopatías hiliares
bilaterales. La espirometría es normal, incluyendo la determinación de DCLO. ¿Cuál es
el tratamiento más adecuado para la patología que presenta?:

1. 1. Azatioprina.
2. 2. Cloroquina.
3. 3. Colchicina.
4. 4. Observación periódica.
Gráfico de respuestas
Comentario

Se trata de una sarcoidosis estadio radiológico II con muy poca clínica y exploración funcional
normal. Por tanto, lo indicado es observación periódica, realizando radiografía de tórax,
espirometría con difusión y, eventualmente, prueba de la marcha, pues sólo si aparece clínica o
deterioro funcional en el seguimiento está indicado tratar con corticoides la sarcoidosis torácica
estadio II o superior.(R4)

141. Las tricomonas se transmiten por contacto directo, y no lo hacen a través de


fómites, como toallas, artículos de aseo, etc. Esta característica es debida a que:

1. 1. No forman quistes, por lo que no son viables en el medio exterior.


2. 2. No afectan clínicamente la uretra del hombre y por ello el inóculo es mínimo.
3. 3. Son inmóviles y no alcanzan el cérvix vaginal.
4. 4. Son anaerobias estrictas y mueren por acción del oxígeno.
Gráfico de respuestas
Comentario

Las infecciones vulvovaginales se preguntan con cierta frecuencia, pero no hasta este nivel de
profundidad, que nos exige conocer detalles muy específicos del microorganismo en cuestión.

Trichomonas vaginalis es un protozoo que puede trasmitirse sexualmente. Sin embargo, esta
pregunta podría considerarse mal planteada, ya que lo que dice el enunciado no es cierto.
Aunque de forma excepcional, se ha descrito la transmisión de esta enfermedad a través de
fómites (artículos de baño, prendas de vestir), aunque efectivamente este mecanismo está
limitado por la escasa viabilidad de los trofozoítos, ya que no forma quistes.(R1)

142. A 2-year-old child is brought to the pediatrician's office by her worried mother who
reports that the child is not startled by loud noises and she seems to be a slow learner.
She says he only notices her presence when she sees her, but never when she calls
her name. The girl had severe meningitis during the first month of her life, which was
cured with antibiotherapy. Which would be the next step in the management of this
child?

1. 1. Audiology evaluation.
2. 2. Schedule a psychiatrist consultation.
3. 3. Reassure the parents: this is a kid with normal development.
4. 4. Perform an MRI of the brain.

!
!
!
!

Gráfico de respuestas
Comentario
Correct: Audiology evaluation. This girl has experienced several risk factors for deafness.
Meningitis at an early stage of life is a potential cause, ototoxic antibiotics are another. The first
step is to make an adequate diagnosis and quantification of the severity by audiology evaluation.
A brain MRI is not useful because the girl does not present added neurological deficits that would
make us suspect a brain tumor.(R1)

143. ¿Cuál de las siguientes afirmaciones sobre los anticuerpos antinucleares es


CORRECTA?

Los pacientes con lupus inducido por fármacos suelen tener anticuerpos antihistonas y
1. 1.
anti-DNA de doble cadena positivos.
Los anticuerpos anti-DNA de cadena única son específicos del lupus eritematoso
2. 2.
diseminado.
El título de anticuerpos anti-DNA de doble cadena puede guardar correlación con la
3. 3.
actividad de la glomerulonefritis lúpica.
La positividad de los anticuerpos anticentrómero prácticamente descarta el diagnóstico de
4. 4.
esclerosis sistémica (esclerodermia).
Gráfico de respuestas
Comentario

Es una pregunta muy fácil con respecto a los autoanticuerpos que debe conocer. Los anti-DNA
doble cadena son los específicos del lupus. El lupus inducido por fármacos no los positiviza; de
hecho, en el lupus inducido por fármacos (procainamida, quinidina, hidralacina, d-
penicilamina…) son los antihistona, y no los anti-Sm, los que se positivizan en el lupus inducido
por fármacos. Los anticuerpos anti-DNA se correlacionan con la actividad de la enfermedad y la
presencia de nefritis. En la tabla siguiente aparecen los anticuerpos asociados al lupus.(R3)

Autoanticuerpos

!
!
!
!

144. En el registro cardiotocográfico, la variabilidad (oscilación de la línea de base)


muestra el estado del sistema nervioso vegetativo. ¿Cuál de los siguientes registros
supone un PEOR pronóstico?

1. 1. Sinusoidal.
2. 2. Normal.
3. 3. Baja.
4. 4. Silente.
Gráfico de respuestas
Comentario

Pregunta sobre la monitorización fetal no estresante de cierta dificultad. La frecuencia cardíaca


fetal está regulada por el sistema nervioso autónomo (simpático y parasimpático), y nos
determinará la variabilidad. Debemos conocer los valores de dicha variabilidad. Es normal si es
de 10 a 25 lpm. Baja si es de 5 a 10 lpm; su significado es prepatológico. La saltatoria (>25 lpm)
significa buena respuesta fetal ante una situación estresante no muy intensa, como una hipoxia
leve, hipovolemia o la fiebre materna. La silente cuando es menor de 5 lpm y nos obligaría a
realizar una microtoma de sangre fetal. No obstante, la más patológica (por ser premortem) es la
sinusoidal. Habitualmente, es posterior a un período de variabilidad silente. Implica una
desconexión de la frecuencia cardíaca fetal del sistema nervioso autónomo. Así, debemos
escoger la opción 1.(R1)

145. Recién nacido de dos días de vida. Es llevado al hospital por presentar
hipoactividad, pobre succión, con antecedente de madre sin control prenatal y ruptura
de membranas de 20 horas ¿Cuál es el diagnostico mas probable?

1. 1. Sepsis neonatal temprana.


2. 2. Sepsis neonatal tardia.
3. 3. Asfixia perinatal.
4. 4. Hiperglicemia.
Gráfico de respuestas
Comentario

Pregunta sencilla sobre sepsis neonatal. en un RN de 2 días con hipoactividad, y antecedente


RPM el diagnóstico más probable es la número 1 sepsis neonatal temprana, que se presenta
hasta el 5º día, mientras que la sepsis neonatal tardía se presenta entre el 7º y el 28 día.
Repase la siguiente tabla.(R1)

!
!
!
!

146. At what age is a baby able to pick up objects with pincer grasp?

1. 1. 10 months.
2. 2. 8 months.
3. 3. 5 months.
4. 4. 15 months.
Gráfico de respuestas
Comentario

Pregunta memorística sin demasiada importancia sobre un aspecto muy concreto del desarrollo
infantil. No hay que preocuparse demasiado. Recordemos que en este tipo de preguntas, la
respuesta correcta suele ser una de las respuestas intermedias: en este caso, 10 meses.(R1)

147. Masculino de 60 años con EPOC que ha ingresado por neumonía y reagudización
grave de la insuficiencia respiratoria, precisando tratamiento con esteroides e ingreso
en UCI. En el 5º día de ingreso, el paciente presenta episodio de hemorragia digestiva
alta. Lo más probable es que la endoscopia muestre:

1. 1. Erosiones gástricas que afectan a cuerpo y fundus.


2. 2. Erosiones gástricas en antro.
3. 3. Ulcera duodenal.
4. 4. Tumor gástrico.
Gráfico de respuestas
Comentario

Las gastritis agudas de estrés suelen expresarse clínicamente en forma de hemorragia


digestiva, sea en forma de hematemesis, melenas o anemización súbita. Aunque su localización
puede ser difusa, predominan en el fondo gástrico. El mecanismo de la lesión es por isquemia y
alteración de la barrera mucosa gástrica, como sucede en las úlceras de Curling (grandes
quemados) y en las relacionadas con otras causas. La excepción a esta regla son las úlceras de
Cushing. Éstas se relacionan con la hipertensión intracraneal, sobre todo cuando es aguda, ya
que produce una hiperestimulación vagal y, como resultado, hipersecreción ácida gástrica.

Debe recordar que, aunque se habla coloquialmente de “úlceras de estrés”, en realidad se trata
de erosiones (la profundidad de la lesión no sería, propiamente, una úlcera).(R1)

148. Entre los efectos de la adicción a la cocaína en una mujer gestante, es FALSO que
exista:

1. 1. Mayor riesgo de desprendimiento de placenta.


2. 2. Mayor incidencia de parto post-término.
3. 3. Mayor incidencia de hipertensión materna.
4. 4. Mayor incidencia de retraso del crecimiento intrauterino y bajo peso al nacer.
Gráfico de respuestas
Comentario

!
!
!
!

La cocaína no suele producir síndrome de abstinencia en el recién nacido, a diferencia de los


opiáceos. Sin embargo, pueden aparecer complicaciones obstétricas, como el parto pretérmino,
el desprendimiento prematuro de placenta, etc.

Por otra parte, puede afectar directamente al feto, manifestándose como CIR, microcefalia,
hemorragias intracraneales, anomalías digestivas o renales, muerte súbita, alteraciones
conductuales o neurológicas.

La respuesta correcta, por tanto, es la 2. Lo que produce la cocaína no es parto postérmino, sino
todo lo contrario: pretérmino.(R2)

149. El aumento en la concentración plasmática de FSH y LH en una paciente con ciclos


anovulatorios indica:

1. 1. Anovulación.
2. 2. Insuficiencia ovárica o perimenopausia.
3. 3. Insuficiencia hipofisiaria.
4. 4. Ninguna.
Gráfico de respuestas
Comentario

El aumento de la FSH y LH son indicativos de fallo ovárica, por lo que la respuesta correcta es la
2.(R2)

150. Un paciente de 53 años, con cervicalgia ocasional desde hace 6 años, refiere un
cuadro progresivo de 2 meses de evolución consistente en debilidad en miembros
superiores y disfagia. En la exploración se aprecian amiotrofias en ambas
extremidades superiores, con reflejos bicipitales exaltados, así como fasciculaciones
en los músculos cuádriceps y gemelo derechos, con reflejos rotuliano y aquíleo
presentes. Una RM medular demostró estenosis de canal a nivel C4-C7, sin
alteraciones a otros niveles. ¿Cuál de los siguientes diagnósticos le parece el más
probable?

1. 1. Mielopatía cervical por estenosis de canal.


2. 2. Neuropatía multifocal con bloqueos.
3. 3. Esclerosis lateral amiotrófica.
4. 4. Enfermedad de Gerstmann-Straussler.
Gráfico de respuestas
Comentario

En neurología, el enfoque de los casos clínicos es mucho más sencillo si conoce bien el primer
tema del Manual (semiología neurológica). Vamos a ir descartando las opciones una a una:

- Si fuera una mielopatía cervical, sería posible encontrar amiotrofias en MMSS e hiperreflexia
bicipital. Sin embargo, no existirían fasciculaciones en cuádriceps. Esto indica patología de
segunda motoneurona a nivel lumbar, que no tendría por qué aparecer.

!
!
!
!

- En una neuropatía (sin especificar el tipo), los reflejos osteotendinosos estarían abolidos. Las
amiotrofias y las faciculaciones podrían aparecer, pero en un estadio muy avanzado.

- En tumor medular intraaxial, la disfagia no estaría justificada (ya que implica afectación de
pares craneales). Igual que en la mielopatía cervical, se afectaría un nivel concreto, pero por
debajo no tendría por qué haber clínica de segunda motoneurona.

- La enfermedad de Gerstmann-Straussler en una enfermedad priónica que debuta en la edad


adulta y que se caracteriza principalmente por disfunción cerebelosa progresiva.

La respuesta correcta es la eslcerosis lateral amiotrófica (ELA). Esta enfermedad casi siempre la
presentan como afectación de primera y segunda motoneurona, sin afectación sensitiva ni
deterioro intelectual.(R3)

151. Respecto al delirium, la demencia y el síndrome confusional agudo, señale la


opción FALSA:

La demencia se caracteriza por un deterioro crónico de las funciones superiores, adquirida


1. 1.
y con un nivel de conciencia y atención normales.
2. 2. Tanto en el delirium como en la demencia la orientación del paciente está alterada.
3. 3. En la demencia y el delirium la memoria reciente y la remota están alteradas.
En el delirium son típicos los síntomas vegetativos con preservación de la memoria
4. 4.
inmediata.
Gráfico de respuestas
Comentario

Típica pregunta que se falla por falta de motivación al final del examen. Hay que perseverar y
luchar cada pregunta, por que podemos encontrarnos con respuestas sencillas ante preguntas
aparentemente complicadas. Es un diagnóstico diferencial demencia vs delirium, en el cual hay
dos datos claves que siempre debe tener en cuenta: inicio insidioso vs agudo, afectación de la
memoria inmediata en el delirium y no en la demencia.(R4)

152. Un marinero jubilado presenta en la frente múltiples lesiones


eritematodescamativas de pequeño tamaño sobre una base de eritema, telangiectasias
y manchas hipo e hiperpigmentadas. El paciente suele rascarse con frecuencia las
lesiones, y acude hoy a consulta porque una de ellas, de aspecto escamocostroso, ha
sangrado con el rascado. ¿Qué es cierto acerca de dicha lesión?:

1. 1. Es un tumor que no aparece en mucosas.


2. 2. Es el cáncer más frecuente en la especie humana.
3. 3. Lo más frecuente es que aparezca sobre una precancerosis previa.
4. 4. El tronco es una localización más frecuente que la cabeza.
Gráfico de respuestas
Comentario

Debe conocer muy bien las características diferenciales entre el basocelular y el epidermoide. El
caso del marinero que nos presentan es muy probablemente un carcinoma epidermoide. Este
tumor puede aparecer en mucosas, a diferencia del basocelular, que no las afecta. El cáncer
más frecuente de la especie humana es el basocelular, y no el epidermoide. El carcinoma
epidermoide sí que tiene capacidad metastatizante, sobre todo vía linfática, a diferencia del

!
!
!
!

basocelular, que es excepcional que metastatice. La afirmación correcta es la 3, puesto que el


carcinoma epidermoide casi siempre aparece sobre lesiones precancerosas, como queratosis
actínicas, que es lo que presenta este marinero en la frente. En cambio, el basocelular suele
aparecer en piel sana. Por último, el tronco no es una localización más frecuente que la cabeza,
porque el carcinoma epidermoide tiene predilección por áreas fotoexpuestas, que son las más
susceptibles de sufrir lesiones solares crónicas.(R3)

153. ¿Cuál de estos virus NO es RNA?:

1. 1. Virus de la rubéola.
2. 2. Virus de la fiebre amarilla.
3. 3. Parvovirus.
4. 4. Enterovirus.
Gráfico de respuestas
Comentario

No debe tratar de saber sobre todos los virus, cúales son DNA y cúales RNA. Solo de los más
importantes (Herpes, HIV, hepatitis...). Los virus de la hepatitis son todos RNA salvo el de la
hepatitis B. También son RNA el de la rubéola, la fiebre amarilla (y la mayoría de los que
provocan fiebres hemorrágicas), y los enterovirus. Los parvovirus (eritema infeccioso; crisi
aplásicas; abortos) son DNA virus. Recuerde que los herpes son DNA virus también.(R3)

154. Marque la respuesta INCORRECTA respecto del efecto tópico del corticosteroide
intranasal, por spray, en el tratamiento de la rinitis alérgica:

1. 1. Reduce la secreción mucosa.


2. 2. Reduce el estímulo alfa – adrenérgico.
3. 3. Reduce la permeabilidad vascular.
4. 4. Reduce la inmediata y tardía de la respuesta alérgica.
Gráfico de respuestas
Comentario

La respuesta incorrecta es la 3, ya que no es un alfa bloqueante adrenérgico, el resto de


respuestas es correcta, ya que disminuye la secreción de las mucosas, además disminuye la
permeabilidad vascular y reduce de manera inmediata y tardía la respuesta alérgica.(R2)

155. Mujer 38 años consulta por deseo de anticoncepción prolongada e hipermenorrea.


Como antecedentes presenta: 2 partos eutócicos previos y un recambio valvular hace
2 años por insuficiencia mitral por lo que toma de forma crónica dicumarínicos. La
revisión ginecológica es normal. De los siguientes métodos anticonceptivos ¿cuál
estaría indicado?:

1. 1. DIU liberador de levonorgestrel.


2. 2. Anticonceptivo combinado monofásico.
3. 3. Anticonceptivo combinado bifásico.
4. 4. Mifepristona.
Gráfico de respuestas
Comentario

!
!
!
!

Por la hipermenorrea que presenta la paciente el DIU-LNG es el método anticonceptivo más


adecuado ya que además de proporcionar una anticoncepción eficaz y prolongada, permite
tratar la hipermenorrea por la atrofia del endometrio que induce y una disminución del sangrado
menstrual en un 90% de las mujeres. Además no existe contraindicación para la utilización de
este tipo de DIU por el hecho de que la paciente esté utilizando anticoagulación oral con
dicumarínicos.(R1)

156. Femenino
de 56 años, con molestias abdominales inespecíficas continuas desde hace unos
meses, que acude a su revisión ginecológica. En el examen ginecológico muestra unos
genitales externos normales, cuello sano macroscópico, útero normal, anexo izquierdo
normal, mientras que en el lado derecho se palpa una tumoración móvil de 7 cm de
diámetro. El USG ginecológico demuestra la formación ovárica que se muestra en la
imagen. ¿Cuál sería la actitud más adecuada?:

1. 1. Sospecho endometrioma y realizo anexectomía laparoscópica.


2. 2. Sospecho teratoma y realizo anexectomía laparoscópica.
3. 3. Sospecho disgerminoma y realizo laparotomía exploradora y cirugía radical.
Sospecho adenocarcinoma mucoso, realizo anexectomía con estudio intraoperatorio de la
4. 4.
lesión y, si es positivo, completo con cirugía radical.
Gráfico de respuestas
Comentario

Una pregunta difícil, pero no imposible. Lo que más llama la atención es la imagen tabicada que
nos muestra el ultrasonido, por lo que no deberíamos pensar en endometriomas ni
disgerminomas, que formarían masas de consistencia sólida. Bajo este punto de vista, un
teratoma podría ser una opción más razonable, ya que puede ser quístico. Sin embargo, es más
frecuente en mujeres jóvenes, y nuestra paciente tiene 56 años. Por otra parte, más que
tabiques, lo que muestra un teratoma quístico en el ultrasonido son bolas móviles que pueden
cambiar con la posición de la paciente. Aunque este movimiento no puede apreciarse en una
imagen estática como la que nos presentan, sí podemos apreciar que se trata de tabiques, y no
imágenes esféricas individuales.(R4)

!
!
!
!

157. Supongamos que el diagnóstico anatomopatológico de la paciente del caso


anterior fue cistoadenocarcinoma seroso de ovario de bajo grado de malignidad, con
células malignas en el lavado peritoneal. En la exploración de la cavidad abdominal no
se detectan otras lesiones. ¿En qué estadio se encuentra el tumor?:

1. 1. IC.
2. 2. IIA.
3. 3. IIC.
4. 4. III.
Gráfico de respuestas
Comentario

Es natural que no conozca el estadiaje a estas alturas del curso (nuestro consejo es que lo
revise cuando esté a muy pocas semanas del ENARM, porque ese tipo de detalles se olvidan
con frecuencia). Por el momento, basta que recuerde que el estadio I hace referencia a tumores
limitados al ovario, como en este caso. Lo que sucede es que, cuando existe un lavado
peritoneal positivo, se trata de un estadio IC. En general, el IC se define como la afectación de
uno o los dos ovarios, con la cápsula rota, tumor en la superficie, ascitis tumoral o lavado
peritoneal positivo. Es importante darse cuenta de que un lavado peritoneal positivo, en este
tumor, no nos ubica directamente en el estadio IV.(R1)

158. Una vez recuperada la paciente del tratamiento quirúrgico, acude a la consulta
para evaluar la necesidad de tratamiento complementario. ¿Qué tratamiento debe
indicarse por el estadiaje que presentó?:

1. 1. Quimioterapia adyuvante.
2. 2. Quimioterapia y radioterapia adyuvante.
3. 3. Radioterapia adyuvante.
4. 4. Quimioterapia y, una vez completada, cirugía de estadificación.
Gráfico de respuestas
Comentario

En el cáncer de ovario, los estadios IA y IB, si su grado histológico es G1 o G2, no se precisa


ningún tratamiento adyuvante. Sin embargo, el resto del estadio I (IA y IB en grado G3, y todos
los IC) precisan tres ciclos de quimioterapia adyuvante, que incluya derivados del platino
(cisplatino). Del IIA en adelante, también se emplea este tipo de adyuvancia (derivados del
platino y taxol).(R1)

159. Usted remite un paciente suyo al Servicio de Neumología de su hospital de


referencia, por un cuadro de disnea e hipoxemia de origen no filiado y largo tiempo de
evolución. En la exploración física, destacaba la presencia de acropaquias. Teniendo
en cuenta este último dato, señale cuál de los siguientes diagnósticos NO le parece
probable:

1. 1. Fístulas arteriovenosas intrapulmonares.


2. 2. Bronquiectasias.
3. 3. EPOC.
4. 4. Fibrosis pulmonar idiopática.
Gráfico de respuestas

!
!
!
!

Comentario

Las acropaquias son un signo sugestivo de todas las patologías expuestas menos de la EPOC.
Recordemos que también pueden aparecer en el cáncer de pulmón, por lo que ante un EPOC
con acropaquias lo que se debe descartar es el cáncer.(R3)

160. Un paciente intervenido hace unos años por una úlcera duodenal acude a consulta
por la aparición de dolor abdominal. Ante la sospecha de una nueva úlcera se le
practica una endoscopia que muestra úlceras en bulbo duodenal y antro pilórico. La
gastrina sérica del paciente está elevada, y se eleva tras la administración de secretina.
¿Cuál sería el tratamiento óptimo de este paciente teniendo en cuenta su sospecha
diagnóstica?

1. 1. Antagonistas H2 en dosis altas.


2. 2. Omeprazol.
3. 3. Resección completa del tumor.
4. 4. Omeprazol más antagonistas H2.
Gráfico de respuestas
Comentario

La primera sospecha diagnóstica ante un cuadro clínico con úlceras múltiples e hipergastrinemia
debe ser el síndrome de Zollinger Ellison (gastrinoma), que además se confirma con la elevación
de la gastrina tras la estimulación con secretina. En relación al tratamiento de elección es
siempre quirúrgico, con extirpación del tumor, si se puede (ya que tiene una gran tendencia a la
diseminación).(R3)

161. Todas las siguientes son características clínicas del chancroide EXCEPTO:

1. 1. Presentación inicial como una pápula dolorosa.


2. 2. Aparición de úlceras genitales dolorosas.
3. 3. Ganglios linfáticos inguinales dolorosos y aumentados de tamaño.
4. 4. Respuesta al tratamiento con ampicilina.
Gráfico de respuestas
Comentario

Con esta pregunta podemos repasar uno de los agentes etiológicos responsables de las
ulceraciones genitales. El chancro blando o chancroide es una enfermedad de transmisión
sexual causada por el Haemophilus ducreyi. Tiene un período de incubación de 1-3 días, tras el
que aparece una lesión de consistencia blanda y dolorosa, muchas veces múltiple, no
sobreelevada, de fondo sucio, acompañada de adenopatías dolorosas unilaterales, con
tendencia a ulcerarse. El tratamiento consiste en 250 mg IM de ceftriaxona en monodosis o
eritromicina durante 7 días. Opción 4 falsa.(R4)

162. Which of the following statements is FALSE in relation to chickenpox-related


complications?

1. 1. Skin lesions may develop secondary infections.


Reye's syndrome has been reported to be associated with aspirin intake during chickenpox
2. 2.
infection.
3. 3. Cerebellar involvement confers a poor prognosis in patients with encephalitis.

!
!
!
!

4. 4. Bleeding blisters may be related to thrombocytopenia.


Gráfico de respuestas
Comentario

Pregunta de dificultad media-alta acerca de las complicaciones de la varicela. La opción correcta


es la 3 ya que la presencia de datos de afectación cerebelosa en la encefalitis post-infecciosa le
confiere buen pronóstico a dicha complicación. La sobreinfección de las vesículas es la
complicación más frecuente de este cuadro. La neumonía es más frecuente en los adultos,
apareciendo en un 30% de los casos, y evoluciona favorablemente en la mayoría de los casos.
El síndrome de Reye tiene como antecedente en el 10% de los casos una varicela tratada con
ácido acetilsalicílico y las vesículas hemorrágicas aparecen en una variante de la varicela que
cursa con trombopenia.(R3)

163. Paciente femenino de 26 años, nulípara con amenorrea de 3 meses, acude por
hemorragia vaginal moderada. En la exploración se observa un útero mayor del que
correspondería al tiempo de amenorrea. La HCG es fuertemente positiva y el
ultrasonido muestra una imagen semejante a los copos de nieve en la cavidad uterina.
¿Qué es FALSO en relación a esta patología?:

1. 1. Puede ser causa de preeclampsia antes de la 20 semana.


2. 2. Puede cursar con clínica de tirotoxicosis.
3. 3. El tratamiento de elección es la histerectomía.
4. 4. La determinación seriada de HCG es útil para su seguimiento.
Gráfico de respuestas
Comentario

Para decidir el tratamiento necesario, tenemos que llegar a un diagnóstico. Dentro de las
hemorragias del primer trimestre, los datos clínicos (útero mayor que amenorrea) y
ultrasonográficos (imagen en “copos de nieve”) nos ayudan a sospechar una enfermedad
trofoblástica. La imagen ultrasonográfica que presentan estas pacientes es una imagen
multivesicular, en ocasiones asociada a la presencia de quistes tecaluteínicos bilaterales.

El tratamiento preferido es el legrado por aspiración frente al legrado con legras cortantes, para
intentar minimizar los riesgos de perforación uterina y la persistencia de restos. El estudio
anatomopatológico nos confirmará el diagnóstico y precisará el tipo histológico.

La histerectomía (respuesta 3) la reservaremos para las pacientes de más de 40 años con los
deseos de descendencia ya cumplidos, situación muy distinta a la que nos describen en el
enunciado (nulípara de 26 años).(R3)

164. Con respecto a la tuberculosis señale lo CORRECTO:

1. 1. La respuesta a la tuberculina es un proceso independiente de la capacidad de defensa.


La respuesta a la tuberculina y la capacidad de defensa se dan separadamente pero ocurren
2. 2.
casi simultáneamente.
3. 3. Ambos procesos son mediados por linfocitos.
4. 4. Todo lo anterior es verdadero.
Gráfico de respuestas
Comentario

!
!
!
!

El principal sistema defensivo contra M. tuberculosis está constituido por la inmunidad celular
específica (mediada por linfocitos T), que se pone de manifiesta en la prueba de reactividad
cutánea a la tuberculina. Debe tenerse muy presente lo que significa una prueba positiva a la
tuberculina. Esta prueba sólo traduce la existencia de inmunidad cutanea (hipersensibilidad
retardada o de tipo IV) frente a M. tuberculosis que a su vez puede haberse adquirido tras la
infección previa (aún sin haber desarrollado sintomatología de enfermedad activa), mediante
vacunación o tras el contracto con determinadas micobacterias ambientales distintas de M.
tuberculosis (habitualmente no patógenas).(R4)

165. Los focos de atrofia coriorretinianos, los estafilomas posteriores y las manchas
de Fuchs son característicos de:

1. 1. Retinitis por citomegalovirus.


2. 2. Uveítis posteriores.
3. 3. Miopía magna.
4. 4. Estadio cicatrizal de una obstrucción de arteria central de la retina.
Gráfico de respuestas
Comentario

La miopía magna es un tema poco preguntado en el ENARM, comienza en edad escolar,


progresa rápidamente durante el desarrollo y hacia los 40 años da lugar a cambios
degenerativos y complicaciones. Su fondo de ojo TIPICO incluye los estafilomas
POSTERIORES y las manchas de Fuchs (que consiste en una hemorragia subretiniana macular
con pigmento, debida a una neovascularización coroidea antigua) que suelen producir una
pérdida de la visión central.(R3)

166. Recién nacido pretérmino ingresado en la UCIN pediátrica por enfermedad de


membrana hialina, permanece intubado durante las primeras 24 horas, presentando
notable mejoría en los 2 días posteriores. Al cuarto día se incrementan sus
necesidades de oxígeno y en la gasometría se observa mayor retención de CO2. Usted
sospecha una persistencia de conducto arterioso. Señale lo CORRECTO:

1. 1. En la exploración es frecuente encontrar pulsos saltones y fuertes.


2. 2. El tratamiento inicial consiste en intervención quirúrgica.
3. 3. Suelen mejorar con administración generosa de líquidos.
4. 4. Las prostaglandinas son el tratamiento de elección para los RNPT con clínica de ductus.
Gráfico de respuestas
Comentario

La dificultad de esta pregunta está en reconocer el caso clínico. Se trata de una PCA, que
siempre tiene que sospechar en un contexto como éste: enfermedad de membrana hialina que
súbitamente empeora, cuando ya comenzaba a estabilizarse, y esto es debido a la caída de las
resistencias pulmonares cuando se instaura el tratamiento con la oxigenoterapia. Es entonces
cuando se pone de manifiesto el conducto arterioso, que semiológicamente se caracteriza por
los pulsos saltones y por un soplo continuo muy típico (soplo de Gibson).(R1)

167. Señale la localización del cefalohematoma fetal:

1. 1. Entre el periostio y el hueso.

!
!
!
!

2. 2. Entre el hueso y las meninges.


3. 3. En el parénquima cerebral.
4. 4. En alguno de los ventrículos cerebrales.
Gráfico de respuestas
Comentario

El cefalohematoma es una hemorrágiasubperíostica, una colección de sangre entre el periostio y


el hueso. Por el contrario, el caput es edema del tejído celular subcutaneo. La tabla que hay a
continuación es vital para diferenciarlos, sobretodo recuerda que el caput no respeta las suturas
mientras que el cefalohematoma sí.

168. Atiende a un paciente que ha sufrido un accidente. Tras comprobar la


permeabilidad de la vía aérea, la adecuada ventilación, la estabilidad hemodinámica y
la ausencia de lesiones viscerales, comprueba que presenta múltiples fracturas. De las
siguientes, la que debe tratar en primer lugar es:

1. 1. Fractura de cúbito y radio.


2. 2. Fractura abierta de tibia.
3. 3. Fractura de calcáneo.
4. 4. Fractura de clavícula.
Gráfico de respuestas
Comentario

En el control de un politraumatizado, el último paso se basa en el control de las extremidades,


pelvis y columna. Ante una serie de fracturas se debe valorar cuál de ellas es más importante y
puede dar más complicaciones para tratar en primer lugar. Para ello se debe conocer el tema
más importante de traumatología, el tema de fracturas y sus complicaciones. De todas las
fracturas que nos dan por respuesta, se debe saber que cualquier fractura abierta constituye una
urgencia, por el riesgo de infección que comporta. La tibia es la localización mas frecuente de
las fracturas abiertas. El tratamiento se basará en desbridar la herida, administrar antibiótico I.V.,
profilaxis antitetánica y estabilizar rígidamente la fractura.(R2)

169. Un paciente de 35 años es traído al Servicio de Urgencia tras sufrir un accidente


de tráfico. No abre los ojos ante estímulos dolorosos. No obedece órdenes y emite
sonidos incomprensibles. Su brazo derecho aparece deformado y no responde a
estímulos dolorosos; sin embargo, su mano izquierda responde en un movimiento
intencionado hacia el estímulo. Su escala de coma de Glasgow (GCS) es:

1. 1. 12
2. 2. 9

!
!
!
!

3. 3. 5
4. 4. 8
Gráfico de respuestas
Comentario

Una pregunta de dificultad media sobre el cáncer de pulmón metastásico.

La escala de Glasgow es útil para hacer una aproximación al nivel de conciencia de los
pacientes con traumatismo craneoencefálico (TCE). Valora tres parámetros clínicos, que son la
apertura de ojos, la respuesta motora y la respuesta verbal, puntuando entre un máximo de 15 y
un mínimo de 3. De acuerdo con la puntuación alcanzada, el TCE puede clasificarse según su
gravedad.

•! TCE grave: Glasgow 3-8.


•! TCE moderado: 9-13.
•! TCE leve: 14-15.

En este caso clínico, la puntuación sería:

•!
•! No abre los ojos ante estímulos dolorosos, 1 punto.
•! Respuesta verbal: emite sonidos incomprensibles, 2 puntos.
•! Existe movimiento intencionado hacia un estímulo, es decir, localiza el dolor (mano
izquierda), 5 puntos.

De acuerdo con esto, la puntuación sería de 8, lo que traduce un TCE grave.

(R4)

170. La placenta es el órgano de conexión fetomaterna. Tiene función de barrera,


función endocrina e intercambiadora de sustancias. Respecto a esta última, es FALSO
que:

1. 1. El agua y los electrolitos utilizan la difusión simple según gradiente.


2. 2. Los aminoácidos necesitan de un transporte activo.
3. 3. Las moléculas grandes, como la IgM, pasan por pinocitosis.
4. 4. El hierro pasa por transporte activo.
Gráfico de respuestas
Comentario

Una pregunta fácil de responder.

La respuesta falsa es la 3, ya que:

- La inmunoglobulina que atraviesa la placenta es IgG, no IgM (su tamaño es enorme como
para poder pasar por la placenta) (respuesta 3 falsa).

El resto de las opciones son válidas y hacen referencia a algunos mecanismos de intercambio
de sustancias que debe conocer.(R3)

!
!
!
!

171. A 57-year-old man


presents to the emergency department with dark stools. Gastrointestinal endoscopy
reveals a protruding mass occupying the duodenum, measuring abour 13 cm in
diameter, with central ulceration and necrosis. CT scan suggest the presence of several
hepatic metastatic lesions. Biopsy is performed and pathology report describes
fusiform cells that stained positive for CD117 (c-Kit). Which of the following is the most
likely diagnosis?

1. 1. Stage IV small bowel adenocarcinoma.


2. 2. Gastrointestinal neuroendocrine tumor (Carcinoid).
3. 3. Gastrointestinal stromal tumor (GIST).
4. 4. Gastric MALT lymphoma.
Gráfico de respuestas
Comentario

Los tumores del estroma gastrointestinal (GIST) son cánceres relativamente raros que se
originan en las células de Cajal, que son células especializadas que tapizan el tubo digestivo y
son responsables de iniciar la peristalsis intestinal. Los GIST se caracterizan por anomalías
genéticas específicas que producen hiperactividad de un gen concreto, denominado proto-
oncogén c-kit y la sobreproducción de determinadas proteínas. Juntas, estas anomalías
conducen a un crecimiento incontrolado y a limitación de la muerte celular entre las células
malignas del GIST. Los GIST malignos pueden producirse en cualquier parte del tubo digestivo
– desde el esófago al recto – pero suelen producirse en el estómago y el intestino delgado. Los
GIST se pueden extender a localizaciones distantes del cuerpo, fundamentalmente la cavidad
peritoneal y el hígado.

El Mesilato de imatinib es un tratamiento dirigido que funciona inhibiendo una proteína llamada
tirosín cinasa. Las células del GIST producen tirosín cinasa en exceso, que es responsable de la
replicación tumoral. Dicho tratamiento se ha convertido en el tratamiento estándar para el GIST.
Los resultados de los ensayos clínicos indican que más de la mitad de los pacientes con GIST
localmente recidivantes o metastásicos experimentaron una respuesta anticancerosa con
imatinib.(R3)

!
!
!
!

172. Regarding the previous case, which of the following options is the most
appropriate treatment?
1. 1. Imatinib.
2. 2. Lapatinib.
3. 3. Bevacizumab.
4. 4. Paclitaxel plus cisplatine
Gráfico de respuestas
Comentario

Los GIST se originan por mutación del gen c-kit que produce la sobreexpresión del receptor de
la tirosina-cinasa (KIT). El imatinib es un inhibidor de la tirosin-cinasa que se emplea
fundamentalmente en estadios avanzados (como terapia neoadyuvante, previo a la cirugía en
función de la respuesta). En casos localizados o en los que se pueda realizar una resección R0
de entrada, la cirugía será el tratamiento de elección.

El imatinib también se emplea como tratamiento de elección en la leucemia mieloide crónica,


también en algunas mastocitosis sistémicas y en algunos síndromes hipereosinofílicos con el
reordenamiento FIP1L1/PDGFRa.

Lapatinib es otro inhibidor de la tirosin-cinasa acoplada a los receptores del fator de crecimiento
epidérmico EGFR y HER2, aprobado su uso en combinación con capecitabina en cáncer de
mama metastásico HER2 positivo.(R1)

173. Paciente de 55 años, con antecedentes de cirrosis hepática por VHC. Ya ha


padecido un episodio de hemorragia digestiva alta, por sangrado de várices esofágicas
de gran tamaño, por lo que está siguiendo tratamiento de profilaxis secundaria con
propranolol y nitratos. Aunque ha tenido algún episodio de ascitis, actualmente está
bien controlada con espironolactona. Una USG rutinario nos muestra una lesión focal
de aproximadamente 3 cm, lo que hace sospechar un hepatocarcinoma. Esta lesión se
confirma con una TC. La alfafetoproteína está elevada. Señale cuál de las siguientes
afirmaciones es INCORRECTA:

En las pruebas de imagen, el patrón hipovascular y el lavado venoso tardío son datos
1. 1.
sugestivos de esta entidad.
La elevación de la alfafetoproteína no es un hallazgo constante en estos tumores, aunque
2. 2.
sí frecuente.
Dado que la presencia de varices y ascitis indica la presencia de hipertensión portal
3. 3.
significativa, la resección de la lesión no es aconsejable.
4. 4. Cuando existe invasión vascular, el propósito del tratamiento es paliativo.
Gráfico de respuestas
Comentario

Un patrón hipervascular y un lavado venosos precoz en las técnicas de imagen nos harían
sospechar un hepatocarcinoma (respuesta 1 incorrecta). Tenga cuidado con esta pregunta,
porque nos están mencionando dos características contrarias.(R1)

174. Respecto al hijo de madre diabética, señale la opción FALSA:

1. 1. Pueden ser macrosómicos.

!
!
!
!

Pueden presentar crecimiento intrauterino retardado y ser pequeños para la edad


2. 2.
gestacional.
3. 3. Tienen menor riesgo de enfermedad de membrana hialina.
4. 4. Presentan riesgo de ictericia.
Gráfico de respuestas
Comentario

El hijo de una madre diabética mal controlada pueden ser macrosómicos o con bajo peso. En el
primer caso es secundario al efecto anabolizante de la hiperglucemia y del hiperinsulinismo
reactivo, y el segundo es consecuencia del daño vascular placentario que ocasionaría un CIR
asimétrico.

Entre otros hallazgos metabólicos no hay que olvidar la hipoglucemia, como consecuencia de la
respuesta insulínica fetal y la falta de aportes tras el pinzamiento del cordón umbilical, la
policitemia que puede conllevar ictericia (opción 4 correcta) y el riesgo más elevado de
enfermedad de membrana hialina de estos pacientes (opción 3 falsa).(R3)

175. Una respuesta es INCORRECTA referente a la hemoglobinuria paroxística


nocturna:

1. 1. La transfusión de sangre total puede aumentar la hemólisis.


2. 2. El tratamiento con heparina puede aumentar la hemólisis.
3. 3. La prueba de Ham es patognomónica.
4. 4. La acetil-colinesterasa eritrocitaria está disminuida.
Gráfico de respuestas
Comentario

Si analizamos una a una las opciones de esta pregunta: - No es frecuente que las transfusiones
de sangre completa exacerben la hemólisis pero para evitarla se hacen transfusiones con
hematíes marcados. - La heparina, al ser una sustancia ácida, puede facilitar la activación del
complemento y por tanto la hemólisis. - Tanto la hemólisis ácida (prueba de Ham) como la
hemólisis con sacarosa orientan hacia su diagnóstico pero no son fidedignas ni, por tanto,
patognomónicas (opción 3 incorrecta). - Es típico el descenso de la acetil- colinesterasa
eritrocitaria y de la fosfatasa alcalina leucocitaria.(R3)

176. Respecto al transporte de sustancias transplacentario, ¿cuál de los siguientes


mecanismos NO es el verdadero?

El paso de O2 al feto está facilitado por la menor concentración de hemoglobina en los


1. 1.
hematíes fetales.
2. 2. Mayor avidez de hemoglobina fetal por el oxígeno que la materna.
3. 3. La glucosa es el principal sustrato energético del feto para el corazón y el cerebro.
4. 4. El agua pasa al feto por gradiente osmótico.
Gráfico de respuestas
Comentario

Pregunta sobre fisiología placentaria nada fácil. La clave está en que la concentración de
hemoglobina en los eritrocitos fetales (17 g/dL) es mayor que en los eritrocitos maternos (11
g/dL). Por otra parte, el paso de oxígeno también es facilitado por la mayor avidez de la
hemoglobina fetal al oxígeno que la hemoblobina materna. Por ello, la opción incorrecta es la 1.

!
!
!
!

El principal sustrato energético del feto es la glucosa, por eso pasa por difusión facilitada (opción
3). La opción 4 también es verdadera, como explicamos en el Manual CTO.(R1)

177. ¿Cuál de las siguientes situaciones clínicas es una contraindicación absoluta para
anticoncepción hormonal?

1. 1. Hipertensión arterial bien controlada.


2. 2. Historia personal de tromboembolismo previo.
3. 3. Diabetes mellitus.
4. 4. Infección urinaria.
Gráfico de respuestas
Comentario

Los anticonceptivos orales tienen una serie de contraindicaciones que merece la pena saber.

Tenga cuidado con este tipo de preguntas, puesto que es fácil tender trampas con pequeños
cambios como los que ahora va a ver. Son contraindicaciones absolutas:

•! Pacientes con riesgo cardiovascular: fumadoras de más de 35 años y no fumadoras de


más de 40 años.
•! Antecedentes de TVP o TEP (respuesta 2 correcta).
•! Cirugía mayor con previsibles períodos prolongados de inmovilización.
•! Mutaciones de carácter trombogénico (factor V de Leyden, protrombina 20210, déficit de
proteínas C y S…).
•! HTA mal controlada (respuesta 1 falsa).
•! Diabetes con afectación vascular (respuesta 3 falsa).
•! Vasculopatía inflamatoria.
•! Cardiopatías graves.
•! Enfermedad hepática importante.
•! Porfiria aguda intermitente.
•! Antecedentes de ictericia durante la gestación, colestasis intrahepática.
•! Embarazo, confirmado o sospechado.
•! Cáncer de mama y otros tumores hormonodependientes.
•! Discrasia sanguínea, anemia de células falciformes.
•! Sangrado genital anormal de origen no filiado.

178. Señale la respuesta FALSA, de entre las siguientes, con respecto a los espasmos
infantiles:

Se trata de una epilepsia generalizada secundaria, que suele comenzar a los 4 - 8 meses de
1. 1.
edad.
2. 2. Suelen clasificarse en idiopáticos, criptogénicos y secundarios.
3. 3. En el EEG aparece un patrón de ondas lentas hipervoltadas, bilaterales y asincrónicas.
4. 4. Los casos idiopáticos tienen peor pronóstico que los secundarios.
Gráfico de respuestas
Comentario

El síndrome de West aparece en el primer año de vida, más frecuentemente entre el 4° y 7°


mes, y predomin a en hombres (1.5:1). Puede ser de origen criptogénico o sintomático (más
frecuente); de hecho, cualquier daño cerebral importante que pueda generar epilepsia a esta

!
!
!
!

edad, probablemente lo hará en la forma de este síndrome, con lo que el listado de


posiblescausas es muy extenso. La tríada que define el síndrome consta de :

Espasmos infantiles

Detención del desarrollo psicomotor.

Hipsarritmia intercrítica.

El tratamiento se basa en: ACTH , corticoides , clonazepam , vigabatrina.

Respuesta correcta 4.(R4)

179. Paciente de 18 años acude al servicio de Urgencias por dolor abdominal tipo cólico
en mesogastrio. Tiene el antecedente de apendicetomía (peritonitis por apendicitis
perforada). Al examen: abdomen distendido, timpánico con contractura muscular en
mesogastrio, ruidos intestinales fuertes y metálicos. La causa más frecuente es:

1. 1. Apendicitis recurrente.
2. 2. Divertículo de Meckel.
3. 3. Gastroenteritis.
4. 4. Bridas y adherencias.
Gráfico de respuestas
Comentario

La respuesta correcta es bridas y adherencias, al tratarse de una probable oclusión intestinal en


un paciente con antecedente de cirugía abdominal. Respuesta 4 correcta.(R4)

180. La alteración del ritmo cardíaco que aparece más frecuentemente en la


intoxicación digitálica es:

1. 1. Taquicardia supraventricular.
2. 2. Bloqueo AV completo.
3. 3. Extrasístoles ventriculares.
4. 4. Flutter auricular.
Gráfico de respuestas
Comentario

Recuerde los efectos ECG de la digoxina:

- Prolongación del PR

- Acortamiento del QT

- Aplanamiento o inversión de la onda T

- Depresión del ST

!
!
!
!

- ESV (aleración del ritmo más frecuente en la intoxicación digitálica)

- Bloqueos AV

- Taquicardia supraventricular no paroxística con bloqueo AV variable

(R3)

181. Enferma de 62 años que consulta por astenia moderada crónica. Su biometría
hemática es el siguiente: leucocitos 3.5 x 109/L con fórmula normal, hemoglobina 6.5
g/dL, VCM 120 fL, plaquetas 80 x 109/L, frotis con presencia de macroovalocitos. El
valor sérico de vitamina B12 es de 100 pg/mL y el de folato superior a 4 ng/mL. La
frecuencia cardíaca es de 80 lpm y la tensión arterial de 120/70 mmHg. El tratamiento
más correcto de la anemia es:

1. 1. Vitamina B12.
2. 2. Vitamina B12 y ácido fólico.
3. 3. Vitamina B12 y transfusión de concentrados de hematíes.
4. 4. Vitamina B12, ácido fólico y concentrados de hematíes.
Gráfico de respuestas
Comentario

Es evidente que se trata de una anemia megaloblástica por deficiencia de vitamina B12, por lo
que el tratamiento exige su administración, siendo también aconsejable añadir ácido fólico al
tratamiento para evitar una crisis megaloblástica que empeore el cuadro (podría producir mayor
aborto intramedular). Respecto a la transfusión de concentrados de hematíes, no está
justificada, ya que el síndrome anémico es moderado (las anemias de larga evolución, como la
megaloblástica, suelen ser bastante bien toleradas) y no hay repercusión hemodinámica. La
transfusión de plaquetas carece de sentido; la cifra de plaquetas est ámuy por encima de los
límites de riesgo de sangrado, y se emplean en situaciones agudas con sangrado activo dada su
escasa vida media (horas).(R2)

182. Paciente femenino de 42 años de edad con diagnóstico de lupus eritematoso


sistémico en tratamiento prolongado con varios medicametos que, no recuerda en este
momento. Acude a la consulta por referir moretones frecuentes, estrias violáceas, cara
redonda a la exploración física aprecias obesidad centrípeta y una TA de 145/90. Por
su sospecha diagnóstica el primer estudio a realizar será.

1. 1. Bioquímica general, LDH, proteinograma y beta-2-microglobulina.


2. 2. Cortisol basal.
3. 3. Cortisol libre urinario.
4. 4. Prueba de suspresión con prueba de 2mg de dexametasona.
Gráfico de respuestas
Comentario

Pregunta compleja en relación a una patología que si le preguntan será de una forma mucho
más fácil en el ENARM: el síndrome de Cushing (que debe sospechar por el consumo crónico
de cortiesteroides por su patología de base).

!
!
!
!

Recuerde que es más frecuente el síndrome de Cushing que es el resutado de una


administración exógena de esteroides y la enfermedad de Cushing es por la producción interna
ya sea por tumor supra-renal, hipósifis.

Generalemente es preguntado en forma de caso clínico.

Nos hablan del Cushing por hiperplasia suprarrenal bilateral, que se caracteriza por un aumento
en la secreción de cortisol y pérdida del ritmo circadiano en su secreción, siendo independiente
de la ACTH. Si administrásemos metirapona, que es un inhibidor de la esteroidogénesis
suprarrenal, se produciría una disminución en la secreción de cortisol.

El cortisol libre urinario de 24 horas es el primer estudio que solicita en caso de sospecha y sus
metabolitos (hidroxiesteroides) se encontrarán aumentados. La determinación basal del cortisol
no resulta de utilidad en esta enfermedad, ya que existen personas sanas con cifras elevadas
(sería un pico de secreción), y pacientes con Cushing con cifras basales normales.

Evindentemente, con la prueba de 2mg de DXM (dexametasona) no se apreciará supresión en


el cortisol, ya que esta es la prueba de elección para el diagnóstico bioquímico. Con la prueba
fuerte de DXM (8mg/24 h/2 días) no existe supresión del cortisol, a diferencia de lo que ocurre
con los microadenomas hipofisarios.(R3)

183. Ante un niño que presenta lesiones purpúricas en extremidades inferiores, dolor
abdominal cólico y artralgias, el diagnóstico más frecuente es:

1. 1. Enfermedad de Kawasaki.
2. 2. Mononucleosis infecciosa.
3. 3. Púrpura de Schönlein-Henoch.
4. 4. Artritis crónica juvenil.
Gráfico de respuestas
Comentario

Describen el cuadro clínico característico de la púrpura de Schönlein Henoch, que es una


vasculitis de pequeño vaso que afecta predominantemente a vasos de pequeño tamaño y
clínicamente cursa con una púrpura palpable en miembros inferiores, dolor abdominal tipo
cólico, artralgias y/o artritis y alteraciones del sedimento urinario como proteinuria y hematuria
con aparición raramente de sde nefrótico que afecta generalmente a niños. Aunque en el resto
de enfermedades que se dan como otras opciones puede aparecer algún dato clínico similar
(como las artralgias en la ACJ, en la FR o en el Kawasaki...), el conjunto de todos estos datos
clínicos en un niño es altamente sugestivo de esta enfermedad.(R3)

184. En una enferma de 40 años, con insuficiencia renal crónica secundaria a


nefrectomía bilateral y antecedentes personales de gastrectomía por úlcera péptica,
apendicectomía e histerectomía, ¿cuál sería el tratamiento sustitutivo más adecuado?

No es candidata a tratamiento sustitutivo por tener un pronóstico vital ominoso a corto


1. 1.
plazo.
2. 2. Hemodiálisis periódica.
3. 3. Trasplante renal urgente.
Hemodiálisis diaria durante una semana, seguida de tratamiento conservador de la
4. 4.
insuficiencia renal crónica.

!
!
!
!

Gráfico de respuestas
Comentario

Una pregunta de dificultad media sobre el tratamiento de la insuficiencia renal crónica.

Dado que se trata de una paciente joven, habría que ofrecerle un tratamiento sustitutivo para su
función renal, que será la hemodiálisis periódica, el trasplante renal o la diálisis peritoneal. En
este caso, por supuesto podría realizarse un trasplante renal, pero no es necesario realizarlo de
forma urgente (respuesta 4 falsa). Por otra parte, en esta paciente no podemos plantearnos la
diálisis peritoneal, puesto que tiene antecedentes de cuatro intervenciones quirúrgicas
abdominales, por lo que es de suponer que la capacidad de intercambio de su peritoneo no será
óptima. Teniendo en cuenta todo esto, la única opción posible es la respuesta 2.(R2)

185. Señale la opción FALSA:

1. 1. La vacuna anti-polio oral está formada por virus muertos.


2. 2. La vacuna frente al sarampión está formada por virus atenuados.
3. 3. La vacuna antineumocócica 23-valente es polisacárida.
4. 4. La vacuna antitetánica no está formada por bacterias enteras.
Gráfico de respuestas
Comentario

La vacuna anti-polio oral o Sabin está formada por virus atenuados. Recuerde que su
inmunogenicidad es algo mayor, pero se han descrito casos de poliomielitis asociada a su
administración, por lo que ha quedado relegada en beneficio de la vacuna parenteral o Salk.(R1)

186. Acude a la consulta un paciente con parálisis facial izquierda. A la anamnesis, el


paciente refiere cuadro catarral los días previos. Ha presentado otalgia, sensación de
entumecimiento facial y trastornos del gusto los días previos. En su exploración física,
la otoscopia revela membranas timpánicas íntegras traslúcidas y ambarinas, sin
niveles hidroareos, con arquitectura conservada. La audiometría es normal
bilateralmente. Determina una parálisis facial grado IV en la escala de House
Brackmann. Usted sospecha una parálisis de Bell, ¿cual no es una causa de parálisis
facial periférica?

1. 1. Enfermedad de Lyme.
2. 2. Síndrome de Ramsay Hunt.
3. 3. Otitis media colesteatomatosa.
4. 4. Infarto lacunar motor puro
Gráfico de respuestas
Comentario

Una pregunta sencilla. El enunciado en este caso es innecesario para responder a la pregunta,
es una estrategia para hacerle perder tiempo.

Existen multitud de causas de parálisis facial, con son las opciones que figuran en la pregunta,
además de otra multitud de posibilidades como son la tuberculosis, sífilis, difteria, intoxicación

!
!
!
!

por plomo, aneurisma intratemporal de la carótida interna, HIV, otitis media aguda, otitis externa
maligna, entre otras.

El dato clínico fundamental para poder diferencial clínicamente una parálisis facial periférica de
una parálisis facial central es observar la contracción del músculo corrugador de la frente y el
orbicular de los párpados. Si la lesión es de origen central, tendrá su función conservada, ya que
estas fibras tienen información bilateral. Otro dato fundamental, es realizar una adecuada
exploración neurológica.

Entre las opciones de esta pregunta, se descarta la opción 4, aún sin conocer el resto de
opciones, pues sabemos que un infarto lacular motor puro se asienta en la cápsula interna. Es
por tanto una lesión supranuclear, una lesión central.(R4)

187. A 35-year-old male patient comes to his physician complaining of a pulsatile mass
in his neck, intermittent otorrhagia and pulsatile tinnitus. Otoscopy reveals an
intratympanic reddish and pulsatile mass. Regarding the most likely diagnosis, which
of the following is INCORRECT?

1. 1. It is a tumor derived from cells of the embryonic neural crest.


2. 2. It is a benign tumor, may be multicentric and neurosecretory. It's more common in men.
If the tumor is in the head and neck area, the most common location is carotid artery,
3. 3.
followed by jugular vein and tympanic cavity.
4. 4. The diagnosis is made with arteriography or angio MRI.
Gráfico de respuestas
Comentario

Los parangliomas, quemodectomas o glomus son tumores originados de las células


paraganglionares o paraganglios derivados de la cresta neural. Pertenecen al sisneuroendocrino
difuso, por lo que pueden ser tumores neurosecretores (1-3%), y asociarse a otros tumores. Son
de carácter benigno aunque multicéntricos. La gran mayoría (90%) son suprarrenales y sólo en
un 0.6% tienen localización OR (intracarotídeos, orbitarios, nasosinusales, timpánicos,
yugulares, vagales, mediastínicos, supraclaviculares). Presentan una prevalencia mayor en
mujeres con una tasa 5:1. La patología que presenta el paciente se trata de un glomus
yugulotimpánico (derivado del crecimiento de las células paranglionares de la adventicia yugular,
y del nervio de Jacobson y de Arnold) que cursa con tumoración de crecimiento lento, la más
frecuente de oído medio.(R2)

188. Una paciente de 60 años presenta ictericia de varios días de evolución, con coluria
y acolia. No refiere ninguna otra sintomatología acompañante, excepto prurito intenso,
que le ha llevado a presentar, en la exploración, lesiones cutáneas de rascado. Entre
los antecedentes destaca colecistectomía por colelitiasis hace 20 años. En el
ultrasonido abdominal urgente se observa dilatación de la vía biliar intrahepática. La
vía biliar distal parece normal y no se visualiza el páncreas por obesidad. El
diagnóstico más probable de esta paciente es:

1. 1. Estenosis iatrogénica de la vía biliar.


2. 2. Hepatitis colestásica.
3. 3. Colangiocarcinoma hiliar.
4. 4. Colangitis esclerosante.
Gráfico de respuestas

!
!
!
!

Comentario

Un caso clínico difícil, en el que nos describen un colangiocarcinoma hiliar, conocido también
como tumor de Klatskin. Este tumor aparece en el hepático común, cerca de la bifurcación, por
lo que clínicamente suele ser más precoz que el resto de los colangiocarcinomas, dando lugar a
una dilatación de la vía biliar intrahepática, sin afectar a la extrahepática. Lógicamente, al ser
proximal al conducto cístico, éste no se afecta y, por tanto, no habría distensión vesicular. Al ser
más proximal que otros colangiocarcinomas, el pronóstico es peor.

Descartaremos algunas opciones:

•! Sería difícil justificar la respuesta 1, puesto que la intervención quirúrgica se realizó hace
20 años. No parece probable una estenosis iatrogénica.
•! En caso de tratarse de una hepatitis aguda colestásica, debería existir clínica de
hepatitis (fiebre, elevación de las transaminasas, malestar general, dolor en hipocondrio
derecho…). Sin embargo, aquí nos hablan de un síndrome colestásico puro.
•! La colangitis esclerosante hubiera mostrado estenosis y dilataciones a varios niveles.
No obstante, dado que el ultrasonido puede no ser lo suficientemente sensible para
evidenciar esta imagen, debemos atender a más detalles. Y falta lo fundamental: el
antecedente de colitis ulcerosa, que aparece en más del 70% de los pacientes con
colangitis esclerosante. Además, recuerde que casi siempre se trata de varones de
mediana edad.

(R3)

189. La manifestación clínica más frecuente del embolismo pulmonar es:

1. 1. Disnea.
2. 2. Fiebre.
3. 3. Taquiarritmia supraventricular con síncope.
4. 4. Hemoptisis.
Gráfico de respuestas
Comentario

El TEP es muy importante en el ENARM, y no debe permitirse dudar ante preguntas como ésta.
El síntoma más frecuente es la disnea de aparición brusca, siendo el signo más frecuente la
taquipnea. Generalmente, la presencia de disnea grave, síncope o cianosis indican un TEP
masivo, en tanto que el dolor pleurítico, la tos o la hemoptisis sugieren un pequeño embolismo
periférico que se acompaña de infarto pulmonar. La detección de TVP es un signo excelente que
apoya el diagnóstico de TEP, pero está presente en menos de la mitad de los casos.(R1)

190. ¿Cuál de los siguientes tumores de ovario es de origen epitelial?:

1. 1. Tumor de Krukenberg.
2. 2. Androblastoma.
3. 3. Quiste dermoide.
4. 4. Tumor de Brenner.
Gráfico de respuestas
Comentario

!
!
!
!

A la hora de estudiar el cáncer de ovario es importante tener un esquema claro de los diferentes
tipos, vamos a repasar lo más importante no profundizan mucho en el ENARM pero no vamos a
confiarnos.

Los tumores de ovario de origen epitelial SON LOS MÁS FRECUENTES serosos, mucinosos,
endometrioides y su variante de celulas claras, y tumor de Brenner.

Los derivados de células germinales, que son los más frecuentes en mujeres jóvenes, son los
teratomas, disgerminomas, tumor del seno endodermico, coriocacarcinoma, carcinoma
embrionario, gonadoblastoma y poliembrioma.

Los tumores de los cordones sexuales-estroma: tumor de la granulosa, tecafibroma,


androblastoma (tumor de células de Leydig-Sertoli) y ginandroblastoma.

El tumor de Krukenberg es la metástasis ovárica de tumor digestivo, habitualmente gástrico,


caracterizado por la presencia de células en anillo de sello. Recuerde que el quiste dermoide o
teratoma quístico maduro es el tumor germinal más frecuente.(R4)

191. Señale la afirmación FALSA respecto a la angioplastia coronaria:

1. 1. El índice de éxitos es del 90%.


2. 2. La mortalidad es menor del 1%.
Si se implanta un stent tras el procedimiento se ha demostrado disminución en la
3. 3.
incidencia de reestenosis hasta sólo el 20%.
Tras sufrir una reestenosis, si se vuelve a realizar ACTP, se mantienen los índices de
4. 4. riesgo, pero el éxito de la intervención se ve mermado por al alteración anatómica que
origina el procedimiento previo.
Gráfico de respuestas
Comentario

Existe un espectacular avance en las técnicas de revascularización percutánea, que tiene un


porcentaje de éxito del 90% con una mortalidad inferior al 1%. El principal inconveniente de la
angioplastia es la reestenosis, que se produce en los seis meses siguientes al procedimiento. El
índice de reestenosis es del 30 al 45% cuando se realiza angioplastia simple, reduciéndose al
15-25% cuando se implantan endoprótesis (stents). En los últimos tiempos, se están empezando
a implantar stents recubiertos con fármacos inmunosupresores o antiproliferativos (rapamicina,
taxol, etc.) que reducen significativamente la tasa de reestenosis a corto y largo plazo. Cuando
se produce una reestenosis, el porcentaje de éxitos del procedimiento posterior es igual que en
el procedimiento inicial.(R4)

192. Habitualmente la fecundación tiene lugar en:

1. 1. Superficie del ovario.


2. 2. Porción ampular de la trompa.
3. 3. Porción ístmica de la trompa.
4. 4. Porción intersticial de la trompa.
Gráfico de respuestas
Comentario

!
!
!
!

Pregunta fácil y básica sobre la fisiología de la fecundación. El óvulo se fecunda en la trompa,


con lo que descartamos la opción 1. Las demás opciones nos indican distintas partes de la
trompa. La correcta es la opción 2, ya que es en la zona ampular donde se produce la
fecundación, y tardará 7 días en recorrer su camino hasta implantarse en el endometrio.
Recordemos que éste es el fundamento por el cual la gran mayoría de los embarazos ectópicos
ocurren en la zona ampular de la trompa.(R2)

193. Una mujer de


26 años consulta por la aparición de las lesiones observadas en la imagen, que se
localizan en codos, rodillas y nalgas. El cuadro tiene 6 meses de evolución y la paciente
refiere un prurito intenso en las áreas afectadas y episodios diarreicos esporádicos.
Responda la FALSA con respecto a esta patología:

Es habitual que se produzca afectación oral en forma de erosiones y, en raras ocasiones,


1. 1.
ampollas tensas.
2. 2. En los exámenes de laboratorio puede observarse un descenso de la ferritina y del calcio.
Es recomendable realizar una endoscopia alta con toma de muestras de la mucosa
3. 3.
intestinal.
4. 4. Habitualmente los pacientes con esta dermatosis no tienen clínica digestiva.
Gráfico de respuestas
Comentario

La imagen clínica corresponde a una dermatitis herpetiforme (observe el carácter vesiculoso de


algunas lesiones en el codo de la izquierda, sobre base eritematosa). El caso clínico, donde se
habla de zonas típicas (codos, rodillas, nalgas) y localización simétrica puede resultar también
de utilidad. Si además integramos el dato sobrante (diarreas esporádicas), nos hablaría a favor
de una posible asociación a enfermedad celíaca.

!
!
!
!

Sabiendo el diagnóstico, no resulta difícil adivinar la respuesta falsa, que es la opción 1.


Recuerde que, entre las enfermedades ampollosas autoinmunes, la afectación oral es muy típica
del pénfigo vulgar, pero absolutamente extraordinaria en la dermatitis herpetiforme.(R1)

194. En relación al aspecto histológico de las lesiones de la imagen, marque la que


considere INCORRECTA:

En el estudio histológico destaca la acumulación de polimorfonucleares en las papilas


1. 1.
dérmicas.
Se produce una típica acumulación granular de IgA en las papilas de la dermis en la
2. 2.
inmunofluorescencia directa.
La dermatitis de la interfase con daño de la membrana basal orienta el diagnóstico
3. 3.
histológico aunque no es definitoria.
4. 4. Las vesículas o ampollas son subepidérmicas.
Gráfico de respuestas
Comentario

Desde el punto de vista histológico, la dermatitis herpetiforme está clasificada dentro del grupo
de los penfigoides, es decir, es una enfermedad ampollosa autoinmune cuyas ampollas se
encuentran a nivel subepidérmico. Más que ampollas, realmente cabría hablar de vesículas, que
mostrarán depósitos granulares de IgA con la inmunofluorescencia directa.

Lo que sería insólito encontrar en esta enfermedad sería una dermatitis de interfase, típica de
entidades como el liquen plano, el lupus eritematoso sistémico o la enfermedad injerto contra
huésped, por mencionar algunas. La respuesta incorrecta es la 3.(R3)

195. Un paciente sufrió una herida por proyectil de arma de fuego, que penetró en el
sexto espacio intercostal izquierdo por delante y salió en el mismo espacio por detrás.
No hay hipotensión ni signos de irritación peritoneal. El tratamiento de elección es:

1. 1. Ingreso en el hospital para vigilancia estrecha.


Exploración local de ambas heridas, e ingreso en el hospital para observación si no se
2. 2.
puede comprobar la penetración de la cavidad peritoneal.
3. 3. Lavado peritoneal, y si resulta positivo, laparotomía exploratoria.
4. 4. Laparotomía exploratoria.
Gráfico de respuestas
Comentario

Es una pregunta que no se volverá a fallar en cuanto estudiemos el manual o se te repita. Todas
las heridas por asta de toro o por arma de fuego necesitan laparotomía exploradora (opción 4),
sea evidente o no la perforación e independientemente de los síntomas. Se deben incluir
aquellas de tórax bajo, espalda y flancos. Esto es debido a que hay lesión en el 90% de los
casos. Además, trayectos extraperitoneales en una herida por arma de fuego pueden originar
lesiones intraperitoneales por el efecto de onda expansiva, de ahí la necesidad de la
laparotomía exploradora.(R4)

196. Niña de 6 años que presenta desde hace 3 días fiebre de 38º C que no cede con
antipiréticos, tos seca y facies congestiva. Acude por aparecer exantema
maculopapuloso, rojo intenso, no puntiforme, confluente, que no se blanquea a la
presión en la parte superior del tórax, cara y parte proximal de extremidades

!
!
!
!

superiores. A la exploración. T 40.3ºC, adenopatías bilaterales en ángulo mandibular y


lesiones blanquecinas sobre base eritematosa en mucosa subyugal. Respecto a la
enfermedad que padece esta paciente, es FALSO:

La neumonía de células gigantes de Hecht es menos frecuente que la neumonía por


1. 1.
sobreinfección bacteriana.
La gravedad de la enfermedad está directamente relacionada con la intensidad y
2. 2.
confluencia del exantema.
3. 3. El prurito suele ser intenso.
Entre las posibles complicaciones se encuentra la anergia cutánea y la reactivación de una
4. 4.
tuberculosis preexistente.
Gráfico de respuestas
Comentario

Hay que saber reconocer cuadro clínicos de enfermedades exantemáticas, son preguntadas año
con año.

Caso clínico sobre una sarampión.

La neumonía de células gigantes de Hecht es una complicación en pacientes inmunosuprimidos


y es menos frecuente que la sobreinfección bacteriana.

El prurito en el sarampión no suele ser intenso, por lo que la respuesta correcta es la número 3.

Recuerde que puede causar una reactivación de una TB latente.(R3)

!
!
!
!

197. La microcefalia es una manifestación de todos los procesos siguientes,


EXCEPTO:
1. 1. La rubéola congénita.
2. 2. La infección congénita por citomegalovirus.
3. 3. Alcoholismo de la madre.
4. 4. Ingestión de talidomida por la madre.
Gráfico de respuestas
Comentario

La microcefalia infantil se define como la presencia de un perímetro cefálico menor de dos


desviaciones estándar con respecto a la media para edad. Dentro de las microcefalias infantiles,
ha de considerar cuadros de muy diversa índole, como la que aparece secundariamente a
encefalopatías hipóxico-isquémicas por atrofia de parénquima cerebral, y otras como la que
acompaña a la rubéola congénita, la secundaria a CMV, alcoholismo y fenilcetonuria. Respuesta
correcta 4.(R4)

198. En una de las siguientes situaciones NO suele observarse disminución en la


difusión pulmonar del CO:

1. 1. Asma.
2. 2. Sarcoidosis.
3. 3. Anemia.
4. 4. Fibrosis intersticial idiopática.
Gráfico de respuestas
Comentario

Hay disminución de la DLCO en patología con perdida de superficie alveolar (enfisema, FPI, o
sarcoidosis), con aumento del espacio alveolo-capilar (FPI o sarcoidosis), cuando hay bajo flujo
sanguíneo, o hay destrucción capilar (FPI y sarcoidosis), y cuando existe anemia. En principio,
en el asma no tiene por que haber alteraciones en la DLCO.(R1)

199. ¿Cuál es el diagnóstico correcto de un paciente hipertenso y con enfermedad


arteriosclerótica que acude por pérdida brusca e indolora de visión en ojo derecho,
presentando trastorno de la conducción pupilar aferente, y en cuya exploración
oftalmoscópica la retina aparece blanca y edematosa, con estrechamiento arterial,
segmentación de la columna sanguínea y “mancha rojo cereza”?:

1. 1. Neuropatía óptica isquémica.


2. 2. Desprendimiento de retina.
3. 3. Obstrucción de la vena central de la retina.
4. 4. Obstrucción de la arteria central de la retina.
Gráfico de respuestas
Comentario

Esta es una pregunta que debe dominar en todo momento. La obstrucción de la arteria central
de la retina cursa como una ceguera de instauración BRUSCA E INDOLORA cuyo fondo de ojo
es característico: la retina aparece pálida por ausencia de irrigación salvo un área descrita como
"mancha rojo cereza" que se corresponde a la mácula, irrigada por la coriocapilar.(R4)

!
!
!
!

200. Sobre las anemias refractarias señale la respuesta FALSA:

La anemia refractaria con exceso de blastos tipo 2 tiene un porcentaje de blastos en


1. 1.
médula ósea entre 11-19%.
El pronóstico de vida de la anemia refractaria con sideroblastos en anillo oscila entre 30-
2. 2.
60 meses.
Las leucemias que surgen de estos síndromes tienen mejor pronóstico si conseguimos
3. 3.
tratar el proceso subyacente.
4. 4. El único tratamiento definitivo es el trasplante de médula ósea.
Gráfico de respuestas
Comentario

Pregunta de dificultad media, fácil de responder con un poco de sentido común. Hay un
concepto que no debe olvidar, toda leucemia que surja de otro proceso hematológico (leucemias
secundarias), tiene un pronostico peor (opción 4 falsa) que las que son de origen idiopatico. Los
síndromes mielodisplásicos se clasifican en cinco subtipos según la FAB (clasificación Franco-
Americano-Británico); la anemia refractaria con sideroblastos en anillo (con un porcentaje de
sideroblastos en anillos>15%) es el subtipo de mejor pronostico, con una sobrevida superior a
los 30 meses. En cambio la anemia refractaria con exceso de blastos tipo 2 es el subtipo de
peor pronóstico, con un porcentaje de blastos en médula ósea entre 11-19%.(R3)

201. La existencia de dolor acompañado de un cuadro agudo de irritación peritoneal


en una paciente portadora de un mioma nos debe orientar hacia:

1. 1. Degeneración maligna.
2. 2. Torsión de un mioma pediculado.
3. 3. Parto del mioma.
4. 4. Mioma intraligamentario.
Gráfico de respuestas
Comentario

En mujeres con antecedentes de miomas que acuden por dolor abdominal y signos de irritación
perioneal, lo primero que debe pensar es la torsión. Respueta 2.(R2)

202. Señale lo CIERTO con respecto a la exploración del pulso venoso yugular:

1. 1. La onda a tiene lugar poco antes de la sístole.


Normalmente al hacer una inspiración profunda aumenta considerablemente la presión
2. 2.
venosa yugular.
3. 3. En la insuficiencia tricuspídea, la onda v es pequeña.
4. 4. En la fibrilación auricular frecuentemente hay ondas a cañón.
Gráfico de respuestas
Comentario

Ésta es una pregunta de dificultad moderada acerca del pulso venoso yugular, dentro de la
semiología cardíaca.

!
!
!
!

La onda “a” se debe a la contracción auricular, que tiene lugar al final de la diástole, por lo que
es una onda presistólica (respuesta 1 correcta). Es muy importante conocer las variaciones de
las ondas del pulso en las distintas patologías.

En la fibrilación auricular, hay una actividad eléctrica desorganizada (sin ondas P). Por ello, en
ella no existen ondas “a”.

En la insuficiencia tricuspídea, existe un volumen de sangre que pasa a la aurícula derecha


durante la contracción ventricular, por lo que la onda “v” está aumentada.

De la misma forma, la descendente “x” está invertida, ya que la válvula tricúspide no se desplaza
hacia el ventrículo durante la relajación de la aurícula.

En condiciones normales, la presión venosa yugular disminuye durante la inspiración, al haber


en el tórax presiones negativas. Si aumentara la presión venosa durante la inspiración, se habla
de signo de Kussmaul, que aparece en situaciones de insuficiencia cardíaca derecha.(R1)

203. Respecto a la relación entre gota y nefrolitiasis, es CIERTO que:

1. 1. El riesgo de litiasis úrica es muy alto cuando la uricosuria está entre 400 y 600 mg diarios.
Los pacientes con gota sufren una mayor incidencia de litiasis cálcica que la población
2. 2.
general.
3. 3. La mayoría de los pacientes con litiasis cálcica son, presentan además gota.
Cuando un individuo con gota presenta litiasis úrica, puede asegurarse que ya sufre un
4. 4.
deterioro significativo de la función renal.
Gráfico de respuestas
Comentario

La hiperuricemia, además de provocar lesiones en las articulaciones, produce aumento de la


concentración de ácido úrico en la orina, lo que facilita la formación de pequeños cálculos que
sirven de catalizador para la formación de litiasis cálcicas.(R2)

204. Paciente de 2 meses que, al introducirle la lactancia artificial con fórmula


humanizada de PLV, presenta una reacción anafiláctica, detectándose en el estudio
alérgico hipersensibilidad inmediata frente a leche y huevo. A los 3 meses, a la hora de
vacunarle con la triple vírica, ¿cuál sería la actitud más correcta?:

1. 1. Esperar al año de vida y repetir el estudio alérgico.


2. 2. Administrarle la triple vírica obtenida en cultivo de células diploides humanas.
3. 3. Desensibilizarle frente a proteínas de huevo y a continuación vacunarle.
4. 4. No existe ninguna contraindicación para administrarle la vacuna triple vírica.
Gráfico de respuestas
Comentario

Pregunta complicada pero que debe intentar acertar con conocimientos básicos y un poco de
lógica. Nos encontramos ante un paciente que es alérgico a las proteínas del huevo y al que se
propone la vacunación con la triple vírica. Este es una vacuna de virus vivos atenuados en
cultivo sobre proteína de huevo, por lo que aquí se deducen sus principales contraindicaciones:
inmunodeficiencias, embarazo y alergia la huevo. Evidentemente el beneficio de administrarla es
muy alto, por lo que se debe intentar conseguir de otra manera, en este caso en cultivo diploide

!
!
!
!

humano. Por ello esta es la solución, conseguir una inmunización adecuada sin problemas con
su alergia al huevo.(R3)

205. Como muchas otras enfermedades sistémicas, la afección por el virus de la


inmunodeficiencia humana (VIH) se ha relacionado con manifestaciones oculares: se
han descrito alteraciones intraoculares, tanto en el segmento anterior como en el
segmento posterior, alteraciones neurooftalmológicas y alteraciones a nivel de la
órbita. La manifestación oftalmológica más frecuente del VIH es:

1. 1. Retinopatía del SIDA.


2. 2. Necrosis retiniana aguda.
3. 3. Retinitis por CMV.
4. 4. Coriorretinitis por Toxoplasma.
Gráfico de respuestas
Comentario

La alteración, que se produce con mayor frecuencia en este segmento posterior, con apenas
repercusión visual, es la retinopatía del SIDA.

En la retinopatía del SIDA se observa, fundamentalmente la presencia de exudados


algodonosos (microinfartos) y hemorragias retinianas, indistinguibles de las observadas en otras
vasculopatías (HTA, Diabetes Mellitus,etc), que se localizan alrededor de las arcadas vasculares
y que suelen desaparecer al cabo de 4 a 8 semanas, sin implicar ningún déficit visual, pudiendo
reaparecer luego en otras localizaciones.

Tenga mucho cuidado con esta pregunta. La retinitis por CMV es más conocida, por su grave
repercusión visual (a diferencia de la retinopatía SIDA). Sin embargo, eso no significa que sea
más frecuente.(R1)

206. With respect to cerebral palsy, which of the following is FALSE:

It is defined as any non-progressive intellectual deficit, which is caused by events that


1. 1.
occurred in the perinatal period.
2. 2. Brain malformations and vascular obstructions are etiologic factors.
3. 3. Spastic form is the most common type.
4. 4. In the extrapyramidal form, dystonic movements usually appear after 6 months of age.
Gráfico de respuestas
Comentario
La parálisis cerebral es un déficit de la neurona motora superior secundaria a un evento pre o
perinatal. Muchas veces se asocia éste a un déficit intelectual, si bien esta condición no es
necesaria. La forma más frecuente es la espástica, si bien hay otras variedades posibles, como
la distónica y la atáxica. A veces el aspecto macroscópico del cerebro es rigurosamente
normal.(R1)

207. A 5-month-old infant is brought to your outpatient clinic due to liquid stools (4 per
day), occasional vomiting and fever up to 38.3ºC, coinciding with a respiratory infection
in the upper airways. He is fed with breastmilk and bottle. Physical examination reveals
good general condition. He is well nourished and hydrated and his current weight is
4730 g (70 g less than last week). The abdomen is normal, and the fontanelle is soft and

!
!
!
!

relatively flat. Except for the presence of watery rhinorrhea, the rest of the examination
of is normal. Which of the following statements is TRUE:

You must estimate losses, recommend a 4-hour fast and rehydrate with oral rehydration
1. 1.
solution.
2. 2. The introduction of rice cereal is recommended, due to its astringent effect.
3. 3. You should recommend the use of lactose-free formula.
You should replace ongoing fluid losses after each deposition with oral rehydration
4. 4.
solution, and continue with regular diet.
Gráfico de respuestas
Comentario

Pregunta sencilla acerca de la gastroenteritis infecciosa. Fácil de recocnocer al tratarse de un


lactante que tiene vómitos, diarrea y fiebre. Al ser la exploración física normal, con un buen
estado de hidratación y sin pérdida significativa de peso (< 3 % del peso previo), estará indicado
continuar con su alimentación habitual, reponiendo las pérdidas tras cada vómito o deposición
con una solución de rehidratación oral (opción 4 correcta). La modificación de la dieta habitual
no está indicada al no modificar el curso de la sintomatología (opciones 1, 2 y 3 incorrectas).(R4)

208. Paciente de 30 años que acude "muerto de miedo" porque su padre ha fallecido
recientemente de un adenocarcinoma de colon y le han comentado que es hereditario.
En sus familiares cercanos también un abuelo ha padecido esta enfermedad y a su tía
le acaban de diagnosticar un adenocarcinoma de endometrio. Usted le recomienda:

Que se tranquilice; aunque existe un condicionante genético, todavía no está en la edad de


1. 1.
riesgo.
2. 2. Colectomía profiláctica debido a su alto riesgo.
Sólo es hereditario el adenocarcinoma asociado a poliposis, así que, si no es así, no hace
3. 3.
nada.
4. 4. Colonoscopia completa.
Gráfico de respuestas
Comentario

El cáncer de colon tiene un componente hereditario incluso cuando aparece de forma


esporádica… Pero en esta ocasión puede que sea algo más. Observe que tiene varios
familiares con el mismo problema y una tía padece un cáncer de endometrio, por lo que
posiblemente se trate de una persona con síndrome de Lynch tipo II. En estos casos, se
recomienda realizar colonoscopias periódicas, a pesar de la juventud del paciente: a partir de los
25 años, o tres años antes del debut del familiar más joven afecto.(R4)

209. Un paciente masculino de 52 años es diagnosticado de adenocarcinoma gástrico


a nivel del antro. No se detecta enfermedad metastásica. Se realiza una laparoscopia
estadificadora donde se encuentra que el tumor afecta por contigüidad al segmento III
hepático. ¿Qué opción terapéutica le parece más adecuada para este paciente?:

1. 1. Gastrectomía paliativa.
2. 2. Gastrectomía, linfadenectomía y resección hepática.
3. 3. Quimioterapia neoadyuvante.
4. 4. Radioterapia neoadyuvante.
Gráfico de respuestas

!
!
!
!

Comentario

En principio el paciente presenta un tumor localmente avanzado que no presenta criterios de


irresecabilidad. Es importante que reconozca los criterios de irresecabilidad con intencionalidad
curativa que son:

-La invasión peritoneal o metástasis a distancia, incluida la citología positiva del líquido ascítico.

-La imposibilidad de realizar una resección completa.

-La afectación de los niveles ganglionares 3 y 4 confirmada por biopsia o muy sugestiva por
pruebas de imagen y la invasión de estructuras vasculares mayores.

La quimioterapia perioperatoria en los adenocarcinomas gástricos y de la unión esófago-gástrica


(y tercio inferior del esófago) aumenta la supervivencia global y libre de progresión respecto a la
cirugía sola, sin aumentar las complicaciones postoperatorias ni la mortalidad quirúrgica, por lo
tanto, se debe contemplar la administración de quimioterapia neoadyuvante en todos los
pacientes con enfermedad localmente avanzada, con comorbilidad y situación funcional
aceptables, salvo que sea obligada la cirugía de entrada por síntomas obstructivos o
hemorragia.

El tratamiento estándar consiste en administrar tres ciclos de 21 días basados en la combinación


de un derivado del platino (cisplatino u oxaliplatino), una fluoropirimidina (capecitabina o 5- FU)
y, frecuentemente, una antraciclina (en general epirrubicina), seguidos tras 3 a 6 semanas de
cirugía, siempre que no haya enfermedad metastásica y comenzando por una laparoscopia
diagnóstica, a criterio del equipo quirúrgico, si existen dudas sobre la resecabilidad.(R3)

210. A 65-year-old asymptomatic patient is found to have elevated serum alkaline


phosphatase with normal liver function. Bone gammagraphy shows abnormal uptake
in the upper half of right hemipelvis. Thickened cortical bone is seen in the pelvic X-
ray film. Which of the following is the most appropriate treatment?

1. 1. Alendronate. 40 mg per day.


2. 2. Naproxen 500 mg twice daily. (b.i.d)
3. 3. Calcium and Vitamin D.
4. 4. No medical treatment is necessary.
Gráfico de respuestas
Comentario

El caso clínico representa un paciente con enfermedad de Paget. Observa el patrón radiológico,
las alteraciones gammagráficas y la elevación de la fosfatasa alcalina, que orientan hacia este
diagnóstico. No obstante, la primera línea del enunciado deja muy claro que está asintomático.
Ésta es la clave de la pregunta.

La enfermedad de Paget asintomática no precisa sino observación. Cuando aparecen


complicaciones (dolor, deformidades óseas, repercusión renal…), estaría indicado instaurar
tratamiento, normalmente con bifosfonatos, que son más potentes y eficaces que la
calcitonina.(R4)

!
!
!
!

Indicación de tratamiento farmacológico

211. De entre los siguientes factores que se nombran indique el que no influye
negativamente, de forma directa, en el proceso de cicatrización:

1. 1. Estado nutricional.
2. 2. Obesidad.
3. 3. Insuficiencia renal.
4. 4. Infección.
Gráfico de respuestas
Comentario

Existen unos factores generales y locales que modulan la cicatrización y conviene reconocer:

–Estado nutricional (hipoproteinemia) y vitamínico (fundamentalmente vitamina C)

–Diabetes mal controlada

–Obesidad: existe más tensión en la herida, necrosis grasa y déficit de riego sanguíneo, lo que
favorece la infección.

–Oxigenación/perfusión: anemia

–Corticoides: alteran la epitelización y la síntesis de colágeno.

–Quimioterapia, radioterapia (que alteran la división celular)

–Infección: es el factor local que más influye, alterando la angiogénesis, la actividad fibroblástica
y aumentando la actividad proteolítica.

Como vemos, la insuficiencia renal no se encuentra entre estos factores (respuesta 3


incorrecta).

(R3)

!
!
!
!

212. Todos los siguientes medicamentos deben ser dados con precaución en la
insuficiencia renal crónica avanzada EXCEPTO:

1. 1. Inhibidores de la enzima conversora de angiotensina.


2. 2. Diuréticos distales.
3. 3. Soluciones hiperosmolares.
4. 4. Diuréticos tiazidicos.
Gráfico de respuestas
Comentario

Los efectos adversos de los fármacos son pregunta típica del ENARM. Dentro de las reacciones
adversas de los IECAs recuerde la tos, la disgeusia y la neutropenia (ésta es infrecuente), la
insuficiencia renal aguda ante una estenosis bilateral de la arteria renal (o unilateral en
monorrenos) y la teratogenia. Sin embargo los IECAs son altamente beneficiosos en la HTA
principalmente de origen renal y tienen efecto nefroprotector de la nefropatía diabética, también
están indicado en la IC y en el IAM. Sólo si el paciente toma simultáneamente IECAs,
betabloqueantes y es diabético, habría tendencia a la hiperpotasemia. Sin embargo los
diuréticos distales (ej. espironolactona) hay que administrarlos con mucho cuidado en la IRC por
el alto riesgo de hiperpotasemia. Las soluciones hiperosmolares tienen riesgo en la insuficiencia
renal avanzada porque cuando el filtrado glomerular es menor del 10% hay retención de agua y
sal y aumentaría la osmolaridad del medio interno. Recuerde que la administración de tiazidas a
un paciente con filtración glomerular menor de 40ml/min no es útil, y sólo tenemos los efectos
adversos (ej. hipercalcemia).(R1)

213. Si al realizar una tinción de Gram, olvidamos decolorar la extensión:

1. 1. Podemos ver teñida de azul una bacteria gramnegativa.


2. 2. Podemos ver teñida de rosa una bacteria gramnegativa.
3. 3. No veremos teñidas las bacterias grampositivas.
4. 4. No veremos teñidas las bacterias gramnegativas.
Gráfico de respuestas
Comentario
La tinción de Gram es un procedimiento simple que da importante información sobre la
naturaleza de una infección: identifica al germen como bacilo o coco, grampositivo o
gramnegativo. El colorante básico es el violeta de genciana que tiñe las bacterias de azul, se
utiliza como mordiente el lugol. Tras tratar con alcohol o alcohol- acetona las bacterias
grampositivas permanecen azules, mientras que las gramnegativas se decoloran. Tras la
decoloración, la preparación se trata con fucsina o safraina y las bacterias gramnegativas toman
una coloración rosa [las grampositivas permanecen azules]. En caso de omitir el paso de
decoloración, las bacterias gramnegativas permanecerían azules y aparecerían en la extensión
como bacterias grampositivas [azules].(R1)

214. Paciente femenino de 55 años, fumadora habitual, a la que se le descubre en una


revisión anual un nódulo de 1.5 cm duro, fijo e indoloro en el cuadrante superoexterno
de la mama. En la exploración física no se palpan adenopatías axilares ni
supraclaviculares. La mastografía muestra imágenes sugestivas de malignidad. Se

!
!
!
!

realiza biopsia intraoperatoria, cuyo resultado es carcinoma ductal infiltrante. La


actitud terapéutica más adecuada es:

1. 1. Mastectomía simple.
2. 2. Tumorectomía + radioterapia + biopsia del ganglio centinela.
3. 3. Quimioterapia + hormonoterapia si receptores estrogénicos positivos.
4. 4. Mastectomía radical (Halsted).
Gráfico de respuestas
Comentario

Debe dominar el tratamiento del cáncer de mama o si o si.

La conducta más adecuada en este caso es la realización de tumorectomía + RT + biopsia de


ganglio centinela (RC- 2).

Se puede realizar una tumorectomía cuando el nódulo es < 5 cm, siendo posible la RT posterior.
Además, el tumor no debe ser multicéntrico y ni un carcinoma inflamatorio. La paciente de este
caso clínico cumple todos estos requisitos

La radioterapiaestá indicada en todos los casos tras la cirugía conservadora.

También es necesario realizar la biopsia del ganglio centinela. El ganglio centinela es el primer
ganglio de una cadena linfática que drena un territorio tisular determinado. Si la biopsia es
negativa, no es necesario realizar una linfadenectomía.(R2)

215. Acude a su consulta un niño de 4 años con un síndrome nefrótico, ¿cuál de los
siguientes NO es un pilar del tratamiento?

1. 1. Diuréticos.
2. 2. Inmunosupresores.
3. 3. Aspirina.
4. 4. IECAS.
Gráfico de respuestas
Comentario

Para contestar a esta pregunta hay que saber que la causa más frecuente de síndrome nefrótico
en niños es la nefropatía por cambios mínimos, que se trata con esteroides, y por otro lado el
tratamiento del síndrome nefrótico: IECAs, diuréticos y aspirina, para evitar complicaciones. No
se utilizan inmunosupresores, sino corticoides (respuesta 2). En caso de no responder a los
corticoides, se procedería a la realización de una biopsia renal, y se pautaría tratamiento en
función del resultado.(R2)

216. ¿Cuál de las siguientes afirmaciones respecto a los antidiabéticos orales es


CIERTA?

Las biguanidas inhiben la neoglucogénesis y aumentan la actividad o el número de


1. 1.
receptores de la insulina.
Los diabéticos tipo 2 que no responden a la dieta y el ejercicio, a veces se controlan con
2. 2. sulfonilureas o con combinación de antidiabéticos orales, antes de probar el tratamiento
con insulina.

!
!
!
!

Las tiazolidinedionas reducen la resistencia a la insulina a nivel periférico, aunque


3. 3.
aumentan los niveles plasmáticos de ácidos grasos libres.
Las biguanidas pueden usarse en monoterapia en casos de diabéticos tipo 2 obesos, pero
4. 4.
no deben utilizarse con sulfonilureas.
Gráfico de respuestas
Comentario

En el tratamiento de la diabetes tipo 2, cuando falla el tratamiento inicial de dieta y ejercicio, se


debe comenzar con el antidiabético oral más adecuado en monoterapia según las
características del paciente. El siguiente escalón sería la asociación de otro u otros
antidiabéticos orales antes de iniciar tratamiento con insulina. Si el paciente es obeso, es de
elección la metformina (biguanida). Estos fármacos producen una disminución de la producción
hepática de glucosa (inhibe la neoglucogénesis hepática), aumenta la captación de glucosa a
nivel periférico y disminuye su absorción gastrointestinal. La mayor actividad de los receptores, o
el incremento de su número, sería uno de los mecanismos de las sulfonilureas. Las
tiazolidinedionas reducen la resistencia a la insulina a nivel periférico aumentando la captación y
utilización de la glucosa mediada por insulina, su efecto a nivel de la producción hepática de
glucosa es mínimo. Además, reducen los ácidos grasos libres y triglicéridos en plasma.(R2)

217. Mujer de 53 años,


nuligesta y sin antecedentes personales de interés, que acude al servicio de urgencias
por presentar dolor abdominal inespecífico de varios meses de evolución, más intenso
en los últimos quince días. Se encuentra afebril y normotensa. En la exploración, el
abdomen está aumentado de tamaño y es doloroso a la palpación en hipogastrio. Se
solicita un USG abdominal donde se observa hígado y vías biliares sin alteraciones,
bazo dentro de la normalidad, ambos riñones de morfología normal, con leve dilatación
pielocalicial derecha. Se observa moderada cantidad de líquido libre intraabdominal y
una masa a nivel pélvico. Se solicita interconsulta a ginecología para completar la
evaluación de la paciente. En la exploración ginecológica, mediante tacto bimanual se
palpa la masa a nivel de fondo de saco de Douglas y un útero de tamaño y consistencia
normal. En el doppler realizado se observan las imágenes que se muestran en la
imagen. NO esperaría encontrar en la ecografía:

1. 1. Presencia de septos gruesos y partes sólidas en el interior de la tumoración.


2. 2. Alto índice de resistencia Doppler en los vasos tumorales.
3. 3. Moderada cantidad de líquido libre en Douglas.
4. 4. Multilocularidad.
Gráfico de respuestas

!
!
!
!

Comentario

Actualmente, el ultrasonido es una de las técnicas de imagen más efectivas en el diagnóstico del
cáncer de ovario, con alta sensibilidad diagnóstica. Dentro de las características
ultrasonográficas de una masa ovárica que sugieren malignidad, se encuentran: el tamaño
tumoral superior a 5-10 cm, el contenido heterogéneo, la presencia de septos o tabiques
gruesos, la multilocularidad, la presencia de líquido libre en pelvis o ascitis, altos índices de
pulsatilidad doppler y bajos índices de resistencia doppler. Por ello, la respuesta incorrecta es la
número 2.(R2)

218. La paciente del caso anterior fue remitida a consulta de ginecología para manejo
y tratamiento adecuado de su patología, en la que se solicitó un estudio de extensión
mediante TC. Los hallazgos que se observan son: imagen anexial derecha compleja de
aproximadamente 8 cm de diámetro, anexo izquierdo de apariencia normal, útero
regular, ascitis en cantidad moderada y pequeños implantes peritoneales de
aproximadamente 1 cm de tamaño, sin observarse afectación de cadenas
ganglionares. No hay afectación de parénquima hepático. El resto de exploración no
presenta hallazgos de interés. Dada la sospecha diagnóstica del estudio de extensión,
que se confirma posteriormente mediante cirugía, se trataría de un estadio tumoral:

1. 1. IC.
2. 2. IIB.
3. 3. IIIA.
4. 4. IIIB.
Gráfico de respuestas
Comentario

La respuesta correcta es la número 4, ya que cuando existen implantes peritoneales


macroscópicos pero de tamaño inferior a 2 cm, se considera un estadio III B de cáncer de
ovario. En el caso de ser implantes superiores a 2 cm o de existir adenopatías retroperitoneales,
sería un estadio IIIC.(R4)

219. Un hombre de 57 años acude a urgencias por debilidad y melenas durante 3 días.
Dice no haber tenido dolor abdominal importante y no ha tenido ninguna hemorragia
digestiva anterior. En la exploración aparenta más edad de la que refiere y hay un
descenso de 20 mmHg en la presión arterial al ponerse en pie, conjuntivitis pálidas y
anictéridas, "spiders" angiomatosos en parte superior del torso, atrofia muscular,
hepatoesplenomegalia y ruidos intestinales aumentados sin hipersensibilidad local en
la palpación abdominal. Las heces son melénicas. La aspiración nasogástrica
demuestra un material en posos de café, que desaparece rápidamente con el lavado.
El hematócrito es del 30 por 100 y el volumen corpuscular medio es de 105 fl. Se inicia
un lavado gástrico con sulución salina. Lo primero que debe hacerse a continuación
es:

1. 1. Colocar un tubo de Sengstaken-Blackmore y comenzar un goteo endovenoso de pitresina.


2. 2. Ordenar un estudio radiológico gastroduodenal seriado.
3. 3. Ordenar una angiografía visceral urgente.
Insertar una amplia vía de acceso intravenoso, determinar el grupo sanguíneo y realizar
4. 4.
pruebas cruzadas con la sangre del enfermo.
Gráfico de respuestas

!
!
!
!

Comentario
Se trata de un paciente con una hemorragia digestiva alta ya que presenta melenas y posos de
café en la aspiración nasogástrica. La ausencia de dolor abdominal, la hepatoesplenomegalia y
los "spiders" orientan hacia una hemorragia por rotura de varices esofágicas en un paciente con
una posible hepatopatía crónica. Sin embargo, determinar la causa de la hemorragia no es lo
primero que se debe hacer en este paciente: presenta hipotensión ortostática (descenso de 20
mmHg de la presión arterial al ponerse de pie) por lo que lo prioritario, antes de tomar ninguna
otra medida, es la estabilización hemodinámica: es decir, canalizar un acceso venoso y reponer
la volemia.(R4)

220. A 50-year-old male patient presents to his dermatologist with concerns regarding
recent episodes of blistering skin lesions after sun exposure. On further questioning,
it is known that his urine is occasionally dark. Laboratory studies are significant for a
slight elevation of transaminases and an increased urinary excretion of uroporphyrin
(over 200 μg/dl). Mark the CORRECT diagnosis and most appropriate treatment:

1. 1. Doss porphyria. Prophylaxis with lead compounds.


2. 2. Acute intermittent porphyria. Prevention of episodes with chlordiazepoxide.
3. 3. Porphyria cutanea tarda. Chloroquine is a possible treatment.
4. 4. Erythropoietic protoporphyria. Periodic phlebotomies are a possible treatment.
Gráfico de respuestas
Comentario

Es una porfiria autosómica dominante o bien adquirida, relacionándose con el consumo de


alcohol, estrógenos o exposición a tóxicos. Es debida a la deficiencia de uroporfirinógeno
descarboxilasa que provoca un acúmulo de uroporfirinógeno III. Comienza con hiperfragilidad
cutánea en dorso de manos (ampollas, erosiones, quistes de millium) más hiperpigmentación
facial e hipertricosis malar relacionada con una marcada fotosensibilidad.

La afectación hepática suele ser asintomática. El diagnóstico se confirma mediante la presencia


de uroporfirina I y III. El tratamiento reside en evitar los desencadenantes, flebotomías
periódicas o dar cloroquina oral.(R3)

221. Mujer de 44 años, usuaria activa de drogas por vía parenteral, consulta por fiebre
con escalofríos en las últimas 72 horas. En la exploración física presenta buen estado
general, TA 110/65 mmHg, FC 89 lpm, FR 12 rpm, saturación basal de O2 96%, estigmas
de venopunción en ambos miembros superiores, y ausencia de soplos a la
auscultación cardíaca. La radiografía simple de tórax es normal. A las 25 horas de
instaurar tratamiento antibiótico empírico, le informan desde el Servicio de
Microbiología del aislamiento de Staphylococcus aureus sensible a la oxacilina (SAOS)
en todos los hemocultivos extraídos inicialmente. Tras confirmar mediante un
ecocardiograma transtorácico su sospecha diagnóstica, ¿cuál de las siguientes
considera que debe ser la actitud terapéutica más apropiada?:

1. 1. Cloxacilina y gentamicina durante 6 semanas, sin reparación quirúrgica.


2. 2. Cloxacilina y gentamicina durante 4 semanas, seguidas de recambio valvular.
3. 3. Vancomicina y gentamicina durante 4 semanas, sin reparación quirúrgica.
4. 4. Cloxacilina y gentamicina durante 2 semanas, sin necesidad de reparación quirúrgica.
Gráfico de respuestas

!
!
!
!

Comentario

Esta paciente presenta un cuadro clínico-microbiológico muy sugerente de endocarditis


tricuspídea por S. aureus oxacilín- sensible, tal y como parece confirmar el resultado de la
ecocardiografía. Recuerda que tales casos suelen cursar sin soplo cardíaco. La normalidad de la
radiografía de tórax y la ausencia de insuficiencia respiratoria descartan de forma razonable la
presencia de embolias sépticas pulmonares como complicación asociada. Las endocarditis
producidas por S. aureus oxacilín-sensible deben tratarse con cloxacilina más gentamicina; por
regla general, este tratamiento se mantiene de 4 a 6 semanas, suspendiendo el aminoglucósido
a la tercera semana. Sin embargo el tratamiento de las endocarditis tricuspídeas no complicadas
por S. aureus, como es el caso, puede limitarse a tan sólo 2 semanas (respuesta 4 correcta).
Recuerde que la drogadicción activa constituye una contraindicación formal para el recambio
valvular quirúrgico en un paciente con endocarditis.(R4)

222. Durante su guardia en un servicio


de neonatología, le avisan por el parto de un RN a término con bajo peso para su edad
gestacional que presenta una llamativa malformación en el momento del parto, como
se observa en la imagen. ¿Cuál es su diagnóstico?:

1. 1. Onfalocele.
2. 2. Granuloma umbilical.
3. 3. Gastrosquisis.
4. 4. Pólipo umbilical.
Gráfico de respuestas
Comentario

Las masas umbilicales se pueden clasificar según su tamaño en grandes o pequeñas. Dentro de
las grandes reciben un nombre distinto en función de cuál sea su envoltura externa.

En el caso de que las vísceras abdominales se externalicen sin ningún tipo de cubierta externa
como es el caso en la imagen que nos muestran recibe el nombre de gastrosquisis. En este tipo
de malformaciones los intestinos típicamente se hernian a través de los músculos rectos
abdominales, en un defecto que casi siempre está a la derecha del cordón umbilical, y su
manejo es quirúrgico (respuesta correcta la 3).

!
!
!
!

En cambio hablamos de hernia umbilical cuando la masa está cubierta por piel; desaparece
espontáneamente al cerrarse el anillo umbilical fibromusucular durante los primeros 3 años de
vida en la mayoría de los casos. La hernia umbilical es una tumoración blanda, no dolorosa y
fácilmente reducible y su tratamiento de entrada es conservador (sólo precisará cirugía si
persiste más allá de los 3 o 4 años o la hernia se complica).

El onfalocele es una masa umbilical cubierta por peritoneo (respuesta 1 falsa) que puede
asociarse a malformaciones cardíacas, síndrome de Down y síndrome de Beckwith-
Wiedemann, siendo su tratamiento siempre quirúrgico.

Dentro de las masas umbilicales pequeñas destacan el granuloma umbilical y el pólipo. Este
último es rojo brillante y duro, se produce por persistencia de la mucosa intestinal en el ombligo
y su tratamiento es quirúrgico (respuesta 4 falsa).(R4)

223. Respecto a la patología que presenta el paciente de la pregunta anterior, señale la


respuesta FALSA:

1. 1. En este tipo de malformaciones, el intestino suele ser más corto de lo normal.


2. 2. La anomalía congénita más frecuentemente asociada es la atresia intestinal.
3. 3. Es recomendable el uso de antibióticos profilácticos.
El cierre del defecto debe realizarse siempre de forma primaria y urgente, estando
4. 4.
contraindicado el uso de mallas protectoras.
Gráfico de respuestas
Comentario

Se trata de una pregunta de alta dificultad que requiere conocimientos especializados en el


campo de la neonatología.

Tanto la gastrosquisis (que vemos en la imagen) como el onfalocele pueden asociar otras
malformaciones congénitas, siendo estas menos frecuentes en el caso de la gastrosquisis, pero
siendo la atresia intestinal la más frecuentemente asociada (respuesta 2 verdadera).

En ambos casos el intestino suele ser más corto de lo normal y tienen mayor predisposición a
volvularse (respuestas 1 verdadera).

Dado el evidente riesgo infeccioso que supone la puerta de entrada de gérmenes a través de la
pared abdominal con los órganos expuestos, es recomendable la utilización de antibióticos
profilácticos (respuesta 3 verdadera).

En cambio la respuesta 4 es FALSA ya que no siempre el cierre del defecto puede realizarse de
forma primaria, sólo si las condiciones lo permiten, y si no es así debe hacerse por etapas
recurriendo al uso de mallas protectoras que permiten proteger el intestino mientras se logra su
reducción completa a la cavidad abdominal.(R4)

224. Lactante de tres meses de vida que desde hace un mes presenta episodios
intermitentes de distensión abdominal, dolores de tipo cólico y algunos vómitos.
Tendencia al estreñimiento. Entre sus antecedentes personales hay que destacar que
fue prematuro, pesó 900 g al nacimiento y tuvo dificultad respiratoria importante que

!
!
!
!

precisó ventilación asistida durante 15 días. ¿Cuál es el diagnóstico más probable de


su cuadro clínico?

1. 1. Estenosis cólica secundaria a enterocolitis necrotizante.


2. 2. Megacolon congénito.
3. 3. Vólvulo intestinal intermitente.
4. 4. Adenitis mesentérica secundaria a neumopatía crónica.
Gráfico de respuestas
Comentario

La enterocolitis necrotizante suelen presentarla en el ENARM como un cuadro de distensión


abdominal y evacuaciones sanguinolentas. Sin embargo, esta pregunta va un poco más allá.
Observa que nos dan tres antecedentes que implicarían un mayor riesgo para esta enfermedad:
prematuridad, bajo peso, necesidad de ventilación asistida... Si a esto añadimos un cuadro
clínico compatible con obstrucción intestinal (estreñimiento, vómitos, distensión y dolor cólico), la
opción que mejor encaja es la 1.

De hecho, una de las complicaciones más habituales de esta entidad son las estenosis
intestinales, que generan cuadros suboclusivos, como en este lactante que nos comentan.(R1)

Enterocolitis necrotizante. Radiología

!
!
!
!

225. Un paciente de 42 años de edad consulta por molestias faríngeas y cierta afonía
de dos meses de evolución. NO es fumador ni tiene una profesión en que deba forzar
la voz. Una exploración laríngea revela un ligero edema de las cuerdas vocales y un
ligero eritema de la región interaritenoidea. Interrogado el paciente no refiere pirosis ni
regurgitación ácida. ¿Cuál de las siguientes afirmaciones es CIERTA?

La presencia de mínimas lesiones laríngeas indican que muy probablemente el paciente


1. 1. tenga también lesiones de esofagitis y por tanto hay que indicar una endoscopia digestiva
alta.
En ausencia de síntomas de broncoaspiración (tos y sibilancias), las molestias laríngeas no
2. 2.
pueden ser atribuidas a enfermedad por reflujo gastroesofágico.
La ausencia de síntomas de reflujo (pirosis y regurgitación) no descarta la enfermedad por
3. 3.
reflujo.
Puede averiguarse si la causa de los síntomas es una enfermedad por reflujo
gastroesofágico con una prueba corta administrando ranitidina 150 mg al día durante dos
4. 4.
semanas. La ausencia de mejoría sintomática descarta el reflujo como causa de los
síntomas laríngeos.
Gráfico de respuestas
Comentario

Pregunta de dificultad media sobre las complicaciones de la enfermedad por reflujo


gastroesofágico (ERGE). Es uno de los temas más preguntados dentro de la patología
esofágica, y éste a su vez del ENARM. Por eso, es primordial su estudio.

La enfermedad por reflujo gastroesofágico (ERGE) se define por la clínica o por daño histológico
en la mucosa esofágica. Es decir, puede existir la posibilidad de episodios de reflujo sin que por
ello implique enfermedad. Asimismo, puede no tener síntomas y sí existir cambios inflamatorios
en la mucosa esofágica y sí estaríamos ante una ERGE. La enfermedad surge por un
desequilibrio entre factores agresores (reflujo ácido, potencia del reflujo) y factores defensivos
(resistencia de la mucosa, aclaramiento del ácido por el esófago por peristalsis, saliva,
gravedad...).

El RGE suele ser asintomático si no existe esofagitis. Es la causa más común de dolor torácico
de origen esofágico. Sin embargo, la pirosis es el síntoma más frecuente de la ERGE. Otras
manifestaciones son regurgitación ácida, disfagia, hemorragias por ulceraciones en la mucosa.
En el ámbito extraesofágico puede ocasionar faringitis, laringitis posterior, broncoespasmo,
neumonía aspirativa, fibrosis pulmonar o asma crónico debido a microaspiraciones de ácido al
árbol respiratorio. En general, todos los síntomas son independientes entre sí y, por ejemplo,
aunque haya laringitis no implica por necesidad tener esofagitis o asma y laringitis, etc.

Cuando la clínica es típica (pirosis) es tan alta la probabilidad de ERGE que se da un


tratamiento empírico. Sólo en caso de refractariedad o complicaciones (estenosis, hemorragias,
úlceras, Barrett...) se indicaría una endoscopia (recuerda que actualmente la endoscopia es la
primera prueba a realizar, si bien la más sensible y específica es la pHmetría). Ésta diagnostica
y cuantifica el RGE. Se puede realizar un esofagograma para demostrar alteraciones
anatómicas como hernia de hiato.

El tratamiento incluye medidas higiénico-dietéticas: elevar la cabecera de la cama, aumentar las


proteínas de la dieta, disminuir la ingesta de grasas, chocolate, alcohol, tabaco... En el ámbito
farmacológico son de elección los inhibidores de la bomba de protones (IBP) como omeprazol,
lansoprazol, pantoprazol, rabeprazol... por ser los más eficaces. Si hay falta de respuesta o

!
!
!
!

existen ya complicaciones, se opta por una cirugía antirreflujo, siendo la más empleada la
funduplicatura de Nissen o posterior 360º.(R3)

A continuación esquematizamos lo expuesto en el siguiente algoritmo.

Esofagitis por reflujo. Diagnóstico y tratamiento

226. Si no aparece menstruación tras dar estrógenos y progestágenos durante tres


meses, pensará en:

1. 1. Anovulación.
2. 2. Endometritis tuberculosa.
3. 3. Hiperplasia de endometrio.
4. 4. Patología cérvico-uterina.
Gráfico de respuestas
Comentario

Esta pregunta puede parecerle difícil pero es muy importante que conozca el diagnóstico
diferencial de las amenorreas.

Ante una amenorrea en primer lugar se realiza un test de embarazo, si es negativo seguimos el
estudio con TSH y PRL, en caso de que sean normales damos una pequeña cantidad de
progesterona (5 días) si la paciente menstrua es porque existía una anovulación que impedía el
aumento de la progesterona. Si no aparece la regla administramos estrógenos y progestágenos
durante 3 meses, si no menstrua a pesar de una adecuada secuencia hormonal debemos
pensar en alteraciones genitales estructurales (como es este caso). Si menstrua tras la
administración hormonal combinada debemos pensar en un problema ovárico o hipotálamo-
hipofisário y para ello ya determinaremos gonadotropinas.(R4)

!
!
!
!

227. Respecto a la hipoglucemia del hijo de madre diabética, señale cuál de las
siguientes afirmaciones es FALSA:

Se observa en el 75% de los hijos de diabéticas y en el 25% de los hijos de madres con
1. 1.
diabetes gestacional.
2. 2. Aparece en las primeras 24 horas de vida y suele ser asintomática.
3. 3. Se piensa que es debida a hiperinsulinismo fetal.
Se trata mediante infusión i.v. de bolos de glucosa y debe retrasarse la alimentación oral
4. 4.
para minimizar el riesgo de hipoglucemia de rebote.
Gráfico de respuestas
Comentario

El tratamiento de la hipoglucemia del hijo de madre diabética consiste en:

- Control de la DM materna.

- Durante el parto, control de la glucemia materna lo que reduce el riesgo de hipoglucemia


neonatal (que a su vez es secundaria a la hiperinsulinemia neonatal).

- Inicio precoz de la alimentación.

- Si, a pesar de realizar una adecuada nutrición enteral, no se remonta la hipoglucemia, se


deberá proceder a la infusión de glucosa i.v.(R4)

228. La alteración del ritmo cardíaco que aparece más frecuentemente en la


intoxicación digitálica es:

1. 1. Taquicardia supraventricular.
2. 2. Extrasístoles ventriculares.
3. 3. Taquicardia ventricular.
4. 4. Flutter auricular.
Gráfico de respuestas
Comentario

Recuerda los efectos ECG de la digoxina:

- Prolongación del PR

- Acortamiento del QT

- Aplanamiento o inversión de la onda T

- Depresión del ST

- ESV (aleración del ritmo más frecuente en la intoxicación digitálica)

!
!
!
!

- Bloqueos AV

- Taquicardia supraventricular no paroxística con bloqueo AV variable

(R2)

229. Un niño de 18 meses presenta nistagmo pendular, cabeceo y tortícolis. En una RM


Craneal los hallazgos son normales. Lo más probable es que este niño tenga:

1. 1. Epilepsia.
2. 2. Neuroblastoma.
3. 3. Dismetría.
4. 4. Espasmo nutans.
Gráfico de respuestas
Comentario

Spasmus nutans es una enfermedad pediátrica que consiste en la triada de nistagmo, cabeceo y
tortícolis. La edad de aparición es alrededor de los 6 y 12 meses de edad. Esta condición
generalmente es benigna y autolimitada. Algunos casos se asocian con problemas retinianos o
neurológicos. Respuesta 4 correcta.(R4)

230. Todos los siguientes procesos MENOS uno son comunes al síndrome autoinmune
poliglandular tipo I y II ¿Cuál de ellos?:

1. 1. Addison.
2. 2. Hipoparatiroidismo.
3. 3. Hipogonadismo.
4. 4. Anemia perniciosa.
Gráfico de respuestas
Comentario

El síndrome tipo I comienza en la edad pediátrica. Produce la triada candidiasis mucocutánea,


hipoparatiroidismo e insuficiencia suprarrenal (Addison). El síndrome tipo II o de Schmidt
presenta diabetes tipo I, insuficiencia suprarrenal y tiroiditis, de inicio en la edad adulta. Puede
unirse a miastenia gravis. Cualquiera de los dos síndromes puede asociarse a otras
manifestaciones autoinmunes que no son características como hipogonadismo por fallo gonadal
primario, hipotiroidismo, alopecia areata, vitíligo, distrofia ungueal, queratopatía, anemia
perniciosa, celiaquía o hipoplasia del esmalte dental. Por tanto, de las opciones el
hipoparatiroidismo es la única manifestación característica del síndrome PGA tipo I.(R2)

231. ¿Cuál de los siguientes le hace pensar en existencia de sufrimiento fetal?:

1. 1. pH en sangre fetal de 7.28.


2. 2. FCF 150 lpm.
3. 3. DIPs I no mantenidos.
4. 4. Meconio en líquido amniótico.
Gráfico de respuestas
Comentario

!
!
!
!

Pregunta sobre el sufrimiento fetal de las fáciles e importantes.

La pHmetría menor de 7.20 significa acidosis fetal, y por tanto es sufrimiento fetal. La FCF de
150 es normal, y para ser patológica debe ser >160 ó <120 lpm. Los dips I no implican mal
pronóstico, sobretodo si no son mantenidos; si fueran mantenidos nos obligarían a realizar
microtoma fetal. Por descarte, la opción a escoger es la 4, el LA meconial. Decir que el meconio
nos induce a pensar que existe sufrimiento fetal, pero aclarar que no siempre que haya
sufrimiento fetal habrá meconio. También decir que no siempre que haya meconio quiere decir
que haya sufrimiento fetal (pero sí nos hace pensar en él), porque por ejemplo en las
presentaciones podálicas es mucho más frecuente el meconio y no hay sufrimiento fetal.(R4)

232. Masculino de 28 años, agricultor y ganadero, que ingresa por un cuadro


confusional progresivo de 24 horas de evolución. En la exploración: Tª 40°C, 38
respiraciones/minuto, 120 latidos/minuto, TA 100/70, en mano izquierda se observa una
lesión elevada, dura, de unos 4 cm, con zona ulcerada y necrótica central, crepitantes
en bases pulmonares, abdomen normal, confuso, falta de atención, tendencia al sueño,
no realiza operaciones matemáticas simples, paresia del VI par izquierdo, rigidez de
nuca. Entre los datos de laboratorio destacan: Hb 16 gr/dl, Leucocitos 16,500 (80%
Neutrófilos, 10% Linfocitos, 10% Bandas), plaquetas 100,000, creatinina 2.5 mg/dl,
líquido cefalorraquídeo con 400 polimorfonucleares, proteínas 120 mg/dl, glucosa 60
mg/dl (en plasma 110) y presencia de bacilos gram positivos. ¿Cuál de los siguientes
agentes le parece responsable del cuadro?:

1. 1. Corynebacterium pseudotuberculosis.
2. 2. Bacillus anthracis.
3. 3. Listeria monocytogenes.
4. 4. Actinomyces israelii.
Gráfico de respuestas
Comentario

Caso clínico difícil en el que hay que saber desechar los datos inútiles y aprovechar las pistas
claves. Sintetizando mucho estamos ante un síndrome meníngeo (fiebre, rigidez de nuca,
somnolencia y afectación de pares craneales) en un paciente con afectación pulmonar y
cutánea. Curiosamente en este caso el LCR no nos ayuda mucho porque todos los
microorganismos son bacilos grampositivos que producen incremento de PMN en LCR. Pero si
nos fijamos la afectación cutánea es muy característica: una úlcera necrótica con edema
perilesional en un paciente ganadero. Con este dato podemos orientar el diagnóstico hacia el B.
anthracis. Casi el 95% de los casos de carbunco humano son cutáneos y el resto por inhalación.
La meningitis carbuncosa se produce en un pequeño porcentaje de todos los casos, pero es una
complicación frecuente en la bacteriemia fulminante por B. anthracis.(R2)

233. La lesión hepática MÁS frecuente en un traumatismo es:

1. 1. Hematoma subcapsular sin hemorragia activa.


2. 2. Hematoma subcapsular con hemorragia activa.
3. 3. Rotura hepática con hemorragia que rompe la cápsula.
4. 4. Lesión del hilio hepático.
Gráfico de respuestas
Comentario

!
!
!
!

Pregunta muy directa. No si la has fallado, pero intente recordarla porque puede preguntarse,
aunque este tema no es uno de los más preguntados en el ENARM. La lesión hepática más
frecuente es el hematoma subcapsular sin hemorragia activa (opción 1) en el nacional del 2013
hicieron una pregunta similar con una imagen que no podía apreciar nada, pero podría deducir
por conocimiento. Para el tratamiento del trauma hepático existen diversas técnicas según el
grado de lesión: drenaje simple, sutura directa de los vasos sangrantes, técnicas hemostáticas,
desbridamiento con resección, lobectomía. Actualmente se realiza sobre todo el control de
daños, limitándose en la primera cirugía a asegurar la hemostasia mediante la introducción de
compresas (empaquetado).(R1)

234. Mujer de 65 años de


edad que sufre un dolor brusco en su región lumbar al realizar un esfuerzo levantando
un objeto pesado. Se le presenta a usted el estudio radiológico y de TAC. ¿Qué prueba
complementaria que la paciente trae a urgencias, por habérsela realizado unos meses
antes en un control de salud, puede confirmar la etiología más frecuente de esta
lesión?:

1. 1. Una resonancia magnética.


2. 2. Una biometría hemática con química sanguínea.
3. 3. Una gammagrafía ósea.
4. 4. Una densitometría ósea.
Gráfico de respuestas
Comentario
En las imágenes se presenta una radiografía simple y una reconstrucción de TAC que muestran
una fractura aplastamiento de vértebra lumbar. La causa más frecuente en una mujer a esta
edad es una osteoporosis postmenopáusica. Este tipo de fracturas se denominan también de
manera muy ilustrativa “fracturas por fragilidad”. Es muy probable que, en ese contexto, la
paciente esté en seguimiento con un Reumatólogo o con su Ginecólogo, y la prueba que nos
traiga sea una densitometría (como señala la respuesta 4) para ver el grado de
osteopenia/osteoporosis de la paciente. A pesar de todo, habrá que realizar una historia clínica
exhaustiva para descartar problemas metabólicos y/o tumorales.(R4)

!
!
!
!

235. Si el aplastamiento vertebral se cuantifica en un 75%, el tratamiento más indicado


será:

1. 1. Medicación analgésica y rehabilitación.


2. 2. Corsé de yeso con hiperextensión lumbar.
3. 3. Cirugía mediante artrodesis vertebral lumbar.
4. 4. Cirugía mediante discectomía mini-invasiva de los discos afectados.
Gráfico de respuestas
Comentario

Ante una fractura-aplastamiento lumbar, uno de los criterios de tratamiento ortopédico vs


quirúrgico es el grado de aplastamiento vertebral. En la radiografía simple en proyección lateral
medimos la altura del muro posterior y del muro anterior. Por encima del 50% de diferencia de
altura (grado de aplastamiento), existe indicación quirúrgica porque las secuelas de no realizar
una cirugía son muy graves (pérdida de lordosis, sufrimiento discal, cuadros compresivos
medulares y radiculares). Además, es mucho más sencilla y tiene menos complicaciones la
cirugía de una fractura aguda que la cirugía de secuelas. Por debajo del 50% de aplastamiento,
el tratamiento ortopédico mediante un corsé de termoplástico estaría indicado. Otros criterios de
tratamiento quirúrgico incluyen una angulación/acuñamiento mayor de 25º, y una ocupación del
canal raquídeo/medular mayor del 50%. La cirugía consiste en realizar una reducción del
aplastamiento para restablecer una lordosis lo más cercana posible a la fisiológica, y la
realización de una artrodesis vertebral incluyendo la vértebra facturada, la vértebra superior y la
inferior.(R3)

236. Niña de un mes que es estudiada


por su pediatra por estreñimiento pertinaz desde el nacimiento. Presentó retraso en la
eliminación del meconio en el periodo neonatal. Actualmente presenta estado de
desnutrición leve-moderada y llamativa distensión abdominal. En las últimas horas
asocia vómitos que se están volviendo fecaloideos. Es estudiada por su pediatra, quien
realiza un enema opaco que se muestra en la imagen. Respecto al cuadro clínico
citado, señale la FALSA:

!
!
!
!

La imagen muestra un segmento del colon estenótico con dilatación del segmento
1. 1.
proximal a éste.
2. 2. El diagnóstico más probable es el de enfermedad de Hirschprung.
3. 3. Sería de gran utilidad la realización de una manometría ano-rectal.
Con la clínica que presenta la paciente, si una manometría fuera compatible no sería
4. 4.
necesaria la realización de una biopsia para obtener el diagnóstico.
Gráfico de respuestas
Comentario
Tal como dice la respuesta 1, en esta imagen podemos observar una estenosis (sección más
inferior de la imagen) y una dilatación proximal a ella, muy marcada, a nivel del colon. Se nos
habla también de un cuadro de estreñimiento desde el comienzo de la vida, que está
empeorando poco a poco y últimamente ha llegado a presentar vómitos fecaloideos. Cuando
nos encontramos con un cuadro de este tipo, deberíamos considerar una posible enfermedad de
Hirschprung (megacolon agangliónico). El patrón típico en la manometría será una ausencia de
relajación del esfínter anal interno al aumentar la presión en la ampolla rectal. No obstante, la
confirmación del cuadro (aunque la manometría es bastante típica) requiere una muestra de
tejido, para evidenciar la ausencia de células ganglionares en la región de la estenosis.(R4)

237. En referencia a la paciente de la pregunta anterior, señale cuál cree que sería el
manejo más adecuado:

Se debe realizar una manometría ano-rectal que mostrará probablemente dilatación del
1. 1.
esfínter anal interno con la distensión de la ampolla rectal.
Debemos descartar vólvulo intestinal como causa de su obstrucción intestinal mediante
2. 2.
realización de una TAC abdominal.
Con la imagen del enema opaco podemos hacer el diagnóstico de megacolon agangliónico
3. 3.
e indicar cirugía urgente.
En la medida de lo posible, completaremos el estudio con una manometría ano-rectal y lo
4. 4. confirmaremos con una biopsia rectal que debe mostrar ausencia del sistema nervioso
parasimpático intramural.
Gráfico de respuestas
Comentario

Como se ha explicado en el comentario de la pregunta anterior, la respuesta correcta es la 4. El


diagnóstico de sospecha es clínico y la manometría ano-rectal es de gran ayuda para enfocar el
caso, pero lo que confirma el diagnóstico de enfermedad de Hirschprung es la ausencia de
células ganglionares en la anatomía patológica.(R4)

238. En la enfermedad de Wilson es INFRECUENTE la afectación de:

1. 1. Hígado.
2. 2. Riñón.
3. 3. Sistema nervioso.
4. 4. Páncreas.
Gráfico de respuestas
Comentario

En la enfermedad de Wilson, el depósito tisular de cobre tiene apetencia fundamentalmente por


el parénquima hepático, provocando hepatitis o elevación crónica de las transaminasas y
raramente cirrosis. En el sistema nervioso central, su depósito puede inducir trastornos en la

!
!
!
!

conducta o en la movilidad. En el ojo es característico el depósito que forma el anillo de Kayser-


Fleischer. Se ha descrito el depósito renal, y es excepcional en páncreas.(R4)

239. La valoración del desarrollo psicomotriz constituye una exploración importante


para detectar trastornos del desarrollo durante la primera infancia. ¿A qué edad el 50%
de los lactantes debe mantenerse sentado sin ayuda?

1. 1. A los 4 - 5 meses.
2. 2. A los 5 - 6 meses.
3. 3. A los 6 - 7 meses.
4. 4. A los 8 - 9 meses.
Gráfico de respuestas
Comentario

Una pregunta aparentemente difícil sobre el desarrollo psicomotor que seguramente sera
preguntado en el nacional, que resulta mucho más sencilla si la analiza con lógica.

La duda razonable estaría entre las opciones 2 y 3.

En el Manual CTO, la solución aparecía claramente, y citamos: “6º mes: inicia la sedestación,
que se completa a los 8 meses”. Por tanto, si la sedestación se inicia en el 6º mes, difícilmente
habrá un 50% de los lactantes que ya puedan sentarse antes de ese momento. La respuesta
correcta sería entonces la 3.

Por otra parte, ante una duda como ésta,es posible que, en el momento del examen, no
dispongamos de ningún argumento teórico y haya que arriesgar entre dos opciones. En tal caso,
ante una pregunta cuyas opciones son numéricas, la apuesta más segura sería el valor
intermedio, ordenándolos todos de menor a mayor.(R3)

240. Uno de los siguientes microorganismos pertenece al grupo HACEK:

1. 1. Haemophilus influenzae.
2. 2. Actynobacillus actynomicetencomitans.
3. 3. Citrobacter freundii.
4. 4. Klebsiella rhinoscleromatis.
Gráfico de respuestas
Comentario

Esta pregunta es dificililla porque o se sabe o no se sabe. Uno de los microorganismos del grupo
HACEK es el Haemophilus, pero el parainfluenzae, no el influenzae (pregunta 1) - recuerde que
este microorganismo es causante de otras patologías muy graves en niños, por lo general
menores de 5 años, como la epiglotitis, meningitis, otitis media, etc- . El resto de
microorganismos de este grupo son: con la c no es Citrobacter (opción 3) sino Cardiobacterium,
la e, es de Eikenella corrodens, con la K no es Klebsiella (opción 4) sino Kingella kingae. Y por
último Actynobacillus (opción 2: resp. Correcta). Recuerde que son bacterias que dan
hemocultivo negativo porque necesitan medios enriquecidos para crecer por eso no podría ser
Klebsiella que suelen crecer en medios de cultivo normales (y que no son bacterias de la
cavidad oral, como las HACEK)(R2)

!
!
!
!

241. El padecimiento de parotiditis en el primer trimestre de la gestación se asocia


típicamente con una alteración cardíaca, ¿cuál?:

1. 1. Prolapso de la válvula mitral.


2. 2. Fibroelastosis endocárdica.
3. 3. Aneurismas de las arterias coronarias.
4. 4. Comunicación interventricular múltiple.
Gráfico de respuestas
Comentario

Las asociaciones que debe recordar de la parotiditis son: el aumento de la amilasa (sin correlato
clínico con pancreatitis) y la orquitis/ooforitis. Recuerde, además, que la parotiditis es la causa
más frecuente de hipoacusia neurosensorial unilateral adquirida. A veces se asocia con
fibroelastosis endocárdica pero es casi una anécdota.(R2)

242. Ante un paciente diabético de 60 años que acude a Urgencias con disminución del
nivel de conciencia, oliguria, presentando en los laboratorios una glucemia de 800
mg/dl, sodio de 150 mEq/l, bicarbonato de 20 mEq/l y creatinina de 3 mg/dl, la actitud
prioritaria será:

1. 1. Intubación orotraqueal.
2. 2. Realizar una TC.
3. 3. Reposición de líquidos.
4. 4. Antibióticos.
Gráfico de respuestas
Comentario

Este paciente presenta una descompensación hiperglucémica hiperosmolar. Los diabéticos que
presentan esta complicación son los de tipo 2. Este cuadro suele producir niveles muy altos de
glucosa, (generalmente > 600 mg/dl), deshidratación importante con aumento de los productos
nitrogenados y sodio sérico normal o bajo, por el efecto de la hiperglucemia. No suele aparecer
acidosis. La cetonuria también es rara y, si aparece, es leve. Para el tratamiento, lo fundamental
es la hidratación del paciente, ya que lo que predomina es el déficit de líquidos, que a veces
llega a ser de hasta 10 litros. A pesar que que la hiperglucemia suele ser severa, la insulina no
es el tratamiento fundamental, aunque ayuda a corregir el cuadro.(R3)

243. A 27-year-old male comes to the office complaining of urinary urgency and dysuria
for the past week. It happened once before but the symptoms disappeared with time.
He has also had interruption of flow and increased urinary frequency. His temperature
is 98.6ºF (37ºC), blood pressure is 140/95 and puse is 85/min. Rectal examination shows
periprostatic tenderness, indurated prostate and increased tone of the anal sphincter.
Urinalysis and urine culture show no abnormalities. Expressed prostatic secretions
show a leukocyte count of 40-50+ WBC/hpf (normal is <10WBC/hpf) but culture of these
secretions is negative. Serum PSA is 2ng/mL. Which of the following is the most likely
diagnosis?

1. 1. Chronic bacterial prostatitis.


2. 2. Inflammatory chronic prostatitis.
3. 3. Non inflammatory chronic prostatitis.

!
!
!
!

4. 4. Prostatic cancer.
Gráfico de respuestas
Comentario
Inflammatory chronic prostatitis. Inflammatory chronic prostatitis or inflammatory pelvic pain
syndrome is usually caused by atypical bacteria such as Ureaplasma or Mycoplasma. Urine and
prostatic secretion cultures are negative but leukocyte count is positive(R2)

244. A woman who is 36 weeks' pregnant comes to the ER after a car accident on the
highway. She refers both neck and back pain. While you’re examining her, she starts
with severe abdominal pain, dark vaginal bleeding and sustained increase in uterine
tone. Which of the following is the most likely diagnosis?

1. 1. Splenic rupture with hemoperitoneum.


2. 2. Abruptio placentae.
3. 3. Preterm labor.
4. 4. Uterine rupture.
Gráfico de respuestas
Comentario
En esta pregunta nos piden el diagnóstico más probable, teniendo en cuenta el dolor con
aumento del tono uterino y el sangrado vaginal oscuro la opción correcta sería el
desprendimiento de placenta. Una duda razonable sería con la rotura uterina, pero tiene menor
probabilidad ya que no hay ningún antecedente de cirugía sobre el útero, el sangrado debería
ser rojo intenso y no oscuro como en nuestra paciente, y en la exploración no hay ningún dato
que nos lo sugiera.(R2)

245. Hablamos de “depresión doble” cuando:

1. 1. El paciente presenta un trastorno orgánico severo asociado a su depresión.


2. 2. Se sobreimponen episodios depresivos mayores sobre un trastorno distímico.
Existe un riesgo de viraje a fase maníaca, ya que en realidad se trata de un trastorno
3. 3.
afectivo bipolar.
La depresión incide sobre un paciente esquizofrénico tras la remisión de los síntomas
4. 4.
psicóticos propios de la fase aguda de su enfermedad.
Gráfico de respuestas
Comentario

Esta pregunta puede resolverse por lógica. Se habla de distimia cuando persisten síntomas
depresivos de intensidad leve durante más de dos años. La depresión mayor es la que muestra
un predominio de sintomatología endógeno/melancólica (anhedonia, despertar precoz, mejoría
vespertina...). La coexistencia de estas dos entidades es lo que se conoce como depresión
doble. Entre las opciones que nos presentan, sólo la 3 nos habla de dos enfermedades que
producen alteraciones de tipo depresivo, por lo que es la única en la que encajaría.(R2)

!
!
!
!

246. En relación al
tratamiento de un paciente con hiperuricemia y la afectación aguda que aparece en la
imagen adjunta, indique cuál de las siguientes afirmaciones es FALSA:

La eficacia de la colchicina y los antiinflamatorios aumenta cuando se administran de


1. 1.
forma precoz.
2. 2. La toxicidad más habitual de la colchicina es la hematológica.
3. 3. Los corticoides intraarticulares son eficaces.
4. 4. Los antiinflamatorios se deben administrar a dosis altas.
Gráfico de respuestas
Comentario

Una pregunta muy sencilla, ya que el propio enunciado de la pregunta le da el dato clave:
hiperuricemia. Lo que muestra la imagen resulta, por tanto, muy predecible: eritema y
tumefacción en la primera articulación metatarsofalángica, lo habitual en una crisis de podagra.

Naturalmente, la respuesta incorrecta es la 2. La principal toxicidad de la colchicina es digestiva


(diarrea, flatulencia, otras molestias gastrointestinales…). Es muy importante darse cuenta de
que la respuesta 3 es cierta. Si el paciente tomaba alopurinol, debe mantenerse, lo mismo que si
no lo tomaba no se debe introducir en fase aguda. Esto es debido a que las modificaciones de la
uricemia pueden agravar el cuadro, tanto si aumenta como si disminuye.(R2)

247. Respecto al tratamiento con cistectomía radical de los tumores infiltrantes de


vejiga NO es cierto que:

La cistectomía radical es una de las múltiples opciones de tratamiento curativo del cáncer
1. 1.
de vejiga con invasión muscular localizado.
El estado funcional y la edad influyen en la elección del tratamiento primario, así como en
2. 2.
el tipo de derivación urinaria con cistectomía.
El retraso de la cistectomía no sólo afecta al pronóstico, sino también al tipo de derivación
3. 3.
urinaria.
La cistectomía radical supone la extirpación de la vejiga y los órganos adyacentes, es
4. 4. decir, próstata y vesículas seminales en los varones y útero y anejos uterinos en las
mujeres.
Gráfico de respuestas

!
!
!
!

Comentario
La cistectomía radical es el tratamiento de referencia del cáncer de vejiga con invasión muscular
localizado y el único que puede considerarse de forma aislada curativo a día de hoy.(R1)

248. Paciente de 42 años de edad que se realiza una citología cervical, en la cual se
reporta lo siguiente: lesión intraepitelial de alto grado de malignidad (NIC III) ¿Cuál es
el procedimiento a seguir?

1. 1. Test de ácido acético y nueva citología.


2. 2. Test de Schiller y nueva citología.
3. 3. Colposcopía y biopsia dirigida.
4. 4. Biopsia de cuatro cuadrantes de cuello uterino.
Gráfico de respuestas
Comentario

Tema extremadamente importante. Ante una citología positiva, se debe proceder a la realización
de una colposcopía y biopsia de la lesión sospechosa. Respuesta 3 correcta.(R3)

249. Una mujer de 70 años consulta por astenia y anemia ferropénica que no
presentaba en los laboratorios del año anterior. En el estudio se demuestra un cáncer
intestinal. ¿En qué lugar estará más frecuentemente localizado?

1. 1. Duodeno.
2. 2. Yeyuno.
3. 3. Ciego.
4. 4. Sigma.
Gráfico de respuestas
Comentario

!
!
!
!

El 70-80% de los cánceres colorrectales aparecen en colon descendente, sigma y recto. No


obstante, los tumores que aparecen en estas localizaciones se manifiestan con síntomas
obstructivos, lo que no encaja con la clínica que nos describen en esta pregunta. Sin embargo,
los cánceres de localización más proximal (ciego, colon ascendente) se manifiestan con signos y
síntomas derivados del sangrado (anemia ferropénica, en este caso), así que la respuesta
correcta es la 3. Importante: el colon derecho sangra; el izquierdo, se obstruye.

Las opciones 1 y 2 son muy improbables, dado que los tumores primarios de esa localización
son mucho menos frecuentes que el cáncer de colon.(R3)

250. Una niña de 10 años con atresia biliar, tratada con la técnica de Kasai cuando era
lactante, presenta ahora torpeza progresiva, disminución de los reflejos tendinosos
profundos y ataxia. El diagnósitco más probable es:

1. 1. Encefalopatía hepática.
2. 2. Deficiencias de vitamina A.
3. 3. Encefalitis.
4. 4. Deficiencia de vitamina E.
Gráfico de respuestas
Comentario

Pregunta de dificultad moderada. Teniendo en cuenta la clínica descrita en el caso sería


razonable dudar entre las dos últimas opciones, pero considerando los antecedentes expuestos,
no queda opción a duda.

La vitamina E es una vitamina liposoluble cuya deficiencia aparece unida a enfermedades


malabsortivas graves y prolongadas: fibrosis quística, colestasis (y dentro de éstas la atresia de
vías biliares), etc. La carencia nutricional no se ha descrito. Se manifiesta por un síndrome
potencialmente reversible caracterizado por ataxia (cerebelosa y por afectación de cordones
posteriores) y neuropatía periférica.

Con respecto al resto de opciones, es interesante comentar la 4. Esta patología debuta en la


infancia o adolescencia con ataxia progresiva de la marcha y, ya que su diagnóstico es
eminentemente clínico (apoyándonos en los antecedentes familiares, EMG, etc.), es
fundamental descartar previamente un déficit de vitamina E.

Las opciones 1 y 3 son incorrectas, puesto que en el caso no se describe ninguna alteración del
estado mental. El déficit de vitamina A cursa con ceguera nocturna, xeroftalmia, retraso del
crecimiento, apatía, sequedad cutánea, etc.

Como comentario final reseñar la importancia de la reposición periódica de vitaminas


liposolubles en las colestasis neonatales.(R4)

251. En cuanto al tabaco como factor de riesgo de cardiopatía isquémica, señale la


respuesta CORRECTA:

1. 1. El riesgo guarda relación con el tipo de tabaco.


Cuando se abandona el hábito tabáquico, el riesgo de enfermedad coronaria aumenta un
2. 2.
50% en el primer año.

!
!
!
!

El mecanismo a través del cual el tabaco favorece la aterogénesis es la disminución del


3. 3.
óxido nítrico.
4. 4. El tabaco produce también una disminución de la agregabilidad plaquetaria.
Gráfico de respuestas
Comentario
La respuesta 3 es la correcta. En este tipo de preguntas debemos actuar con sentido común.
Debemos partir de la premisa de que el tabaco es malo en cualquiera de sus formas, y que si se
deja el mismo el riesgo disminuye. Si aplicamos esta premisa, la única respuesta que nos
quedaría como correcta es la que en efecto es correcta, la número 3. Hay que resaltar que para
la enfermedad coronaria es igual de factor de riesgo el fumar cigarrillos, pipa o puros.(R3)

252. Paciente de 42 años,


con menarquia a los 10 años, G1P1, con ciclos regulares y portadora de DIU de cobre como
método de planificación familiar. Antecedentes familiares: madre intervenida de cáncer de mama
bilateral, antes de los 50 años, que falleció a los 56 años sin realizarse estudio genético.
Exploración mamaria dentro de la normalidad sin telorrea asociada. En una mastografía de
seguimiento a los 6 meses por una mastografía anterior informada como BI-RADS 3 se observa
un aumento de las alteraciones observadas, siendo la mastografía actual la que se muestra en la
imagen. ¿Cuál de las siguientes afirmaciones es CORRECTA?:

1. 1. La mastografía se informará como BI-RADS 4.


2. 2. Son hallazgos de características benignas.
3. 3. Se recomiendan controles cada 6 meses durante 2 años.
4. 4. Deberemos realizar una galactografía para descartar un papiloma intraductal.
Gráfico de respuestas
Comentario

Tal vez no lepongan a clasificar un BIRADS por imagen pero si debe dominar la clasificación. La
mastografía muestra microcalcificaciones de aspecto benigno, aunque el hecho de que tiendan
a agruparse junto con el aumento progresivo en la siguiente mastografía obliga a realizar estudio
histológico que podría ser realizado por esterotaxia o previo marcaje con arpón. La ecografía no
es una técnica de imagen útil en la evaluación de las microcalcificaciones. La galactografía sería

!
!
!
!

úitl si existiera sospecha de papiloma intraductal pero no es el caso ya que la paciente no


presenta telorrea asociada.(R1)

253. Se realiza biopsia que revela la existencia de un carcinoma lobulillar in situ en


varios focos. ¿Cuál de las afirmaciones siguientes NO es correcta respecto a la
patología que presenta la paciente del caso clínico anterior?:

El carcinoma lobulillar in situ tiende a ser bilateral, no simultáneo, en un 35-60% de las


1. 1.
pacientes.
Esta paciente tiene un 20-35% de posibilidades de padecer un cancer invasivo en el
2. 2.
seguimiento a 20 años, con riesgo similar en ambas mamas.
El tratamiento quirurgico indicado es la mastectomía simple si bien puede considerarse la
3. 3.
mastectomía bilateral con o sin reconstrucción inmediata.
4. 4. El tratamiento quirurgico será la mastectomía más la linfadenectomía axilar.
Gráfico de respuestas
Comentario

Cáncer de mama debes dominar TODOS sus aspectos para fines del ENARM, año con año se
pregunta sobre detección y prevención.

Recuerde que el tipo de cáncer de mama más frecuente es el ductal infiltrante se pregunta casi
año con año.

El carcinoma lobulillar in situ (CLIS) es multicéntrico en un elevado porcentaje de pacientes (60-


90%) y tiende a ser bilateral no simultáneo. Se trata de un proceso silente. Toda paciente con
CLIS tiene altas posibilidades de padecer un cancer invasivo en el seguimiento a 20 años. En el
tratamiento es posible realizar mastectomía uni o bilateral con o sin reconstrucción inmediata, si
bien no procede ni la biopsia del ganglio centinela ni la linfadenectomia axilar, así como la
radioterapia, ya que no es infiltrante.

No olvide reviar la guía de práctica clínica de cáncer de mama.

http://www.cenetec.salud.gob.mx/descargas/gpc/CatalogoMaestro/001_GPC_CadeMama/(R4)

254. Un paciente fumador de 35 años consulta por fiebre, disnea, tos, dolor torácico y
pérdida de peso. La radiografía de tórax muestra infiltraciones intersticiales con
pequeños quistes aéreos que afectan a los campos pulmonares superiores y medios,
con conservación del volumen pulmonar. En las pruebas de función respiratoria se
aprecia un patrón restrictivo con disminución de la capacidad de difusión para el

!
!
!
!

monóxido de carbono. ¿Cuál considera que es, entre los siguientes, el diagnóstico más
probable?

1. 1. Los datos clínicos no suelen ser expresivos.


2. 2. Proteinosis alveolar pulmonar.
3. 3. Histiocitosis X.
4. 4. Fibrosis pulmonar idiopática.
Gráfico de respuestas
Comentario

La histiocitosis X es una enfermedad pulmonar que normalmente afecta a varones jóvenes y


fumadores. Afecta preferentemente a lóbulos superiores. Desde el punto de vista funcional
podemos encontrar tanto un patrón restrictivo como uno obstructivo, aunque en los casos más
crónicos predomina la obstrucción. El pronóstico está muy influenciado por el hábito tabáquico
(si se abandona, tiende a mejorar). Recuerde que, a diferencia de otras intersticiales, los
corticoides no son de utilidad.

Las otras opciones no encajarían por los motivos siguientes:

•! R1. No justifica el patrón radiológico ni las alteraciones de la difusión.


•! R2 La proteinosis alveolar produce una radiografía muy distinta, condensaciones
perihiliares, semejante a un edema pulmonar (el alvéolo está lleno de un material
proteináceo).
•! 4. La fibrosis pulmonar idiopática afecta más a las bases y suele tratarse de hombres de
mediana edad.

(R3)

255. En relación al síndrome de ovario poliquístico, es cierto que:

Existe un mayor riesgo a largo plazo de desarrollar diabetes mellitus y carcinoma


1. 1.
endometrial.
2. 2. Clínicamente, es típico en este síndrome la polimenorrea, obesidad e hirsutismo.
3. 3. Habitualmente la concentración sérica de FSH es mayor que la de LH.
4. 4. No existe una imagen ecográfica ovárica característica.
Gráfico de respuestas
Comentario

Concepto importante para el ENARM, ya fue preguntado. Es característico del Síndrome del
ovario poliquístico la resistencia insulínica y los ciclos anovulatorios. Debido a la resistencia
insulínica hay un riesgo aumentado de desarrollar DM, y los ciclos anovulatorios son uno de los
motivos por los que los niveles de estrógenos se mantienen siempre altos, este ambiente
hiperestrogénico favorece la aparición de carcinoma endometrial. La clínica típica es
oligomenorrea, no hipermenorrea. La concentración sérica de LH es mayor que la de FSH.(R1)

256. En la hipertensión pulmonar persistente del recién nacido, ¿qué NO esperaría


encontrar?:

1. 1. Radiografía de tórax normal.


2. 2. Presión estimada en la arteria pulmonar elevada.

!
!
!
!

3. 3. Gradiente pre-postductal de PaO2 menor de 20 mmHg.


4. 4. Buena respuesta al óxido nítrico inhalado.
Gráfico de respuestas
Comentario

La hipertensión pulmonar persistente del recién nacido o persistencia de la circulación fetal es


un cuadro de distrés respiratorio típico del recién nacido a término.

Se debe a la persistencia de resistencias vasculares pulmonares elevadas que perpetúan la


circulación fetal, con shunt derecha-izquierda a través del conducto arterioso y el foramen oval.

Se manifiesta como distrés con cianosis e hipoxemia refractarias a la administración de oxígeno


(shunt).

La radiografía de tórax suele ser normal y el gradiente pre-postductal de PaO2 es mayor de 20


mmHg.

El ultrasonido sirve para descartar una cardiopatía congénita cianosante y para estimar las
presiones pulmonares.(R3)

257. Una mujer embarazada de 12 semanas consulta por intensas náuseas y vómitos.
En sus dos embarazos previos también padeció estos síntomas, pero de intensidad
mucho menor. Por otra parte, últimamente duerme menos horas (aunque no por ello
está más cansada) y ha perdido algo de peso, aunque la ingesta no ha disminuido;
incluso tiene más hambre que de costumbre. En la exploración se observa un útero
mayor que el que corresponde al tiempo de amenorrea. Sobre la enfermedad que
sospecha, señale la respuesta FALSA:

1. 1. Sería esperable encontrar niveles elevados de beta-HCG.


En algunos casos, se producen quistes tecaluteínicos en el ovario, que regresan
2. 2.
espontáneamente con el tratamiento de la enfermedad.
3. 3. La progesterona y el estriol estarán muy probablemente aumentados.
4. 4. El riesgo de preeclampsia es claramente superior al de un embarazo normal.
Gráfico de respuestas
Comentario

Se trata de una enfermedad trofoblástica gestacional: hay que guiarse por el útero mayor que
amenorrea. En efecto son frecuentes la hiperemesis, las manifestaciones de hipertiroidismo, y
un riesgo aumentado de preeclampsia.

La respuesta 3 es parcialmente falsa: la progesterona estará alta, cierto, pero el estriol está muy
descendido en la ETG porque su síntesis requiere la participación fetal.(R3)

258. Un lactante de 10 meses que presenta vómitos y diarreas, taquicardia, TA normal,


mucosas secas, llenado capilar menor de 2 segundos e irritabilidad. ¿Qué porcentaje
de peso ha perdido?

1. 1. 13-15%.
2. 2. 3-5%.

!
!
!
!

3. 3. 10-12%.
4. 4. 6-9%.
Gráfico de respuestas
Comentario

Se habla de deshidratación cuando existe una situación en la que el niño presenta un balance
hidroelectrolítico negativo. Los RN y los lactantes son los pacientes con una mayor tendencia a
presentar alteraciones hidroelectrolíticas, ya que proporcionalmente tienen más agua corporal,
mayor superficie corporal y una menor capacidad para regular la concentración de la orina. El
grado de deshidratación se corresponde con el porcentaje de peso corporal que se ha perdido,
pues se asume que una pérdida aguda de peso corresponde a agua y electrolitos y no a masa
corporal. En ocasiones no se dispone del dato de peso previo, por lo que es necesario recurrir a
signos clínicos que, de forma indirecta, permiten estimar el grado de deshidratación. Se puede
clasificar clínicamente en tres grados:

• Leve o de primer grado: pérdida < 5 % (niños) o < 3 % (lactantes).

• Moderada o de segundo grado: pérdida 5-10% (niños) o 3-7 % (lactante).

• Grave o de tercer grado: pérdida de > 1 0 % (niños) o > 7 % (lactantes).(R4)

259. ¿Cuál es la medida prioritaria a realizar en un paciente politraumatizado?:

1. 1. Detener hemorragias.
2. 2. Colocar una vía venosa central.
3. 3. Conseguir una vía aérea permeable.
4. 4. Administrar drogas inotrópicas positivas.
Gráfico de respuestas
Comentario

Esta pregunta es muy importante ya que el manejo del politraumatizado es un tema muy
importante en el ENARM. Para valorar el orden de priorización ante un accidentado utilizamos el
acrónimo ABCDE. Lo prioritario es mantener la vía aérea permeable (airway) y en segundo lugar
controlar la ventilación (breathing), por lo que se deben tratar aquellas lesiones que la alteren,
como es el caso del neumotórax a tensión. El siguiente paso seria control de las hemorragias
(circulation), y coger una vía central. Posteriormente se controlan las lesiones viscerales y NRL
(disability). El último paso es el tratamiento y control de las fracturas.(R3)

260. Paciente de 2 meses de edad acude por presentar convulsiones tónico clónicas
generalizadas de 2 minutos de duración, durante la evaluación no se determina fiebre.
Los exámenes de laboratorio y la punción lumbar son normales, los estudios de
imágenes no muestran mayor alteración. Durante la hospitalización en los días
siguientes se presentaron convulsiones de corta duración. El diagnóstico probable
será?

1. 1. Convulsión neonatal.
2. 2. Epilepsia.
3. 3. Convulsión asociada a fiebre.
4. 4. Convulsión febril compleja.
Gráfico de respuestas

!
!
!
!

Comentario

Si le dicen que el paciente no ha presentado fiebre nunca, debe de descartar inmediatamente la


opción 3 y la 4; y si le dicen que tiene 2 meses de edad, por lo tanto es un lactante menor,
deberá eliminar también la opción 1, por lo que la opción correcta es la número 2.(R2)

261. En el linfoma de células T del adulto NO es cierto:

1. 1. Frecuente expresión leucémica.


2. 2. Buena respuesta a la quimioterapia.
3. 3. Lesiones osteolíticas.
4. 4. Hipercalcemia.
Gráfico de respuestas
Comentario

Es un linfoma de muy mal pronóstico con mala respuesta a tratamientos, en los que en
ocasiones se emplea medicación antiuretroviral como la Zidovundina, ya que está causado por
el virus HTLV-1. Se presenta preferentemente en pacientes del Caribe y de Japón. Es típico la
presencia de lesiones osteolíticas e hipercalcemia.(R2)

262. La captación tiroidea de yodo radiactivo está disminuida en todas las causas de
hipertiroidismo que se mencionan a continuación, EXCEPTO una:

1. 1. Estruma ovárico.
2. 2. Adenoma productor de T4 y T3.
3. 3. Hipertiroidismo facticio.
4. 4. Hipertiroidismo inducido por yodo.
Gráfico de respuestas
Comentario

Una pregunta muy sencilla, puesto que el adenoma tiroideo es una de las causas más
conocidas de hipercaptación gammagráfica. Naturalmente, la captación se limita al adenoma en
cuestión, quedando abolida el resto de la glándula. Sin embargo, en el resto de las opciones no
existe captación de ningún tipo.(R2)

!
!
!
!

263. Paciente femenino


de 27 años, nuligesta, fumadora de 20 cigarillos/día. Usuaria de anticoncepción
hormonal (anillo vaginal desde hace más de 5 años). Antecedentes familiares: madre
intervenida de cáncer de mama a los 44 años, actualmente libre de enfermedad. Acude
a la consulta por tumoración en mama izquierda, que se notó hace un año, pero por la
que no había consultado hasta ahora por miedo. A la exploración se palpa una
tumoración de 2 cm en CSE de la mama izquierda, no dolorosa, móvil, no adherida, sin
telorrea ni retracción del pezón. ¿Cuál de las siguientes respuestas NO es correcta?:

1. 1. El hallazgo de un carcinoma en el interior de esta tumoración es muy raro.


2. 2. Su diagnóstico se basa en la clínica, confirmándose por las pruebas de imagen.
3. 3. La ecografía es la técnica de elección en las mujeres jóvenes.
4. 4. Nunca se extirpan quirúrgicamente, a no ser que la anatomía patológica sea dudosa.
Gráfico de respuestas
Comentario

El fibroadenoma es el tumor benigno mas frecuente de la mama. Su diagnóstico se basa en la


clínica y en la ecografía: nodulo sólido, regular, de bordes bien limitados, hipoecoico y con
sombra posterior. Se trata de una tumoración benigna. Su extirpación quirúrgica se recomienda
en los siguientes supuestos: tamaño superior a 2 cm, crecimiento rápido, duda diagnóstica,
cancerofobia, si provoca dolor, o si la mujer es mayor de 35 años. No presenta riesgo de
recidiva local.(R1)

264. Con respecto a la paciente del caso clínico anterior, ¿cuál de las siguientes
opciones terapéuticas NO es correcta?:

1. 1. Está contraindicada la cirugía por la edad (menor de 35 años).


2. 2. Puede plantearse la cirugía por la ansiedad de la paciente
3. 3. La observación periódica es una opción totalmente válida.
Se debe tranquilizar a la paciente, indicándole que el fibroadenoma no le aumenta el riesgo
4. 4.
de padecer un cáncer de mama.
Gráfico de respuestas

!
!
!
!

Comentario

En este caso que la paciente tiene antecedentes familiares de cáncer de mama debemos
informar a la paciente que el fibroadenoma no aumenta el riesgo como factor independiente. El
manejo de este tipo de tumoración puede ser conservador, con controles periódicos. En algunos
supuestos (tamaño > 2 cm, crecimiento rápido, clínica asociada, duda diagnóstica o
cancerofobia) se puede optar por cirugía. Respecto a la edad, recuerda que en mujeres mayores
de 35 años, se recomienda la cirugía, ya que la aparición de estas lesiones no es tan frecuente y
además la incidencia de cáncer de mama aumenta por lo que es importante disponer de
confirmación histológica.(R1)

265. Gestante de 10 semanas que presenta náuseas y vómitos matutinos,


acompañados de pirosis, no refiere antecedentes de patología digestiva previa. ¿Cuál
es la respuesta INCORRECTA?:

1. 1. El tratamiento mas utilizado, por carecer de efectos demostrados es la doxilamina.


2. 2. Pueden utilizarse sin riesgo, antisecretores como el omeprazol.
Los antihistaminicos como la metoclopramida, son seguros: no se ha podido demostrar
3. 3.
efecto teratogénico en humanos.
Pueden utilizarse casi todos los antiácidos de uso habitual: hidroxido de aluminio y sales
4. 4.
de magnesio.
Gráfico de respuestas
Comentario

El Omeprazol pertenece a la categoria C de toxicidad fetal: no se puede descartar la existencia


de riesgo. La Doxilamina es de categoria A: Sin efectos indeseables demostrados sobre el feto.
Con la Metoclopramida, no existen estudio de riesgo en humanos (categoria B). Los antiacidos
prácticamente no se absorben.(R2)

266. La confirmación de la hemorragia uterina disfuncional se realiza principalmente


por:

1. 1. Sólo con ecografía pélvica.


2. 2. Examen físico, biopsia de endometrio, ultrasonido pélvico e historia clínica.
3. 3. Sólo con biopsia de endometrio.
4. 4. Por el método de supresión de progestágenos previo dosaje de hormonas.
Gráfico de respuestas
Comentario

Generalmente la respuesta correcta siempre será la más completa. En este caso la respuesta 2
engloba el examen físico, biopsia y ultrasonido.(R2)

267. Paciente de 26 años con metrorragias desde hace 8 días y dolor en fosa iliaca
derecha. Tuvo su última regla hace 9 semanas. Es sexualmente activa, pero no utiliza
anticonceptivos de manera eficaz. Tiene antecedente de enfermedad inflamatoria
pélvica y de apendicectomía. En la exploración sólo se detecta engrosamiento del
anexo derecho. ¿Cuál es la sospecha más probable?:

1. 1. Embarazo ectópico.
2. 2. Carcinoma de trompa.

!
!
!
!

3. 3. Rotura de un folículo de de Graaf.


4. 4. Endometrioma ovárico.
Gráfico de respuestas
Comentario

Ante esta pregunta, se debe sospechar un posible embarazo ectópico. Los antecedentes que
nos mencionan (EIP y cirugía abdominal previa) resultan muy orientativos. Por otra parte, la
clínica es compatible (dolor y amenorrea, con un anexo engrosado, que es donde
probablemente se localiza).

El diagnóstico se confirmaría realizando un test de embarazo y, a continuación, un ultrasonido.


Los hallazgos característicos sería la ausencia de vesícula embrionaria intrauterina, con un
endometrio decidualizado (signo de Arias-Stella). Suponiendo que el embarazo ectópico se
localizara a nivel tubárico, que es lo más frecuente, podría observarse una estructura
embrionaria a ese nivel, aunque esto no ocurre en todos los casos.(R1)

268. Señale el factor que NO supone mayor riesgo en la aparición de un cuadro severo
de bronquiolitis aguda en un lactante:

1. 1. Bajo peso al nacer.


2. 2. Cardiopatía congénita.
3. 3. Vivienda con hacinamiento.
4. 4. Procedencia rural.
Gráfico de respuestas
Comentario

Esta pregunta de bronquiolitis no tiene mucha relevancia. Se deben considerar niños de alto
riesgo de infección grave por VRS los recién nacidos prematuros o de bajo peso al nacer, edad
inferior a 6 semanas, enfermedad crónica pulmonar, déficit inmunitarios, cardiopatía congénita y
síndromes polimalformativos, patología neuromuscular o metabólica especialmente cuando han
precisado algún tipo de tratamiento en los meses previos a la epidemia. La exposición pasiva al
tabaco produce alteración del epitelio respiratorio de base. Recuerde que en estos niños puede
considerarse el uso de ribavirina en su tratamiento. La fuente de transmisión suele ser familiar
por lo que en viviendas con hacinamiento el riesgo es mayor. La procedencia urbana o rural no
influye en el riesgo.(R4)

269. Entre las causas de hemolisis del recién nacido debemos considerar, menos:

1. 1. Enfermedad materna (por ej. lupus).


2. 2. Defectos de la membrana eritrocitaria, hemoglobinopatías.
3. 3. Hemólisis adquiridas: infección, coagulación intravascular diseminada.
4. 4. Enfermedad membrana hialina, déficit de surfactante.
Gráfico de respuestas
Comentario

Entre las causas de hemólisis no se encuentra secundario a EMH, por lo que la respuesta
correcta es la número 4.(R4)

!
!
!
!

270. Entre las siguientes afirmaciones en relación con el abdomen agudo, señale cuál
de ellas es INCORRECTA:

La ecografía puede ser útil para valorar vasos y lesiones retroperitoneales, aunque es
1. 1.
superada por el TAC.
2. 2. El estreñimiento y la falta de eliminación de gases es sugerente de obstrucción intestinal.
La progresión de un dolor sordo y difuso hacia uno agudo, constante y bien localizado
3. 3.
orienta a un abdomen agudo no quirúrgico.
Un cuadro extraabdominal, como un infarto inferior o una neumonía basal, pueden ser
4. 4.
causa de abdomen agudo.
Gráfico de respuestas
Comentario

La progresión de un dolor sordo y difuso hacia uno agudo, constante y bien localizado orienta a
un abdomen agudo quirúrgico, una apendicitis por ejemplo.(R3)

271. ¿Cuál de las siguientes afirmaciones referidas a los virus herpes humanos es
CORRECTA?

1. 1. Son virus RNA de simetría icosaédrica y desnudos (sin envoltura).


2. 2. La queratitis herpética casi siempre afecta a ambos ojos.
El virus de Epstein-Barr presenta una relación etiológica con el linfoma endémico de
3. 3.
Burkitt, la enfermedad de Hodgkin y el carcinoma nasofaríngeo.
4. 4. La vía frecuente de transmisión de Citomegalovirus es la aérea.
Gráfico de respuestas
Comentario

Los Virus Herpes Humanos son virus DNA (Opción 1 FALSA) que se subdividen en 3
subfamilias:

Alfa: incluye los Virus Herpes Simple 1 y 2 (también denominados Herpes 1 y 2), el virus
Varicela-Zóster (también denominado Herpes 3) y el Virus Herpes B. Tienen mecanismos
patogénicos parecidos: el lugar de latencia son las neuronas de ganglios sensitivos y la
enfermedad más habitual es la mucocutánea.

•! Beta: incluye el Citomegalovirus (también denominado Herpes 5) y los Virus Herpes 6 y


7.
•! Gamma: incluye el Virus de Epstein-Barr (también denominado Herpes 4) y el Virus
Herpes 8 (asociado con el sarcoma de Kaposi).
•! La queratitis por herpes es mayoritariamente unilateral (bilateral en alrededor del 5% de
los casos).

El citomegalovirus se transmite por múltiple rutas, incluidas la vía aérea, contacto sexual,
productos hemáticos, trasplante de órganos, perinatal, sin que ninguna de ellas tenga una
llamativa preponderancia.

La opción 3 es CORRECTA. El virus de Epstein-Barr se encuentra implicado en el desarrollo de


varias neoplasias, entre las que podemos destacar: linfomas en pacientes trasplantados y
pacientes VIH (incluido el linfoma cerebral primario del paciente VIH), linfoma Burkitt en

!
!
!
!

cualquiera de sus tipos (endémico, esporádico, asociado a VIH) con porcentaje de asociación
variable según los subtipos, linfoma Hodgkin, carcinoma nasofaríngeo; también lo está en el S.
linfoproliferativo ligado a X (que en algún momento de su evolución puede desarrollar linfoma) y
en algunos tipos de linfomas de la línea media.(R3)

272. En la nefronoptisis, se he descrito mutación de varios genes, ¿Cuál es el que


produce manifestaciones clínicas más precoces?

1. 1. NPHP1.
2. 2. NPHP2.
3. 3. NPHP3.
4. 4. NPHP5.
Gráfico de respuestas
Comentario
Pregunta muy difícil. De entre todas las mutaciones descritas en pacientes con nefronoptisis, la
del NPHP 2 es en la que antes suele aparecer las alteraciones renales.(R2)

273. Acude al servicio de


urgencias una mujer de 28 años, brasileña, con un cuadro de hemiparesia derecha de
horas de evolución. Durante los últimos meses se había quejado de entumecimiento y
sensación de cansancio en extremidades superiores durante su utilización. En la
exploración destaca la auscultación de un soplo diastólico en foco aórtico y una
disminución muy marcada de ambos pulsos radiales. Aporta una arteriografía realizada
en otro centro (ver imagen). En relación con la patología que parece presentar esta
mujer, una de las siguientes afirmaciones es FALSA:

Probablemente la paciente presente hipertensión arterial debida a la afectación de las


1. 1.
arterias renales.

!
!
!
!

El tratamiento quirúrgico debe posponerse al control de la inflamación con corticoides a


2. 2.
dosis de 40-60 mg/día.
3. 3. No es propia de la enfermedad la elevación de la VSG o la anemia de trastornos crónicos.
4. 4. La arteriografía mostrará estenosis y dilataciones postestenóticas.
Gráfico de respuestas
Comentario

Clínicamente, el cuadro que nos describen corresponde a una arteritis de Takayasu, que es una
vasculitis cuya afectación preferente ocurre a nivel de los troncos supraaórticos. Esta sospecha
se corrobora con la arteriografía que aporta, donde se observa un claro adelgazamiento de
estos vasos. Recuerde que esta patología es más frecuente en países asiáticos, pero también lo
es en determinadas zonas de Latinoamérica, como sucede en este caso.

La respuesta incorrecta es la 3, puesto que se trata de una vasculitis sistémica, con lo que es
frecuente el aumento de la VSG y alteraciones anémicas tipo trastorno crónico.(R3)

274. ¿Cuál sería la opción más adecuada a llevar a cabo en un RN cuya madre ha
presentado lesiones compatibles con varicela a las 24 horas después del parto?:

1. 1. Poner vacuna frente a la varicela al RN.


2. 2. Administración de aciclovir al RN.
3. 3. Administrar gammaglobulina al RN.
4. 4. Actitud expectante.
Gráfico de respuestas
Comentario

Concepto muy importante. Las indicaciones de profilaxis pasiva con gammaglobulina


antivaricela zoster son en las primeras 72 horas postexposición en:

- Immunodeprimidos sin el antecedente de vacunación o infección previa de varicela.

- Recién nacidos de madre cuya varicela comenzó entre los 5 días previos al parto y 48 horas
después.

- Prematuro de mayor o igual a 28 semanas cuya madre carece de antecedentes de varicela o


serología negativa.

- Prematuros menores de 28 semanas o de 1000g de PRN sea cual sea el estado serológico
materno.(R3)

275. ¿Cuál es el tratamiento de elección para la Ascaris lumbricoides?:

1. 1. Metronidazol.
2. 2. Atovacuona.
3. 3. Albendazol.
4. 4. Praziquantel.
Gráfico de respuestas
Comentario

!
!
!
!

Pregunta de dificultad media.

El Metronidazol se usa en infecciones por anaerobios, como por ejemplo infecciones colónicas,
pero también es tratamiento de elección para la giardiasis.

El Praziquantel sería tratamiento de elección para las teniasis.

El Pamoato de pirantel es un tratamiento de segunda elección para la oxiuriasis (primera


elección: Mebendazol)

Atovacuona sería el tratamiento alternativo para la infección por Pneumocystis jirovecii


asociadas al VIH (primera elección: TMP/SMX).

El Albendazol es de primera elección para Ascaris y para Estrongiloides.(R3)

276. El tratamiento de elección para la tosferina es:

1. 1. Eritromicina.
2. 2. Eritromicina e inmunoglobulina de la tosferina.
3. 3. Penicilina.
4. 4. Inmunoglobulina de la tosferina.
Gráfico de respuestas
Comentario

Pregunta sencilla y directa sobre el tratamiento de la tos ferina. Debemos saber desde la
primera vuelta que el antibiótico de elección para la Bordetella pertussis es la Eritromicina
(macrólidos).(R1)

277. ¿Cuál de las siguientes situaciones sería más favorable para obtener una
respuesta al tratamiento de la infección crónica por VHB con interferón?

1. 1. ALAT >100 miliunidades/ml, DNA del VHB <200 picogramos.


2. 2. ALAT normal y DNA del VHB <200 picogramos.
3. 3. ALAT mayor de 100 y DNA VHB >200 picogramos.
4. 4. ALAT normal y DNA VHB <200 picogramos.
Gráfico de respuestas
Comentario

Para acertar esta pregunta deben recordarse los criterios de buen pronóstico para respuesta al
tratamiento son:

- Sexo femenino.

- Edad joven.

- Niveles bajos de carga viral.

!
!
!
!

- Niveles altos de transaminasas (al contrario de lo que uno puede pensar inicialmente).

- Ausencia de infección VIH.

- Escasa fibrosis en la histología hepática.

Por tanto, la opción 1 es la que más se adapta a estas condiciones.(R1)

278. Una de las siguientes situaciones NO se considera englobada en la categoría de


angina de pecho inestable. Señale cuál:

1. 1. Angina de esfuerzo clase III de la CSC en paciente que nunca antes la había presentado.
Angina que acontece en las circunstancias habituales en cuanto a desencadenantes,
2. 2.
duración y frecuencia de los episodios.
3. 3. Angina típica con incremento de la frecuencia de episodios en los últimos días.
4. 4. Crisis de angina típica que sucede una semana después de un infarto de miocardio.
Gráfico de respuestas
Comentario

Debe dominar este tema o si o si para el Nacional y para la vida. Para contestar esta pregunta,
hay que saber cuándo definimos una angina como inestable. Toda angina que comienza en
reposo es inestable. También el angor postinfarto es inestable, por definición. En cuanto al
angor de esfuerzo, se considera inestable el de reciente comienzo (menos de 2 meses), siempre
que sea al menos clase III de la CSC - angor que produce limitación marcada de la actividad
física, es decir, que aparece al caminar una o dos manzanas o al subir un piso de escaleras- y el
progresivo - aumento de la intensidad, duración o número de los episodios o aparición de los
mismos a un umbral de esfuerzo cada vez menor. Con esto, hemos eliminado todas las
respuestas menos la 2. Es importante recordar que la duración no es, per se, un criterio de
inestabilidad.(R2)

279. Un niño de 4 años había sufrido resfriado común, permaneciendo propenso a los
hematomas y las petequias en los últimos 3 días. En la actualidad permanece afebril.

!
!
!
!

280. La biometría hemática revela una hemoglobina de 12.5 g/dl con VCM de 85 fl; 8,700
leucocitos por mm3 con 60% de neutrófilos, 35% de linfocitos y 5% de monocitos;
recuento plaquetario de 8,000 por mm3. El diagnóstico más probable es:

1. 1. Anemia aplástica.
2. 2. Síndrome hemolítico urémico.
3. 3. Meningococemia.
4. 4. Fiebre botonosa.
Gráfico de respuestas
Comentario

El síndrome hemolítico urémico es una microangiopatía trombótica que afecta a diversos


órganos, con predominio sobre el riñón, el tubo digestivo y el SNC. Es una de las causas más
frecuentes de IRA en niños menores de cinco años. Suelen existir antecedentes de
gastroenteritis enteroinvasiva por E. coli serotipo O157:H7 aunque también se describen casos
precedidos por Shigella.

Entre uno y 15 días tras el episodio desencadenante, se producen:

- Alteraciones hematológicas (preceden a fallo renal): anemia hemolítica microangiopática y


trombopenia. No alteraciones coagulación, ni CID.

- Disfunción renal: oligoanuria, hematuria, HTA.

- Afectación SNC (más frecuente PTT).

- Afectación gastrointestinal.

(R2)

281. Paciente de 52 años que consulta por notar desde hace una semana una
tumoración en su mama izquierda durante un autoexamen. Menopausia hace 4 años.
Estuvo recibiendo terapia de reemplazo hormonal con estrógenos que lo descontinuó
hace 1 año, niega antecedentes familiares de cáncer de mama. El examen revela un
nódulo palpable de 2 cm, indoloro, móvil en el cuadrante supero externo de mama
izquierda, no secreción por pezón a la compresión. Examen de mama derecha
negativo. Señale el siguiente paso más apropiado en el manejo de este caso:

1. 1. Reevaluación en 3 meses.
2. 2. Mastografía.
3. 3. Tomografía computarizada de tórax.
4. 4. Mastectomía.
Gráfico de respuestas
Comentario

Siempre el paso siguiente después de detectar una tumoración en mama es la realización de


mastografía. Respuesta 2 correcta.(R2)

!
!
!
!

282. En una lactante que luego de presentar fiebre le aparecen lesiones


maculopapulares centrífugas que el duraron dos días, el diagnóstico más probable es:

1. 1. Sarampión.
2. 2. Eritema infeccioso.
3. 3. Exantema súbito.
4. 4. Escarlatina.
Gráfico de respuestas
Comentario

Debes dominar este tema para fines del ENARM. Es el caso típico del exantema SÚBITO: las
lesiones maculopapulares aparecen después de un periodo febril, cuando éste cede.(R3)

283. Es llevada a urgencias una niña de 3 años por presentar, en el curso de 24 horas,
fiebre de 39ºC, dolor de garganta y, progresivamente, estridor afónico y dificultad
respiratoria. En la exploración se aprecia disnea leve, tiraje intercostal, aspecto tóxico,
postura en trípode y babeo. ¿Cuál sería su diagnóstico?:

1. 1. Crup espasmódico.
2. 2. Epiglotitis.
3. 3. Laringitis.
4. 4. Laringotraqueítis bacteriana.
Gráfico de respuestas
Comentario

Esta pregunta es importante porque nos describen una caso típico de epiglotitis, que no
debemos tener problemas en reconocer.

Es una causa de dificultad respiratoria alta que, aunque poco frecuente muy grave. Es una
inflamación de los tejidos supraglóticos de instauración brusca y rápidamente progresiva que
puede provocar una obstrucción total de la via aérea superior que cause la muerte.

La introducción de la vacuna contra Haemofilus influenziae tipo B ( Hib) ha reducido su


incidencia.

El término laringitis puede ser encontrado en la literatura como crup, laringotraqueitis y laringitis
subglótica. Todos ellos describen un cuadro caracterizado por afonía, tos, estridor inspiratorio y
dificultad respiratoria. Al contrario que en las laringitis agudas el estridor en la epiglotitis no es
tan ruidoso y es rara la tos.

Las laringitis son producidas en su mayoría por agentes virales pero pueden complicarse por
sobreinfección bacteriana. Existen dos entidades laringitis aguda viral (LAV) y crup
espasmódico, ambas con el mismo tratamiento.(R2)

284. Niña de 2 años de edad que, a su llegada a Urgencias, presenta mal aspecto con
fiebre de 40.2ºC con estridor inspiratorio y babeo intenso. Destaca una posición del

!
!
!
!

cuello en hiperextensión con protrusión de la lengua. Sobre la patología que sospecha,


¿cuál de las siguientes opciones es INCORRECTA?:

1. 1. Empeora con el llanto y con el decúbito prono.


2. 2. La etiología más frecuente es bacteriana.
3. 3. Se debe evitar la manipulación de la cavidad oral.
4. 4. El tratamiento será asegurar la vía aérea y tratamiento antibiótico.
Gráfico de respuestas
Comentario
Estamos ante un caso típico de epiglotitis aguda con una niña con aspecto tóxico y posición en
trípode, además presenta fiebre alta e intenso babeo. La respuesta menos correcta es la
primera ya que empeora con el llanto y sobre todo con el decúbito supino aunque estos niños
hay que mantenerlos sentados y tranquilos para que no se colapse la vía aérea.(R1)

285. A 16-year-old girl goes to her physician because she has been feeling very tired
for the past 2 years with poor appetite and trouble concentrating in school. She sleeps
more than 12 hours a day, and still gets up tired. When asked about her friends, she
says that she does not feel like going out anymore. She gets good grades at school,
and she does not have any problems with her family. What is the most likely diagnosis?

1. 1. Dysthymia.
2. 2. Schizotypal personality disorder.
3. 3. Avoidant personality disorder.
4. 4. Borderline personality disorder.
Gráfico de respuestas
Comentario
Dysthymia. This patient is suffering from dysthymia. The essential feature of persistent
depressive disorder (dysthymia) is a depressed mood for most of the day, more days than not,
for at least 2 years (at least 1 year for children and adolescents). During periods of depressed
mood, at least two of the following six symptoms are present: poor appetite or overeating,
insomnia or hypersomnia, low energy or fatigue, low self-esteem, poor concentration or difficulty
making decisions and feelings of hopelessness. In order to meet the diagnostic criteria for
Dysthymic Disorder, the symptoms must not be due to the direct physiological effects of the use
or abuse of a substance (for instance, alcohol, drugs, or medications) or a general medical
condition (e.g., cancer or a stroke). The symptoms must also cause significant distress
orfunctional impairment.(R1)

286. Mujer de 45 años que, al igual que en múltiples ocasiones anteriores, es traída a
urgencias por presentar, tras una discusión familiar, un cuadro de incosciencia.
Presenta oposición manifiesta a la apertura de ojos y se recupera por técnicas
sugestivas. Probablemente se trata de:

1. 1. Crisis de inhibición histérica.


2. 2. Síndrome de Briquet.
3. 3. Catatonia.
4. 4. Síndrome neuroléptico maligno.
Gráfico de respuestas
Comentario

!
!
!
!

El término clásico de histeria hace referencia a la aparición de síntomas objetivos sin sustrato
anatómico que los justifique (por ejemplo, una parálisis sin lesión muscular ni nerviosa, una
sordera con el oído y las vías auditivas íntegras…). En todos estos casos de aparente
enfermedad orgánica, se supone que la causa es un conflicto psicológico, que se convierte en
un síntoma orgánico que lo simboliza. Por ello, también se denomina neurosis de
conversión o alteraciones somatomórficas.

Existe otra forma de histeria en que el problema psicológico no se simboliza a través de la


imitación de una enfermedad corporal, sino de una enfermedad psíquica, en estos casos se
habla de trastornos disociativos, pues la identidad está disociada. Un ejemplo sería una amnesia
histérica, en la que el sujeto olvida quién es, y en algunos casos altera su propia identidad con la
de nuevos personajes que va creando o que acepta, como en el trastorno de personalidad
múltiple.

En el caso que nos plantean, existen varios datos que orientan al carácter conversivo de los
síntomas:

- Le ha ocurrido en múltiples ocasiones.

- Situación estresante como causa aparente (discusión familiar).

- Oposición manifiesta a la apertura de los ojos.

- Recuperación por técnicas sugestivas.

Uno de los datos típicos de las crisis histéricas es la “bella indiferencia” hacia los síntomas. Por
ejemplo, un paciente con una sordera de origen histérico no manifestaría especial desagrado
con su situación, cuando lo lógico sería todo lo contrario. La paciente de la pregunta va todavía
más lejos: intentamos abrirle los ojos y los cierra voluntariamente.(R1)

287. Paciente de 55 años de edad que fue intervenido de un tumor del colon sigmoides.
Se trataba de un adenocarcinoma de 3 cm que invadía la pared de colon sobrepasando
la muscular; también presentaba un ganglio epicólico metastatizado (Asher-Coller B2).
¿Cuál de las opciones terapéuticas que a continuación se mencionan es la más eficaz
en este caso?

1. 1. Bastaría con el tratamiento quirúrgico realizado.


2. 2. Radioterapia coadyuvante.
3. 3. Realizar quimioterapia postoperatoria (5- fluorouracilo y ácido fólico).
4. 4. Asociar 5-fluorouracilo y radioterapia.
Gráfico de respuestas
Comentario

Se trata de una pregunta de dificultad media.

Estamos ante una pregunta sobre el cáncer de colon en la que debemos saber estadificar el
cáncer y decidir en función del estadiaje la necesidad o no de tratamiento coadyuvante. Así
pues, se trata de un cáncer del colon sigmoides que sobrepasa la muscular, por lo que hemos

!
!
!
!

de asumir que llega a la serosa, de modo que localmente sería un estadio B2. Sin embargo,
añaden el dato de la existencia de un ganglio epicólico metastásico, lo que lo convertiría en un
C2. Así que podría ser impugnable porque el enunciado afirma que el estadio es un B2.

En cualquier caso, sea B2 o C2, este paciente tendría indicación de tratamiento adyuvante que,
por la localización, se tendría que hacer con quimioterapia exclusivamente (lo que excluye las
opciones 1, 2 y 4). El protocolo clásico de quimioterapia que ha demostrado un aumento de
supervivencia a largo plazo incluye la utilización de 5-FU.(R3)

Clasificación de Astler-Coller (Dukes modificado)

288. ¿Qué tipo de pacientes se pueden beneficiar del tratamiento con talamotomía en
la enfermedad de Parkinson?:

1. 1. Aquéllos con bradicinesia invalidante.


2. 2. Aquéllos con temblor bilateral que no responde a fármacos.
3. 3. Aquéllos con temblor unilateral que no responde al tratamiento médico.
4. 4. Aquéllos con temblor unilateral a pesar de que el tratamiento sea efectivo.
Gráfico de respuestas
Comentario

Pregunta de extrema dificultad y que no le debe preocupar si la ha fallado. La palidotomía o


talamotomía posteroventral unilateral puede mejorar la rigidez, la bradicinesia y el temblor de los
pacientes parkinsonianos en fases avanzadas en los que el tratamiento farmacológico no es
eficaz o es muy mal tolerado. Hasta en el 90% de los pacientes existe una respuesta positiva a
la cirugía con un predominio del efecto beneficioso en el lado conteralateral al intervenido. La
cirugía bilateral tiene una morbilidad mucho mayor por lo que no suele aconsejarse, pues las
tres necesitarían intervenciones bilaterales.(R3)

289. Con relación al Test de Apgar, señale la respuesta INCORRECTA:

1. 1. Mide cinco parámetros: Respiración, frecuencia cardiaca, color, tono e irritabilidad refleja.
2. 2. Se realiza al 1 er. y 5 to. minuto después del parto.
3. 3. La valoración del Apgar no interfiere con la reanimación neonatal.
Un puntaje menor a 7 requiere que al neonato se le de asistencia probablemente con
4. 4.
oxigenoterapia.
Gráfico de respuestas

!
!
!
!

Comentario

Debes dominar el APGAR para el día del ENARM.(R4)

Recuerde

El test de Apgar valora cinco parámetros: esfuerzo respiratorio, respuesta a la estimulación (o a


la introducción de una SNG), frecuencia cardíaca, tono muscular y color.

Cada parámetro se valora entre 0 y 2 puntos. La situación óptima de cada parámetro se evalúa
con dos puntos; la situación más desfavorable ("ausencia de"), con cero puntos. La situación
intermedia se valora con 1 punto.

El test de Apgar se realiza al minuto de vida y a los 5 minutos (y cada 5 minutos hasta los 20
minutos de nacido si la puntuación a los 5 minutos es inferior a 7).

Un test de Apgar bajo en los primeros minutos de vida no sirve para establecer valoraciones
pronósticas. No obstante, una puntuación inferior a 3, mantenida más allá de los 20 minutos de
vida, sí puede predecir una elevada morbi-mortalidad.

290. Niña de 6 meses con el antecedente


de cuadro catarral de 8 días de evolución, sin fiebre, que su pediatra del centro de
salud ha tratado con mucolíticos y lavados nasales con suero fisiológico. En las

!
!
!
!

últimas 12 horas ha presentado varias crisis de llanto y un vómito, siendo las


evacuaciones normales, la última hace 24 horas. Durante su estancia en la sala de
espera presenta una evacuación con sangre fresca. A la exploración tiene todas las
constantes normales para su edad, presenta un aceptable estado general, aunque
usted la nota decaída, pálida y con tendencia al sueño. La auscultación cardíaca y
pulmonar son normales. En la palpación abdominal no se evidencian masas ni
megalias, el abdomen es blando y depresible, y no presenta signos de irritación
peritoneal, aunque usted sospecha dolor a la palpación en hemiabdomen derecho.
Tras la palpación abdominal la niña presenta una crisis de llanto agudo, con
encogimiento de piernas de 2 minutos de duración, quedándose posteriormente
tranquila y con tendencia al sueño. Decide pedir un ultrasonido urgente que puede
observar en la imagen que acompaña a esta pregunta. Respecto al cuadro que
sospecha, indique cuál de los siguientes enunciados le parece menos correcto:

Este cuadro clínico es la causa más frecuente de obstrucción intestinal entre los 3 meses y
1. 1. los 6 años de edad, con un pico de máxima incidencia entre los 4 y los 12 meses, siendo
raro en menores de un mes.
En la mayoría de los casos la causa es desconocida, aunque en mayores de 2 años es más
2. 2.
probable que exista una causa anatómica que desencadene el cuadro.
Algunas causas relativamente frecuentes en los casos secundarios son la púrpura de
3. 3.
Schönlein-Henoch, infecciones por adenovirus y el divertículo de Meckel.
En el ultrasonido abdominal, prueba diagnóstica de primera elección, son típicos el signo
4. 4.
de la rosquilla y el signo del muelle enrollado.
Gráfico de respuestas
Comentario

La lectura de un ultraosinido es muy difícil, pero en este caso no es necesario saber


interpretarla, ya que con la información del caso clínico puede hacer el diagnóstico.

Se trata sin duda de una invaginación intestinal. La clínica típica de este cuadro son episodios
de llanto intenso, con palidez facial y encogimiento de piernas que alternan con episodios de
normalidad o de letargia.

La evacuación con sangre son“heces en jalea de grosellas”. Todas las opciones son correctas,
excepto la 4 ya que el signo del muelle enrollado NO es un signo ultrasonográfico, sino un
hallazgo que se observa en el enema opaco.

El enema opaco se realiza sólo en casos en los que existan dudas después del ultrasonido.

Recuerde que la prueba diagnóstica de primera elección es el ultrasonido abdominal en el que


se observa el sigo de la dona o de la rosquilla.(R4)

!
!
!
!

291. Una mujer de 32 años en su tercer


mes de embarazo comienza a sangrar por lo que acude al Servicio de Urgencias. En la
exploración se confirma la metrorragia en cantidad menor que regla, abdomen blando
con un útero más grande que el esperado para su edad gestacional. Usted le realiza
una ecografía en la que encuentra los hallazgos mostrados en la imagen. ¿Cuál es el
diagnóstico?

1. 1. Amenaza de aborto
2. 2. Aborto incompleto.
3. 3. Gestación ectópica.
4. 4. Mola completa.
Gráfico de respuestas
Comentario

Pregunta sencilla sobre hemorragia del primer trimestre del embarazo. Se trata de una mola
completa ya que menciona que el útero es más grande que el esperado para su edad
gestacional y en la ecografía se puede observar los típicos copos de nieve.(R4)

292. ¿Cuál es el tratamiento más adecuado para la paciente del caso clínico anterior?

1. 1. Reposo relativo y abstinencia relaciones sexuales.


2. 2. Legrado por aspiración.
3. 3. Administración de prostaglandinas y legrado si persistencia de restos.
4. 4. Tratamiento con metotrexate.
Gráfico de respuestas
Comentario

En pacientes con enfermedad trofoblástica, el tratamiento de elección es el legrado por


aspiración. Respuesta 2 correcta.(R2)

!
!
!
!

293. Una mujer de 65 años consulta por metrorragias escasas desde hace tres meses.
La exploración ginecológica es normal. En un USG transvaginal se observa un útero
de 7 x 3 x 4 cm con un endometrio de 14 mm de espesor, así como unos ovarios
atróficos. La citología cervicovaginal informa de un frotis atrófico sin otras
alteraciones celulares. En una biopsia endometrial realizada por aspiración con una
cánula flexible de tipo Cornier o Pipelle (microlegrado) se informa de escaso material
endometrial de tipo atrófico, insuficiente para un diagnóstico endometrial adecuado.
¿Cuál de las siguientes indicaciones es la más adecuada?:

Tratamiento hemostático con estrógenos + progestágenos, seguido de progestágenos


1. 1.
cíclicos cada mes, durante 6 meses.
Repetición de la biopsia si vuelve a sangrar, y en caso contrario, repetición de la ecografía
2. 2.
a los 4-6 meses.
3. 3. Histeroscopía.
Completar el estudio con determinación de marcador tumoral CA 12.5 y otras pruebas de
4. 4.
imagen como RNM o TAC.
Gráfico de respuestas
Comentario

Nos presentan un caso sospechoso de carcinoma endometrial (mujer postmenopaúsica con


metrorragias). Recordemos que el método ideal para obtener una biopsia endometrial es realizar
una histeroscopía y biopsia dirigida. EL USG TV es orientativo pero no diagnostica y la muestra
obtenida por aspiración con cánula flexible no es suficiente para realizar un diagnóstico
concluyente.(R3)

294. ¿Cuál de las siguientes afirmaciones sobre los anticuerpos antinucleares es


INCORRECTA?

Los anticuerpos más específicos del lupus eritematoso sistémico son los anti-Sm y los anti-
1. 1.
DNA nativo.
2. 2. Los anticuerpos anti-RNP se asocian a la presencia de fenómeno de Raynaud.
El título de anticuerpos anti-DNA de doble cadena no guardan relación con la actividad de
3. 3.
la glomerulonefritis lúpica.
4. 4. Los anticuerpos anti-plaquetarios y los anti-neuronales son relativamente poco específicos.
Gráfico de respuestas
Comentario

Los anticuerpos anti- DNA de doble cadena no sólo son útiles en el diagnóstico de LES al formar
parte de los criterios diagnósticos, sino que nos informan sobre la actividad y del desarrollo de
nefritis. Respuesta 3 incorrecta.(R3)

295. Los análogos de la GnRH se utilizan como tratamiento para todos los siguientes
EXCEPTO:

1. 1. Endometriosis.
2. 2. Miomas uterinos.
3. 3. Fallo ovárico precoz.
4. 4. Estimulación ovárica en técnicas de reproducción asistida.
Gráfico de respuestas

!
!
!
!

Comentario
Los análogos de GnRH desensibilizan las células por internalización de sus receptores,
inhibiendo la LH y la FSH, provocando un estado de hipoestrogenismo, que los hace útiles para
todos menos para el fallo ovárico precoz, en el que ya hay una situación de hipoestrogenismo, y
el tratamiento será justo el contrario, con terapia hormonal sustitutiva.(R3)

296. Niño de 4 meses de edad, que acude a valoración por contacto cercano con
hermano de 3 años con diagnóstico de sarampión. ¿Cuál sería el manejo adecuado en
beneficio del lactante?:

1. 1. Inmunización inmediata con vacuna de virus muertos.


2. 2. Dosis de terapia de gamma globulina.
3. 3. Dosis preventiva de gamma globulina.
Dosis de terapia de gamma globulina junto con vacuna de virus atenuados en el mismo
4. 4.
momento pero en lugares diferentes.
Gráfico de respuestas
Comentario

Dado que la vacuna triple viral no se recomienda hasta los 12 meses, antes de ese tiempo el
niño tiene riesgo de contagio. La administración de inmunoglobulina profiláctica después de la
exposición al virus puede reducir el riesgo de desarrollar el sarampión o puede hacer que la
enfermedad sea menos severa, aunque no protege a los que se encuentren ya infectados y en
el periodo de incubación.(R3)

297. When evaluating a patient who complains of a visual defect, you discover a left
homonymous hemianopia with macular sparing. Select the CORRECT statement about
the case:

1. 1. The lesion is located in the left parietal optic radiation.


2. 2. The lesion is located in or around the left calcarine fissure, in the occipital lobe.
3. 3. The lesion is located in or around the right calcarine fissure, in the occipital lobe.
The most likely origin of the lesion is an embolic occlusion of the right middle cerebral
4. 4.
artery.
Gráfico de respuestas
Comentario

Una pregunta como ésta no se debe fallar ya que el tema de las alteraciones campimétricas en
la patología de la vía óptica es muy preguntado en el ENARM. La hemianospisa homónima
congruente con visión macular respetada debe hacernos pensar en una lesión en el lóbulo
occipital (cisura calcarina). En este caso, la lesión es en el lóbulo occipital derecho porque la
hemianopsia homónima es izquierda (el campo visual izquierdo depende de la retina nasal
izquierda y la retina temporal derecha; las fibras de la retina nasal izquierda cruzan la línea
media a nivel del quiasma y se juntan ya con las procedentes de la retina temporal derecha para
seguir ipsilateralmente hacia el córtex occipital).(R3)

298. La hipertensión es una complicación común de todas las patologías siguientes


MENOS una, señálela:

1. 1. Embarazo.

!
!
!
!

2. 2. Anafilaxia.
3. 3. Síndrome de Cushing.
4. 4. Tumor de fosa posterior.
Gráfico de respuestas
Comentario

Se trata de una pregunta sencilla que puede contestar con el conocimiento de otras asignaturas.
Como ha estudiado en neurología, el tumor de fosa posterior puede desencadenar un cuadro de
hipertensión intracraneal que se manifiesta con la tríada de Cushing (HTA, bradicardia y
alteraciones del ritmo respiratorio). De obstetricia debe saber que un posible complicación del
embarazo es la preeclampsia, caracterizada por HTA más proteinuria después de la semana 20
del embarazo. En endocrino ha estudiado que una de las manifestaciones más características
del síndrome de Cushing. En la anafilaxia se produce una situación de choque distributivo con
vasodilatación e hipotensión.(R2)

299. ¿Cuál es el tratamiento correcto para una pareja estéril por una obstrucción
bilateral de las trompas?

1. 1. La insuflación de las trompas.


2. 2. La inseminación artificial con semen del marido.
3. 3. La inseminación artificial con semen de donante.
4. 4. La fecundación in vitro.
Gráfico de respuestas
Comentario

Una obstrucción bilateral de las trompas no podría tratarse con inseminación artificial, ya del
marido o de un donante, porque el gameto femenino encontraría las mismas dificultades para
progresar por las trompas y alcanzar la cavidad uterina. Inducir la ovulación con gonadotropinas
también es absurdo, puesto que el problema persiste, ovule o no. Por último, la insuflación de
trompas se emplea para comprobar la permeabilidad tubárica, y ya nos han dicho que están
obstruidas bilateralmente.

En conclusión, la única alternativa válida es la fecundación in vitro, que es la técnica más


adecuada en la obstrucción tubárica bilateral. Implica obtener ovocitos (normalmente, mediante
punción folicular vía vaginal). Se ponen en contacto con los espermatozoides y, obtenida la
fecundación, se transfieren a la cavidad uterina. Sobre esta técnica, has de saber que aumenta
el riesgo de gestación ectópica o abortos, pero no el de malformaciones congénitas.(R4)

300. Mujer de 45 años que acude a Urgencias por debilidad generalizada, náuseas y
dolor en las articulaciones. En sus antecedentes personales destacan episodios de
púrpura cutánea fundamentalmente al exponerse al frío, fiebre y artralgias. En el
interrogatorio, la paciente nos revela que hace dos días que orina muy poco y lo poco
que orina tiene un color oscuro como el "cognac". En la exploración presenta
hepatoesplenomegalia importante. En examen general de orina destaca intensa
hematuria y proteinuria moderada. Respecto al cuadro que usted sospecha, señale la
respuesta FALSA:

1. 1. Puede evolucionar a una glomerulonefritis con semilunas.


2. 2. En el tratamiento se usan corticoides e inmunosupresores y, en ocasiones, plasmaféresis.
3. 3. El riñón se afecta en el 75% de los casos.

!
!
!
!

4. 4. No se conoce con seguridad su etiología.


Gráfico de respuestas
Comentario

Esta pregunta es complicada, por lo que tranquilo. Esta mujer presenta por sus síntomas y por la
presencia de una púrpura cutánea una vasculitis. Es importante darse cuenta de que ésta
aparece cuando la paciente se expone al FRIO, esto nos orienta a una Crioglobulinemia. Otros
datos son orinas colúricas, seguramente por hematuria por una GN y una hepatoesplenomegalia
que podemos suponer que puede deberse a una hepatitis posiblemente C. Finalmente, la opción
4 es la falsa.(R3)

301. A 17-year-old male patient comes to the emergency department with a 30-minute
history of shortness of breath, nonproductive cough and wheezing. He also complains
of chest pain. X-ray of the chest shows no abnormal findings. After stabilisation and
discharge, he is refered to a pneumologist for further studies. Espirometry shows FEV1
of 65% of the average value. Which of the following is the most appropriate treatment?

1. 1. Pro re nata inhaled short-acting beta2-agonists.


2. 2. Medium dose inhaled budesonide and salmeterol. Pro re nata salbutamol.
3. 3. Low dose inhaled budesonide and pro re nata salbutamol.
4. 4. Low dose oral prednisone and pro re nata inhaled salbutamol.
Gráfico de respuestas
Comentario

Pregunta sencilla sobre el tratamiento de un paciente con diagnóstico de asma. En el este caso
el tratamiento es un corticoides inhalados, asociado a b2 agonistas de larga duración y con
salbutamol PRN, respuesta correcta 2.(R2)

302. En el manejo del paciente asintomático infectado por el VIH se aconseja la


realización de serología para los siguientes gérmenes, EXCEPTO:

1. 1. Toxoplasma.
2. 2. Criptococo.
3. 3. Treponema pallidum.
4. 4. Virus C de hepatitis.
Gráfico de respuestas
Comentario

Podemos plantearnos la necesidad de realizar serología frente a CMV, ya que muchas personas
son positivas, con independencia de su infección por VIH… Pero, desde luego, lo que no
podemos pedir es una serología que no es útil, como sucede con la criptocócica. Aunque esta
prueba existe, carece de utilidad debido a que la sensibilidad es muy escasa.(R2)

303. Un muchacho de 17 años de edad con enfermedad de Crohn con afectación


colónica de 2 años de evolución, en tratamiento de mantenimiento con azatioprina,
consulta por la aparición desde hace 5 días de nódulos subcutáneos rojo-violáceos,
calientes, dolorosos, de localización pretibial, bilaterales asociados a un incremento

!
!
!
!

del número de evacuaciones y dolor abdominal. La actitud más adecuada en este caso
es:

Recomendar reposo relativo y paños calientes en ambas piernas y añadir tratamiento


1. 1.
antidepresivo.
Biopsiar zonas de piel alejados de las áreas lesionadas y pautar analgésicos opioides de
2. 2.
entrada.
3. 3. Sospechar isquemia bilateral de extremidades inferiores de origen medicamentoso.
4. 4. Ajuste del tratamiento de la enfermedad intestinal.
Gráfico de respuestas
Comentario

El eritema nodoso, es un tipo característico de lesión de la piel que consiste en la presencia de


nódulos dolorosos que presentan signos inflamatorios (enrojecimiento y aumento de
temperatura) y afectan predominantemente a la zona pretibial (delante de la tibia), en las
extremidades inferiores, aunque puede tener otras localizaciones. Las lesiones suelen
desaparecer en un plazo de entre 1 y 3 semanas.

La existencia de lesiones cutáneas, diarrea y dolor es sugestivo de brote de su enfermedad de


Crohn, por lo que es necesario ajustar el tratamiento de la misma e iniciar tratamiento con
esteroides.(R4)

304. Masculino de 46 años de edad, con antecedentes personales de HTA y colitis


ulcerosa corticodependiente de larga evolución. En esta última se indicó tratamiento
quirúrgico por refractariedad al tratamiento médico. Realizado el estudio preoperatorio
que no contraindicaba la cirugía, se llevo a cabo una panproctocolectomía con
reservorio ileoanal e ileostomía temporal de protección. El postoperatorio transcurrió
de forma normal, por lo que al tercer día fue dado de alta. En seguimiento en las
consultas de Cirugía General, y dada su buena evolución, se decidió el cierre de la
ileostomía transcurridos 2 meses de la primera intervención. De nuevo, el
postoperatorio inmediato cursó sin incidencias, por lo que fue dado de alta a los 4 días.
En la revisión al mes de la cirugía, el paciente refiere estar satisfecho, pues según nos
cuenta, tiene buena continencia y el ritmo defecatorio lo considera normal. Dos meses
después de esta visita, el paciente acude a Urgencias por cuadro de rectorragia,
aumento de la frecuencia defecatoria, dolor abdominal y fiebre. De la complicación que
sospechamos ha desarrollado, ¿cuál considera el tratamiento más CORRECTO?:

1. 1. Cirugía programada de entrada.


2. 2. Corticoides orales.
3. 3. Antibioterapia y enemas de corticoides.
4. 4. No precisa tratamiento.
Gráfico de respuestas
Comentario
Se trata de una reservoritis aguda y suelen responder bien al tratamiento médico con
antibióticos. Conviene diferenciarla de la reservoritis crónica en la que existe buena respuesta
inicial al tratamiento convencional, pero recidiva precoz: en estos casos el tratamiento asocia 5
ASA, corticoides, inmunosupresores o infliximab, y a veces es preciso plantearse extirpar el
reservorio y hacer una anastomosis ileoanal convencional.(R3)

!
!
!
!

305. A un niño febril de 16 meses de edad se le realizó una punción lumbar por
sospecha de meningitis. ¿Qué característica se considera anormal en el líquido
cefaloraquídeo?

1. 1. Proteínas de 15 mg/dl.
2. 2. Xantocromía.
3. 3. Glucorraquia equivalente a la mitad de la glicemia.
4. 4. Dos leucocitos.
Gráfico de respuestas
Comentario

En la sospecha de meningitis, uno de los hallazgos de sospecha de origen bacteriano es el


consumo de glucosa por las bacterias en el LCR, que suele encontrarse por debajo del 50% del
valor sanguíneo al momento de la punción.(R3)

306. En relación con el virus de la hepatitis tipo B, indique cuál de las siguientes le
parece INCORRECTA:

1. 1. Está claramente establecido el carácter citopático directo del virus de la hepatitis B.


2. 2. El virus de la hepatitis de tipo B se ha identificado a nivel extrahepático.
El HBeAg y las proteínas pre-S1 y pre-S2 se expresan durante los períodos de replicación
3. 3.
máxima.
En el 1 al 5 % de las infecciones agudas no se detecta HBsAg por tener concentraciones
4. 4.
demasiado bajas.
Gráfico de respuestas
Comentario

Cuando se padece una hepatitis B, la lisis de los hepatocitos no es responsabilidad directa del
propio VHB, sino del sistema inmune, que ataca a las células infectadas. Por ello, la opción falsa
es la 1. Respecto al resto de las opciones, merece la pena recordar lo que nos dicen sobre el
HBeAg y su relación con la replicación vírica.(R1)

307. ¿Qué patología obstétrica se asocia con más frecuencia a las distocias dinámicas
del parto?:

1. 1. Desproporción pélvico-cefálica.
2. 2. Hiperdistensión uterina (gemelar, polihidramnios).
3. 3. Causa yatrógena.
4. 4. Patología cervical.
Gráfico de respuestas
Comentario

Las distocias dinámicas son aquellas situaciones en las que no existe una adecuada dinámica
uterina y por ello no progresa de forma normal el periodo de dilatación. Podemos hacer dos
grandes grupos de distocias dinámicas: - Disminución de la contractilidad uterina (hipodinamia).
Es la alteración más frecuente. Se caracteriza por la presencia una intensidad contráctil baja
(<25mmHg) o una frecuencia contractil baja (inferior a 2 contracciones cada 10 minutos). La
hipodinamia puede ser primaria o secundaria (más frecuente). En cuanto a la etiología, en la
primaria muchas veces no tiene una etiología conocida. La secundaria se debe a un

!
!
!
!

agotamiento muscular uterino y en lo primero que debemos pensar es en una desproporción


pelvicocefálica no diagnosticada. Aumento de la contractilidad uterina (hiperdinamia). Dentro de
este grupo consideramos tres entidades que pueden aparecer aisladas o mezcladas que son la
hipersistolia (intesidad >50mmHg), polisistolia (frecuencia contractil superior a 5 contracciones
cada 10 minutos) e hipertonía (tono basal >12mmHg). La hiperdinamia en muchas ocasiones
aparece como reacción a un obstáculo mecánico durante el parto como puede ser la
desproporción pelvicocefálica.(R1)

308. Es un indicador de mal pronostico en pancreatitis aguda:

1. 1. Amilasa mayor de 1000.


2. 2. Obesidad.
3. 3. Bilirrubina total mayor de 5.
4. 4. Lipasa mayor de 1000.
Gráfico de respuestas
Comentario

Debe mostrar con soltura conocimientos sobre pancreatitis, es preguntada año con año en todos
sus aspectos. La respueta es la número 2, ya que se ha observado que los pacientes obesos
presentan un peor pronóstico.

Nunca debe de olvidar que los niveles de amilasa y de lipasa, NO sirven como pronóstico, sin
importar el nivel de elevación.(R2)

309. La primera opción de entre las siguientes de tratamiento en un niño con un petit
mal (ausencias típicas) debería ser:

1. 1. Topiramato
2. 2. Acido valproico.
3. 3. Levetiracetam
4. 4. Fenitoina
Gráfico de respuestas
Comentario

Para el tratamiento de las ausencias típicas del niño se nos tienen que venir a la cabeza dos
tratamientos en primer lugar: la Etosuximida (fármaco que solamente se usa en el tratamiento de
las ausencias) y el Ácido Valproico. Como el primero no está entre las respuestas, la verdadera
es el ácido valproico. El fenobarbital y la fenitoina no son muy adecuados para los niños por sus
efectos secundarios (síndrome de hiperactividad e hiperplasia gingival e hirsutismo
respectivamente). El topiramato y el levetiracetam son dos fármacos muy nuevos que no serían
de elección para un tratamiento inicial.(R2)

!
!
!
!

310. A 53
years old man with active smoking and hypertension became to the consulting room
of the general practitioner because of recurrent episodes of chest pain feeling like a
pressure spreading to his left arm triggered by exercise and relieved by rest in 5 or 10
minutes. Last episode occurred two days ago. What is CORRECT?

The patient presents an acute coronary syndrome without ST segment elevation


1. 1.
(NSTEMI).
The patient must initiate double antiplatelet therapy and low molecular weight heparin
2. 2.
adjusted to the patient’s weight.
3. 3. Nitrates administration may reduce symptoms and improve life quality.
4. 4. The best choice for hypertension treatment in this patient is an alpha-blocker.
Gráfico de respuestas
Comentario

Ante un cuadro clínico de un varón de mediana edad, con factores de riesgo cardiovascular, que
presenta dolor torácico ante una situación que implica "estrés" (ej: ejercicio) y desaparece con el
reposo, deberemos pensar en una cardiopatía isquémica presentada como un cuadro de angina
(en este caso, inestable porque, según parece, es el primer episodio que presenta).

De las opciones disponibles, la respuesta correcta es la número 3: Los nitratos son


vasodilatadores que disminuyen el dolor de causa coronaria e incrementan la calidad de vida de
estos pacientes.(R3)

311. La presencia de punteado basófilo en los hematíes sugiere el diagnóstico de:

1. 1. Esferocitosis hereditaria.
2. 2. Sepsis por gramnegativos.
3. 3. Saturnismo.
4. 4. Insuficiencia renal.
Gráfico de respuestas
Comentario

!
!
!
!

Los hallazgos en el frotis sanguíneo suelen ayudarnos a encontrar el diagnostico sobre todo en
los casos clínicos, por lo que conviene aprenderse algunos de ellos, como el punteado basófilo
en los hematíes. Estos se observa en los trastornos de la síntesis del hem, como el saturnismo
(opción 3 correcta), los estados diseritropoyéticos (anemias sideroblasticas), así como en
algunas enzimopatías (déficit de pirimidina 5' nucleotidasa).(R3)

312. Una de las siguientes NO es complicación frecuente del abruptio:

1. 1. Coagulación intravascular diseminada.


2. 2. Insuficiencia renal.
3. 3. Choque hipovolémico.
4. 4. Pielonefritis.
Gráfico de respuestas
Comentario

Como complicaciones del abruptio, se han descrito todas las respuestas, excepto la pielonefritis
(opción 4). Las más frecuentes son la CID y la insuficiencia renal. La CID también se presenta
en abortos diferidos y la patogenia es similar en ambos casos: liberación y paso de
tromboplastinas fetales a circulación materna. El tratamiento consiste en una adecuada
reposición de factores de coagulación y fibrinógeno, al mismo tiempo procedemos a la
evacuación inmediata del útero, mediante la realización de una cesárea. El choque hipovlémico
y el síndrome de Sheehan son complicaciones más raras y se deben a una atonía uterina
secundaria a un útero de Couvelaire.(R4)

313. Paciente de 35 años, diagnosticado previamente de colitis ulcerosa, que acude al


hospital por presentar en el último mes astenia, prurito, pérdida de peso, dolor
abdominal y episodios recurrentes de ictericia. En la exploración física presenta
hepatoesplenomegalia. Los exámenes de laboratorio muestran aumento de fosfatasa
alcalina y de la IgM con anticuerpos antimitocondriales negativos. El cuadro es
sugerente de:

1. 1. Colangitis esclerosante primaria.


2. 2. Cirrosis biliar primaria.
3. 3. Hepatitis crónica autoinmune.
4. 4. Sarcoidosis.
Gráfico de respuestas
Comentario

Nos están describiendo un caso clínico de colangitis esclerosante primaria (hallazgos típicos en
la CPRE), en un paciente que además padece una colitis ulcerosa, lo que refuerza aún más
nuestra sospecha. La colitis ulcerosa y la colangitis esclerosante primaria se relacionan en
muchos casos.

Esta entidad es de base autoinmune (expresión de p-ANCA) y aparece con mayor frecuencia en
hombres. La siguiente prueba que deberíamos realizar es una CPRE, para observar las lesiones
de las vías biliares, donde encontraremos múltiples estenosis y dilataciones.(R1)

314. Masculino de 70 años, que nos es remitido por presentar imágenes


osteocondensantes en D9 y L1. Presenta clínica llamativa de dolor lumbar y

!
!
!
!

prostatismo leve. En el tacto rectal, se aprecia una induración pétrea de la próstata que
ocupa más del 60% de la glándula, con los márgenes borrados. La actitud más
CORRECTA sería:

1. 1. Ecografía transrectal, biopsia prostática y comenzar tratamiento con agonistas LHRH.


2. 2. No hacer nada, ya que tiene metástasis.
Ecografía transrectal, biopsias prostáticas, gammagrafía ósea y comenzar tratamiento con
3. 3.
antiandrógenos y agonistas LHRH.
Realizar prostatectomía radical, si los estudios de extensión son negativos y la ecografía
4. 4.
transrectal es informada como sospechosa de cáncer de próstata.
Gráfico de respuestas
Comentario

El caso que nos ofrecen corresponde a un cáncer de próstata diseminado, que habrá que
confirmar histológicamente (biopsia prostática). Esta biopsia no sólo tiene sentido desde el punto
de vista diagnóstico, sino también pronóstico, ya que podemos obtener el score de Gleason a
través de biopsias múltiples en la glándula.

Desde el punto de vista del tratamiento, existen dos posibilidades: castración quirúrgica o
farmacológica. En este último caso, debemos asociar antiandrógenos con los agonistas de la
LHRH, ya que de lo contrario se produciría un importante pico androgénico al comienzo del
tratamiento, que sería contraproducente.(R3)

315. Paciente de 32 años, con taquipnea y sensación disnéica. Toma anticonceptivos


orales y es consumidora habitual de drogas de síntesis. La gasometría muestra una
PaCO2 de 36 mmhg, una PaO2 de 87 mmhg y un pH de 7.43, con una diferencia alveolo-
arterial normal. La radiografía de tórax es normal. Esta mareada, sudorosa y con
palpitaciones; usted sospecha:

1. 1. Hiperventilación psicógena o por ansiedad.


2. 2. Tromboembolismo pulmonar.
3. 3. Neumonía.
4. 4. Intoxicación por salicilatos.
Gráfico de respuestas
Comentario

Es una pregunta de dificultad media, prototipo de una típica pregunta sobre los síndromes de
hiperventilación. En este caso clínico nos presentan a una mujer con un síndrome de
hiperventilación: taquipnea, disnea (síntoma más frecuente, por aumento del impulso respiratorio
y del esfuerzo muscular ), mareos (por alcalosis), sudoración y palpitaciones; en la gasometría,
la PaCO2 está en el límite inferior de la normalidad y el pH casi en rango de alcalosis. Las
situaciones que con más frecuencia producen hiperventilación inexplicable son el
tromboembolismo crónico o recurrente y la ansiedad; descartamos en principio la opción 3 (Rx
tórax normal, gradiente alveoloarterial de O2 normal) y las opción 4 ( no se mencionan dichos
fármacos en el enunciado). La hiperventilación por enfermedad vascular pulmonar se caracteriza
por disnea de esfuerzo y gradiente alveolo-arterial de O2 elevado; sin embargo, en la
hiperventilación psicógena la disnea es en reposo y el gradiente alveoloarterial es normal.
Opción 1 correcta.(R1)

!
!
!
!

316. Gestante primigesta de 36 años, que acude al hospital en la semana 39ª de


gestación por inicio de trabajo de parto. Después de un periodo de dilatación de 8
horas y varias exploraciones iguales sin cambios, llega a 8 cm de dilatación y la
presentación se encuentra en un II plano de Hodge. El RCTG es rigurosamente normal
y presenta buena dinámica uterina. Ante la no evolución y el cansancio materno, ¿qué
actitud adoptaría?:

1. 1. Realizar una ventosa.


2. 2. Realizar una cesárea.
3. 3. Realizar una microtoma fetal (pHmetría).
4. 4. Dejar evolucionar 6 horas más para intentar un parto por vía vaginal.
Gráfico de respuestas
Comentario
Si tras varias exploraciones iguales (2 exploraciones en aproximadamente 6 horas) una paciente
se queda estancada con una dilatación inferior a completa, es decir, no ha ido aumentando de
dilatación entre una exploración y otra, estamos ante una no progresión de parto (NPP), por lo
que habría que indicar una cesárea. Los partos instrumentales están contraindicados en este
caso porque no ha llegado a dilatación completa. No hay que realizar microtoma fetal porque en
ningún momento nos sugiere la pregunta que estemos ante una sospecha de pérdida de
bienestar fetal.(R2)

317. An otherwise healthy 16-year-old girl is brought to the emergency department


complaining of progressive lower abdominal pain for the last 4 days. The patient says
that she also has fever and a smelly vaginal discharge. Her last menstrual period was
18 days ago. She has one male partner with whom she is sexually active. She takes
contraceptive pills. Her temperature is 37.9ºC, pulse is 80/min, respirations are 22/min
and blood pressure is 115/80 mm hg. Severe bilateral lower quadrant tenderness is
noted on abdominal examination. Pelvic examination shows whitish cervical discharge,
cervical motion tenderness and bilateral adnexal tenderness. Her haemoglobin
concentration is 19.5 g/dL, leukocyte count is 14,200/mm3 and platelet count is
320,000/mm3. Pregnancy test is negative. Which of the following is the most adequate
treatment for this patient?

1. 1. Oral doxycycline and intramuscular ceftriaxone.


2. 2. Vaginal clotrimazole.
3. 3. Vaginal clindamycin.
4. 4. Intravenous cephalosporin and vancomycin.
Gráfico de respuestas
Comentario
Oral doxycycline and intramuscular ceftriaxone. Pelvic inflammatory disease (PID) is an infection
of the female reproductive organs and one of the most serious complications of a sexually
transmitted disease in women. A typical antibiotic therapy prescribed to patients with this
condition is doxycycline and ceftriaxone, since they are effective against Chlamydia and
Gonorrhea infections, respectively.(R1)

318. Masculino de 70 años de edad, que acude a urgencias con un cuadro de dolor
abdominal difuso, náuseas y vómitos. En la anamnesis nos cuenta que desde hace

!
!
!
!

medio año nota un cambio de ritmo intestinal. En la exploración se observa un


abdomen distendido, sin visceromegalias y sin signos de rebote. En la radiografía se
aprecia dilatación del colon. ¿En qué cuadro clínico deberíamos pensar?:

1. 1. Colitis isquémica.
2. 2. Cáncer de sigma.
3. 3. Cáncer de intestino delgado.
4. 4. Vólvulo de ciego.
Gráfico de respuestas
Comentario

La presencia de distensión abdominal junto con cambios del hábito intestinal sugieren
fuertemente un problema obstructivo. Los hallazgos radiológicos también respaldan esta
posibilidad. Por lo tanto, de las opciones que nos ofrecen, debemos elegir entre las que
produzcan algún tipo de obstrucción, que son la 2, 3 y 4. Entre ellas, es fácil salir de dudas. Si
se tratase de un cáncer de intestino delgado o de un vólvulo de ciego, no habría distensión del
colon. La obstrucción debería ser distal al mismo para que pudiera existir esta distensión, por lo
que la respuesta correcta es la 2.(R2)

319. Paciente de 6 años consulta por fiebre de hasta 40ºC, dolor abdominal y dolor
faringeo. Usted aprecia en la exploración física, enantema en paladar con adenopatía
cervical anterior dolorosa y exudado en amígdalas. A las 16 horas consulta por
exantema papuloso ("rasposo") que afecta a axilas y a ingles. Además, presenta
eritema en mejillas y frente, con zona perioral blanquecina. Señale lo FALSO:

El cultivo faríngeo es el método de elección para el diagnóstico rápido de la patología que


1. 1.
usted sospecha.
2. 2. Sería frecuente encontrar elevación de ASLO.
3. 3. En este proceso se produce descamación laminar en dedos y furfurácea en tronco.
4. 4. El periodo de incubación oscila entre 1-7 días.
Gráfico de respuestas
Comentario

Pregunta de dificultad media sobre la escarlatina. En el enunciado, nos presentan un caso con
todos los datos característicos de esta enfermedad: fiebre elevada, adenopatía cervical anterior
dolorosa (no la confundamos con las retrorauriculares y occipitales de la rubéola), el exantema
rasposo y la característica facies de Filatov. La opción falsa es la 1, ya que hasta un 20% de la
pobación es portadora asintomática delS. pyogenes en la faringe y por lo tanto su aislamiento, si
bien es sugerente, no es lo que CONFIRMA la enfermedad, además de que el cultivo NO es una
técnica de diagnóstico rápido. Este es clínico, no microbiológico.(R1)

320. Retomando el tema de las funciones placentarias, centrándonos en la denominada


función de barrera, término tal vez un tanto inapropiado, ya que son pocas las
sustancias que no atraviesan la membrana placentaria de forma mensurable (aunque
algunas de las que la atraviesan no alcanzan concentraciones suficientes para afectar
al feto o embrión) ¿Cuál de las siguientes sustancias NO atraviesa la placenta?

1. 1. Insulina.
2. 2. Proteínas complejas.
3. 3. TRH.

!
!
!
!

4. 4. Virus.
Gráfico de respuestas
Comentario

Pregunta nada fácil sobre una de las funciones de la placenta como es la transferencia
placentaria. Las vitaminas hidrosolubles atraviesan la barrera placentaria por transporte activo
con consumo de energía. Los virus, las proteínas complejas y la TRH atraviesan la placenta por
pinocitosis. Es la insulina (opción 1) la que no traspasa la barrera placentaria. Recuerde que la
heparina tampoco atraviesa la placenta.(R1)

321. Acude a Urgencias


un paciente de 66 años por haber iniciado 24 horas antes un cuadro de dolor
abdominal, fiebre, ausencia de evacuaciones y dolor a la palpación de fosa ilíaca
izquierda. La radiografía simple de abdomen muestra lo que aparece en la imagen nº
##. El estudio de laboratorio inicial revela únicamente una leucocitosis con desviación
a la izquierda de 17,200/mm3. De las siguientes opciones, ¿cuál NO considera
adecuada?

1. 1. El desarrollo de rigidez abdominal difusa justificaría un abordaje quirúrgico urgente.


2. 2. La realización de una TC abdominal puede ser de utilidad diagnóstica.
La colonoscopia nos puede permitir la confirmación diagnóstica y la detección de
3. 3.
complicaciones para plantear el tratamiento más adecuado.
La dieta absoluta, el aporte de líquidos y cobertura antibiótica constituyen el esquema
4. 4.
terapéutico inicial.
Gráfico de respuestas
Comentario

!
!
!
!

Nos plantean un caso tipo de diverticulitis aguda. La radiografía no suele mostrar alteraciones
definitivas para el diagnóstico, como sucede en esta imagen. Recuerde que, en la apendicitis
aguda, tampoco se suelen encontrar alteraciones importantes en la radiografía simple de
abdomen.

Lo más importante sobre esta patología es recordar el riesgo de perforación que genera el
enema opaco y la colonoscopia, por lo que deben evitarse durante la fase aguda (opción 3
incorrecta). La TC será la técnica de elección en la fase aguda. En cuanto al tratamiento del
paciente, el planteamiento inicial será conservador, con reposo intestinal y líquidos + antibióticos
intravenosos. La sospecha de peritonitis justificaría la laparotomía urgente. Por último, en casos
recurrentes se debe valorar la cirugía de forma electiva.(R3)

322. En el síndrome de Goodpasture, es FALSO:

1. 1. La inhalación de hidrocarburos volátiles produce un cuadro similar.


El nivel de Acs circulantes antimembrana basal se correlaciona bien con la gravedad de la
2. 2.
clínica renal y pulmonar.
3. 3. Más del 50% de los pacientes son HLA-DR2 positivos.
4. 4. A menudo se detectan depósitos de C3 en la biopsia renal.
Gráfico de respuestas
Comentario

Esta pregunta es puramente teórica y su dificultad es media. El síndrome de Goodpasture o


síndrome hemorrágico reno-pulmonar es una entidad que se caracteriza por la asociación de
hemoptisis y hematuria y cuya principal causa es la Enfermedad de Goodpasture producida por
la presencia de anticuerpos anti-membrana basal glomerular. Con respecto a la etiología de la
enfermedad, las opciones 1 y 3 son correctas (aunque no está clara la relación entre la
inhalación de hidrocarburos y la nefritis). La opción 4 también es correcta, añadiendo también un
depósito lineal de inmunoglobulinas. Finalmente, la opción 2 es FALSA porque el nivel de
anticuerpos circulantes es útil para diagnosticar los periodos de actividad pero NO sirve para
evaluar la gravedad de la clínica desarrollada.(R2)

323. A 17-year-old boy who was diagnosed with Crohn disease 2 years ago presents to
his physician with a 5-day history of tender red lumps on both shins. He also says that
he has worsening diarrhea and abdominal pain. Current medications include
azathioprine and over-the-counter analgesics. Which of the following is the most
appropriate next step in the management of this patient?

1. 1. Bedrest, wet dressings and antidepressant drugs.


2. 2. Biopsy of healthy skin and directly prescribe opioid analgesics.
3. 3. Suspect the presence of intestinal cancer as the cause of the skin lesions.
4. 4. Optimize the treatment of the inflammatory bowel disease.
Gráfico de respuestas
Comentario
Optimize the treatment of the inflammatory bowel disease. This patient presents with erythema
nodosum, which is a disorder that can appear as an extraintestinal manifestation of inflammatory
bowel diseases (IBD) such as Crohn disease and/or ulcerative colitis. Erythema nodosum usually
parallel IBD flare-ups, which means that if the inflammatory activity of the disease is controlled,

!
!
!
!

the manifestation will also improve. Other "parallel disorders" are: peripheral arthritis, episcleritis,
aphthous stomatitis, erythema nodosum, and pyoderma gangrenosum.(R4)

324. Realiza usted una


esofagoscopia a un niño de 11 años que ha presentado 2 episodios de impactación
esofágica con alimentos, refiere disfagia desde hace 1 año, presenta antecedentes
personales de asma alérgico y dermatitis atópica, a la exploración física no se aprecia
nada relevante. En la esofagoscopia se observa la siguiente imagen con una mucosa
esofágica atigrada. ¿Cuál de los siguientes procesos sospecharía?

1. 1. Esofagitis cáustica.
2. 2. Esofagitis eosinofílica.
3. 3. Enfermedad por reflujo gastroesofágico.
4. 4. Esófago de Barrett.
Gráfico de respuestas
Comentario

La enfermedad que padece este niño es una esofagitis eosinofílica. Esta entidad se relaciona
con procesos alérgicos a otros niveles, su prevalencia está aumentando durante los últimos
años y el principal diagnóstico diferencial es el reflujo gastroesofágico. Por eso nos dicen que
padece asma alérgico y dermatitis atópica.

El tratamiento se realiza con corticoides, y es típica la imagen endoscópica con “traquealización”


del esófago (se vuelve blanquecino y su imagen endoscópica recuerda la textura del interior de
la tráquea).(R2)

325. Respecto a la patología que usted ha sospechado en la pregunta anterior, la toma


de biopsias es fundamental ya que le aportará información respecto a:

Pronóstico: si encontramos menos de 20 eosinófilos por campo de gran aumento es de


1. 1.
esperar que se resuelva espontáneamente en unas semanas.
Tratamiento: si encontramos infiltrado eosinófilo en la mucosa habrá que tratar con
2. 2.
inhibidores de la bomba de protones.

!
!
!
!

Diagnóstico: si encontramos más de 20 eosinófilos por campo de gran aumento podemos


3. 3.
confirmar el diagnóstico.
Diagnóstico: si encontramos menos de 20 eosinófilos por campo de gran aumento
4. 4.
podemos confirmar el diagnóstico.
Gráfico de respuestas
Comentario

El término de “esofagitis eosinofílica” debería haberle hecho sospechar que el diagnóstico


definitivo será histológico. De lo contrario, difícilmente podrá ver los eosinófilos que justifican el
nombre de la enfermedad. Por ello, la respuesta correcta es la 3.(R3)

326. Una mujer de 45 años, con cirrosis grado A de Child por una hepatitis autoinmune,
presentó un episodio de hemorragia digestiva. En la endoscopia únicamente se
observaron varices esofágicas con estigmas de hemostasia reciente sin sangrado
activo, por lo que se realizó ligadura con bandas. Tras la realización de esta
terapéutica, ¿cuál de las siguientes afirmaciones es CIERTA?

1. 1. Se debe realizar además escleroterapia.


2. 2. No se recomienda repetir la ligadura con bandas.
3. 3. La tasa de mortalidad es superior a la que se da tras la aplicación de escleroterapia.
El porcentaje de recidiva hemorrágica con este tratamiento es igual o inferior al que se
4. 4.
produce tras escleroterapia.
Gráfico de respuestas
Comentario

El tratamiento endoscópico de las varices esofágicas se puede realizar mediante ligadura con
bandas o con escleroterapia. La ligadura con bandas tiene mejores resultados que la
escleroterapia, pero es más laboriosa de realizar y es difícil cuando existe sangrado activo. La
tasa de mortalidad no es mayor, ni se precisa realizar escleroterapia. Se puede repetir la
ligadura si es necesario.(R4)

!
!
!
!

327. Lactante de 5 meses que acude a urgencias con


clínica de fiebre e irritabilidad desde hace 48 horas. Los padres además refieren que
durante las últimas 24 horas rechaza la alimentación. A la exploración el niño se
encuentra bien hidratado y perfundido, con una exploración normal. Se solicita un
examen general de orina donde se observa piuria y bacteriuria. ¿Cuál sería la actitud
adecuada?

1. 1. Ingreso y antibioticoterapia intravenosa con ampicilina y gentamicina.


2. 2. Ingreso con antibioticoterapia intravenosa con gentamicina.
3. 3. Tratamiento ambulatorio con cefixima v.o durante 14 días.
4. 4. Tratamiento ambulatorio con ciprofloxacino v.o durante 7 días.
Gráfico de respuestas
Comentario

La infección urinaria en el lactante es un cuadro que típicamente puede aparecer con


sintomatología muy variada.

Ante un lactante con un cuadro de fiebre sin foco claro con irritabilidad, vómitos o rechazo de la
alimentación, debemos sospecharla y solicitar un EGO.

Ante una IVU febril de un neonato se debe emplear doble antibioticoterapia con ampicilina y
gentamicina iv durante 5 días para después completar el tratamiento vía oral hasta completar 14
días.

Si el niño es mayor de 3 meses, no será necesaria la antibioticoterapia intravenosa, salvo que


presente mal estado general, criterios de sepsis o deshidratación, o signos de insuficiencia
renal.

!
!
!
!

Si requiere de ingreso hospitalario se iniciará gentamicina IV, mientras que si no cuenta con
criterios de iresgo se podrá utilizar tratamiento ambulatorio con cefixima VO durante 10 a 14
días.(R3)

328. El paciente de la pregunta anterior es derivado a consultas de nefrología donde


se realiza un ultrasonido renal y una CUMS (cistoureterografía miccional seriada) que
se puede ver en la imagen adjunta. ¿Qué se observa en la imagen?

1. 1. Sistema excretor normal.


2. 2. Reflujo vesicoureteral grado II.
3. 3. Reflujo vesicoureteral grado IV.
4. 4. Reflujo vesicoureteral grado V.
Gráfico de respuestas
Comentario

En la imagen se observa que el contraste llega a los cálices sin dilatarlos, por lo que nos
encontramos ante un reflujo vesicoureteral grado II.

Ante la presencia de una IVU febril en lactantes, se debe realizar una evaluación posterior para
valorar la presencia de alteraciones renales que expliquen la infección.

Se realiza en primer lugar una ecografía renal y posteriormente la realización de una CUMS.(R2)

329. Señale lo INCORRECTO en cuanto a los genitales en la niñez:

1. 1. El pene puede parecer pequeño si está rodeado de grasa en los obesos.


2. 2. Si el escroto parece grande se sospecha en hernia o hidrocele.
3. 3. El hidrocele se debe operar antes de los 2 años de edad.
4. 4. La galactorrea en la edad neonatal se presenta con relativa frecuencia.
Gráfico de respuestas
Comentario

Normalmente el hidrocele congénito sólo se vigila si se ha descartado patologia importante. Y se


suelen operar cuando el paciente tiene molestias. Respuesta 3 incorrecta.(R3)

330. A 42-year-old male comes to emergency department with intense anal pain for the
last 24 hours. He denies any bleeding. He has suffered from chronic constipation for
many years. His vital signs are heart rate 98/min, respiratory rate 17/min, blood
pressure 135/90 mmHg and temperature 36.5ºC. Laboratory workup reveals RBC 4
mill./mm3, WBC 5,500/mm3 and platelets 290,000/mm3. Digital rectal examination is not
possible because of the intense pain that it produces but anal inspection gives the
diagnosis. Which of the following is the most likely diagnosis?

1. 1. Thrombosed external hemorrhoid


2. 2. Prolapsed internal hemorrhoid
3. 3. Anal fissure
4. 4. Pilonidal cyst
Gráfico de respuestas
Comentario

!
!
!
!

Pregunta sencilla sobre hemorroides. La respuesta correcta es la 1 una hemorroide externa


trombosada.(R1)

331. La displasia broncopulmonar es el resultado de :

1. 1. Daño inflamatorio pulmonar en un feto en desarrollo.


2. 2. Falla en el desarrollo de las arteriolas pulmonares en el recién nacido.
3. 3. Una infección viral intrauterina.
El uso de oxígeno y presión positiva durante el tratamiento de síndrome de dificultad
4. 4.
respiratoria del recién nacido.
Gráfico de respuestas
Comentario

La displasia broncopulmonar aparece generalmente posterior al uso de oxígeno y presión


positiva y es una complicación que se observa en RN.(R4)

332. El CD23 es un marcador de superficie celular que corresponde a:

1. 1. Receptor de baja afinidad de la IgE.


2. 2. Receptor de la Fc de la IgG.
3. 3. Receptor de C3b.
4. 4. Receptor de las rosetas T.
Gráfico de respuestas
Comentario

Pregunta muy difícil y a la vez poco útil para el ENARM. El estudio de los diferentes CDs no es
nada útil por lo que debería centrarse sólo en el estudio de aquellos que le son útiles para
identificar las diferentes poblaciones celulares o que poseen alguna importancia inmunológica.
En este caso el CD23 no reúne estas características, así que no le importe. El CD23 es una
glicoproteína de 45Kda que actúa como receptor de baja afinidad para IgE. Se expresa en los
linfocitos B activados por la IL-4 y también en los macrófagos activados y una población
importante de células dendríticas. Clínicamente se ha demostrado su utilidad en la
diferenciación de los linfomas linfocítico de célula pequeña y el de células del manto.(R1)

333. Niña de 4 años asintomática con antecedentes de ingreso neonatal durante dos
meses por prematuridad. Presenta un buen estado general y de desarrollo
ponderoestatural, tiene pulsos arteriales aumentados y se le asculta un soplo continuo
en región subclavicular izquierda. ¿Cuál es, de los siguientes, el diagnóstico más
probable?

1. 1. Comunicación interventricular.
2. 2. Tetralogía de Fallot.
3. 3. Conducto arterioso persistente.
4. 4. Coartación de aorta.
Gráfico de respuestas

!
!
!
!

Comentario

Pregunta fácil. Tema de cardiopatías congénitas.

Se trata de un conducto arterioso persistente, cardiopatía congénita debida a la persistencia de


comunicación entre aorta y arteria pulmonar que se cierra en las primera horas de vida, pero
que en niños prematuros (como es el caso) puede permanecer abierto durante períodos
prolongados de tiempo. En la clínica destaca un soplo continuo (en maquinaria o de Gibson) en
foco pulmonar que puede irradiarse hacia región infraclavicular izquierda, los pulsos arteriales
suelen ser normales o estar aumentados, y el desarrollo pondoestatural suele ser adecuado a la
edad.

En cambio, en la coartación de aorta (diagnóstico diferencial a tener en cuenta en este caso) se


ausculta un soplo mesosistóico interescapular, suele haber un mayor desarrollo de tórax y
extremidades superiores, y los pulsos arteriales están disminuidos en los miembros inferiores.
En la siguiente figura encontrará una imagen de esta patología.(R3)

Ductus arterioso persistente

334. Síntomas o signo más frecuente en el adenocarcinoma de endometrio.

1. 1. Aumento de tamaño del útero.


2. 2. Sangrado post menopáusico.
3. 3. Flujo vaginal.
4. 4. Ninguna de las anteriores.
Gráfico de respuestas
Comentario

En mujeres postmenopáusicas en las que se sospeche de adenocarcinoma de endometrio el


signo clínico más frecuente es el sangrado transvaginal post-menopáusico, respuesta 2
correcta.(R2)

!
!
!
!

335. ¿Cuál de las siguientes medidas terapéuticas es INCORRECTA en el tratamiento


de la cardiomiopatía hipertrófica?:

1. 1. Marcapasos bicameral definitivo.


2. 2. Beta-bloqueantes.
3. 3. Diuréticos.
4. 4. Miectomía del tabique interventricular.
Gráfico de respuestas
Comentario

Debe recordar que esta enfermedad produce fundamentalmente una disfunción diastólica. Por
eso, los fármacos inotrópico positivos, como la digoxina, están contraindicados, ya que
aumentan la contractilidad del VI y producen un deterioro hemodinámico con un incremento de
la sintomatología. También deben evitarse los antagonistas del calcio dihidropiridínicos (como el
nifedipino) o los vasodilatadores (como la hidralacina). Los diuréticos deben usarse con mucho
cuidado para no disminuir la precarga. Los fármacos que sí están indicados son los que
disminuyen la contractilidad miocárdica, como los betabloqueantes o los antagonistas del calcio
no dihidropiridínicos (verapamil y diltiacem), y algunos antiarrítmicos, como la amiodarona y la
disopiramida. De hecho, el tratamiento más usado son los betabloqueantes, que mejoran la
sintomatología y el gradiente ventrículo- aórtico (preguntado en dos ocasiones). La cirugía
(miomectomía y/o sustitución valvular mitral) puede estar indicada cuando hay síntomas
importantes y cuando hay un gradiente significativo a pesar del tratamiento médico. Por último,
la implantación de un marcapasos bicameral (DDD) es una posibilidad terapéutica, con la cual
se puede producir una mejoría hemodinámica y sintomática.(R3)

336. Si un paciente que ingresa por hemorragia subaracnoidea patente en la TC


cerebral y con grado 3 de Hunt y Hess, pasa a grado 4 en el quinto día de evolución,
pensará sobre todo que la causa principal del empeoramiento clínico ha sido:

1. 1. Hidrocefalia.
2. 2. Diabetes insípida.
3. 3. Resangrado.
4. 4. Vasoespasmo.
Gráfico de respuestas
Comentario

De las complicaciones de la hemorragia subaracnoidea (HSA), las neurológicas son las más
importantes. Para saber frente a cuál de ellas nos enfrentamos, hay que tener en cuenta la
clínica y el tiempo de evolución desde el sangrado.

La hidrocefalia se desarrolla de forma aguda en las primeras 24-48 hs y producirá síntomas de


hipertensión intracraneal. El resangrado puede producirse en las primeras 24-48 horas o a la
semana, y la clínica suele ser la misma que en el episodio inicial. El vasoespasmo es la causa
principal de morbimortalidad en las HSA. La clínica se desarrolla en horas o en días,
produciendo un empeoramiento del estado neurológico debido a la isquemia de los territorios
afectados. Se produce entre los días 4º y 12º después del sangrado. Recuerdae que se produce
en más del 75% de los pacientes, pero se manifiesta clínicamente en sólo un 30%. El fármaco
empleado como profilaxis es el nimodipino, un antagonista del calcio.(R4)

!
!
!
!

337. A 9-year-old child with a history of recurrent respiratory infections and three otitis
media episodes during the past two months is brought to the emergency department
presenting with fever and sleepiness. Neurological examination shows a mydriatic
right pupile with no response to light and left hemiparesia. What is the most likely
diagnosis?

1. 1. Brain abscess.
2. 2. Acute encephalitis.
3. 3. Acute meningitis.
4. 4. Toxoplasmosis.
Gráfico de respuestas
Comentario
Brain abscess. The child has a history of recurrent otitis so a complication of these infections
must be considered as a cause of the current situation. Fever and neurological defects are
suggestive of a brain abscess formation.(R1)

338. Usted pensaría en una etiología tuberculosa de un cuadro de meningitis, en uno


de los siguientes casos:

LCR con 1000 células, con 70% de linfocitos, glucosa de 20 mg/dl y proteinorraquia de
1. 1.
600 mg/dl.
LCR con 780 células, con 80% de PMN, glucosa de 19 mg/dl y proteinorraquia de 350
2. 2.
mg/dl.
LCR con 800 células, con 85% de linfocitos, glucosa de 70 mg/dl y proteinorraquia de 40
3. 3.
mg/dl.
LCR con 10000 células, con 65% de linfocitos, glucosa de 55 mg/dl y proteinorraquia de
4. 4.
35 mg/dl.
Gráfico de respuestas
Comentario

Pregunta fácil si se domina el cuadro de características del LCR que resulta verdaderamente
útil. Hay que saber que la tuberculosis es una de las excepciones de las bacterias, ya que cursa
con predominio de linfocitos en LCR (excepto en la fase inicial). Por tanto sólo con ese dato ya
podemos descartar la opción 2. Sin embargo cumple la otra característica de las bacterias y es
que se alimenta de la glucosa, así que podemos eliminar también las opciones 3 y 4. Nos queda
la opción 1, ya que además también es característica la presencia de una proteinorraquia entre
100-800 mg/dL.(R1)

339. A 5-month-old infant presents with a high-pitched cry (similar to a baby cat) and
marked microcephaly. A chromosomic defect is diagnosed. What abnormality is finally
diagnosed?

1. 1. Deletion of Xq.
2. 2. Deletion of 5p.
3. 3. 21 trysomy.
4. 4. Deletion of 23p.
Gráfico de respuestas
Comentario

!
!
!
!

Deletion of 5p. The Cri du chat syndrome is a chromosomal disorder caused by a type of
autosomal terminal or interstitial deletion of the short arm of chromosome 5, characterized by a
cry that resembles a cat's meow.(R2)

340. ¿Qué afirmación es FALSA sobre la neumonía por Mycoplasma pneumoniae?:

1. 1. El período de incubación suele durar 2-3 semanas.


2. 2. Presenta un pico de incidencia entre 3 y 4 años.
3. 3. La afectación más frecuente es en los lóbulos inferiores.
4. 4. Un título de crioaglutininas mayor de 1/64 apoya el diagnóstico.
Gráfico de respuestas
Comentario

Mycoplasma pneumoniae es la causa más frecuente de neumonía atípica. Su incidencia es


mayor en adolescentes y adultos jóvenes, no en niños. Es una bacteria difícil de cultivar, por lo
que habitualmente se diagnostica por serología. Un fenómeno muy típico de esta bacteria es
que se acompañe de múltiples fenómenos relacionados con la autoinmunidad: crioaglutininas,
artritis, anemia hemolítica, eritema multiforme, miopericarditis, miringitis bullosa,
meningoencefalitis… La eritromicina es un excelente antibiótico para su tratamiento.(R2)

341. A 31-year-old man is brought to the emergency department with fever, chills and
malaise for the past 4 days. He also complains of vomiting and headaches. The patient
denies a history of sexually transmitted diseases but does use IV drugs occasionally.
Physical examination reveals painful, raised and red lesions on his hands and feet. A
holosystolic murmur is noted on auscultation. Which of the following is the most
appropriate next step in management?

1. 1. Echocardiogram.
2. 2. Blood cultures.
3. 3. Chest MRI.
4. 4. Cardiac catheterization.
Gráfico de respuestas
Comentario
Echocardiogram. Infective endocarditis must be suspected in this case. Therefore, the most
appropriate test would be an echocardiogram.(R1)

342. A 6-year-old child diagnosed with sickle cell anemia is brought to the pediatrician's
office presenting with recurrent episodes of high intensity pain located on her ankle,
chest, ribs and back. She has been admitted to the hospital five times for osteomyelitis
and severe infections. Which of the following would reduce the intensity of these
painful episodes?

1. 1. Imatinib.
2. 2. Folic acid supplements.
3. 3. Periodic bloodletting.
4. 4. Hydroxyurea.
Gráfico de respuestas
Comentario

!
!
!
!

Hydroxyurea. A feared complication of sickle cell anemia is a vaso occlusive crisis. When this
occurs, there is no treatment appart from adequate analgesia, even with opioids if they are
required. Long term treatment with hydroxyurea has also proven useful. It increases the
proportion of fetal hemoglobin vs abnormal S hemoglobin, thus preventing crises and reducing
intensity when they do occur.(R4)

343. Un RNPT de 30 semanas comienza a los 15 días de vida con decaimiento, restos
gástricos, distensión abdominal y determinación de sangre en heces positiva. En la Rx
simple de abdomen se observa edema de asas y neumatosis intestinal. Todo lo
siguiente es correcto desde un punto de vista terapeútico, SALVO:

1. 1. Iniciar tratamiento con clindamicina más gentamicina.


2. 2. Repetir las placas de abdomen en un plazo menor de 24 horas.
3. 3. Abrir y resecar el segmento intestinal afecto.
4. 4. Reiniciar la alimentación con prudencia las 72 h de la normalización radiológica.
Gráfico de respuestas
Comentario

La enterocolitis necrotizante tiene una clínica muy típica (neonato con aspecto de gravedad que
presenta distensión abdominal llamativa y evacuaciones hemorrágicas) y una radiología con
signos muy propios (edema de asas, neumatosis, miga de pan, gas portal). Su tratamiento
depende de la presencia de neuomperitoneo (perforación de víscera hueca); si lo hay, se
empleará cirugía. En caso contrario, se instaurará reposo digestivo, nutrición parenteral y
antibioterapia que cubra gram negativos y anaerobios.(R3)

344. A previously healthy 44-year-old-woman comes to the physician because of


menorrhagia. Pelvic examination shows a firm, nontender, irregular enlarged uterus.
Diagnosis is confirmed by an ultrasound that showed three submucous myomas. A
complete blood count is remarkable for ferropenic anemia. Which of the following
would be the best treatment in this particular case?

1. 1. Surgery.
2. 2. GnRH analogs.
3. 3. First treat ferropenic anemia and then surveillance of myomas.
4. 4. Total hysterectomy after treating ferropenic anemia.
Gráfico de respuestas
Comentario

La paciente del caso clínico presenta una patología orgánica (miomas uterinos) con repercusión
clínica (menorragias o sangrado cíclico abundante) y laboratorio (anemia ferropénica). Requiere,
por tanto, un tratamiento causal además de sintomático. Los análogos de la GnRH sólo los
usamos antes de la cirugía, pues una vez interrumpido el tratamiento, los miomas regresan a su
tamaño anterior. Se nos presenta además una paciente de cierta edad, a la que se puede
realizar una cirugía más completa que evite las menorragias, como la histerectomía, dejando la
miomectomía para pacientes más jóvenes con deseos genésicos no cumplidos. Por último,
habría que añadir el tratamiento necesario para remontar la anemia ferropénica, por lo que la
opción correcta es la 4.(R4)

!
!
!
!

345. Un neonato de 21 días presenta ictericia, hepatomegalia y acolia. En los exámenes


de laboratorio destaca una cifra de bilirrubina total de 19 mg/dl con un 50% de
bilirrubina directa, así como GGT de 500 UI/L, con el resto normal. La gammagrafía
hepática muestra captación normal, pero no hay excreción biliar. ¿Qué diagnóstico se
corresponde con este cuadro?

1. 1. Hepatitis neonatal idiopática.


2. 2. Atresia biliar extrahepática.
3. 3. Enfermedad de Byler.
4. 4. Déficit de alfa-1-antitripsina.
Gráfico de respuestas
Comentario

Dada la captación normal del contraste por parte del hígado, sin que éste pase a la vía biliar,
debemos considerar una posible atresia biliar extrahepática. Recuerde que esta enfermedad es
la causa más frecuente de trasplante hepático en la infancia.(R2)

346. Paciente
tercigesta, con antecedente de dos partos eutócicos en las semanas 35 y 34,
respectivamente, acude en la semana 31 a urgencias del hospital por percibir
contracciones. Exploración: cérvix permeable 1 dedo, borrado 50%, consistencia
media, posición media. Feto longitudinal podálica. Se realiza RCTG que se muestra en
la imagen. Longitud cervical por USGtransvaginal 20 mm. Test de la fibronectina
positivo. ¿Cuál sería la actitud más correcta en este caso?:

1. 1. Mantener a la paciente en observación y reevaluar en 2 horas.


2. 2. Iniciar tocólisis con betamiméticos y maduración pulmonar con betametasona.
3. 3. Iniciar tocólisis con atosibán y maduración pulmonar con betametasona.
4. 4. Ya se ha iniciado el parto; realizar una cesárea, ya que la presentación es podálica.
Gráfico de respuestas
Comentario

A pesar de que la dinámica uterina es de parto, el cérvix todavía no lo es. Teniendo en cuenta
que el embarazo todavía no ha llegado a su semana 34, debemos tratar de frenar el parto. A
partir de la semana 34, no sería necesario utilizar corticoides para la maduración pulmonar fetal,

!
!
!
!

pero antes de este momento sí lo es, como en este caso. Por último, entre los betamiméticos y
el atosibán, en la actualidad se prefiere este último.

No olvide que el atosibán es un inhibidor competitivo de la oxitocina, de ahí su carácter


tocolítico. Es tan eficaz como la ritodrina (betamimético clásicamente utilizado), pero sus efectos
adversos son menores, por lo que resulta preferible.(R3)

347. La desconjugación del movimiento de los ojos junto a la abolición de los reflejos
oculocefálicos y oculovestibulares indica lesión de:

1. 1. Fascículo longitudinal medial.


2. 2. Núcleo intersticial rostral del fascículo longitudinal medial.
3. 3. Núcleo de VI par.
4. 4. Centros corticales que controlan los movimientos oculares horizontales.
Gráfico de respuestas
Comentario

No se preocupe si ha fallado esta pregunta porque el tema de coma y muerte encefálico no es


del todo útil para el ENARM. Una pregunta que hacia referencia a los reflejos oculocefálicos y
óculovestibulares. La abolición del reflejo oculocefálico indica que existe una lesión estructural a
nivel troncoencefálico (pontomesencefálico). Recuerde que el núcleo del sexto par craneal se
encuentra en la protuberancia y que de él sale el fascículo longitudinal medial, cuya integridad
es necesaria para permitir la mirada conjugada en el plano horizontal. Por lo tanto, una lesión a
nivel protuberancial puede lesionar el FLM ocasionando la desconjugación de la mirada y la
abolición de los refelejos oculocefálicos y oculovestibulares.(R1)

348. A newborn is being evaluated at the delivery room. The mother had a complicated
pregnancy and she did not follow scheduled controls. The child begins with
progressive respiratory distress half an hour after delivery. Lung auscultation shows
hypoventilation of the left hemithorax with a shift of the cardiac sounds to the right.
The child is suspected to be affected by a congenital diaphragmatic hernia. What
should be done as a first measure?

1. 1. Start fluid reposition.


2. 2. Echocardiography.
3. 3. Orogastric tube placement.
4. 4. Chest X ray.
Gráfico de respuestas
Comentario
Orogastric tube placement. The occupation of the chest by the abdominal contents in congenital
diaphragmatic hernia is a vital emergency. The first step to take to ensure adequate ventilation of
the neonate is proceed to place a nasogastric tube. The nasogastric tube will prevent the
distention of gastrointestinal tract by air occupation and thus greater compression of the healthy
lung. Chest radiography and echocardiography are useful but not essential as a first step to
take.(R3)

349. Masculino de 57 años, alcohólico crónico e indigente que es traído por el servicio
de urgencias por encontrarlo tirado en la calle inconsciente. El paciente está
desconectado del medio, sin ningún tipo de respuesta a estímulos,

!
!
!
!

hemodinamicamente estable. Al realizarle un control de glucemia capilar se encuentra


un valor de 25 mg/dl. ¿Cuál es la causa de la hipoglucemia?:

1. 1. Estimulación de la célula beta inducida por el alcohol.


Inhibición de la gluconeogénesis por el alcohol en un paciente sin reservas hepáticas de
2. 2.
glucógeno por el ayuno prolongado.
3. 3. Hipoglucemia cetósica inducida por el alcohol.
La hepatopatía es la causa fundamental, pero el alcohol no juega ningún papel
4. 4.
directamente.
Gráfico de respuestas
Comentario

Los pacientes alcohólicos crónicos tienen mayor riesgo de hipoglucemia por dos motivos: el
etanol bloquea la gluconeogénesis pero no la glucogenolisis y además suelen ser pacientes que
llevan varios días consumiendo alcohol sin otro tipo de alimento, por lo que su reserva de
glucógeno hepático están agotadas. Las hipoglucemias en estos pacientes pueden ser severas
y mortales. Recuerde que los pacientes alcohólicos pueden presentar cuadros de acidosis
metabólica anión gap positivo con cetonuria como consecuencia del ayuno (diagnóstico
diferencial con la cetoacidosis diabética por los niveles de glucosa).(R2)

350. Masculino de 29 años que acude a Urgencias por presentar pérdida de fuerza en
la mano derecha de instauración brusca y caídas frecuentes sin clara lateralización en
los últimos 6 meses. En la exploración de los movimientos oculares se aprecia
limitación de la adducción del ojo derecho con nistagmus en la abducción del ojo
izquierdo. La alteración de la motilidad ocular que presenta el paciente se denomina:

1. 1. Oftalmoplejía internuclear.
2. 2. Nistagmus de retracción-convergencia.
3. 3. Nistagmus pendular.
4. 4. Oftalmoplejía externa progresiva.
Gráfico de respuestas
Comentario
A pesar de que nunca se ha preguntado nada a cerca la oftalmoplejia internuclear en el examen,
esta pregunta resulta útil para repasar este concepto. Este fenómeno consiste en la parálisis de
la aducción de un ojo con nistagmus del ojo abducente como consecuencia de la lesión del
fascículo longitudinal medial (interneuronas originadas en el núcleo del VI par craneal
protuberancial, que cruzan la línia media para hacer sinapsis con el núcleo del III y VI par
contralateral., permitiendo así la mirada conjugada). El ojo que no aduce (el ojo paralizado) es el
que define la oftalmoplejia internuclear. Las causas más frecuentes de dicha lesión son la
esclerosis múltiple (en jóvenes), los infartos cerebrales ( en ancianos), los tumores y los
traumatismos.(R1)

351. A 7-year-old boy is brought to her pediatrician by her parents, complaining of a 3-


week history of intermittent pruritic rash on his arms. His past medical history includes
a bronchiolitis a year ago treated with albuterol. The rash is itchy, located on his ante-
cubital fossae and erythematous. Which of the following is the most adequate
treatment?

1. 1. Biospy first and then medical therapy accordingly.


2. 2. Topical corticosteroid therapy.

!
!
!
!

3. 3. Oral corticosteroid therapy.


4. 4. Topical antibiotic therapy.
Gráfico de respuestas
Comentario

Se trata de un cuadro de dermatitis atópica, por lo que el tratamiento inicial es con


cortcoesteroides tópicos. Respuesta 2 correcta.(R2)

352. El bloqueo auriculoventricular completo puede encontrarse asociado a todas


EXCEPTO una de las siguientes situaciones:

1. 1. Madre con lupus eritematoso sistémico.


2. 2. Madre con síndrome de Sjögren.
3. 3. L-transposición de los grandes vasos.
4. 4. Ingesta materna de digoxina.
Gráfico de respuestas
Comentario

El bloqueo auriculoventricular completo (BAVC) en niños es una entidad poco frecuente. Puede
ser adquirido (cirugía o miocarditis) o congénito. La incidencia de BAVC quirúrgico es baja y se
produce por trauma sobre el nódulo auriculoventricular o sobre el haz de His, habitualmente
durante la corrección de comunicaciones interventriculares, tetralogía de Fallot o defectos del
canal AV. El BAVC congénito (BAVCC) puede ser secundario a un desarrollo embrionario del
nodo AV anormal o a la destrucción del sistema de conducción normal. El BAVCC puede
producirse en asociación con anormalidades del colágeno de la madre. Por otro lado, hay una
clara asociación entre los anticuerpos SSA/Ro y SSA/La en la madre y el desarrollo de BAVCC
en el niño. Por último, el BAVCC puede ser visto en corazones absolutamente normales y en
pacientes con cardiopatías congénitas, siendo la L-transposición de los grandes vasos el defecto
anatómico más frecuente.(R4)

353. ¿Cuál de las siguientes manifestaciones de la fibrosis quística y su tratamiento


NO es apropiada?:

1. 1. Colonización por Pseudomonas aeruginosa tipo mucoide: antibióticos nebulizados.


2. 2. Esteatorrea: suplementos de enzimas pancreáticas, aportes de vitaminas liposolubles.
3. 3. Íleo meconial: enemas.
4. 4. Hemoptisis: vitamina K, segmentectomía.
Gráfico de respuestas
Comentario

La fibrosis quística del páncreas es una enfermedad multisistémica, de elevada morbilidad,


siendo las complicaciones derivadas de las bronquiectasias las que dictan la mortalidad de esta
enfermedad. Entre ellas tenemos la hemoptisis, agravada generalmente por la depleción de
vitamina K por la malabsorción intestinal.

Recuerde que el tratamiento ideal es la embolización de la arteria bronquial sangrante (opción 4


falsa), suspender la fisioterapia, ofrecer cobertura antibiótica antipseudomona y suplementar la
vitamina K. El resto de opciones son correctas, entre ellas no olvide la asociación fibrosis
quística-íleo meconial.(R4)

!
!
!
!

354. Masculino de 77 años, con una calidad de vida previa adecuada a su edad, es
traído a Urgencias por su familia con un cuadro de 24 horas de evolución de confusión
mental, disartria, alucinaciones olfativas y fiebre, sin focalidad infecciosa aparente.
¿Usted cree que lo prioritario en este caso es?:

1. 1. Tomar hemocultivos.
2. 2. Punción lumbar.
3. 3. Aciclovir intravenoso.
4. 4. Antibióticos de amplio espectro.
Gráfico de respuestas
Comentario
Pregunta difícil sobre un caso clínico, pero que hay que saber reconocer porque es muy típico y
preguntado. Estamos ante un cuadro de fiebre sin foco aparente pero con síntomas
neurológicos, por lo que deberíamos pensar que el foco infeccioso se encuentra a ese nivel. La
clave para resolver la pregunta es saber que las alucinaciones olfativas son características de la
focalidad del lóbulo temporal y ésta, a su vez, típica de las encefalitis herpéticas. Por lo tanto, lo
más importante en este caso sería la administración empírica y precoz de aciclovir iv, que ha
demostrado reducir la mortalidad en esta patología. Por supuesto que el resto de opciones
también se aplicarían, pero lo más importante en este caso, ante la sospecha etiológica es el
tratamiento.(R3)

355. ¿Cuál de las siguientes afirmaciones es cierta respecto al tratamiento con


lovastatina?

1. 1. Actúa inhibiendo la absorción de las sales biliares.


Inhibe la HMG-CoA reductasa, que es la enzima limitante en la síntesis de los triglicéridos
2. 2.
endógenos.
3. 3. Su utilización junto con gemfibrozilo previene la aparición de miopatía.
Disminuye la producción de LDL y aumenta la depuración hepática de LDL al aumentar el
4. 4.
número de receptores.
Gráfico de respuestas
Comentario

Debe conocer el tratamiento farmacológico de la dislipemia.

La lovastatina es una estatina. Es un inhibidor de la HMG- CoA reductasa, la enzima limitante


para la síntesis de colesterol endógeno. Esta disminución de síntesis estimula el aumento de
receptores de LDL, que favorece la eliminación plasmática de colesterol.

Los principales efectos secundarios son molestias abdominales, aumento de las transaminasas
y rabdomiolisis (sobre todo en combinación con fibratos).(R4)

356. ¿Cuál es la causa más frecuente de taquicardia ventricular?:

1. 1. Displasia arritmogénica de ventrículo derecho.


2. 2. Miocardiopatía dilatada.
3. 3. Infarto de miocardio crónico.
4. 4. Intoxicación digitálica.

!
!
!
!

Gráfico de respuestas
Comentario

Las taquicardias ventriculares monomórficas suelen estar producidas por reentrada y la


cardiopatía predisponerte más frecuente en la cicatriz ventricular post-infarto. También pueden
aparecer en otras cardiopatías como la miocardiopatía dilatada o la hipertrófica (recuerde que
esta es la causa más frecuente de muerte súbita de causa cardiaca en pacientes jóvenes. Las
taquicardias ventriculares de la displasia arritmogénica del ventrículo derecho también son por
reentrada mientras que las de la intoxicación digitálica suelen ser por actividad desencadenada
(postpotenciales). Las taquicardias ventriculares se dividen en sostenidas (cuando duran más de
treinta segundos o producen inestabilidad hemodinámica) o no sostenidas (cuando duran menos
de 30 segundos y son bien toleradas).(R3)

357. Masculino de 59 años, alcohólico crónico, ingresa en la UCI tras sufrir un IAM.
¿Cuál de las siguientes medidas NO considera apropiado aplicar a este paciente?:

1. 1. Acido acetilsalicílico.
2. 2. Benzodiacepinas para prevenir el síndrome de abstinencia.
3. 3. Disulfiram mientras esté ingresado.
4. 4. Beta-bloqueantes.
Gráfico de respuestas
Comentario

Esta pregunta es muy sencilla. Nos dan un caso clínico de un alcohólico con un IAM ingresado y
nos preguntan lo que no hay que darle. Claramente, en principio, el paciente no consumirá
alcohol en la UCI y lo que nos interesa es que no haga una deprivación que nos complicaría el
IAM , por lo que habría que administrarle benzodiacepinas (opción 2). La deshabituación con
disulfiram se planteará al alta como coadyuvante en el tratamiento de deshabituación. Por lo que
la medida No apropiada para el caso es la opción 3, el disulfiram. Como alcohólico deberemos
administrar complejo vitamínico B para evitar las secuelas neurológicas. De cardiología
sabemos que un IAM está indicado el ácido acetilsalicílico y los betabloqueantes.(R3)

358. ¿Cuál de los siguientes métodos es más útil para el diagnóstico precoz de
osteomielitis?:

1. 1. Gammagrafía con Ga67.


2. 2. Gammagrafía con I125.
3. 3. Gammagrafía con leucocitos marcados con In111.
4. 4. Gammagrafía con IgG policlonal marcada con I125.
Gráfico de respuestas
Comentario

La gammagrafía ósea con Tecnecio 99 es una prueba con una gran sensibilidad, ya que es
positiva en el 95% de los casos dentro de las primeras 24 horas del inicio de los síntomas de
una osteomielitis. Sin embargo, dada su baja especificidad, siempre que la prueba sea positiva y
exista una fuerte sospecha de infección, debe completarse el estudio con una gammagrafía con
leucocitos marcados con In111, que presenta una sensibilidad del 80% y una especificidad
superior al 95%.(R3)

!
!
!
!

359. En relación a patología anal, lo siguiente es verdadero. EXCEPTO:

1. 1. Los linfáticos del conducto anal drenan en los ganglios inguinales.


2. 2. La arteria hemorroidal inferior es rama terminal de la mesentérica inferior.
3. 3. Las hemorroides de II grado pueden manejarse con ligaduras.
4. 4. Ninguna de las anteriores.
Gráfico de respuestas
Comentario

Pregunta muy complicada. La respuesta erronea es la 2.

La arteria hemorroidal inferior o rectal inferior, proviene de la pudenda interna y esta última de la
iliaca interna.(R2)

360. Señalar (V) o (F) en lo siguiente: ( ) El cuello uterino está constituido por
abundantes fibras musculares. ( ) Las glándulas endometriales obstruidas forman 10x5
Huevos de Naboth. ( ) El engrosamiento de la base del ligamento ancho forma el
ligamento cardinal o de Mackenrodt. ( ) La arteria uterina cruza por detrás del uréter a
nivel de la base del ligamento ancho. ( ) La relación del cuerpo uterino con el cuello
cervical, en nulíparas jóvenes, es de mitad y mitad.

1. 1. VVFFV.
2. 2. FFVFV.
3. 3. FVVFV.
4. 4. VVFVF.
Gráfico de respuestas
Comentario

El cuello uterino está formado sobre todo por tejido conjuntivo con escasas fibras musculares; a
diferencia del cuerpo uterino, que posee abundante tejido muscular formando el miometrio.

Los huevos o quistes de Naboth derivan de glándulas endocervicales, no endometriales.

La base del ligamento ancho se llama, en efecto, ligamento cardinal. Están a ambós lados del
útero, sujetándolo.

La arteria uterina cruza por delante del uréter.

En efecto, cuerpo y cuello del útero miden lo mismo en nulíparas; en mujeres que ya han tenido
gestaciones previas, el cuerpo se queda un poco más grande.(R2)

361. Señale la FALSA sobre la artritis reactiva:

1. 1. Suele haber úlceras bucales muy dolorosas.


2. 2. El 60-85% son HLA B27 positivo.
3. 3. En pacientes con SIDA puede haber una forma grave de artritis reactiva.
4. 4. Una colitis por Shigella flexneri puede desencadenar un brote de artritis reactiva.
Gráfico de respuestas

!
!
!
!

Comentario

La artritis reactiva se puede producir tras una infección gastrointestinal (Shigella, Salmonela,
Yersinia, Campilobacter) y en este caso no muestra predominio sexual o tras una infección
genitourinaria (Chlamydia) siendo entonces más frecuente en varones. La mayoria de los
pacientes son HLA B27. La artritis reactiva es la cusa más frecuente de artritis en pacientes con
infección por VIH. Entre las manifestaciones extrarticulares se encuentran las úlceras orales que
debemos buscar ya que al ser indoloras el paciente puede no haberlas percibido.(R1)

362. A 30-year-old woman presents with malaise, asthenia and skin lesions on her left
forearm and hand. She denies any chills, nausea or vomiting. Vital signs are within
normal limits, except for her body temperature, which is 38.2ºC. Her skin lesions are
mostly papulopustular, purplish and well demarcated. Some microabscesses are seen
surrounding the purple rash. Which of the following is the best initial test in this
patient?

1. 1. Wet preparation of purulent material from skin lesions.


2. 2. Skin biopsy.
3. 3. Bacterial and viral cultures of purulent material from skin lesions.
4. 4. Bacterial and fungal cultures of purulent material from skin lesions.
Gráfico de respuestas
Comentario

Pregunta un tanto complicada, pero tendría que notar que solicitan la prueba inicial que se
realizaría, por lo que la opción correcta sería la 1, con una preparación humeda del material
purulento de las lesiones cutáneas.(R1)

363. Con respecto a la aparición de fiebre (temperatura superior a 38 ºC) en el


postoperatorio, y suponiendo que no haya ninguna causa preoperatoria que justifique
la hipertermia, señale cuál de las siguientes afirmaciones NO es cierta:

1. 1. La aparición de fiebre en el postoperatorio es muy frecuente.


La fiebre por infección de la herida quirúrgica suele aparecer entre el 7º y el 10º días del
2. 2.
postoperatorio.
Las reacciones febriles entre las 24 y 72 horas postoperatorias generalmente se deben a
3. 3.
infecciones urinarias.
Un absceso intraabdominal o una fuga anastomótica suelen causar fiebre entre el 4º y el 7º
4. 4.
días del postoperatorio.
Gráfico de respuestas
Comentario

Una pregunta que puede tener dos respuestas ya que existen dos opciones falsas:

- R2: La causa más frecuente de infección de herida quirúrgica son los estafilococos, y esto
suele ocurrir hacia el 4º- 5º día.

- R3: Las infecciones urinarias producirían fiebre a partir de las 72 horas.

!
!
!
!

Este tipo de preguntas las contesta pero nunca sabe cual se dio como correcta y esto no debe
generar ansiedad.(R3)

364. Mujer de 63 años e índice de masa corporal > 30, con menopausia hace 11 años,
que acude a consulta por sangrado vaginal de 6 días de evolución. En la exploración
se detecta una metrorragia "en agua de lavar carne", con un cérvix sin lesiones
macroscópicas. Se realiza un USG transvaginal donde se observa un endometrio
engrosado sin lesiones sólidas en su interior. El siguiente paso que realizaría sería:

1. 1. Tranquilizar a la paciente y nuevo control a los 6 meses.


2. 2. Histeroscopía diagnóstica y biopsia.
3. 3. PET.
4. 4. Legrado endocervical.
Gráfico de respuestas
Comentario
La paciente que nos presentan en el caso presenta una metrorragia “en agua de lavar carne”,
característica del cáncer de cérvix y endometrio. En la ecografía nos describen un endometrio
engrosado que en la menopausia debería estar atrófico, por lo que hay que descartar la
posibilidad que nos encontremos ante un carcinoma de endometrio. Hoy en día, la prueba de
elección en el diagnóstico de la patología endometrial es la histeroscopia y posterior biopsia
endometrial, quedando relegado el legrado fraccionado de la cavidad endometrial. El resto de
las pruebas se deberían pedir como estudio de extensión en el caso que se confirmara el
carcinoma de endometrio.(R2)

365. A 4-year-old child is brought to the emergency department presenting with pain
and bulging of the right knee. He is afebrile. Articular examination shows a restriction
of movement with signs of intraarticular fluid. The mother says that the child has
frequent bruises and nosebleeds. What is the most likely diagnosis?

1. 1. Haemophilia.
2. 2. Acute arthritis.
3. 3. Ruptured cruciate ligaments.
4. 4. Osteonecrosis of the knee.
Gráfico de respuestas
Comentario
Haemophilia. The disorder described is a knee hemarthrosis. Joint haemorrhages are rare. They
can occur in case of high-energy trauma or coagulative disorders. In this case, the history of
nosebleeds favours haemophilia.(R1)

366. Nos llega a la consulta un niño de 7 años diagnosticado de crisis de ausencias


típicas. ¿Qué dato clínico de los abajo expuestos NO esperaría encontrar en el
paciente?

1. 1. Confusión poscrítica.
2. 2. Descargas generalizadas punta-onda a 3 Hz en el electroencefalograma durante la crisis.
3. 3. Pérdida completa de la conciencia de segundos de duración.
4. 4. Con maniobras de hiperventilación podemos provocar las crisis del niño.
Gráfico de respuestas

!
!
!
!

Comentario

Las crisis de pequeño mal son bastante conocidas, por lo que no se trata de una pregunta difícil.

Las crisis de ausencia típicas se producen muchas veces al día, duran unos segundos y, sobre
todo, no existe confusión posterior, a diferencia de otros tipos de epilepsia. El resto de las
respuestas son claramente ciertas.

Sobre este tipo de crisis debe recordar en especial los siguientes detalles:

•! Patrón típico en el electroencefalograma: descargas generalizadas punta-onda a 3 Hz.


•! Algunas maniobras, como la hiperventilación, pueden desencadenar las crisis.
•! Existen dos tratamientos posibles. El más típico es la etosuximida, antiepiléptico que
sólo es eficaz en este tipo de crisis. No obstante, recuerde que también se podría
emplear el ácido valproico.

(R1)

367. Si va a realizar una mediastinoscopia para explorar la afectación ganglionar


paratraqueal y subcarínica en un paciente con un carcinoma epidermoide de pulmón
debe saber que la complicación de esta técnica más frecuente entre las siguientes es:

1. 1. Mediastinitis.
2. 2. Hemorragia.
3. 3. Parálisis del recurrente.
4. 4. Fístula bronquial.
Gráfico de respuestas
Comentario

Pregunta de dificultad elevada, sobre las complicaciones de la mediastinoscopia,una de las


técnicas quirúrgicas de estadificación. En la mediastinoscopia se introduce el mediastinoscopio
a través de una incisión en yugulum hastala región paratraqueal,explorándose las cadenas
paratraqueales y subcarínicas. Las complicaciones más graves son la hemorragia y la
mediastinitis, pero las más frecuentes son la parálisis del nervio recurrente izquierdo y la
infección de la herida quirúrgica. Por lo tanto la opción corecta es la 3.(R3)

368. ¿Cuál es la complicación extraintestinal más típica en hombres con colitis


ulcerosa?

1. 1. Pielonefritis.
2. 2. Colangitis esclerosante.
3. 3. Artropatía periférica.
4. 4. Anemia hemolítica.
Gráfico de respuestas
Comentario

Aunque en una colitis ulcerosa podríamos padecer artritis periférica, y de hecho es más
frecuente que la colangitis esclerosante, nos están preguntando la complicación más TÍPICA en

!
!
!
!

HOMBRES. No confunda, por tanto, lo típico con lo frecuente. La colangitis esclerosante


primaria debemos sospecharla en todo paciente con colitis ulcerosa que comienza con un
cuadro de colestasis. Habitualmente, se trata de un varón de unos 40 años.(R2)

369. En la decisión terapéutica de un paciente neoplásico es fundamental determinar


el estadio tumoral. En los últimos años se ha extendido la práctica de biopsia del
denominado “ganglio centinela” para conocer la existencia de diseminación linfática
en un tumor determinado. El concepto de ganglio centinela es:

El primer ganglio linfático aumentado de tamaño en un territorio que drena un tumor


1. 1.
primario.
El primer ganglio linfático con metástasis que aparece en un paciente con un carcinoma
2. 2.
primario.
El primer ganglio en un territorio linfático que recibe el flujo linfático de un tumor
3. 3.
primario.
4. 4. Se refiere a todos los ganglios del territorio axilar en pacientes con carcinoma de mama.
Gráfico de respuestas
Comentario

Pregunta de dificultad baja, puesto que la técnica del ganglio centinela es un recurso novedoso
para localizar el primer ganglio de drenaje linfático de un determinado tumor primario mediante
una técnica de medicina nuclear.

Es una técnica que suele valorar la necesidad de tratamiento sistémico en una determinada
neoplasia. En el Manual CTO, podrá ver que se realiza en el cáncer de mama y en el
melanoma.(R3)

370. Señale la respuesta CORRECTA en relación a las complicaciones de la


enfermedad diverticular del colon:

1. 1. La diverticulitis es más frecuente en el colon izquierdo que en el derecho.


2. 2. En la fase aguda de la diverticulitis, el método diagnóstico de elección es el enema opaco.
3. 3. La hemorragia diverticular es más frecuente a nivel de divertículos del sigma.
4. 4. En la mayoría de los casos de hemorragia diverticular se necesita cirugía.
Gráfico de respuestas
Comentario

La diverticulitis, también llamada "apendicitis izquierda", se debe a la inflamación de un


divertículo (más frecuentes también ellos en colon descendente). Su diagnóstico es clínico (dolor
abdominal agudo) y laboratorio (leucocitosis con neutrofilia); no estaría indicada la realización de
colonoscopia o enema opaco durante el episodio de diverticulitis, por el riesgo de perforación.
Los divertículos son la primera causa de hemorragia digestiva baja (sangran más los divertículos
de colon derecho), que se suelen resolver con tratamiento médico conservador.(R1)

371. Femenino de 25 años de edad, con pareja estable desde hace 2 años, con
antecedentes ginecoosbtétricos: G1-A1-P0, un USG reciente con resultado de quistes
en ovario derecho, antecedentes personales de alergia a betalactámicos y a
sulfamidas, antecedentes familiares de cáncer de endometrio, consulta por el método
anticonceptivo más adecuado para ella. Se le recomienda utilización de
anticonceptivos orales combinados (estrógenos + progesterona). De las siguientes

!
!
!
!

opciones, todas son contraindicaciones absolutas para el uso de anticonceptivos


orales, EXCEPTO una. Señálela:

1. 1. Dismenorrea.
2. 2. Historia de tromboembolismo previo.
3. 3. Antecedentes personales de cáncer de mama.
4. 4. Antecedentes de colestasis durante un embarazo.
Gráfico de respuestas
Comentario

Pregunta sencilla sobre los anticonceptivos orales. Realmente está clara la indicación en casos
de dismenorrea, ya que pueden controlar y regulan el ciclo menstrual de la paciente. En el resto
de casos es clara la contraindicación. Recuerde especialmente la relación de los ACO con el
tromboembolismo, ya que este es un aspecto importante.(R1)

372. ¿Cuál de los siguientes hechos NO corresponde a la silicosis?:

1. 1. Coalescencia de nódulos.
2. 2. Calcificaciones en cáscara de huevo.
3. 3. Se complica frecuentemente con neoplasias bronquiales.
4. 4. Se asocia frecuentemente a la tuberculosis.
Gráfico de respuestas
Comentario
En la actualidad aún no se conoce si existe relación entre la silicosis y el cáncer de pulmón. Lo
que sí es más frecuente en esta enfermedad es la tuberculosis (silicotuberculosis) y las
infecciones por micobacterias atípicas. En paciente silicóticos con Mantoux positivo está
indicado realizar quimioprofilaxis. Radiológicamente afecta a lóbulos superiores en forma de
nódulos múltiples bilaterales que tienden a coalescer formando grandes conglomerados. Las
calcificaciones en cáscara de huevo son características de la silicosis simple o crónica.(R3)

373. Pre-escolar de 3 años, presenta dolor abdominal intenso desde hace 1 día. La
madre refiere tos, fiebre y malestar general desde hace 4 dias. Al examen: llenado
capilar adecuado, soplo tubario en base de hemitorax derecho, distensión abdominal
marcada sin signos peritoneales. El diagnostico más probable es:

1. 1. Neumonía basal derecha con ileo secundario.


2. 2. Neumonía basal derecha con peritonitis secundaria.
3. 3. Neumonía basal derecha con pioneumotórax.
4. 4. Neumonía basal derecha y choque séptico mas ileo.
Gráfico de respuestas
Comentario

Pregunta sencilla. Detecte los datos importantes: tos, fiebre, con soplo tubario en hemitórax
derecho, lo que hace que sospeche una neumonía basal derecha y no tiene signos de irritación
peritoneal, la respuesta correcta es la número 1.(R1)

374. ¿Cuál sería el primer cambio hormonal de la premenopausia? Señale la respuesta


CORRECTA:

!
!
!
!

1. 1. FSH y LH disminuidas.
2. 2. FSH y LH aumentadas.
3. 3. FSH aumentada y LH normal.
4. 4. FSH aumentada y LH disminuida.
Gráfico de respuestas
Comentario

Pregunta fácil, contestada claramente en el Manual.

Se denomina climaterio a la etapa en la que se pasa de la edad fértil a la edad no fértil y dura
años. La menopausia es un sólo día, el día de la última regla.

El hecho fundamental del climaterio es el descenso de los niveles de estrógenos, lo que


condiciona la mayoría de los síntomas que aparecen:

•! Neurovegetativos: sofocos, sudoración, palpitaciones, etc.


•! Psicológicos.
•! Genitourinarios y cutáneos: atrofia cutánea y genitourinaria.
•! Sistémicos: aterosclerosis, HTA, osteoporosis, etc.

El cambio endocrino más precoz es la elevación de FSH, seguramente en respuesta al


descenso de inhibina como consecuencia del escaso número folicular. LH suele estar normal en
un primer momento y, posteriormente, puede mantenerse normal o aumentar. Durante el
climaterio el principal estrógeno pasa a ser la estrona, como consecuencia de la conversión
periférica de los andrógenos.

En cuanto al tratamiento, la terapia hormonal sustitutiva (THS) queda reservada para las
pacientes con síndrome climatérico intenso y para el tratamiento de la atrofia genitourinaria. Es
necesario reseñar que la THS no ha demostrado, como se creía antiguamente, protección
contra los eventos cardiovasculares (pudiendo incluso ser perjudicial), y no ha de ser utilizada
como tratamiento de la osteoporosis del climaterio.(R3)

375. Las convulsiones febriles se presentan con mayor frecuencia:

1. 1. En el recién nacido.
2. 2. A los tres meses de edad.
3. 3. De 1 a 5 años de edad.
4. 4. A los 6 años de edad.
Gráfico de respuestas
Comentario

Entre las opciones dadas, la mayor frecuencia está entre el primero y quinto y dentro de estos
aumenta de 1 a los 2 años de vida. Respuesta 3 correcta.(R3)

376. Ante la sospecha de una osteomielitis aguda en un lactante en el que la radiografía


ha sido normal, ¿cuál sería nuestro siguiente paso diagnóstico?:

1. 1. Esperar un tiempo y repetir el estudio radiológico.


2. 2. Resonancia nuclear magnética.
3. 3. Gammagrafía ósea.

!
!
!
!

4. 4. Tomografía axial computerizada.


Gráfico de respuestas
Comentario

El diagnóstico temprano de la osteomielitis es fundamental, ya que se debe iniciar el tratamiento


antibiótico de forma precoz para evitar una necrosis. Se debe pedir un estudio radiológico simple
inicial en todo paciente para descartar otras causas, aunque lo normal es que no se encuentra
ningún hallazgo en los primeros días. La gammagrafía es probablemente la técnica de elección
por su elevada sensibilidad, pese a su escasa especificidad (detecta alteraciones en un 95% de
los casos en las primeras 24 horas tras la aparición de los síntomas).Al utilizar en la gamagrafía
en vez de Tc99 leucocitos marcados con In99 aumenta mucho la especificidad. La ecografía
estará indicada para descartar o confirmar la existencia de abscesos subperiósticos en niños. La
RNM tiene una elevada sensibilidad (similar a la gammagrafía) y mayor especificidad que ésta,
además de una resolución anatómica muy superior, por lo que se convierte en técnica de
elección ante una osteomielitis vertebral para valorar además la presencia de abscesos
epidurales. La RNM o el TAC deben reservarse ante la existencia de una osteomielitis vertebral
o en otros casos cuando el diagnóstico no se puede confirmar por otros métodos, ya que estas
pruebas son menos disponibles y más caras. Además, en lactantes, el evitar radiaciones y el
realizar pruebas sencillas que no precisen sedación es importante.(R3)

377. Mujer de 68 años consulta por presentar una lesión de rápido crecimiento en el
dorso de mano izquierda, asintomatica. Comenzó hace 3 semanas y, actualmente,
presenta una lesión tuberosa, firme, de coloración rojiza de 1 cm de diametro, bien
definida, con fondo infiltrado y superficie queratósica que, al ser retirada, presenta un
cráter central. ¿Cúal es el diagnóstico más probable?:

1. 1. Carcinoma espinocelular.
2. 2. Queratosis actínica hipertrófica.
3. 3. Queratoacantoma.
4. 4. Carcinoma basocelular queratinizante.
Gráfico de respuestas
Comentario

El queratoacantoma es un tumor benigno muy frecuente que aparece en adultos en zonas


fotoexpuestas (cara, manos). Es una lesión de crecimiento muy rápido, morfología peculiar e
histología de carcinoma epidermoide bien diferenciado, aunque hoy día se discute si en realidad
es una variedad más de carcinoma epidermoide. Clínicamente, cursa como una tumoración que
alcanza un gran tamaño en pocas semanas. Su morfología es diagnóstica, pues se trata de una
formación nodular cupuliforme, simétrica, con un cráter córneo central. Su comportamiento suele
ser benigno, con tendencia a la resolución espontánea, aunque lo más frecuente es tratarlo,
para evitar una mayor destrucción del tejido subyacente.(R3)

378. ¿Cuál de las siguientes hormonas tiene mayor similitud con la HCG?:

1. 1. FSH.
2. 2. LH.
3. 3. TSH.
4. 4. PRL.
Gráfico de respuestas
Comentario

!
!
!
!

Pregunta difícil sobre las hormonas femeninas. Podemos tener dudas son entre la FSH, LH y
TSH, que comparten la misma subunidad alfa. La diferencia estará en la subunidad beta.

La única que presenta cierta similitud en cuanto a la subunidad beta es la LH (opción 2


correcta).

Aunque desconozca la opción correcta, se puede llegar a la respuesta correcta detectando las
similitudes funcionales de la LH y HCG. Recuerde que la LH es la hormona de la fase lútea, y la
HCG es luteotropa, ya que es la encargada de mantener el cuerpo lúteo durante el primer
trimestre de la gestación.

A la inversa, recuerde que en el ciclo ovárico justo antes de la ovulación ocurre un pico de LH.

En los ciclos de estimulación ovárica utilizados en la reproducción asistida, precisamente


empleamos LH subcutánea. De esta manera.

En cualquier caso, no se preocupe si la ha fallado, la dificultad de esta pregunta es elevada.(R2)

379. En relación a la masa ósea corporal y menopausia. Marque lo CORRECTO:

1. 1. Se pierde 5% de masa ósea anual en los tres primeros años.


2. 2. Se pierde 1-2% de masa ósea anual en los tres primeros años.
3. 3. Se pierde 50% de hueso compacto hasta los 65 años.
4. 4. Se pierde 3% de masa ósea anual en los cinco primeros años.
Gráfico de respuestas
Comentario

Esta pregunta es sumamente teórica, y no se preocupe si la ha fallado. La respuesta correcta es


la 4. Se pierde 3% de masa ósea anual en los primeros cinco años.(R4)

380. ¿Cuál de las siguientes afirmaciones es FALSA, en relación con la meningitis


neonatal?:

1. 1. El periodo neonatal es la época de la vida con mayor incidencia de meningitis.


2. 2. La meningitis ocurre al menos en el 30% de la sepsis neonatales.
3. 3. Las meningitis causadas por Haemophilus son relativamente frecuentes en el neonato.
4. 4. Tienen peor pronóstico las meningitis por microorganismos Gram negativos.
Gráfico de respuestas
Comentario

Entre los microorganismos más frecuentes como causales de la meningitis neonatal se


encuentran los Gram negativos o Listeria entre otros, pero no el Haemophilus.(R3)

381. En la consulta de control de salud observamos las habilidades de un niño. La


mamá nos dice que el niño sabe decir “mamá”, “papá”, “agua”, “nene” y “popo”.
Comprobamos cómo sabe señalar las partes del cuerpo cuando su mamá le pregunta
dónde están. La mamá nos cuenta que la obedece de inmediato cuando le dice que se
siente para comer y que con señas se hace entender casi siempre, pero que no se

!
!
!
!

enfada si no logra hacerse comprender. ¿Cuál de las siguientes edades sería la más
compatible con el grado de desarrollo del niño?

1. 1. 12 meses.
2. 2. 15 meses.
3. 3. 18 meses.
4. 4. 24 meses.
Gráfico de respuestas
Comentario

Pregunta sobre el desarrollo psicomotor del niño. En Pediatría son típicas estas preguntas, son
bastante difíciles y específicas, ya que los periodos que nos proponen son muy concretos, y
existen diversos hitos del crecimiento y desarrollo que pueden solaparse en diferentes
momentos del tiempo.

La prueba más empleada es el test de Denver, que valora los aspectos personal, social,
lenguaje y motricidad fina y gruesa en los niños desde el nacimiento hasta los 6 años.

La respuesta correcta es la opción 3. Corre, sube escaleras andando tomado de la mano,


construye una torre de cuatro cubos, se sienta en sillas pequeñas, dice 10 palabras (de media),
identifica una o más partes del cuerpo, come solo, pide ayuda en caso de necesitarla, etc.(R3)

382. ¿Qué músculo extraocular de los enunciados a continuación NO es inervado por


el III par o motor ocular común?

1. 1. Elevador párpado superior.


2. 2. Oblicuo inferior.
3. 3. Recto superior.
4. 4. Oblicuo superior.
Gráfico de respuestas
Comentario

Una pregunta muy sencilla sobre los músculos extraoculares.

El músculo oblicuo superior tiene inervación propia a través del IV par (patético). En cambio, el
recto externo es inervado por el VI par. Por su parte, el III par craneal inerva al resto de
músculos extraoculares: recto superior, recto inferior recto medio y oblicuo inferior (de ahí su
nombre, “oculomotor”). También el elevador del párpado superior recibe inervación a través del
III par.(R4)

!
!
!
!

383. Un recién nacido, a los 5 minutos de vida, presenta: F.C. 60 latidos por minuto,
cianosis acra, esfuerzo respiratorio ausente, tono muscular inexistente y ausencia de
respuesta al introducir un catéter por su nariz. La puntuación de Apgar en este niño
será:

1. 1. 4.
2. 2. 3.
3. 3. 2.
4. 4. 1.
Gráfico de respuestas
Comentario

Debe dominar el APGAR para el ENARM es muy importante. Según la escala de Apgar
mostrada arriba, la puntuación es de 2.(R3)

!
!
!
!

384. En relación al tromboembolismo pulmonar, uno de los enunciados siguientes es


FALSO:

1. 1. Se produce un aumento del espacio muerto.


Se produce una reducción del área transversal del lecho arterial pulmonar, lo que produce
2. 2.
un aumento de la resistencia al flujo.
3. 3. Con frecuencia, el infarto acompaña a la embolia pulmonar.
4. 4. La mayoría de los émbolos se resuelven con rapidez.
Gráfico de respuestas
Comentario

El TEP es una patología que preguntan con frecuencia en el ENARM y debe dominarla.
Además, es muy útil porque pueden asegurarse casi todas las preguntas con relativamente poco
esfuerzo (tres páginas del manual). Fisiopatológicamente, lo que sucede es que el émbolo
ocluye los vasos pulmonares, de forma que quedarán alveolos sin perfundir, pero bien
ventilados, con lo que el espacio muerto está aumentado La pérdida del surfactante pulmonar se
debe a la lesión de los neumocitos II, que son las células que lo producen, y se hace máxima a
las 12-15 horas del evento, lo que puede traducirse en la aparición de atelectasias. La opción
claramente falsa es la 3. El pulmón es un órgano realmente difícil de infartar, puesto que cuenta
con una doble circulación: los vasos pulmonares (procedentes de cavidades derechas) y las
arterias bronquiales. Si se ocluyen los vasos pulmonares por el émbolo, aún quedan los
bronquiales, que pueden abastecer de sangre al pulmón y evitar, dentro de unos límites, la
aparición del infarto pulmonar.(R3)

385. Ante una paciente que consulta por amenorrea primaria, con fenotipo femenino,
ausencia de vello axilar y púbico, genitales externos femeninos con vagina corta que
acaba en fondo de saco, y con los siguientes datos de laboratorio: gonadotropinas
elevadas, estrógenos elevados y testosterona muy elevada, ¿cuál de los siguientes
diagnósticos es el más probable?:

1. 1. Hiperplasia suprarrenal congénita.


2. 2. Síndrome de Turner.
3. 3. Síndrome de Rokitansky-Küster-Hauser.
4. 4. Síndrome de Morris.
Gráfico de respuestas
Comentario

Una buena oportunidad para aprender los aspectos más esenciales del síndrome de Morris, o
síndrome de feminización testicular.

En esencia, consiste en una insensibilidad total a los andrógenos. Se trata de pacientes con
cariotipo XY, pero fenotípicamente son mujeres, ya que sus receptores androgénicos están
insensibilizados. Dado que el cariotipo es XY, no existen conductos de Wolff ni van a desarrollar
las estructuras que de ellos derivan (ovarios, útero, trompas, parte superior de la vagina).

Externamente, tienen genitales externos femeninos, con escaso vello axilar y pubiano, sin
clitoromegalia. Todo esto es lógico, ya que no existe ningún tipo de sensibilidad a andrógenos.
Desde el punto de vista hormonal, la testosterona, los estrógenos y la LH estarán elevadas, por

!
!
!
!

resistencia a la acción androgénica en la hipófisis. La principal complicación de este síndrome


es la malignización de las gónadas no descendidas (que son testículos).(R4)

386. Mujer de
67 años, diabética y fumadora, que acude a Urgencias por dolor típico, prolongado,
acompañado de cortejo vegetativo y de disnea, de aparición en reposo. El dolor es
similar al que presentó en episodios anteriores. La paciente tiene historia cardiológica
de infarto sin Q en cara lateral, tratado con stent en la arteria circunfleja hace 5 meses.
En urgencias se realiza ECG que detecta lo que se muestra en la imagen nº 19. Se
obtiene una primera determinación de troponina I de 4 (normal < 0.2). Estaría indicada
una actitud intervencionista urgente en todos los siguientes casos salvo en:

1. 1. El dolor persiste pese a la medicación antianginosa.


2. 2. TA sistólica 70 mmHg.
3. 3. TVNS en el monitor.
4. 4. FEV1 55%.
Gráfico de respuestas
Comentario

La pregunta nos presenta a una mujer con factores de riesgo cardiovascular y antecedentes de
revascularización percutánea, que padece un síndrome coronario agudo sin elevación del ST
(clínica compatible + alteraciones eléctricas sugerentes de isquemia subepicárdica en cara
anterior por presentar T negativas en cara anterior). Por tener los marcadores de necrosis
positivos (troponina I elevada) podemos clasificar ya a este SCA como infarto de miocardio sin
onda Q. Ante este tipo de infartos se puede adoptar una actitud intervencionista urgente o
programada.

Estaría indicada una actitud intervencionista urgente en el caso de que el dolor no ceda con la
medicación antianginosa, cuando aparezcan datos de fallo cardíaco (como una TAS 70 mmHg
y/o un grado III en la clasificación de Killip, es decir, un edema agudo de pulmón), cuando se
monitoricen arritmias ventriculares, y si la FEV1 se encuentra gravemente afectada (menor de
35%). La respuesta: falsa es la 5 porque una FEV1 del 55% se puede considerar dentro de los
límites bajos de la normalidad.(R4)

387. Un paciente de 60 años de edad acude a la consulta por habérsele descubierto un


patrón de tipo intersticial en la radiografía de tórax. Refiere ser fumador de 40

!
!
!
!

paquetes/año y haber trabajado como montador de tuberías. En la tomografía axial


computerizada se observa el mencionado patrón y un engrosamiento pleural. El
diagnóstico en que pensará es:

1. 1. Fibrosis pulmonar.
2. 2. Bronquiolitis respiratoria.
3. 3. Asbestosis.
4. 4. Silicosis.
Gráfico de respuestas
Comentario

Las enfermedades pulmonares intersticiales han aumentado su presencia. No es el tema más


frecuente de Neumología, pero es importante. En el caso clínico que nos comentan, nos hablan
de un montador de tuberías. Esto quiere decir que estará en contacto con algún material que
tendrá relevancia para el diagnóstico... En este caso, se tratará del asbesto. Observe que,
igualmente, tiene una alteración radiológica muy típica, como el engrosamiento pleural (placas
pleurales). Las otras opciones no producirían esta alteración radiológica y no tendrían por qué
asociarse a ser montador de tuberías.(R3)

388. Paciente que presenta nódulos cutáneos, eritrodermia descamativa y placas


infiltrativas. En la exploración se detecta una esplenomegalia moderada. La biometría
hemática muestra una linfocitos de 17,000/m3 y en la punción de médula ósea se
observa una infiltración de células linfoides. Todo ello debe hacernos pensar, en
principio, que nos encontramos ante:

1. 1. Leucemia prolinfocítica.
2. 2. Leucosis linfoide crónica B.
3. 3. Leucosis linfoide crónica T.
4. 4. Síndrome de Sézary.
Gráfico de respuestas
Comentario

La micosis fungoide es un linfoma cutáneo de células T. A veces, se produce una


leucemización, en cuyo caso de habla de síndrome de Sézary. El síndrome de Sézary se define
por la tríada: eritrodermia, linfadenopatías y existencia de más del 10% de células de Sézary en
sangre periférica (linfocitos T atípicos con núcleo cerebriforme). El tratamiento de la micosis
fungoide va encaminado a prolongar la vida y mejorar la calidad de ésta, pero no a curar la
enfermedad, lo que aún no resulta posible. Aunque no es habitual que en el síndrome de Sézary
se infiltre la médula ósea por las células leucémicas, el caso es lo suficientemente típico y no
debería haberte planteado demasiados problemas.(R4)

389. The ECG of a 55-year-old patient who is currently taking some medication to treat
his chronic heart failure shows: peaked T waves, increased QRS amplitude and loss of
P wave amplitude. Which of the following statements is true?

1. 1. The most likely cause is hypokalemia.


2. 2. Spironolactone may be the cause.
3. 3. He must be taking digoxin.
4. 4. These are typical ECG signs of hypocalcemia.
Gráfico de respuestas

!
!
!
!

Comentario

Las alteraciones electrocardiográficas que nos describen, especialmente la T picuda, son


sugestivas de hiperkalemia. Por ello, la respuesta correcta es la 2, dado que la espironolactona
es antagonista de la aldosterona, por lo que podría justificarla (igual que sucede con los
IECAs).(R2)

390. Which of the following signs is not suggestive of anovulation state?

1. 1. A cycle with monophasic basal body temperature pattern.


2. 2. Endometrial proliferation one week before menstruation.
3. 3. Unbroken follicles seen on ultrasound later than 24 hours after LH peak.
4. 4. Absence of progesterone elevation one week after menstruation.
Gráfico de respuestas
Comentario

Para contestar esta pregunta debe conocer que cambios hormonales son los que producen la
ovulación.

La ovulación ocurre como consecuencia directa del pico de LH desencadenado por el efecto
gatillo del pico de estrógenos. Tras la ovulación el folículo se colapsa y se convierte en cuerpo
lúteo, que es el lugar de producción de la progesterona.

La progesterona eleva el metabolismo y la temperatura corporal. En este periodo postovulatorio


el endometrio se encuentra en fase secretora. La progesterona permanece elevada hasta la
luteolisis que ocurre al final del ciclo.

Por todo ello, al no producirse ovulación, no habría elevación de progesterona y tampoco sus
efectos consecuentes.(R4)

391. La exanginotransfusión aplicada al recién nacido afecto de eritroblastosis fetal


sirve, fundamentalmente, para:

1. 1. Disminuir la tasa de complemento circulante.


2. 2. Aportar proteínas.
3. 3. Aportar antígenos Rh sobre los que se fijan los anticuerpos.
4. 4. Controlar la hiperbilirrubinemia.
Gráfico de respuestas
Comentario

La exanguinotransfusión es una técnica que tiene como misión principal depurar de tóxicos el
caudal hemático. A aplicar esta técnica a la isoinmunización Rh, lo que hacemos es depurar la
bilirrubina de la sangre, con lo cual se reduce el riesgo de kernicterus. Para llevar a cabo esta
técnica, lo que hacemos es un recambio casi completo de la volemia del niño por sangre 0
negativo.(R4)

392. Un paciente anciano acude por anuria. En los últimos días el volumen de orina ha
descendido progresivamente. Reinterrogado refiere dolor suprapúbico desde hace
unas semanas. En la exploración física no se aprecian masas abdominales, y presenta

!
!
!
!

dolor a la palpación en columna lumbosacra. Ante este cuadro debe pensarse que el
paciente padece:

1. 1. Estenosis uretral con retención urinaria.


2. 2. Hiperplasia prostática benigna con retención urinaria.
3. 3. Carcinoma prostático con atrapamiento ureteral bilateral y metástasis óseas.
4. 4. Litiasis urinaria bilateral acompañado de cólico renal.
Gráfico de respuestas
Comentario

Situación bastante frecuente en los Servicios de Urgencia. Nos dan algunos datos que tenemos
que interpretar. Cuando nos dicen que esta en anuria y no palpan masas abdominales,
descartamos un globo vesical, es decir una uropatía obstructiva infravesical. El dolor a la
palpación sobre la columna lumbosacra nos puede orientar a lesiones metastásicas a ese nivel,
muy frecuentes en el cáncer de próstata. Este tumor en su crecimiento puede producir un
atrapamiento trigonal (ureteral bilateral) y causar uropatía obstructiva bilateral, y por tanto anuria
con vejiga vacía. El tratamiento en este caso seria la nefrostomía percutánea bilateral o la
abstención terapeútica en caso de mal estado general previo. Posteriomente deberíamos
instaurar tratamiento del tumor primario (en caso de confirmarse mediante biopsia) con un
bloqueo hormonal completo.(R3)

393. Mujer de 24 años, asintomática, que en una biometría hemática rutinaria presenta
los siguientes hallazgos: leucocitos 6.5 x 109/L con fórmula normal, eritrocitos 5.6 x
1012/L, hemoglobina 10 g/dL, VCM 70 fL, plaquetas 250 x 109/L. Las cifras de ferritina
sérica se cuentan en 150 microg/L. En la electroforesis de hemoglobinas, la A2 es de
2% y la F 0.4%. ¿Cuál es el diagnóstico más probable?:

1. 1. Anemia de enfermedad crónica.


2. 2. Rasgo talasémico beta.
3. 3. Rasga talasémico alfa.
4. 4. Drepanocitosis.
Gráfico de respuestas
Comentario

La BH, con cifra de hematíes algo elevada pero con hemoglobina baja y VCM llamativamente
disminuido, es típico de rasgo talasémico, si bien debe descartarse previamente la existencia de
ferropenia, como se ha hecho en esta paciente. Con la electroforesis de hemoglobinas, el
resultado consiste en una elevación del porcentaje de la A2 (por descenso de A1 al disminuir la
síntesis de la cadena beta de la A, mientras en el rasgo alfa se mantienen los porcentajes
normales (al disminuir la síntesis de la cadena alfa, que forma parte de las tres hemoglobinas
normales) En la drepanocitosis se demuestra hemoglobina S.(R3)

394. La rehidratación oral en una gastroenteritis aguda con deshidratación está


indicada en todos los casos siguientes, EXCEPTO:

1. 1. Deshidratación isotónica de 2º grado.


2. 2. Deshidratación hipertónica con natremia sérica de 160 mEq/l.
3. 3. Deshidratación en el recién nacido.
4. 4. Deshidratación y presencia de íleo paralítico.
Gráfico de respuestas

!
!
!
!

Comentario

Aunque las preguntas de deshidrataciones son habituales en el ENARM, no suelen ser de este
tipo, sino como un caso clínico para que reconozca el tipo. De todas formas es importante que
conozca su manejo.

La rehidratación vía oral es la de elección por ser más adecuada y fisiológica, y se suele
emplear en los casos leve-moderados, según la tolerancia oral de paciente (se puede utilizar
sonda). El déficit se debe reponer en 4-6 horas en caso de deshidrataciones hipo-isotónicas y en
12 h en hipertónicas. Posteriormente se administran líquidos y electrolitos según las pedidas que
se sigan produciendo. La vía iv se reserva para los casos graves, inestabilidad hemodinámica,
íleo paralítico o abdomen potencialmente quirúrgico, trastorno de conciencia o fracaso de
rehidratación oral.(R4)

395. Respecto a las diferentes fórmulas de nutrición enteral que hay en el mercado,
señale la afirmación FALSA:

1. 1. El criterio principal para su clasificación es la complejidad de la proteinas.


Las fórmulas específicas para la Insuficiencia Respiratoria aumentan la proporción de
2. 2.
grasas y restringen la de hidratos de carbono.
Se ha demostrado que todos los pacientes oncológicos se benefician de fórmulas
3. 3.
enriquecidas con inmunonutrientes.
4. 4. Ninguna afirmación es correcta.
Gráfico de respuestas
Comentario
En los pacientes oncológicos, la adición de inmunonutrientes (arginina, ácidos grasos omega
3,…) sólo ha demostrado ser beneficiosa en el perioperatorio, reduciendo la tasa de
complicaciones. En pacientes no quirúrgicos no han demostrado ningún beneficio, frente a otras
fórmulas estándares hiperproteicas.(R3)

396. La característica radiológica más comúnmente encontrada en RN con síndrome


de dificultad respiratoria es:

1. 1. Atelectasia lobar con edema intersticial.


2. 2. Opacidades bilaterales y pneumotórax.
3. 3. Cambios reticulogranulares difusos con broncograma aéreo.
4. 4. Cardiomegalia y edema intersticial.
Gráfico de respuestas
Comentario

La respuesta correcta son los cambios reticulogranulares difusos con broncograma aéreo (3).
Repase la siguiente tabla comparativa.(R3)

!
!
!
!

397. A propósito de la hipertensión portal:

1. 1. Para que ésta exista es necesario que el enfermo padezca una cirrosis.
2. 2. Es una complicación no habitual de la pancreatitis aguda.
Para el diagnóstico de la hipertensión portal es necesario medir la presión libre y de
3. 3.
enclavamiento.
4. 4. El estudio con bario del esófago es el mejor medio para detectar varices esofágicas.
Gráfico de respuestas
Comentario

La pancreatitis aguda puede producir una trombosis de la vena porta, con hipertensión portal
secundaria, aunque es una complicación muy infrecuente. El mejor método para detectar varices
esofágicas es la endoscopia. En la profilaxis de sangrado por varices secundarias a hipertensión
portal, se usan betabloqueantes, no betaagonistas. El gradente portocava sería el mejor método
diagnóstico, pero no es imprescindible, ya que se puede realizar el diagnóstico con signos
indirectos de hipertensión portal, como el aumento del diámetro de la vena porta, mediante
ecografía. Puede aparecer hipertensión portal sin cirrosis (por ejemplo, por una trombosis de la
porta, como decíamos antes).(R2)

398. Hombre de 82 años que consulta por presentar en los dos últimos años
numerosos episodios de disfagia al inicio de la deglución, halitosis y, en ocasiones,

!
!
!
!

regurgitación maloliente. Además, describe en varias ocasiones, y sin guardar relación


con la deglución, accesos de tos y disnea. Entre los siguientes, ¿cuál sería el
diagnóstico más probable?:

1. 1. Divertículo por tracción.


2. 2. Divertículo de Zenker.
3. 3. Acalasia.
4. 4. Esófago de Barrett.
Gráfico de respuestas
Comentario
El divertículo de Zenker se desarrolla en la región posterior de la hipofaringe en la unión con el
esófago superior. Se acompaña de disfagia tipo faríngeo por disfunción de la musculatura
cricofaríngea. Además, el acúmulo de alimento condiciona halitosis, e incluso en ocasiones
regurgitación maloliente. La acalasia, las estenosis del esófago de Barrett y la fibrosis esofágica
dan disfagia de esófago inferior, no tipo faríngeo.(R2)

399. Un hombre de 43 años, ingeniero de profesión, ha permanecido durante 2 meses


en el República Democrática del Congo. A los 8 días de su regreso, comienza con un
cuadro de fiebre, escalofríos, cefalea, mialgias, náuseas y dolor abdominal. A la
exploración: esplenomegalia y discreta hepatomegalia. En los exámenes de
laboratorio destaca: Hb 9.8 g/dl; leucocitos 3,465 mm 3, plaquetas 97,000, AST 121, ALT
119 y Na 129. Es FALSO sobre esta patología que:

1. 1. Podría haberse evitado con una vacunación correcta.


2. 2. Es una enfermedad potencialmente mortal.
3. 3. Las medidas de barrera son fundamentales en la prevención.
4. 4. La doxiciclina ha demostrado eficacia en su profilaxis.
Gráfico de respuestas
Comentario

Un paciente con fiebre y estupor, procedente de un lugar exótico, tiene un paludismo hasta que
se demuestre lo contrario. Veamos opción por opción:

•! R1: aunque existe una vacuna frente al paludismo, su eficacia es muy cuestionable, por
lo que la profilaxis es farmacológica (aparte de las medidas de barrera, como las
mosquiteras, que se mencionan en la respuesta 3).

•! R2: el paludismo puede producir la muerte del paciente, sobre todo las formas más
graves, que son las que causa Plasmodium falciparum.

•! R4: aparte de la cloroquina (formas sensibles a la misma) y la mefloquina (para formas


resistentes), existen otras alternativas profilácticas, como la doxiciclina. No obstante,
este fármaco no se utiliza mucho con este fin, dado que puede producir fotosensibilidad,
y el paludismo precisamente aparece en zonas tropicales. También puede emplearse la
azitromicina, si bien su eficacia es menor.

(R1)

!
!
!
!

400. A 45-year-old man comes to the physician for a regular checkup. Laboratory
studies show: Hb 16.5 mg/dL, MCV 80.5 fL, CHM 30 pg, leukocytes 7100/mm3 with 63%
neutrophils, platelets 199,000/mm3, glucose 102 mg/dL, urea nitrogen 23 mg/dL,
creatinine 0.7 mg/dL, total proteins 7.6 g/dL, albumin 4.5 g/Dl, GOT 47 U/L, GPT 45 U/L,
GGT 67 U/L, bilirubin 0.9 mg/dL, Alkaline phosphatase 158 U/L, LDH 190 u/L, sodium
139, potassium 3.9, serum ferritine level 654 ng/mL, IST 49%, ANA -, anti-LKM-, AMA -,
anti-Sm -. Viral serology: antiHBs +, AgHBs -, AntiHBc +, AgHBe -, antiHBe+, anti-VHC
-, AgVHD -, anti-VHD IgM -, anti-VHD IgG -, IgG VHA + . Which of the following is the
most appropriate test to confirm the diagnosis?

1. 1. Genetic testing for HLA DQ2.


2. 2. Genetic testing for HLA63D and/or C282Y.
3. 3. Liver MRI.
4. 4. Genetic testing for CFTR mutations.
Gráfico de respuestas
Comentario
Genetic testing for HLA63D and/or C282Y. Although there are 4 types of hereditary
hemochromatosis, the most common one is type I, due to mutations in gene HFE. Homozygous
C282Y or C282Y/H63D compound heterozygote mutation are the underlying genetic disorder in
up to 80% of type I hemochromatosis. Therefore, the most appropriate test would be focused on
finding such mutations. Here you have a reminder of the 4 types of hereditary hemochromatosis:
Type 1 (CLASSIC HEMOCHROMATOSIS): Mutations of the HFE gene. Type 2 (JUVENILE
HEMOCHROMATOSIS): Mutations in the HJV and HAMP genes. Type 3: Mutations in the TFR2
gene. Type 4 (ferroportin disease): Mutations in the SLC40A1 gene. TREATMENT:
PHLEBOTOMY (especially when serum ferritin levels are over 1000 ng/mL or elevated
transferrin saturation). Treatment of complications. Sometimes iron chelators.(R2)

401. Respecto a la alveolitis alérgica extrínseca, ¿cuál de las siguientes afirmaciones


es cierta?:

1. 1. Es un subtipo de asma bronquial.


2. 2. Su tratamiento de elección son los broncodilatadores inhalados.
3. 3. La radiografía de tórax es generalmente normal.
4. 4. El diagnóstico se basa en la historia clínica.
Gráfico de respuestas
Comentario

Se trata de una pregunta difícil sobre las enfermedades por inhalación de polvos. De este tema,
de cara al ENARM, hay que tener un dibujo claro de la neumonitis por hipersensibilidad, de la
silicosis y de la asbestosis. Con respecto a la alveolitis alérgica extrínseca (neumonitis por
hipersensibilidad), hay que saber que se debe a una exposición intensa y/o repetida a polvos
inorgánicos. La mayoría de agentes etiológicos derivan de exposiciones ocupacionales, pero
también puede relacionarse con los aparatos de calefacción central, humidificadores, aficiones
como la cría de palomas, etc. La reacción inmunológica más relevante es el daño tisular
mediado por inmunocomplejos (tipo III). También puede haber granulomas en la biopsia
pulmonar (tipo IV). La forma aguda presenta un patrón ventilatorio restrictivo, con pérdida
sobretodo de la CV y disminución de la DLCO y la complianza. La forma crónica asocia un

!
!
!
!

patrón restrictivo a un leve patrón obstructivo. Hay hipoxemia inducida por el esfuerzo y a veces
en reposo. Radiográficamente, las formas agudas y subagudas tienen infiltrados nodulares mal
definidos y áreas en vidrio despulido e incluso infiltrados alveolares predominante en lóbulos
inferiores. La forma crónica presenta infiltrados nodulares y lineales de predominio en lóbulos
superiores. El diagnóstico se basa en una historia clínica compatible en un paciente con
exposición a algunas de las etiologías conocidas, junto con la radiografía y la función pulmonar.
Hay que saber que en el lavado broncoalveolar típicamente hay incremento de linfocitos T, con
una relación CD4/CD8 menor a 1 en la forma crónica. Los eosinófilos están ausentes. El
tratamiento se basa en la retirada del agente etiológico , aunque los casos más graves requieren
corticoides.(R4)

402.! A 56 year-old patient comes to the physician


because of hypertension, edema and hematuria. Proteinuria and ANCA are detected.
A renal biopsy is performed and an image is shown. No deposits are seen on
immunofluorescence test. Which of the following is the most likely diagnosis?

1. 1. Post-infectious glomerulonephritis.
2. 2. Pauciimmune rapidly progressive glomerulonephritis.
3. 3. Type 2 membranoproliferative glomerulonephritis.
4. 4. Goodpasture syndrome.
Gráfico de respuestas
Comentario
En la microscopía óptica se observa proliferación extracapilar en forma de semiluna epitelial.
Aunque en la imagen del ovillo no se puede saber a qué tipo de glomerulonefritis extracapilar
corresponde en el enunciado nos informan de que el paciente tiene ANCA positivos (Compatible
con extracapilar III), ausencia de inmunocomplejos (no es una extracapilar II) y ausencia de
anticuerpos antimembrana basal glomerular (No es una extracapilar I).(R2)

403. Paciente de 47 años que acude a urgencias porque, desde hace 15 días, presenta
un sangrado que comenzó como una menstruación normal, pero que no ha cedido,
incluso llegando a sangrar más que una menstruación. Como antecedentes
ginecológicos destacan 2 partos eutócicos y un legrado por aborto diferido;
menstruaciones cada 26 días, de 5 días de duración, no dolorosas. Le solicita una
prueba de embarazo en orina, siendo negativo su resultado. En la exploración se
detecta un cérvix de multípara, con restos hemáticos en vagina en cantidad similar a
una menstruación. Le realiza un USG transvaginal en la que se observa un útero regular
en anteversión de 8 cm, con un endometrio de aspecto secretor de 13 mm sin que
parezca observarse patología. Anexo derecho normal, anejo izquierdo con formación
econegativa de 38 mm en ovario izquierdo de aspecto benigno. Respecto al cuadro
clínico que presenta la paciente, es FALSO que:

1. 1. Para cohibir la hemorragia, se debe pautar tratamiento estrogénico.

!
!
!
!

2. 2. Es recomendable el uso de AINEs y preparados con hierro oral.


3. 3. El cuadro clínico puede explicarse por ciclos disovulatorios.
Si persisten los cuadros de sangrado, se puede plantear el uso de anticonceptivos
4. 4.
hormonales, DIU de levonorgestrel o cirugía.
Gráfico de respuestas
Comentario

Este es un caso clínico muy frecuente en la práctica diaria. Tenemos una paciente en edad
premenopáusica, las cuales son más candidatas a metrorragias disfuncionales, debidas muchas
veces a ciclos anovulatorios. Nuestra paciente presenta una metrorragia que en ecografía se
observa una formación funcional ovárica que puede ser un folículo luteinizado no roto, folículo
persistente, … en tal caso, formación funcional de aspecto benigno, la cual debe vigilarse en
unos tres meses para demostrar su desaparición; esta tumoración ovárica secretora de
hormonas hace que un endometrio secretor pueda sangrar, por ello debemos pautar gestágenos
para cohibir la hemorragia. Si fuera atrófico el endometrio, deberíamos dar estrógenos. También
podemos pautar hierro para prevenir anemia y los AINEs al disminuir las prostaglandinas
disminuirán el sangrado. Para regular el ciclo se pueden usar anticonceptivos hormonales, o se
puede usar DIU de LNG para hacer que el endometrio prolifere menos y tener menos regla.(R1)

404. Se encuentran en una hernia complicada, EXCEPTO:

1. 1. Incarcelamiento.
2. 2. Reductibilidad.
3. 3. Obstrucción.
4. 4. Estrangulamiento.
Gráfico de respuestas
Comentario

Una hernia complicada puede presentar incarcelamiento, obstrucción y estrangulamiento,


excepto reductibilidad, ya que si presenta esta característica no será complicada.(R2)

405. Which is the CORRECT answer with relation to children's development.

1. 1. Cephalic perimeter at birth is usually 40 cm.


2. 2. Lambdoid fontanel usually closes between 2-4 months of age.
3. 3. The first deciduous teeth to appear are the canine teeth.
4. 4. The social smile usually appears during the third month of life.
Gráfico de respuestas
Comentario

En esta pregunta hay ciertos conceptos sobre el desarrollo de el niño que tiene importancia
recordar.

Los dientes empiezan a aparecer a los 6 meses y los primeros que lo hacen son los incisivos
centrales. Los primeros molares son los dientes permanentes que primero aparecen, en torno a
los 6 años. La sonrisa social suele aparecer entre las 4 y 8 semanas de vida. El perímetro
cefálico al nacimiento es de 35 cm, la fontanela anterior o bremática se cierra entre los 12 y 18
meses, la posterior o lamboidea a los 3 meses. La existencia de unas fontanelas anormalmente
grandes o retrasadas en su cierre puede ser debido, entre otras, a hidrocelafia hipotiroidismo
acondroplasia o rubéola congénita. Para completar el repaso de las características del cráneo

!
!
!
!

del neonato recordar que en los pretérminos es posible encontrar craneotabes parietal fisiológica
y que la persistencia de suturas a cabalgadas a la semana de vida se denomina
craneosinostosis.(R2)

406. ¿Cuál de las siguientes enfermedades presenta un período de infectividad que


dura desde el período catarral hasta 4 semanas después?:

1. 1. Rubéola.
2. 2. Sarampión.
3. 3. Tos ferina.
4. 4. Varicela.
Gráfico de respuestas
Comentario

Pregunta sobre enfermedades infecciosas pediátricas que es fácil fallar, ya que hace referencia
a datos muy memorísticos. Merece la pena recordar lo siguiente:

- En general, suele ser de alrededor de dos semanas, salvo excepciones.

- La escarlatina tiene un período muy breve: 3 días.

- En la tos ferina, es muy prolongado: hasta 6 semanas.

(R3)

407. Un paciente de 45 años consulta por dolor abdominal epigástrico. Una endoscopía
demuestra una úlcera gástrica a nivel del antro. Las biopsias descartan malignidad. Se
inicia tratamiento con 20 miligramos al día de omeprazol. A las cuatro semanas se le
hace un control radiológico, que demuestra que el lecho ulceroso persiste, aunque se
ha reducido en un 75%. Se mide la gastrina sérica, que está claramente elevada. ¿Qué
haría con este paciente?

1. 1. Cirugía.
2. 2. Cambiar tratamiento a un anti-H2.
3. 3. Le haría un TAC pancreático.
4. 4. Seguiría con el mismo tratamiento.
Gráfico de respuestas
Comentario

Algunos tumores gástricos pueden debutar como úlceras gástricas, por lo que es obligatorio, al
hacer un diagnóstico por imagen de un úlcera gástrica, tomar biopsias para descartar
malignidad. Además, las úlceras gástricas, en general, suelen ser más grandes que las
duodenales, y por ello su tiempo necesario para la cicatrización es mayor. Los fármacos que se
emplean son los antisecretores, fundamentalmente inhibidores de la bomba de protones, como
el omeprazol. El tiempo estimado de tratamiento con omeprazol necesario para la cicatrización
de una úlcera gástrica es de unas 12 semanas. En este caso, parece que el tratamiento está
siendo correcto, aunque aún habría que mantenerlo 8 semanas más. En cuanto a la gastrina, es

!
!
!
!

normal que se eleve, ya que el efecto del fármaco antisecretor disminuye la secreción ácida en
el estómago. Y esta hipoclorhidria es, precisamente, el estímulo más potente para la secreción
de gastrina.(R4)

408. Le avisan con urgencia para valorar a un recién nacido postérmino, con
antecedentes de sufrimiento fetal, que ha empeorado tras su primera toma de leche.
En la exploración está cianótico, polipneico, con Silverman de 4, abdomen excavado,
hipoventilación de pulmón izquierdo y latido cardíaco desplazado a la derecha.
Respecto a esta patología, ¿cuál es la INCORRECTA?

Debe operarse urgentemente al realizar el diagnóstico, sobre todo si la cianosis y el


1. 1.
compromiso hemodinámico son severos, sin esperar a la estabilización del recién nacido.
2. 2. Es más frecuente en el lado izquierdo.
Puede presentarse después del primer mes de vida en forma de vómitos, estreñimiento o
3. 3. dolor abdominal, e incluso detectarse de forma casual al realizar una radiografía de tórax
en los casos leves.
Puede realizarse diagnóstico prenatal por ultrasonido, debiendo diferenciarse de la
4. 4.
malformación adenomatoidea quística y del enfisema lobar congénito.
Gráfico de respuestas
Comentario

Caso clínico típico de hernia diafragmática congénita de Bochdalek y por tanto desplazamiento
del contenido abdominal hacia la caja torácica. Se manifiesta como cianosis, depresión
respiratoria postnatal, hipertensión pulmonar, cavidad abdominal excavada y desplazamiento del
latido cardíaco hacia la derecha. El diagnostico prenatal se realiza mediante ultrasonido y el
postnatal mediante Rx de tórax. El tratamiento inicial es la estabilización de la hipertensión
pulmonar (mediante hiperventilación controlada, bicarbonato, óxido nítrico o empleo de ECMO =
extracorporeal membrane oxygenation) y posteriormente proceder alcierre quirúrgico en 24- 72
horas tras la estabilización hemodinámica del neonato.(R1)

409. Embarazada en el primer trimestre de gestación (9ª semana), con antecedentes de


hijos fallecidos por una inmunodeficiencia, y cuyo defecto molecular en la familia ya
es conocido. Con el fin de obtener el diagnóstico prenatal lo más precoz posible, se
realiza una biopsia de corion. Señale la afirmación INCORRECTA respecto a este caso:

1. 1. Puede realizarse por vía transabdominal o a través del cuello uterino.


Dado que la biopsia corial es la técnica que implica más pérdidas fetales, hubiera sido
2. 2.
preferible una amniocentesis.
El resultado de las pruebas genéticas puede estar entre 48-72 horas tras la obtención de la
3. 3.
biopsia.
La funiculocentesis, a diferencia de la biopsia corial, además permite realizar maniobras
4. 4.
terapéuticas.
Gráfico de respuestas
Comentario

Además de la utilidad de cada método diagnóstico invasivo de uso prenatal, debé recordar bien
un calendario de cuándo se puede llevar a cabo cada uno: la biopsia corial es aceptable desde
la semana 8, mientras que la amniocentesis sólo se puede realizar entre las semanas 12 y 16, y
la funiculocentesis a partir de la semana 18. En este caso no hubiéramos podido preferir nunca
la amniocentesis (respuesta 2 falsa) porque estamos en semana 9.(R2)

!
!
!
!

410. A 25-year-old man, diagnosed with asthma when he was a child, goes to his
pneumologist complaining of worsening of his symptoms. He is currently being treated
with on-demand salbutamol. The patient reports two asthma attacks per week, two or
three night crisis per month and moderate physical activity limitation. Which of the
following is the most appropriate treatment for him?

1. 1. Adding a long-acting beta-2 agonist inhaler.


2. 2. Adding inhaled corticosteroids.
3. 3. Adding oral theophylline.
4. 4. Adding oral prednisone.
Gráfico de respuestas
Comentario

Adding inhaled corticosteroids, because it is not a controlled asthma.(R2)

411.! Neonato de 32 semanas de edad


gestacional, ingresado en UCIN por prematuridad, presenta a los 16 días de vida,
distensión abdominal, evacuaciones hemorrágicas e inestabilidad hemodinámica. En
sus antecedentes personales, destaca distrés respiratorio inmediato, que requirió
intubación endotraqueal. Inicia alimentación enteral con fórmula adaptada a los 3
días de vida. ¿Cuál sería su actitud inicial?:

1. 1. Colocar una sonda rectal por posible cólico del lactante.


2. 2. Dejar a dieta absoluta, reposición hidroelectrolítica y antibioterapia.
3. 3. Observación y repetir la exploración en unas horas.
4. 4. Mantendría la alimentación oral y recogería coprocultivo.
Gráfico de respuestas
Comentario

!
!
!
!

La combinación de evacuaciones sanguinolentas y distensión abdominal en un lactante es


prácticamente definitoria de una enterocolitis necrotizante. El resto de los datos son compatibles
y los antecedentes apoyan este diagnóstico (enfermedad de membrana hialina). El tratamiento
de esta enfermedad consiste en dieta absoluta y antibioterapia intravenosa.(R2)

412. Se realiza radiografía de abdomen y se observa la siguiente imagen, junto con


exámenes de laboratorio: leucocitos 5000 (neutrófilos 40%, linfocitos 40%, bandas
15%, basófilos 5%), Hb 10%, plaquetas 150 000/mm3. ¿Qué patología le sugiere?:

1. 1. Alergia a las proteínas de vaca.


2. 2. Síndrome hemorrágico del lactante.
3. 3. Megacolón agangliónico.
4. 4. Enterocolitis necrotizante.
Gráfico de respuestas
Comentario

Esta pregunta, en realidad, es un regalo. Le están aportando más datos, desde luego, pero no
los necesita. Para responder a la pregunta anterior necesitaba saber el diagnóstico: enterocolitis
necrotizante, por lo que estos datos que añaden no aportan nada a lo que ya sabía. La imagen
que puede ver en la radiografía muestra distensión de las asas intestinales, muy similar a la que
puedes encontrar en el capítulo correspondiente del Manual CTO, pero insistimos en que no era
necesaria.(R4)

413. Sobre enfermedad diarreica aguda en niños:

1. 1. Más del 90% no requiere tratamiento antibiótico.


2. 2. Todas deberían recibir antibióticos.
3. 3. La reacción inflamatoria es positiva cuando los leucocitos son mayor de 10/campo.
4. 4. El trimetoprim/sulfametoxazoll ha demostrado ser muy útil en el tratamiento.
Gráfico de respuestas
Comentario

Se trata de un concepto importante y básico. La mayoría de diarreas en niños están causadas


por virus y no es necesario tratarlas con antibióticos a no ser que se trate de cuadros con
especial gravedad. Respuesta 1 correcta.(R1)

414. Ante una placa de color blanco de 3 centímetros de diámetro sobre la mucosa de
la lengua que no se desprende con el raspado, debe pensarse como primera
posibilidad diagnóstica en:

1. 1. Micosis por Candida albicans.


2. 2. Leucoplasia.
3. 3. Carcinoma epidermoide bien diferenciado.
4. 4. Eritroplasia erosiva.
Gráfico de respuestas
Comentario

!
!
!
!

La leucoplasia es la lesión precancerosa más frecuente de la mucosa oral.

La leucoplasia oral se ha definido desde un punto de vista fundamentalmente clínico, como una
lesión predominantemente blanca de la mucosa oral, que no puede catalogarse como ninguna
otra lesión definida. Desde el punto de vista etiológico, algunas se relacionan con el tabaco
(factor de riesgo más importante), mientras que en otros casos son idiopáticas. No obstante,
existen otros factores de riesgo.

Desde el punto de vista histológico, en algunos casos existe displasia cervical, pero no en todos.
Cuando ésta existe, puede ser leve, moderada o severa, siendo mayor el riesgo de evolución
maligna cuanto mayor es el grado. En caso de tener esta evolución, lo más frecuente es que se
desarrolle un carcinoma epidermoide.(R2)

415. Ante un paciente con claudicación intermitente a 200 m y una úlcera isquémica
crónica en el talón, nos encontramos, según la clasificación de Fontaine, ante un:

1. 1. Estadio I.
2. 2. Estadio IIa.
3. 3. Estadio III.
4. 4. Estadio IV.
Gráfico de respuestas
Comentario

Es importante que se aprenda los estadios de la clasificación de Fontaine porque en base a ella
se decide el manejo terapéutico de un paciente con isquemia arterial crónica. El estadio II se
define por la presencia de claudicación intermitente y se subdivide en IIa o IIb (claudicación
incapacitante) en función del dolor aparezca a más o menos de 150 metros. Es a partir del
estadio IIb cuando se recomienda la realización de técnicas de revascularización. El estadio III
se define por la aparición de dolor de reposo y el estadio IV por la aparición de lesiones tróficas,
como le ocurre al paciente de esta pregunta.(R4)

416. Un niño de 3 años diagnosticado de Tetralogía de Fallot nos lo traen porque le


encuentran más irritable, más cianótico y con polipnea. De todas las siguientes
medidas terapéuticas, ¿cuál NO estaría indicada?

1. 1. Decir al niño que se coloque de cuclillas.


2. 2. Administrar oxígeno.
3. 3. Administrar bicarbonato.
4. 4. Iniciar digitalización.
Gráfico de respuestas
Comentario

Recuerda el tratamiento de las crisis hipoxémicas de la tetralogía de Fallot:

- Beta bloqueantes

- Morfina

!
!
!
!

- Fenilefrina

- Posición en cuclillas

- Oxigenoterapia (eficacia controvertida, pues la hipoxia es debida a un efecto shunt)

- Bicarbonato en casos de acidosis grave

No existe indicación de digital en esta patología. De hecho, el uso de este fármaco podría
incluso agravar la estenosis pulmonar y producir síncopes.(R4)

417. Ante un neonato a término que tras cesárea presenta precozmente taquipnea y
cianosis que desaparece con pequeñas cantidades de oxígeno, en el que la
auscultación pulmonar es normal y en el que la radiografía muestra marcas vasculares
prominentes y líquido en las cisuras sin broncograma aéreo, sospecharemos:

1. 1. Enfermedad de membrana hialina.


2. 2. Taquipnea transitoria del recién nacido.
3. 3. Síndrome de aspiración meconial.
4. 4. Persistencia de la circulación fetal.
Gráfico de respuestas
Comentario

418. ¿Cuál de estas lesiones oculares NO es típica de la artritis reumatoide?:

1. 1. Uveítis.
2. 2. Escleritis.
3. 3. Queratoconjuntivitis seca.
4. 4. Escleromalacia perforante.
Gráfico de respuestas
Comentario

La uveitis puede aparecer en pacientes con AR con una frecuencia del 1%, por lo tanto
equiparable a la de la escleritis. Sin embargo la uveitis no es más frecuente en estos pacientes

!
!
!
!

que en la población general y por lo tanto no podemos considerarla manifestación propia de la


enfermedad. Recuerde que lo más frecuente es la queratoconjuntivitis seca y lo más típico la
escleritis.(R1)

419. Una de las siguientes sustancias NO produce o agrava el reflujo gastroesofágico:

1. 1. Nifedipino.
2. 2. Levosulpiride.
3. 3. Nicotina.
4. 4. Chocolate.
Gráfico de respuestas
Comentario

Los calcioantagonistas y nitritos, o los fármacos con un comportamiento similar al de los nitritos,
como es el caso del sildenafilo, relajan la musculatura del cuerpo esofágico, alivian el dolor de
las contracturas o espasmos y relajan el esfínter esofágico inferior, por lo que aumenta el reflujo.
Las dietas ricas en grasas, chocolate, café o el uso de nicotina también relajan el esfínter. El
levosulpiride es un potente procinético, que estimula la contractilidad del cuerpo esofágico y
contrae el esfínter esofágico inferior. Por ello, no agravaría el reflujo, sino todo lo contrario:
podría emplearse como tratamiento médico.(R2)

420. A propósito del antígeno prostático específico (APE), todo es cierto EXCEPTO:

1. 1. No es de utilidad en el diagnóstico de reincidencia después de la prostatectomía radical.


La densidad del APE (APE sérico/volumen prostático) es útil en pacientes con próstata
2. 2.
grande, en los que el APE elevado puede deberse a hiperplasia benigna.
Las glándulas prostáticas neoplásicas bien diferenciadas producen más PSA que las
3. 3.
cancerosas no diferenciadas.
4. 4. El APE puede presentar aumentos normales en relación con la edad.
Gráfico de respuestas
Comentario

Pregunta de cierta dificultad. Las opciones 2, 3 y 4 son ciertas. Sólo recordar los límites
normales, entre 0 y 4 ng/mL y que aunque la densidad de APE puede ser útil para diferenciar
entre neoplasia y HBP, no supera en eficacia al APE total. La respuesta 1 es errónea, tras un
prostatectomía radical las cifras de APE deben ser indetectables. Si se produce un ascenso de
APE, puede deber a micrometastásis, recidiva local o ambas cosas. Por tanto, es una
herramienta muy útil para el seguimiento del paciente tras la cirugía.(R1)

421. Mujer de 50 años, menopáusica desde hace uno, sin antecedentes médicos ni
quirúrgicos de interés, que padece un importante síndrome climatérico con sofocos.
Respecto a su posible tratamiento señale la FALSA:

1. 1. Los estrógenos son la primera opción terapéutica, pero asociando progestágenos.


2. 2. Otra opción es la tibolona, que además reduce el riesgo de fracturas vertebrales.
También se pueden utilizar los SERM (raloxifeno), que además previenen el cáncer de
3. 3.
mama.
4. 4. Los fitoestrógenos son una alternativa cuando está contraindicada la terapia hormonal.
Gráfico de respuestas
Comentario

!
!
!
!

La falsa es la 3, ya que, aunque el raloxifeno sí previene la osteoporosis y el cáncer de mama,


empeora los síntomas vasomotores, que es el problema fundamental de esta paciente por lo
que hay que desechar esta opción terapéutica. El tratamiento más efectivo en esta paciente
como bien dice la respuesta número 1 son los anticonceptivos hormonales combinados
manteniéndolos el menor tiempo posible, por asociarse a un incremento de presentación de
cáncer de mama.(R3)

422. Un paciente afecto de la enfermedad de Niemann-Pick presenta un cuadro de


anemia grave que requiere transfusiones. ¿Cuál cree usted que es el mecanismo de la
anemia que presenta este paciente?

1. 1. Anemia hemolítica autoinmune.


2. 2. Anemia perniciosa.
3. 3. Anemia mieloptísica.
4. 4. Anemia por déficit de hierro.
Gráfico de respuestas
Comentario

Pregunta de dificultad moderada sobre las anemias.

La enfermedad de Niemann-Pick se caracteriza por una acumulación de esfingomielina y


colesterol en las células de distintos órganos. El acumulo en la médula ósea puede dar lugar a
una anemia mieloptísica.

Se denomina mieloptisis a la ocupación de la médula ósea por cualquier proceso patológico que
distorsiona la arquitectura normal de dicha médula. Es típico la presencia de dacriocitos y
reacción leucoeritroblástica (células inmaduras en sangre periférica).

La causa más frecuente de mieloptisis son micrometástasis de carcinoma en la médula


ósea.(R3)

423. En una paciente de 68 años, sin historia de patología osteoarticular previa, la


aparición de proliferación ósea y engrosamiento de las articulaciones interfalángicas
distales sin apenas signos inflamatorios nos hará sospechar la presencia de:

1. 1. Gota crónica con presencia de tofos.


2. 2. Artrosis con nódulos de Bouchard.
3. 3. Eritema nodoso.
4. 4. Artrosis con nódulos de Heberden.
Gráfico de respuestas
Comentario

La artrosis en el ENARM es fácil, y esta pregunta recuerda algo típico aunque rebuscado: los
nódulos de Heberden. Son manifestaciones de la artrosis, muchas veces asintomáticos pero que
generan un problema estético por engrosamiento de las articulaciones interfalángicas distales.
Recuerda que la artrosis no es una inflamación, sino una degeneración. Es conveniente
destacar su transmisión genética, especialmente las mujeres que los padecen suelen tener
antecedentes familiares. La localización es bien típica, a diferencia de los de Bouchard, que son
menos frecuentes y se localizan en interfalángicas proximales. Recuerde que la AR muestra

!
!
!
!

inflamación sinovial en muñecas y metacarpofalángicas. Los tofos son granulomas erosivos que
aparecen típicamente en 1ª articulación metatarsofalángica, tendón aquíleo, codo o pabellón
auricular, y suelen aparecer en pacientes con historia de gota.(R4)

424. El carbonato de litio está indicado específicamente en el tratamiento de:

1. 1. Crisis depresivas agudas.


2. 2. Equivalentes depresivos psicosomáticos.
3. 3. Crisis agudas de excitación.
4. 4. Alteraciones maníacas y depresivas crónicas.
Gráfico de respuestas
Comentario
La indicación princeps del litio es la manía aguda y la profilaxis de fases afectivas en la
enfermedad bipolar, con una eficacia del 80%. Esta eficacia baja a menos del 30% en fases y
cursos atípicos, mixtos y en ciclaciones rápidas. Otras indicaciones del litio son posibles, pero
son anecdóticas. Aunque es más eficaz que el placebo en la profilaxis de episodios depresivos
unipolares, esta eficacia no es excesiva (es mejor antimaniaco que antidepresivo, y esto también
con los bipolares), y han aparecido alternativas terapeuticas en los últimos años: ISRSs de
mantenimiento y lamotrigina.(R4)

425. Respecto a la citología cervicovaginal, señale la opción FALSA:

La citología cérvico-vaginal o test de Papanicolaou es el método más efectivo en el


1. 1.
cribado del cáncer cervical.
2. 2. Tiene una alta sensibilidad (80-90%), para el diagnóstico de displasias y cáncer.
3. 3. Tiene un alto porcentaje de falsos negativos, que puede llegar al 30%.
4. 4. La sensibilidad para el adenocarcinoma es menor que para las lesiones escamosas.
Gráfico de respuestas
Comentario

La citología cérvico-vaginal o test de Papanicolau es el método más efectivo en el cribado del


cáncer cervical. Consiste en la triple toma de células del tracto genital (vaginal, endocervical y
ectocervical) con el fin de detectar la existencia de displasias antes de que se haya producido
invasión del estroma por células neoplásicas. Tiene una baja sensibilidad (50-60%), debida
fundamentalmente a errores en la toma de muestra y errores en laboratorio, y un alto porcentaje
de falsos negativos, que puede llegar al 30%. La sensibilidad para el adenocarcinoma es menor
que para las lesiones escamosas. Se dispone también de la citología en medio líquido, la cual
disminuye el número de citologías no valorables y permite la realización de otras técnicas
adicionales, como la determinación de HPV en la muestra. Aumenta la sensibilidad con respecto
a la citología convencional, sin reducir significativamente su especificidad.(R2)

426. Señale cuál de los siguientes NO es un factor de riesgo de cáncer de ovario:

1. 1. Edad.
2. 2. Historia familiar.
3. 3. Multiparidad.
4. 4. Síndrome Swyer.
Gráfico de respuestas
Comentario

!
!
!
!

Los principales factores de riesgo de cáncer de ovario son la edad avanzada, la historia familiar
y las mutaciones BRCA1 y BRCA2. En el síndrome de Swyer también podemos considerar que
hay mayor riesgo de cáncer de ovario ya que este síndrome se asocia con la aparición de
tumores en la gónadas disgenéticas, fundamentalmente el gonadoblastoma. La respuesta
correcta es la multiparidad, ya que la nuligestas tienen mayor indicencia de cáncer de ovario, ya
que la gestación es un periodo de reposo ovulatorio.(R3)

427. Paciente intervenido hace un mes de una hernia inguinal indirecta, que desde
entonces presenta dolor en ingle, base de pene y zona alta del escroto, acompañado a
veces de cortejo vegetativo. ¿En qué complicación pensaría usted?:

1. 1. Sección del nervio femorocutáneo.


2. 2. Sección del nervio femoral.
3. 3. Sección del nervio ilioinguinal.
4. 4. Sección del cordón espermático.
Gráfico de respuestas
Comentario

Se trata de un pregunta muy difícil. Es importante que conozca algunos detalles de la anatomía
inguinal. El conducto inguinal contiene el nervio ilioinguinal o abdominogenital menor en ambos
sexos, pero en el varón no forma parte del cordón espermático sino que lo acompaña por fuera,
es decir, es extrafunicular. Pues bien en la cirugía de la hernia inguinal indirecta es posible
lesionar o simplemente manipular este nervio. Entonces se produce el síndrome que se explica
en este caso clínico.(R3)

428. ¿En qué tumor de ovario podemos encontrar los cuerpos de Call-Exner?:

1. 1. Gonadoblastoma.
2. 2. Cistoadenocarcinoma mucinoso.
3. 3. Coriocarcinoma.
4. 4. Tumor de la granulosa.
Gráfico de respuestas
Comentario

Los cuerpos de Call-Exner se presentan en tumor de la granulosa, y son células que semejan a
la granulosa de los folículos maduros, se disponen en forma de roseta con caivdad central
rellena de material PAS positivo.(R4)

429. Mujer de 48 años, alcohólica desde hace 15, que presenta clínica de disnea de
pequeños esfuerzos, ortopnea de dos almohadas, edemas maleolares que se
incrementan al final del día y debilidad generalizada. En la exploración física hay
ingurgitación yugular, edemas maleolares que dejan fóvea, hepatomegalia, pulso débil
y latido de la punta desplazado hacia abajo y afuera. ¿Cuál de los siguientes datos
obtenido de las exploraciones complementarias NO apoyaría su diagnóstico de
sospecha?:

1. 1. Dilatación de las cavidades cardíacas en el ecocardiograma.


2. 2. Disminución de la función contráctil en el ecocardiograma.
3. 3. Signos de dilatación de las cavidades en el ECG.

!
!
!
!

4. 4. Disminución de la presión telediastólica de las cuatro cavidades por cateterismo cardíaco.


Gráfico de respuestas
Comentario

Nos presentan una paciente con signos y síntomas de IC, tanto izquierda (disnea de esfuerzo)
como derecha (ingurgitación yugular, edemas maleolares, hepatomegalia). La exploración física
y los antecedentes son los que nos orientan a la causa de esta IC. Presenta un latido de la
punta desplazado hacia abajo y hacia fuera y el antecedente del alcoholismo de larga evolución.
Por tanto, el diagnóstico de sospecha sería la Miocardiopatía alcohólica. Se trata de un tipo de
miocardiopatía dilatada secundaria (es la causa más frecuente) y, por ello, esperaríamos
encontrar otros datos como: dilatación de los ventrículos y disminución de la contractilidad en el
ecocardiograma, signos de dilatación de las cavidades en el ECG, cardiomegalia y edema
intersticial y alveolar en la Rx (por la IC izquierda), y aumento (NO disminución) de la presión
telediastólica en las cuatro cavidades en el cateterismo. Recuerde que cuando el VI falla, la
presión telediastólica aumenta. Esto se explica porque durante la sístole no es capaz de
bombear toda la sangre que alberga, y parte del volumen se queda en el ventrículo, de forma
que en la telediástole habrá el volumen residual más el volumen que proviene de la
aurícula.(R4)

430. ¿Qué fenómeno ocurre como consecuencia del aumento de estrógenos producido
por la actividad del folículo dominante?:

1. 1. La menstruación.
2. 2. La ovulación.
3. 3. La transformación del folículo a cuerpo lúteo.
4. 4. El pico de LH que produce la ovulación.
Gráfico de respuestas
Comentario

La FSH estimula en el ovario el crecimiento de la cohorte de folículos primordiales


seleccionados. La bajada de FSH selecciona el folículo dominante y la atresia simultánea del
resto por exceso local de andrógenos. La granulosa del folículo seleccionado contiene
aromatasa que emplea los andrógenos de la teca para producir estradiol. El pico de estradiol
dispara el pico de LH y este pico de LH provoca 10-12 horas después la ovulación.(R4)

431. Señale cuál de los siguientes cuadros cutáneos NO se ha relacionado con


infecciones estreptocócicas:

1. 1. Eritema nodoso.
2. 2. Psoriasis en gotas.
3. 3. Liquen plano.
4. 4. Eritema marginado.
Gráfico de respuestas
Comentario

Pregunta que requiere cierta integración de conocimientos dermatológicos. El eritema nodoso es


una paniculitis, más frecuentemente idiopática, pero que puede ser secundaria a numerosas
causas, entre ellas las infecciones estreptocócicas. Algo similar ocurre con las vasculitis por
hipersensibilidad o leucocitoclásticas. La psoriasis guttata o en gotas, es una forma de psoriasis
generalmente autolimitada, que se suele ver en niños y se relaciona con infecciones

!
!
!
!

estreptocócicas. El eritema marginado forma parte de la clínica cutánea de la fiebre reumática,


complicación no supurada de infecciones estreptocócicas. El liquen plano suele ser idiopático,
pero se le ha asociado también a infección por virus de la hepatitis C, sales de oro, antipalúdicos
y tiazidas, pero no con infecciones estreptocócicas.(R3)

432. Respecto al carcinoma adenoide quístico de senos paranasales señale la


proposición INCORRECTA.

1. 1. Tiene tendencia a la invasión perineural.


2. 2. No son habituales las metástasis al diagnóstico.
3. 3. El tratamiento de elección es la radioterapia.
4. 4. La supervivencia a los 10 años es del 15-55%.
Gráfico de respuestas
Comentario
Además de aparecer en las glándulas salivares mayores, el carcinoma adenoide quístico puede
tener su origen en las glándulas salivares menores repartidas por toda la vía aerodigestiva
superior. Son tumores que característicamente presentan diseminación perineural hacia
estructuras anexas, aunque al diagnóstico no suelen tener metástasis regionales o a distancia.
Otra característica es que tienen gran tendencia a la recidiva local y metástasis tardías (a veces
con años de latencia), por lo que su pronóstico a largo plazo no es bueno. El tratamiento de
elección es la cirugía, reservando la RT para los casos no operables.(R3)

433. Ante un cuadro de macrohematuria, oliguria e HTA en un niño de 7 años que


presentó un impétigo 2 semanas antes, ¿qué dato de laboratorio nos extrañaría
encontrar?

1. 1. Excreción fraccional de sodio (EFNa) en orina superior al 1%.


2. 2. Proteinuria de 2 g/día.
3. 3. Disminución del aclaramiento de creatinina.
4. 4. Anticuerpos antiestreptolisina 0 (ASLO) bajos.
Gráfico de respuestas
Comentario

Nos describen un síndrome nefrótico que aparece después de un impétigo, por lo que
deberíamos pensar en una GN postestreptocócica. Recuerde que, en esta entidad, desciende el
filtrado glomerular, por lo que el riñón trata de reabsorber todo el sodio que puede, por lo que la
excreción fraccional de sodio no estará aumentada, sino disminuida (inferior a 1).(R1)

434. Paciente de 4 años de edad, es llevado a urgencias por tos y polipnea. Antecedente
de síndrome obstructivo bronquial a repetición, rinitis y urticaria. Al exámen físico: FR
64 por minuto, FC 150 por minuto, presenta tirajes, sibilancias y estertores. ¿Cuál sería
su presunción diagnóstica?

1. 1. Bronquiolitis.
2. 2. Insuficiencia cardíaca congestiva.
3. 3. Asma.
4. 4. Reflujo gastroesofágico.
Gráfico de respuestas
Comentario

!
!
!
!

Debe dominar el Asma para el ENARM. El cuadro que nos describen es el típico de asma (tos,
taquicardia, tirajes, sibilancias...). Los antecedentes de rinitis y urticaria apoyan este diagnóstico.
Respuesta 3 correcta.(R3)

435. Un paciente es intervenido quirúrgicamente, practicándose una incisión para


apendicectomía, según McBurney, y encontrándose un apéndice cecal de
características normales. ¿Cuál sería su actitud entonces?:

1. 1. Dejar el apendice y cerrar la incisión de apendicectomía.


2. 2. Realizar la apendicectomía y cerrar la incisión de apendicectomía.
3. 3. Realizar la apendicectomía y revisar al menos 100-150 cm de intestino delgado distal.
4. 4. Revisar el intestino delgado distal.
Gráfico de respuestas
Comentario

Esta pregunta es más práctica que teórica y no le debe preocupar mucho si la ha fallado pues es
un tema controvertido. En la cirugía abierta lo ortodoxo es extirpar el apéndice aun cuando el
aspecto macroscópico sea normal, debido a que la presencia de una cicatriz quirúrgica en la
fosa ilíaca derecha podría causar confusión para diagnósticos futuros. Con el desarrollo del
procedimiento endoscópico ha resurgido la discusión sobre si se debe extirpar el apéndice que
en una evaluación laparoscópica tiene aspecto macroscópico normal. Si se encuentra otra
patología responsable del dolor agudo, ésta debe solucionarse por técnica laparoscópica o
abierta y dejar el apéndice en su sitio. El mayor problema se presenta cuando el apéndice
parece sano y no se demuestra otra patología como causa del dolor. De esta pregunta debes
quedarte con que se debe extirpar el apéndice aunque sea normal siempre que el abordaje sea
abierto.(R3)

436. Nuligesta de 27 años que presenta de dismenorrea desde hace 1 año, que le
impiden trabajar por dos días durante sus menstruaciones. Ocasionalmente presenta
dolor durante las relaciones sexuales. Peso 50 kg. Talla 1.60 m. IMC (Índice de Masa
Corporal) 20 kg/m2. El examen pélvico: vulva y vagina aparentemente normal; cérvix
rosado con mínima eversión endocervical, útero de tamaño normal. Ovario izquierdo
de 2 x 3 cm, ovario derecho de 4 x 6 cm. ¿Cuál de las siguientes es la causa más
probable de esta condición?

1. 1. Endometriosis.
2. 2. Enfermedad pélvica inflamatoria.
3. 3. Síndrome de ovario poliquístico.
4. 4. Síndrome premenstrual.
Gráfico de respuestas
Comentario

Tema bastante importante en el ENARM.

En la endometriosis el dolor es el síntoma más frecuente y característico. Generalmente se


manifiesta en forma de dismenorrea progresiva que no cede con la toma de antiinflamatorios, ni
con anticonceptivos hormonales. Otros síntomas son: alteraciones menstruales, rectorragia,
dismenorrea, disuria e infertilidad.

!
!
!
!

El diagnóstico de sospecha se hará por la clínica y por el ultrasonido, en el que es habitual la


visualización de los quistes endometriósicos, si bien el diagnóstico definitivo es a través de
laparoscopia, que además permitirá tratar a la paciente . Respuesta 1 correcta.

La EPI se descarta ya que no refieren fiebre, leucorrea, ni dolor a la movilización del cérvix.

El SOP se presenta con datos clínicos de hiperandrogenismo, asociado a periodos de


insuficiencia ovárica.(R1)

437. Dentro de las múltiples causas de infertilidad, se consideran las siguientes,


EXCEPTO:

1. 1. Esterilidad de causa inexplicada.


2. 2. Antecedente de EPI.
3. 3. Uso de anticonceptivos orales.
4. 4. Antecedente de TB.
Gráfico de respuestas
Comentario

Dentro de las causas de infertilidad estarían las siguientes: tuberculosis, EPI, idiopática, pero no
por uso de anticonceptivos orales. Respuesta 3 incorrecta.(R3)

438. Tras sufrir una crisis tónico-clónica, un paciente de 33 años aqueja dolor sobre la
articulación glenohumeral derecha, que permanece fija en rotación interna. La única
lesión objetivada en la radiografía es la fractura aislada del troquín. ¿Cuál es el
diagnóstico más probable?

1. 1. Inestabilidad glenohumeral crónica.


2. 2. Lesión del manguito de los rotadores.
3. 3. Luxación glenohumeral posterior.
4. 4. Lesión de Bankart.
Gráfico de respuestas
Comentario

La luxación posterior de hombro se asocia con frecuencia con una fractura aislada de troquín.
Radiológicamente, la presencia de esta fractura nos debe alertar sobre una posible luxación
posterior de hombro, puede ser muy difícil de diagnosticar. Por ello, no suele bastar una una
proyección anteroposterior, siendo necesaria una lateral.

Existe otro detalle muy importante que debe recordar sobre la luxación posterior. Esta luxación
suele relacionarse con situaciones en las que se produzca una contracción violenta y simultánea
de todos los músculos del hombro. Por ello, debe pensar en ella ante un paciente que ha
padecido una crisis convulsiva o una descarga eléctrica.(R3)

439. ¿Cuál es el tratamiento de elección del síndrome de Pancoast?:

1. 1. Cirugía.
2. 2. Quimioterapia.
3. 3. Inmunoterapia.
4. 4. Radioterapia y cirugía.

!
!
!
!

Gráfico de respuestas
Comentario

Pregunta de dificultad media. El síndrome de Pancoast se produce por crecimiento local de un


tumor del vértice pulmonar que penetra en el canal neural y destruye las raices nerviosas octava
cervical y primera y segunda torácicas. La causa más frecuente es el cancer de pulmón,
principalmente el epidermoide. Cursa con dolor de hombro irradiado por todo el borde cubital del
antebrazo.Suele coexisteir con afectación del simpático cervical produciendo el síndrome de
Claude-Bernard-Horner,consistente en miosis,ptosis,enoftalmos y en ocasiones anhidrosis
ipsilaterales. El síndrome de Pancoast se considera T3 para la clasificación TNM, luego podría
pertenecer a un estadio IIb-IIIa. Para el tratamiento del síndrome de Pancoast existen 2
posibilidades: radioterapia en dosis curativas o radioterapia seguida de resección del tumor y
pared torácica 3-6 meses después (por lo tanto la respuesta correcta es la 4). En ambos casos
la supervivencia es similar.(R4)

440. Nos traen a Urgencias a un escolar de 7 años de edad, que estando totalmente
bien, presenta fiebre de 40ºC, acompañada de vómitos, cefaleas, pulso rápido y algo
débil, con sensación de enfermedad grave. A la exploración destacan amígdalas
hipertróficas con exudado blanco-amarillento, así como enantema en paladar blando.
Además presenta adenitis satélite en ángulo mandibular izquierdo. A las 14 horas
aparece exantema puntiforme a la palpación que respeta el ángulo nasogeniano. A los
3-4 días se inicia descamación furfurácea. El tratamiento etiológico de este proceso
es:

1. 1. Claritromicina por 5 días.


2. 2. Tratamiento sintomático.
3. 3. Penicilina oral durante 10 días.
4. 4. Asociar corticoides al antibiótico elegido.
Gráfico de respuestas
Comentario

Nos están describiendo una escarlatina. Esta está producida por el S. pyogenes que es sensible
al tratamiento con penicilina. Respuesta 3 correcta.(R3)

441. Señale la respuesta FALSA respecto a la hepatopatía alcohólica:

El 10% de los casos de cirrosis alcohólica se descubre casualmente durante una


1. 1.
laparotomía o autopsia.
Los cuerpos de Mallory, probablemente constituidos por agregados de filamentos
2. 2.
intermedios, no son específicos de la hepatitis alcohólica.
3. 3. En la cirrosis alcohólica podemos encontrar una hipergammaglobulinemia policlonal.
En la hepatitis alcohólica, las transaminasas están moderadamente elevadas, predominando
4. 4.
la ALT sobre la AST.
Gráfico de respuestas
Comentario

Una pregunta muy sencilla que no tendría que haber ni siquiera dudado. Si tiene que saber algo
para el ENARM sobre la hepatopatía alcohólica, es cómo reconocerla a partir de su patrón
enzimático: AST>ALT, al contrario que en las víricas.(R4)

!
!
!
!

442. A ferropenic anemia is found after a routine blood count in a previously healthy
40-year-old man. A complete colonoscopy reveals an adenocarcinoma at the hepatic
angle. His grandfather died from colon cancer when he was 50 year-old; his father and
two of his father’s siblings also died from colon cancer. He has three children ages 15,
12 and 6. Mark the CORRECT answer.

1. 1. Stool occult blood should be tested annually to the children.


2. 2. Annual colonoscopy should be performed to the children, starting from age 12.
3. 3. Sigmoidoscopy every five years should be performed to the children
4. 4. Colonoscopy every three years should be performed to the children, starting from age 25.
Gráfico de respuestas
Comentario

En los pacientes que cumplen 2 de los criterios de Amsterdam del Síndrome de cáncer
colorrectal hereditario no asociado a poliposis (CCHNP): 3 ó más familiares afectos de cáncer ce
colon (al menos uno que sea familiar de primer grado de los otros 2), uno o más casos en la
familia de cáncer colorrectal antes de los 50 años y afectar al menos a dos generaciones
consecutivas, deberá realizarse en los familiares una colonoscopia periódica (cada 2-3 años) a
partir de los 25 años, o 5 años antes del familiar afecto más joven, para el diagnóstico precoz de
un eventual cáncer de colon.(R4)

443. De las causas de obstrucción laríngea aguda, mencione Ud. cuál o cuáles pueden
ser consideradas como más frecuentes.

1. 1. Traqueitis bacteriana.
2. 2. Laringotraqueitis aguda (crup viral).
3. 3. Uvulitis.
4. 4. Bronquiolitis.
Gráfico de respuestas
Comentario

Pregunta muy fácil y muy preguntada te preguntan por obstrucción laríngea... la única opcion
que habla de laringe es la 2!!!!! por lo que esta es la respuesta correcta!!!(R2)

444. Un paciente de 19 años está siendo estudiado, junto con el resto de sus familiares,
por haber ocurrido dos muertes súbitas de origen cardíaco en sendos familiares hace
un año. No refiere ningún síntoma y la exploración física es normal. En el
electrocardiograma aparece un ritmo sinusal a 70 l.p.m. con un PR de 180 mseg, un
QRS normal y QTc de 500 mseg. En el ecocardiograma no se aprecia nada patológico.
¿Cuál de las siguientes cree que sería la actitud más CORRECTA en este paciente?:

1. 1. Antiarrítmicos del grupo Ia.


2. 2. Betabloqueantes.
3. 3. Antagonistas del calcio.
4. 4. Tratamiento quirúrgico.
Gráfico de respuestas
Comentario

!
!
!
!

El caso clínico ya nos ofrece el diagnóstico, ya que el QTc (c= corregido) es de 500 mseg, y
cuando es mayor de 440 mseg (0.44 seg) se define el síndrome de QT largo. Recuerde que este
síndrome va asociado a la taquicardia en torsión de puntas (o taquicardia helicoidal), y la clínica
consiste en síncopes recidivantes, FV, muerte súbita cardíaca, etc. La etiología es muy variada
(alteraciones electrolíticas como la hipocalcemia, la hipopotasemia, o la hipomagnesemia,
algunos fármacos antiarrítmicos), y existen casos de QT largo congénito, en los que la
enfermedad es hereditaria y puede haber varios casos en una misma familia. El tratamiento de
la taquicardia en torsión de puntas consiste sobre todo en tratar la causa desencadenante. En el
QT largo congénito se pueden dar betabloqueantes (es el fármaco más usado por ser el que
más ha demostrado mejorar el pronóstico).(R2)

445. En relación a la enfermedad hemolítica por isosensibilización ABO, señale la


FALSA:

1. 1. La madre suele ser O.


2. 2. Pueden afectarse los productos del primer embarazo.
3. 3. El antígeno más frecuentemente implicado es A1.
4. 4. La enfermedad se agrava en los embarazos siguientes, si no hacemos profilaxis.
Gráfico de respuestas
Comentario

No se confunda con la isoinmunización por Rh, en la cual es característico que se agrave con
los embarazos subsecuentes en caso de no realizar profilaxis. La isosensibilización por ABO es
la más frecuente, pero menos severa que por Rh. No se agrava en los embarazos siguientes ya
que los anticuerpos estan preformados a diferencia de los del grupo Rh.(R4)

446. Paciente de 59 años con EPOC que acude por aumento de su disnea basal. En la
gammagrafía de perfusión aparecen dos defectos segmentarios, con discordancia con
la Rx de tórax. Al realizar la gammagrafía de ventilación no existen claras zonas
hipoventiladas. ¿Cuál es la actitud más CORRECTA?:

1. 1. Se descarta un TEP.
2. 2. Hacer un TC de tórax.
3. 3. Iniciar anticoagulación.
4. 4. Añadir antiagregantes.
Gráfico de respuestas
Comentario

El algoritmo diagnóstico-terapéutico del TEP es muy importante. La gammagrafía de perfusión


pulmonar es el test de screening más útil para descartar un TEP agudo clínicamente importante.
Los resultados normales casi nunca se asocian con TEP, es decir, tiene valor para descartarlo.

Se describe como gammagrafía de alta probabilidad diagnóstica la que presenta dos o más
defectos de perfusión moderados a extensos (ocupando más del 25% del tamaño de un
segmento pulmonar) con gammagrafía de ventilación y RX tórax normales en las zonas afectas.
A esto se llama discordancia ventilación-perfusión y es típico del TEP.

!
!
!
!

En este caso, dado que la gammagrafía podría definirse como alta probabilidad de TEP y existe
discordancia con la gammagrafía de ventilación, está claramente indicada la instauración de
tratamiento anticoagulante.(R3)

447. ¿Cuál de los siguientes enunciados sobre el trastorno obsesivo-compulsivo es


CIERTO?:

1. 1. Un acto de carácter repetitivo que disminuye la ansiedad del sujeto es una compulsión.
El paciente no es capaz de reconocer que sus obsesiones y compulsiones son excesivas e
2. 2.
irracionales.
3. 3. La mayoría de los pacientes con un TOC presentan rasgos obsesivos premórbidos.
4. 4. El fármaco más habitual que se utiliza en el tratamiento es la imipramina.
Gráfico de respuestas
Comentario

El TOC es la neurosis más preguntada en el ENARM junto con el trastorno de crisis de


ansiedad. R1: es la definición de compulsión R2: si no lo reconoce debemos pensar en una idea
delirante o hipocondriaca. R3: no es necesario, de hecho sería factor de mal pronóstico. R4: casi
caemos, suele ser la clorimipramina, de perfil más serotoninérgico.(R1)

448. A 2-year-old boy is brought to the emergency department after his parents have
noticed that he accidentally swallowed the content of a bottle of paracetamol syrup 30
minutes ago. The boy has no current symptoms and physical examination is
unremarkable. Oral activated charcoal therapy is prescribed. Which of the following
options is the most appropriate next step in medical management?

Obtain blood analysis, and liver function tests. If liver enzymes are within normal limits,
1. 1.
the patient can be discharged.
Keep the patient in the emergency department. 4 hours after the acetaminophen intake,
2. 2. obtain a blood sample and measure acetaminophen blood levels in order to know whether
or not the patient will need intravenous N-acetylcysteine therapy.
Keep the patient in the emergency department. 4 hours after the acetaminophen intake,
3. 3. obtain a blood sample and measure acetaminophen blood levels in order to know whether
or not the patient will need protamine therapy.
Keep the patient in the emergency department. 6 hours after the acetaminophen intake,
4. 4. obtain a blood sample and measure acetaminophen blood levels in order to know whether
or not the patient will need atropine therapy.
Gráfico de respuestas
Comentario

Para comenzar un tratamiento de la intoxicación por paracetamol se debe tomar como punto
guía el nomograma adaptado de Rumack. Por tanto, se deben tomar determinaciones de niveles
de acetaminofeno en plasma y comenzar el tratamiento oportuno antidótico, a partir de las
cuatro horas tras la ingesta en caso de que aquellos se encuentren por encima de la línea del
nomograma: éstos son 150 mg/L a las 4 horas, y 30 mg/L a las 12 horas. El tratamiento de la
intoxicación por paracetamol se basa en dos conceptos: Decontaminación gastrointestinal con
carbón activado y el uso de N-acetilcisteína (NAC).

- Decontaminación: El uso de carbón activado está indicado en todos los pacientes que se
presentan en las primeras 4 horas de la ingestión y después si hay uso de preparados de

!
!
!
!

liberación prolongada o ingesta simultánea de drogas que retarden el vaciamiento gástrico. El


carbón activado adsorbe el paracetamol, disminuyendo su absorción en un 90%.

- N-acetilcisteína (NAC): Es un precursor de glutatión que previene la toxicidad por paracetamol


además de tener propiedades inotrópicas y vasodilatadores. La máxima eficacia de la NAC se
produce cuando se usa dentro de las primeras 10 horas de la ingesta, reduciendo la
hepatotoxicidad de 50 a 5% y la mortalidad global de un 8 a 0%. Sin embargo, el uso más tardío
también ha probado ser beneficioso en estos pacientes.(R2)

449. A 45-year-old male patient diagnosed with follicular thyroid carcinoma comes to
his physician for a follow-up after he was treated with total thyroidectomy 6 months
ago. He also underwent his first radioiodine whole-body total scintigraphy (with usual
dose of 50 mCi). His most recent laboratory studies show elevated thyroglobulin levels.
Which of the following is the most appropriate next step in management?

1. 1. Complete radiological skeletal survey and chest X-ray.


Prescribe TSH-suppression therapy with L-T4 and perform a new radioiodine whole-body
2. 2.
scintigraphy (WBS) in one year.
3. 3. Perform a new radioiodine whole-body scintigraphy with ablating doses (100 mCi).
Measure again his thyroglobulin levels in 3 months. If the levels are still elevated, perform
4. 4.
a new radioiodine whole-body scintigraphy with ablating doses (100 mCi).
Gráfico de respuestas
Comentario
Pregunta importante dado que hace referencia al seguimiento del cáncer diferenciado de
tiroides. Estos pacientes son seguidos mediante la determinación de tiroglobulina y la realización
de rastreos corporales de forma periódica. Si en cualquier momento del seguimiento el paciente,
como en este caso, presenta una elevación de la tiroglobulina, se diagnóstica de recidiva de la
enfermedad. Lo más lógico en este caso, será realizar un rastreo corporal para localizar la
metástasis. Si no se evidencian las metástasis con la dosis del rastreo, se debe conocer que
existen micrometástasis que si que captan dosis más elevadas de yodo, por lo que se
administrará una dosis elevada del mismo (opción correcta 3) La opción 4 es falsa, dado que el
radioyodo se administrará tanto si existen como si no las imágenes sugestivas de recidiva. Este
último hecho hace que en la actualidad cuando un paciente presenta tiroglobulina positiva, se
asume que existe una recidiva y se administra una dosis elevada de radioyodo, realizando el
RCT posteriormente a la administración de éste.(R3)

450. La pauta más consensuada para administrar antibióticos de forma profiláctica en


cirugía es:

1. 1. Cada 8 h el día antes y el día de la intervención.


2. 2. Monodosis en el momento de la inducción anestésica.
3. 3. Tres dosis posoperatorias.
4. 4. Monodosis intraoperatoria.
Gráfico de respuestas
Comentario

!
!
!
!

La profilaxis antibiótica se inicia en la inducción anestésica. En ningún caso debería prolongarse


más allá de 48 horas, aunque las recomendaciones actuales serían de una duración inferior a 24
horas (respuesta 2 correcta).

Si analiza el resto de las opciones, verá que se administra los antibióticos en momentos distintos
a la inducción, o que se utilizan pautas de 48 horas, lo que va en contra de lo explicado.(R2)

!
!
!
!

!
!
!
!

!
!
!
!

!
!
!
!

!
!
!
!

!
!
!
!

!
!
!
!

!
!
!
!

!
!
!
!

!
!
!
!

!
!
!
!

!
!
!
!

!
!
!
!

!
!
!
!

!
!
!
!

!
!
!
!

!
!

S-ar putea să vă placă și